You are on page 1of 603

REMEDIAL

MATHEMATICS

Sudhir Kumar Pundir

Asian Books Private Limited


REMEDIAL
MATHEMATICS

Sudhir Kumar Pundir


Reader, Department of Mathematics and Computer Science
S.D. (P.G.) College, Muzaffar Nagar (UP)

7/28, Mahavir Lane, Vardan House, Ansari Road,


Darya Ganj, New Delhi -110002.
"This page is Intentionally Left Blank"
Remedial Mathematics
S.K. Pundir

Registered and Editorial Office


7/28, Mahavir Lane, Vardan House, Ansari Road, Darya Ganj, New Delhi - 110 002.
E-Mail: asian@asianbooksindia.com ..
World Wide Web: http://www.asianbooksindia.com
Phones: 23287577,23282098,23271887.23259161
Fax: 91 1123262021

©Publisher
First Published 2010
ISBN 978-81-8412-112-4
All rights reserved. No part of this publication may be reproduced, stored in a retrieval system, or
transmitted in any form or by any means, electronic, mechanical, photocopying, recording and/or
otherwise, without the prior written permission of the publishers.

Published by Kamal Jagasia for Asian Books Pvt. Ltd., 7/28, Mahavir Lane, Vardan House, Darya
Ganj, New Delhi - 110 002.
'/jIpesetting at Add Computers
Printed at Compudata Services, N. Delhi
Sales Offices
Bangalore 103, Swiss Complex No. 33, Race Course Road, Bangalore - 560 001
Ph. : 080-22200438 Fax: 91 8022256583
Email: asianblr@blr.vsnl.netin
Chennai Palani Murugan Building No.21, West Cott Road, Royapettah,Chennai-600014
Ph.: 044-28486928,32979058
Email: asianmds@vsnl.net
D~ihi 7/28, Mahavir Lane, Vardan House, Ansari Road, Darya Ganj, New Delhi - 110002
Phones: 011-23287577,23282098,23271887,23259161, Fax: 91 1123262021
E-Mail: asian@asianbooksindia.com
Guwahati 6, G.N .B. Road, Panbazar, Guwahati,Assam-781 001
Ph. :0361-2513020,2635729
Email: asianghyl@sanchamet.in
Hyderabad 3-5-315, Street No.7, Narayanguda, Vittalwadi, Hyderabad - 500 029
Phones: 040-23220112, 23220113, 32927534 Fax: 9~40 24751152
Email: hydasian@gmail.com
Kolkata lOA, Hospital Street, Calcutta - 700 072
Ph. : 033-22153040, 32506706 Fax: 91 3322159899
Email: calasian@vsnl.net.in
Mumbai Shop No.3 & 4 Ground Floor Shilpin Centre
40, GO, Ambekar Marg, Sewree Wadala Estate, Wadala, Mumbai - 400031
Ph.: 022-32510689,24157611-12 Fax 91-22-24157613
Email: asianbk@mtnl.netin
Pune Shop No. 5-8, Ground Floor Shaan Brahma Compo Near Ratan Theatre,
Budhwar Peth, Pune - 02
Ph.: 020-3230455424497208 Fax: 91-20-24497207
Email: asianpune@asianbooksindia.com
Preface

The book "Remedial Mathematics" has been written according to the latest syllabus
of B. Pharma and many other courses related to Bioscience of different technical
colleges of India.
The book is furnished with well defined theoretical background of the subject
followed by well graded set of examples. At the end of each chapter, a set of unsolved
problems as an exercise and chapter revision and learning fundamentals with the name
"Refresher" to revise the chapter quickly to the students has been given. Various
concepts !lnd related theories have been given in simple manner. The language used
in this book is simple and a fairly large number of solved and unsolved problems with
"hints have been added. I believe that the subject matter of this book will be very helpful
to each of B. Pharma students in getting high percentage of marks in this paper.
I gratefully acknowledge my indebtedness to various authors and publishers,
whose books have been freely consulted during the preparation of the book. However,
'No Claim' is made of originality of results but presentation is my own. I have extended
a conscious effort to make the book student friendly. I have tried my best to keep the
book free from misprints. The author shall be grateful to the readers who point out
errors and omissions which in spite of all cares, might have been there.
I wish to sincerely thank Smt. Purobi Biswas, Production Manager, Asian Books
Private Limited, New Delhi, whose encouragement and support enabled me to complete
my book timely. I must also record my appreciation due to my wife Dr. Rimple, daughter
Rijuta and son Shrish for their understanding and love during the long period that I
have taken to complete this book.
Mr. S.P. Singh, MIET Meerut; Mr. Aftab Alam, SDCMS, Muzaffar Nagar;
MI'. Nishutosh, SDCET, Muzaffar Nagar; deserve special attention and thanks for their
kind help and support.
Above all, I am thankful to Almighty, without whose grace nothing is possible for
anyone.
Suggestions for further improvement will be gratefully acknowledged and accepted.

Dr. Sudhir Kumar Pundir


Email: skpundir05@yahoo.co.in
Contents

I. QUADRATIC EQUATIONS ..................................................................................,.......... 1-60


1.1 Introduction .................................................................................................................... I
1.2 Linear Equation ......................................................................................... :.................... I
1.3 Solution of Quadratic Equations .................................................................................. 4
1.4 Equations Reducible to Quadratics ................................................................................ 12
I.S Nature of Roots of a Quadratic Equation ................................................................ 36
1.6 Symmetric Function of The Roots ........................................................................... 36
1. 7 Relation Between Roots and Coefficients ................................................................ 38
1.8 Formation of Equations ...................................................... ........................................ 39
1.9 Common Roots ........................................................................................................... 39
1.10 Application of Quadratic equations ............................................................................. 48
Objective Evaluation ..................................................................................................... S6
Refresher ....................................................................................................................... S9

2. SIMULTANEOUS LINEAR EQUATION ................................................................. 61-89


2.1 Linear Equation in Two Variables ................................................................................. 61
2.2 Solution ofa Given System of Simultaneous Equations ............................................... 61
2.3 Graphical Method for Solving Simultaneous Linear Equations ............................... 62
2.4 Algebraic Methods of Solving a Pair of Linear Equations ............................................. 71
2.5 Word Problems on Simultaneous Linear Equations ....................................................... 81
Objective Evaluation ..................................................................................................... 86
Refresher ....................................................................................................................... 88

3. DETERMINANTS ........................................................................................................ ~1-126

3.1 Introduction .................................................................................................................. 90


3.2 Determinant of Order Two ...................................................... ...................................... 90
3.3 Determinant of Order Three ...................................................... .................................... 91
3.4 Co-factors and Minors of an Element ...................................................... ................ 92
3.5 Properties of Determinants ........................................................................................ 9S
3.6 Cramer's Rule ............................................................................................................. 117
Refresher ............................................. ............................. ... ......... ............. ......... ......... 12S
(vi) Contents

4. MATRICES ............................................................................................................. ;•. 127-210


4.1 Introduction ............................................................................................... ,................ 127
4.2 Types of Matrices .................................................................................................... 127
4.3 Determinant of Square Matrix .................................................................................. 129
4.4 SingularandNon-SingularMatrix ...........................................................~ .................... 129
4.5 Sub- Matrix of a Matrix ............................................................................................ 130
4.6 Minors of a Matrix ................................................................................................... 130
4.7 Transpose of Matrix ....................................................................... ¥ 130
••••••••••••••••••••••••

4.8 Symmetric and Skew-Symmetric Matrices ................................................................... 130


4.9 Complex Matrix ......................................................................................................... 131
4.10 Algebra of Matrices ................................................................................................. 132
4.11 Properties of Matrices Addition ............................................................................ 133
4.12 Multiplication of Matrices ........................................................................................ 136
4.13 Adjoint of a Matrix ............................................................................................... 149
4.14 Inverse or Reciprocal of Matrix ................................................... ,........................ 150
4.15 Orthogonal and Unitary Matrices ................................................... ~ .................... 151
4: 16 Solution of Equations Using Inverse of a Matrix ................................................ 165
4.17 Rank of a Matrix .................................................................................................... 172
4.18 Echelon Form of a Matrix ..................................................................................... 172
4.19 Elementary Transformations of a Matrix ............................................................. 173
4.20 Elementary Matrices ................................................................................................ 173
4.21 Invariance of Rank Under E-transformation .......................................................... 174
4.22 Normal Form ........................................................................................................... 174
4.23 Equivalence of Matrices ........................................................................................... 174
4.24 Rank of Product of Matrices ..................................................................................... 175
4.25 Linear Equations ....................................................................................................... 189
4.26 Nature of the Solution of Equation Ax = 0 ................................................................. 190
4.27 Non-homogeneous Equations .................................................................................. 193
4.28 Conditions for Consistency .................................................................................... 193
4.29 Conditions for a System of N-equations in N-unknowns to have a ......................... .
Unique Solution ........................................................................................................ 194
Objective Evaluation ................................................................................................. 206
Refresher ................................................................................................................... 209
5. MEASURE OF CENTRAL TENDENCY ............................................................. 211-254
5.1 Introduction ................................................................................................................ 211
5.2 Kinds of Statistical Averages .................................................................................. 211
5.3 Arithmetic Mean (A.M.) ............................................................................................ 211
5.4 Methods of Calculating Arithmetic Mean in Individual Series ............................... 212
5.5 Properties of Arithmetic Mean ................................................................................ 220
5.6 Combined Mean ......................................................................................................... 225
5.7 Geometric Mean ......................................................................................................... 227
5.8 Properties of Geometric Mean ............................................................................... 230
5.9 Harmonic Mean .......................................................................................................... 233
Contents (vii)

5.10 Properties of Harmonic Mean .................................................................................. 237


5.11 Median ........................................................................................................................ 242
5.12 Mode ........................................................................................................................... 248
6. TRIGONOMETRY ..................................................................................................... 255-321
6.1 Introduction ................................................................................................................ 255
6.2 Angles and Quadrants ............................................................................................... 255
6.3 Measurement of Angles ............................................................................................ 255
6.4 Various Types of Angeles ......................................................................................... 256
6.5 Trigonometric Ratios or Functions .......................................................................... 264
6.6 TRigonometric Identities and Equations .................................................................. 265
6.7 Fundamental Trigonometric Identities ...................................................................... 265
6.8 Signs of Trigonometric Functions ............................................................................. 275
6.9 Trigonometrical Ratio of Compound Angles (to be Used Directl) .......................... 292
6.10 Trigonometrical Ratio for Half Angles

(Obtained by Replacing A by ~ in the above Formulae) ...................................... 293


6.11 Conditional Identities ................................................................................................ 308
Objective Evaluation ................................................................................................... 315
Refresher ..................................................................................................................... 318
7. LOGARITHMS ........................................................................................................... 322-345
7.1 Introduction ................................................................................................................. 322
7.2 Properties of Logarithms ........................................................................................... 323
7.3 System of Logarithms ............................................................................................... 323
7.4 Standard Form of Decimal ........................................................................................ 324
7.5 Characteristic and Mantissa ...................................................................................... 324
7.6 Method to Determine The Characteristic and Mantissa ......................................... 332
7.7 Antilogarithm .............................................................................................................. 335
7.8 ,Application of Logarithm in Pharmaceutical Problems ........................................... 339
€lbjective Evaluation ................................................................................................... 342
Refi;esher ..................................................................................................................... 344
8. SYSTEMS OF COORDINATES ............................................................................. 346-374
8.1 Introduction ................................................................................................................. 346
8.2 Distance between Two Points ................................................................................. 349
8.3 Collinear Points ......................................................................................................... 352
8.4 Section Formula ........................................................................................................... 354
8.5 Mid Point Formula ..................................................................................................... 355
8.6 Area of a Triangle ..................................................................................................... 360
8.7 Locus and Equation to a Locus ................................................................................ 365
Objective Evaluation ................................................................................................... 370
Refresher ..................................................................................................................... 372
9. THE STRAIGHT LINES ........................................................................................... 375-411
9.1 Introduction ................................................................................................................. 375
9.2 Slope or Gradient of a Line ..................................................................................... 375
(viiO Contents

9.3 Slope of a Line Through Two Points ..................................................................... 376


9.4 Equation of Lines is Standard Form ...................................................................... 380
9.5 Transformation of General Equation in Different Standard Forms ........................ 394
9.6 Point of Intersection of Two Lines ......................................................................... 397
9.7 Condition of Concurrency of Three Given Lines ................................................... 397
9.8 Angle between Two Intersecting Lines .................................................................... 400
Objective Evaluation ................................................................................................... 408
10. FUNCTIONS AND LIMITS .................................................................................... 412-458
10.1 Introduction ............................................................................................................... 412
10.2 Type of Functions .................................................................................................. 413
10.3 Some Particular Functions ...................................................................................... 423
10.4 Algebra of Functions .............................................................................................. 428
10.5 Composition of Functions ....................................................................................... 429
10.6 Concept of Limit ..................................................................................................... 439
10.7 One Side Limits ....................................................................................................... 447
10.8 Limit at Infinity and Infinite Limits ..................................................................... 447
10.9 Continuity ................................................................................................................ 448
Objective Evaluation ................................................................................................. 454
Refresher .............................................~..................................................................... 457
11. DIFFERENTIATION ................................................................................................... 459-514
11.1 Introduction ............................................................................................................... 459
11.2 Method for Finding the Derivative Using First Principle .................................... 459
11.3 Derivative of the Sum of Two Functions ............................................................. 464
11.4 Derivative of the Difference of Two Functions ................................................... 465
11.5 Derivative of the Product of Two Functions ....................................................... 468
11.6 Derivative of the Quotient of Two Functions ..................................................... 469
11.7 Derivative of Functions of aFunction (Chain Rule) ............................................ 472
11.8 Differentiation of Implicit Functions ..................................................................... 483
11.9 Logarithmic Differentiation ...................................................................................... 486
11.10 Second Order Derivatives ........................................................................................ 505
Objective Evaluation ................................................................................................. 510
Refresher ................................................................................................................... 513
12. INTEGRATION ........................................................................................................... 515-586
12.1 Introduction ............................................................................................................... 515
12.2 Indefinite Integral .................................................................................................... 515
12.3 Methods of Integration ........................................................................................... 520
12.4 Integration by Parts ................................................................................................ 530
12.5 Integration by Partial Fractions ............................................................................. 537
12.6 Definite Integral ....................................................................................................... 564
12.7 Properties of Definite Integrals ............................................................................. 564
APPENDIX .................................................................................................................... 587-594
QUADRATIC EQUATIONS

• INTRODUCTION
(i) Polynomial: A function/defined by
lex) ra +a x+azx2+ ... +a/,x
O 1 E R
where ao' a p a2, ... , an E R is called a polynomial of a real variable with real coefficients.
Remarks
*
• If an 0 then the degree of the polynomial is n.
• If ao' aI' ... , an E C, the set of complex number and x E R, then the polynomial is
called complex polynomial.
(ii) Polynomial Equation: Let/(x) be a polynomial, then/ex) = 0 is called the
polynomial equation. Generally, a polynomial equation of degree two is called
quadratic equation.
(iii) Degree of an Equation: The degree of an equation is the index of the highest
power of variable quantity involved in the equation, when the equation has
been expressed to the rational integral form (radical free form).
(iv) Roots of an Equation: Let/(x) = 0 be a quadratic equation. A real or complex
numbers a is said to be a root or solution of a quadratic equation/ex) =
ax2 + bx + c = 0, if/(a) = aa2 + ba + c = 0 i.e. a satisfies the given quadratic
equation.
(v) Solution Set: The set of all roots of an equation, is called the solution set of
the given equation.
(vi) Identity: An expression involving equality and a variable is called an identity,
if it is satisfied by every value of the variable.
Remark
• A root of an equation is also called zero.

III LINEAR EQUATION


An equation of the form ax + b = 0 is called a linear equation of x, where x is unknown
*
variable (quantity) and a and b are any constants. !fere, a 0 because if a ~ 0 then the
equation gives b = 0, which has no unknown.
2 Remedial Mathematics

For example, x - 2 = 0, 3x - 9 = 0, x + ~ = 0, x = 2, x = 0 are the linear equation in x.


Remark
• To find the degree of an equation, the unknown variable x must be in the numerator
only in the equation and power of x must be a positive integer.

Solution of a Linear Equation


All those values of the unknown variable, which is involved in the equation, for which
equation is satisfied, are called the solutions (or roots) of the equation. To find the solution
ofa linear equation we write the equation in standard form (ax + b = 0) ifit is not.
Method: consider a linear equation
ax + b = 0, a*O ...(1.1)
Here, we want to find the value of x, So, add - b to both sides in (1.1) we get
ax+b-b = O-b
=> ax=-b
Now, dividing both sides by a, we get
b
x=--,a*O
a
Hence, the solution of the given equation is x = ~, a * 0
a
For example:
Consider the equation 2x + 3 = 0. First adding - 3 in both sides of the given equation, we get
2x+3-3 =0-3
=> 2x =-3
Now, dividing both sides by 2, we get
3
x = - 2' which is the required solution.

SOLVED EXAMPLES
Example 1: Solve the following equation

11_1::._ y -
10
=y
10 5
Solution: Here, the given equation is

11_1::._
y - 10 =y
...(1 )
10 5
and y is the unknown variable in this equation.
So, multiplying the equation (1) by 10, we get
10
10(11-1::._ Y - ) = lOy
IO 5
QJladratic Equations 3

110_y_1O(y-1O) = lOy
5
=> 11O-y-2y+20 = lOy
=> 130-3y = lOy ... (2)
Now, adding 3y on both sides of (2), we get
130-3y+3y = lOy+3y
=> 130 = 13y
130
y = -=10
13
Hence, y=1O
Example 2: Solve the equation 3 (x + 2) = 5
Solution: Here, the given equation is 3 (x + 2) = 5 and x is unknown variable in the given
equation.
So, 3 (x+2) = 5
=> 3x+3 x2 = 5
=> ~+6=5

Now, adding -6 on both sides of the above equation, we get


3x+6-6 = 5-6
3x =-1
1
x = --
3
Example 3: Solve the equation x + 2 = 2x - 8.
Solution: Here, the given equation is x + 2 = 2x - 8, adding 8 on both sides of the above
equation, we get
x+2+8 = 2x-8+8
=> x+IO =2x
=> 10 = 2x-x
=> 10 = x or x= 10
x x-6
Example 4: Solve -+3 =--
2 8
·
S o IutIOn: H ere, th ·
e gIven . .IS -x + 3 =x-6
equatIOn --
2 8

=> 8(~+ 3) =x-6

8x
=> -+8x3 =x-6
2
=> 4x+24 = x-6
Now, adding -24 on both sides, of the above equation, we get
4x+24-24 = x-6-24
4x = x-30
4 Remedial Mathematics

4x-x = -30
3x = -30
x = - 30 =-10
3
x = -10
Theorem 1: A quadratic equation can not have more than two roots.
Proof: Let, if possible, a, p, y be the three distinct roots of the quadratic equation
ax2 + bx + c = 0, then we have
aa2 +ba+c = 0 ...(Ll )
ap2+bp+c = 0 ...(1.2)
and ar+by+c = 0 ... (103)
Using (Ll) and (1.2), we get
a(a2-p2)+b(a-p) = 0
=> a(a-p)(a+p)+b(a-p) = 0
=> (a-p)[a(a+p)+b] = 0
a(a+p)+b = 0
b
a+p = -- ... (1.4)
a
Similarly, using (1.2) and (1.3), we get
b
P+y = - - ...(1.5 )
a
Equation (1.4) and (1.5) gives
a+p = P+y => a=y
which is a contradiction (:. a and yboth are distinct)
Hence, the quadratic equation cannot have more than two roots

III SOLUTION OF QUADRATIC EQUATIONS


There are three methods to solve a complete quadratic equation:
(a) Method offactorization
(b) Method of Completing the square
(c) Method offormula

SOLVED EXAMPLES
Type I: Method of Factorization :
Example 1: Solve Xl - 4x + 3 = 0
S"ution: Here, we have x 2 - 4x + 3 = 0
=> x 2 - 3x - x + 3 = 0
=> x(x-3)-1 (x-3) = 0
=> (x-3)(x-1) = 0
x = 1,3
Quadratic Equations 5

E~a mple 2. Solve x2 - 2x - 3 = 0


Solution: Here, we havex2-2x-3 = 0
~ x2-3x+x-3 = 0
~ x(x-3)+1(x-3)=0
~ (x-3)(x+ I) = 0
x=-1,3
Type II: Method of Completing the Square:
Remark
• This method can be employed when the factorization method fails i. e. when the
quadratic expression is not factorize or difficult to factorize.
Example 3: Solve 4x2 - 3x - 1 = 0
Solution: Here, wehave4x2-3x-1 =0

~ 4( x2 - ~x) = I
3 I
~ x 2 --x =
4 4
~ + (-~J [By adding both sides (1/2 the coeff. of x)]

1. + ~ = 25 = (~y
4 64 64 8)
[By taking square root of both sides 1

x= l±~
8 8

x= l+~
3 5
or x= - - -
8 8 8 8
x= or
4

Hence, the solution set of the given equation is {I, - ±}


Example 4: Solve 2x2 - 7x + 6 = 0
Solution: Here, the given equation is 2x2 -7x + 6 = 0
~ a2-7x =-6
7 =-3
x2 - -x
2

~ x2_7....x+(_7....)2 = -3 + ( -
2 -4
~y [BY adding both sides,H the coeff. Ofxt
7 49 49
x 2 --x+- = -3+-
2 16 16
6 Remedial Mathematics

=>
16
7
=> x-- =±4 (By taking square root of both sides)
4
Therefore x = 2. ± 1.. = 2. + 1.. = ! = 2 and x = 2. - 1.. = .§. = .l
, 44444 4442

Hence the solution set of the given equation is {%' 2}.


Type III: Method offormula
Consider a quadratic equation ax 2 + bx + c = 0, a,* 0
The solution of equation (l) is given by ... (1)
2
-b± Jb -4ac
x=
2a
Example 5. Solve each a/the/allowing equation by quadratic/ormula.
(i) 5x2 - 15x + 11 = 0 Oi) ~ - 3x + 5 = 0
Solution: (i) Comparing the given equation with ax 2 + bx + c = 0, we get,
a =S,b=-IS,c=ll

- b± Jb 2 - 4ac
Therefore, x=
2a
-C-1S)±J(-IS)2 -4xSxll IS ± ~22S - 220
2xS 10
lS±)5 IS+)5 IS-)5
---or---
10 10 10

. set IS
Hence, the solutlOn . [IS+)5 , - S-)5]
--
10 10
2
(ii) Comparing the given equation x - 3x + S = 0 with the standard quadratic equation
ax 2 + bx + c = 0, we get
a = l,b=-3,c=S
-b ± Jb 2 - 4ac
Therefore, x=
2a
_ (_ 3) ± ~r-(--3-i---4-x-l-x-S
2xI

3±~ =3±FU
2 2
3±i,Ji!
(:.P=-l)
2
Quadratic Equations 7

. d so IuttOn
H ence, th e reqUIre ' set IS +i
" gIven by {3--2--' 3 - i2 m m}
Example 6: Solve the following equations by quadratic method.
(i) 2~ - 4x + 3 = 0 (ii) 25~ - 30x + 11 = 0
Solution: (i) Here, the given equation is 2x2 - 4x - 3 = 0
Comparing with the standard quadratic equation ax2 + bx + c = 0, we get
a = 2, b = - 4, c = 3

Now, x = - b± Jb 2
- 4ac = 4±.J16=24
2a 2x2

= 4±R = 4±W = 4±i)8


4 4 4
4 ± 2i.fi I .
= =I±-l
4 J2
I.
= I +-Ior I - -II.
.fi .fi

Hence, the solution set is {I + ~ i, 1- ~ i}


(ii) Comparing the given equation 25x2 - 30x + 11 = 0 with the standard quadratic
equation, we get
a = 25, b = - 30, c = II

-b±~b2 -4ac 30±J900-1100


Therefore, x=
2a 50

30 ± ROO 30 ± J200ii
50 50
30±IOJi =l±.fii
50 5 5

1 + .fi i or 1_ J2 i
5 5 5 5

Hence, the required solution set is {1


5
+ J2 i,l- J2
5 5 5
i}
Example 7: Solve the following equations:
I 2 4
(i) - - + - - - - - (ii) 2x Jl3 + 2x-i / 3 = 5
x+1 x+2 - x+4
I 2 4
Solution: (i) Here, - - + - - = --
x+1 x+2 x+4
(x + 2) + 2 (x + 1) 4
(x + I)(x + 2) x+4
8 Remedial Mathematics

(3x+4)(x+4) =4 (x+ 1)(x+2)


3.x2+ 16x+ 16 = 4 (.x2 + 3x+2)
x2 -4x-8 = 0

_b±~b2 -4ac
Put all these values in x = 2a ' we get

-(-4) ± ~(-4)2 - 4·1(-8)


x = --'---'-----'--'--'----~
2 ·1

=
4±~16+32 =
2
(1 ± v3{:;)
2
(ii) Here, the given equation is 2xl/3 + 2x- I / 3 = 5
2
So, Put x l/3 = y, we get2y+ - = 5
y
21-5y+2 = 0
(y-2)(2y-l) = 0
1
Y = 2, 2'
Now, x = y=(2)3=8

or
(tY i =

Hence, x = 8 or 8

Example 8: Solve the equation x +..Ix = ;5


So, put ..Ix = y in the given equation, we get 1 + Y = 265
::::> 251-25y-6 = 0
::::> (5y+ 6)(5y-l) = 0
6
y = -- or
5 5

Now, x =1= (-~f = ~~


or (~f =
1
25
36 1
Hence, x = - or x=-
25 25
Quadratic Equations 9

1. Solve the following equations by factorization method


(/) .x2+x+I=O (ii) x2-x-12=0 (iii) x2+1=0 (iv) x2-4x+3=0
2. Solve the following equations by quadratic method.
(l) x2-7x+12=0 (ii) x2-4x+7=0
2
(iii) 2x -3x+ 1 =0 (iv) 27.x2-10x+ I =0
(v) 3x2-x-1O=0 (vi) 9.x2+12x+4=0
.
3. Solve the equatlOn 6"x -"5x = I5
x x
- 3" + 7
4. Solve the following equations:
x 2 -3x
(i) ---+2x =6 'it) 2.x2-lOx=3x-15
7
5. Solve the following equations:
2X2 - 3x - 5 4x + 9
(ii) =--
3 5
6. Solve the following equations:
3x -7 16
(/) --=-
5 x-5
7. Solve the equation: pqx2 - (p2 + l) x + pq = 0
8. Solve: ~{2(x2 - x + I)} = 3x-4

9. Solve: 52x -126· 5x +3 =0


10. Solve the following equations:

(I) J(3x + 10) = 9- J(9x + 7) 2


(ii) J3x +1 + ~
2
3x + I

. . vlpx)
(1/1)
Fl-x)
~ ) vl--;--x-) ="6
+
13

HINTS TO THE SELECTED PROBLEMS

2. (I) .x2-7x+12=0. (ii) x2-4x+7=0.


Here a= I, b =-7, c= 12 x= l,b=-4,c=7.
b ± ~b2 -4ac -b ± ~r-:b2:--_-4a-c
x= x =
2a 2a
7 ± ~(- 7)2 - 4 x I x 12 4 ± ~(- 4)2 - 4 x I x 7
x =
2x 1 2xl
10 Remedial Mathematics

7±.J49-48 7±1 =4±..jl6=2s


2 2 2
7+1 7-1 = 4±..FI2
- -- or
2 2 2

x = 4or3. = 4±2J)i =2± J) i


2
x 2 -3x
3. ~- ~ = 1~ - j +7 4. (i) 7 + 2x = 6

5x -6x x -5x x 2 -3x + 14x


--- =--+7 = 6
30 15 7
-x -4x
= --+7
30 15
4x x
--- =7 => ~+ 14x-3x-42 = 0
15 30
8x-x
=7 => x(x+ 14)-3 (x+ 14) = 0
30

=> -7x =7 => (x+ 14)(x-3) = 0


30
=> x = 30 => x = -14
or x = +3
2X2 - 3x - 5 4x+9
5. ~ll)
3 5
=> 10~-15x-25 = 12x+27
=> 1O~-27x-52 = 0
=> a = 10, b =-27, c=-52

-b± ~b2 -4ac


x =
2a

27 ± ~(- 27)2 - 4 x 10 x (- 52)


2 x 10

27 ± .J729 + 2080 27 ± .J2809 27 ±53


20 20 20
27 +53 27 - 53 -13
or =4or-
20 20 10
3x-7 16
6. (I) --
5 x-5
=> (3x-7)(x- 5) = 80
Quadratic Equations 11

3~-15x-7x+35 = 80
3~-22x-45 = 0
3~-27x+ 5x-45 = 0
3x(x-9)+5(x-9) = 0
(3x+ 5)(x-9) = 0
3x+5 = 0 or x-9=O
-5
x = - or x=9.
3
7. pq~_(p2+q2)x+ pq= 0
=> pqx2 _ p 2x - q2x + pq = 0
=> px (qx-p)-q (qx-p) = 0
=> (qx-p) (px-q) = 0
either qx - p = 0 or px - q = 0
p q
x=- or x=-
q P
8. 2
)2(x - x + 1) = 3x-4
Squaring both sides.
2 (~-x + 1) = (3x-4)2
=> Y-2x+2 = 9~+ 16-24x.
=> 7~-22x+ 14 = 0

+22±) (22)2 -~x7xI4


x=
2x7

= 22±~484-392
14
22 ± J92 22 ± 2 53 11 ± 53
14 14 7

let 5X = y
=> 1-126y+3 = 0

126 ± )(126)2 - 4 x 1 x 3
y=
2xl
= ±126

ANSWERU

(ii) 4,-3 (iii) (i, - i)


12 Remedial Mathematics

2. (I) 3,4 (ii) 2 + J3 i, 2 - J3 i


(iv) ~±J2i (v) 2 --
5
(vi)
2
27 27 ' 3 3
3
3. 30 4. (I) 3,-14 (ii) 2,5

(") 4 -13 -5
5. (z) 2,-3 II , 10 6. (z) 9'"3 (ii) +3,-3

p
7. !L 8. 11±m 9. OJ
q p 7
10. (I) 2 (ii) 0, ±,fS (") 9 4
III 13' 13
III EQUATIONS REDUCIBLE TO QUADRATICS
Type I
Equation of the form aX' +bX'+ c=O, whereXis an expression inx.

STEP KNOWLEDGE
In order to solve such type of equation we use the following steps:
Step 1. Putx.' = y, and obtain the equation. ai + by + c = O.
Step 2. Now solve the obtained equation for y.
Step 3. Finally get the value of x, by using the relation Xn = y.

~~~~~~~I SOLVED EXAMPLES ~I~~~~~~


Example 1: Solve x4 - 9x2 - 10 = 0
Solution: The given equation is x4 - 9x2 - 10= 0 ...(1)
Put, x 2 = yin (1), we get i -
9y - 10 = 0, which is a quadratic equation iny.
i-9y-l0 = 0
=> ;-10y+y-l0 = 0
=> y(y-1J)+ 1(y-l0) = 0
=> (y-lO) (y+ 1) = 0
=> y = -1, lO
Now Y = 10
=> x 2 = 10
=> x=±J1O
and y =-1
=> x 2 =-1
=> x =±i
Hence, the solution set is given by (JlQ, - JlQ, i, -i)
Quadratic Equations 13

2x+I 4- 10 (-
2x+I )2
(
Example 2: Solve - -
x-I J
- + 9 = O.
x-I
rUPTU B. Pharma 20051

2x + 1 2x + I
SolutIOn: We have (- -)4- 10 (- -J2 + 9
.
= 0
x-I x-I

=> y-lOy+9 = 0

=> y-9y-y+ 9 = 0
=> y(y-9)-1 (y-9) = 0
=> (y-1)(y-9) = 0
=> y = 1 or y=9

Now y -_ 1 => ( - + 1)2


2x - 2x + 1 _ I
= 1=>---±
x-I x-I
2x + I 2x + I
=> = I- - =-1
or
x-I x-I
=> 2x+l =x-I or 2x+ 1 =-x+ I
=> x = -2 or x=O.
2
2x + I 2x + 1
Again y = 9 => - - =9 =>-- =±3.
( x-I ) x-I
2x + I 2x+ I
=> =3 or --=-3
x-I x-I
=> 2x+ I = 3x-3 or 2x + I = - 3x + 3
2
x = 4 or x= -
5'

Hence, the solution of the given equation is {-2, 0, 4,~}

Example 3: Solve the equation (x _ _ x_)2 + 2x


x+1 x+1
= 3 ~
Solution: Here, the given equation is (x __x+x_)2 + 2x ~ = 3
I x+ I

x2)2 x2
which can be written as - - + 2 - - = 3
( x+1 x+1
2
Now putting _x_ = y, the equation (I) reduces to y + 2y = 3
x +1
=> y+2y-3 =0=>y+3y-y-3=0
=> y(y+3)-I(y+3) =0=>(v+3)()'-I)=0
14 Remedial Mathematics

=> y = 1,-3
Now, y = 1
x2
- =1
x +1
x 2 = x+ 1
x 2 -x-l =0

x=
1 ± .ji+4 1±
=--
.J5
2 2
Also, y =-3
2
x
- =-3
x +1
x 2 = -3x-3
~+3x+3 = 0
- 3 ± ~9 -12 - 3 ± i J3
x= =----
2 2
· set IS
Hence, th e so IutlOn . {1±J5
-2-' -3±i.J3}
2-

Example 4: Solve the following equation 4x - 5. :]X + 4 = 0


Solution: 4x-5.2x+4 = 0
which can be written as (2 Xi - 5 (2X) + 4 = 0
Put '? = y, we get
y-5y+4 = 0
which is quardrat in y.
'So1ving (l) for y, we get
(y-4)(y-l) = 0
=> y = 1,4
Now, y =1
=> '? = 1
=> '? = 2°
=> x = 0
Also, y =4
=> '? = 4
=> '? = 22
=> x=2
Hence, the solution set of the given equation is (0,,2).
Example 5: Solve the equation ~/3 + x l13 - 2 = 0
Solution: Here the given equation is x 2/3 + X 113 - 2 = 0
=> i
(x ll3 + x l/3 - 2 = 0
Putx l/3 = y, we gety + y-2 = 0, which is quadratic iny,
Quadratic Equations 15

Solving for y, we get


(y+2)(y-l) = 0
=> y = 1,-2
Now,y= 1
=> X l/3 = I
=> x = 1
also, Y =-2
=> x l/3 =-2
x = (_2)3
=> x =-8
Hence, the solution set of the given equation is (1, - 8).
1)rpell

Equation of the form aX + !!.- + C = 0, where X is an expression in x.


X

STEP KNOWLEDGE
In order to solve, such type of equations, we use the following steps
Step I. Put X = Y and obtain the quadratic equation in y.
Step 2. Solve the quadratic equation for y.
Step 3. Finally get the values of x by using the relation X = y.

~~~~~~~I SOLVED EXAMPLES ~I~~~~~~

Example I: Solve ~ x +JX-l = 13, (.'(*I,x*O)


x-I x 6

·
So I utlOD: H
ere, t I '
1e gIven . .IS
equatIOn ~
- - + Fxl-l
- - = -13
x-I x 6

Put ~ x = y, then the above equation reduces to


x-I
I 13 7
Y + - = - => 6y- - 13y + 6 = 0
y 6
Solving for y, we get
(2y-3)(3y-2) = 0
3 2
y = 2'3
Now, y = 12 => ~ x-Ix = 12 => _x_ = (1)2
x-I 2
=> x = 2.
5

also, y=~=>~
3
x
x -I
=~3
16 Remedial Mathematics

Hence, the solution set is given by {~, -~}


3/2 8
Example 2: Solve 8x - -m = 63
x
·
S o IutJon: H ere t h '
e given . .IS 8 x 3/2 -
equatIOn -m
8 = 63
x
8
Putx3/2 = y, we get 8y - - = 63
y
=> 1-63y-8 = 0
=> 81-64y+y-8 =0
=> 8y(y-8)+ 1 (y-8) =0
=> (y- 8) (8y+ I) =0

y = 8,-i
Now, y = 8,x312 =8
=> x = (8)2/3 = (2 3 xl/3i = 22 = 4

also, Y = _k=>x3/2 = -k=>x = (_ky/3 = ±


Hence, the solution set is given by {4, i}
Example 3: Solve the equation 7' + x + 7' -x = 50 IUPTU B. Pharma 2008]
Solution: Here, the given equation can be written as
7·7 x +7·T x = 50
7.7 x +2.=50
x
7
Put r = y, we get

7y+1. = 50
y
=> 71 - 50y + 7 = 0, which is quadratic iny.
Solving for y, we get
71-49y-y+7 = 0
7y(y-7)-1 (y-7) = 0
(y-7) (7y-l) = 0
1
Y = 7,-
7
Now, y = 7
=> 7x = 7
{,)uadratic Equations 17

x = 1 (By putting the value ofy)


1 x 1
also, y = -::::::>7 =-::::::>x =-1
7 7
Hence, the solution set is given by {I, - I}
4x + 1 x + 1 5
Example 4: Jf-- + - - = - = 0 ,jindthevalue a/x. IUPTU B. Pharma 20081
x +I 4x + 1 2
4x + 1 x +I 5
Solution: We have - - + - - = -
x+I 4x + 1 2
4x+1
Let ~=y

I 5
then y+- = -
y 2
5
1+ 1 = -y
2
21+2 = 5y
21-5y+2 = 0
21-4y-y+2 = 0
2y(y-2)- I(y-2) = 0
(y-2)(2y-l) = 0
y = 2, ory= 1/2
Ify= 2 then
4x+ 1
- - =2
x+l
4x+ I = 2(x+ 1)::::::>4x+ 1 =2x+2
2x = I ::::::>x= 112.

EXERCISE 1.2 ~I~~~~~~~~~


Solve thefollowing equations:
1. x 4 -S.x2-9=0

4. (~)2
x+a
-5(~)+6=0
x+a
5. (.x2-3x+3i-(x-l)(x-2)=7 6. (.x2-5xi-30(.x2-5x)-216=0
7. 3x-2 + 7x- 1 +5=0 8. (.x2-5x+7)2_(x-2)(x-3)= 1

10. 3x + 1 + ~ = ~ (x E R)
x + 1 3x+ 1 2
x l+x 13
11. - - + - - = -
l+x x 6
18 Remedial Mathematics

13. 5x+ 1 +5 2- x = 53 + 1 14. 3x+rX-2 =0


4x -I 4x + 1 10
15. 2 2t +8 _8. 2x+2+1 =0 16. - - + - - = -
4x + 1 4x -I 3

17. 8~-f¥=2
x+3 x
18. )3x 2 +1+ ~=5
2
3x + 1

HINTS TO THE SELECTED PROBLEMS


1. x4 - 8~ - 9 = O.
Let x 2 =y =>;-8y-9=0
=> ;-9y+y-9 = 0 =>y(y-9) + 1 (y-9)=0
=> (y-9)(y+ I) = 0 =>y-9=0 or y+ 1 =0
=> y = 9 or y = - 1 => x 2 = 9 or x 2 = - 1
=> x = ± 3 or x = ± i
3. substitute x-I = y. to obtain 3; + 7y + 5 = O.

4. substitute (x -
x+a
a)
= y. to obtain; - 5y + 6 = O.

5. (~-3x+3i-(x-I)(x-2)=7
=> (x 2 -3x+3)2_(x2-3x+2)-7=0
=> (~-3x + 3)2_(~_ 3x+ 3)-6 = O.
=> substitute ~ - 3x + 3 = y. to obtain; - y - 6 = O.
9.2x =42x _1

=> Let
11. substitute x-I = y.
Then ;-12=-y
=> ;+y-12=0
=> ;+4y-3y-12=0
=> (y+4)(y-3) = 0
=> y = 3 or y=-4
=> x-I = 3 or x-I =-4

x=- orx=-l
3

ANSWERS

1 7 Jli.
l. (± 3, ± i) 2. ±-,± 1 3. - - ± - l
2 6 10
Quadratic Equations 19

4. -2a,-3a

5.0,3, - - 2 -
iJII 6. 2,3, - 4,9

5 ±iJ] 1 1
7. ± 2,± 3 8. 2,3, 2 9 ---
. 3' 4
I
10. -- I 11. -3,2 12. -1,1
5'
13. -1,2 14. 0 15. -4
1
16. ±-
2
17. 1 18. 0,0, ± J5
Typem
Equation ofthe form (x + b) (x + b) (x + c)(x + d) = e, where a, b, c, d are constant such that
a+b=c+d.

STEP KNOWLEDGE
In order to solve such type of equations, we use the following steps
Step 1. Put~ + (a + b) x = y and obtain the quadratic equation iny.
Step 2. Solve this quadratic equation for y.
Step 3. Finally get the value of x by putting ~ + (a + b) x = y

~~~~~~~I SOLVED EXAMPLES ~I~~~~~~


Example 1: Solve (x + 1) (x + 2) (x +3) (x + 4) = 120
Solution: Here, the given equation is
(x+ I)(x + 2)(x + 3)(x+ 4) = 120
which can be written as
[(x + I) (x + 4)][(x + 2)(x + 2)] = 120
=> (~+5x+4)(~+5x+6) = 120
2
put x + 5x = y, then equation (I) reduces to
(y+4)(y+6) = 120
=> y+ lOy+24-120 = 0
=> y+ 10y-96 = O. which is quadratic iny, solving fory, we get
y (y + 16) - 6 (y + 16) = 0
=> (y-6) ()It 16) = 0
=> Y =6,-16
Now, Y = 6
=> x 2 +5x = 6
=> ~+5x-6 = 0
=> x = 1,-6
Also, y = -16
=> x 2 + 5x = -16
20 Remedial Mathematics

=> xl+5x+ 16 = 0
-5±iJ39
=> x =
2
· set 0 f th
Hence, the so IutIOn ·
e given . .IS {I , -6, - 5 ±2i
equatIOn J39}
Example 2: Solve (2x - 7) (~ - 9) (2x + 5) = 91
Solution: Here, the given equation is
(2x - 7)(xl - 9)(2x + 5) = 91
which can be written as
(2x-7)(x-3)(x+ 3)(2x+ 5) = 91
=> [(2x-7)(x + 3)][(x-3)(2x+ 5)] =91
=> [2.x2-x - 21] [2x 2 -x - 15] = 91
Put 2x2 -x = y, we get
(y-21) (y-15) = 91
=> ;-36y+315-91 = 0
=> ;-36y+224 = 0
=> (y-8)(y-28) = 0
=> Y = 8,28
Now, Y = 8
=> 2x2 _x = 8
=> 2x2 -x-8 = 0
1±J65
=> x =
4
also, y = 28
=> 2xl-x = 28
2
2x -x-28 = 0
=>
=> x = 1 ± 15 = 4 _2
4 '2

· set 0 f th
Hence, the so Iutlon ·
e given . .IS {4 , - "2'
equatIOn 7 l± 4 M}
Example 3: Solve (~ - 5x + 7l- (x - 2) (x - 3) = 1
Solution: Here, the given equation can be written as
(x 2 - 5x + 7i- (xl - 5x + 6) = 1
Put xl - 5x = y, we get
(y + 7)2 - (y + 6) = 1
=> ;+ 14y+49-y-6-1 = 0
=> ;+ 13y+42 = 0
=> ;+6y+7y+42 = 0
=> y(y+6)+7(y+6) = 0
=> (y + 6) (y + 7) = 0
Quadratic Equations 21

=:> y = -6,-7
Now, Y = 6
=:> xl-5x+6 = 0
=:> (x-2) (x-3) = 0
=:> x = 2,3
also y = - 7 =:> x 2 - 5x + 7 = 0

x=---
5±iJ3
2
. set of the given
Hence, the solution . equation
. .IS {2,3, -5±iJ3}
2-

'JYpeIV
Equation of the type ~ax + b + ~cx + d = ~ex + f where a, b, c, d, e,fare constant.

STEP KNOWLEDGE
In order to solve such Jype of equation we use the following steps.
Step 1. Square both the sides of the given equation.
Step 2. Put the rational terms on one side and irrational terms on other side.
Step 3. Again squaring and obtain the quadratic equation.
Step 4. Solve the obtained quadratic equation.

~~~~~~~ISOLVED EXAMPLES ~I~~~~~~


Example 1: Solve ~x + 5 + ~x + 21 = ~6x + 40

Solution: Here the given equation is ~ x + 5 + ~ x + 21 = .J6x + 40


On squaring both the sides, we get
(x+5)+(x+21)+2 "Jx+5. ~x + 21 =6x+40
=:> 2~(x + 5) (x + 21) = 4x+ 14
=:> ~(x + 5)(x + 21) = 2x+7
Again squaring, we get
(x+ 5)(x+21) = (2x+7i
=:> xl+26x+ 105 = 4x2+28x+49
=:> 3x2+2x-56 = 0
(3x+ 14)(x-4) = 0
(3x + 14) or (x-4) = 0
14
x = --orx=4
3
Now, we check, whether the obtained values x = 4 and x = - 1~
.)
satisfy the given
equation or not.
22 Remedial Mathematics

When x =4,wehave, ~ +~ = J24+40


=> 3 + 5 = 8, which is true
Hence, x = 4 is solution of the given equation.
14
when x = - - then
3'

)_1; +5 +)_1; +21 6(_1;)'+40

=> Jf+f! J¥ => 1 + 7 *- 6, which is not true

Therefore, x = - 1; is not a solution. It is an extraneous root and so reject it. Hence, the
solution is 4.
Remark
A root which is obtained by solving an equation but does not satisfy it, is called an
extraneous root. Such roots enter the equation in the process of squaring because
this process is irreversible.
Example 2. Solve = J(x + 5) + J(x + 12) = J2x + 41
Solution: Here, the given equation is
J(x+5)+Jx+12 = J2x+41
Squaring both the sides, we get
(x+5)+(x+ 12)+ 2J(x + 5)(x + 12) =2x+41

=> 2J(x + 5)(x + 12) = 24

=> Jex + 5)(x + 12) = 12


Again squaring, we get
(x+5)(r+12) = 144
=> .?+ 17x + 60 = 144
=> x 2 + 17x-84 = 0
=> x = 4,-21
Therefore x = 4 is a root, because it satisfy the given equation. Also x = - 21 is an
extraneous root, because it does not satisfy the given equation.

Example 3: Solve ~5x2 - 6x + 8 - ~5x2 - 6x - 7 =~ 1


Solution: Here, the given equation reduce to ~5x2 - 6x + 8 - ~5x2 - 6x - 7 = 1
Let 5.? - 6x = y, then given equation reduce to
JY+8-JY-7 =\
Quadratic Equations 23

Squaring both the sides, we get


(y + S) + (y - 7) - 2,Jr-(y-+-S"--')(y---7-) = 1

=> y = ~y2 + Y _ 56
Again squaring, we get
y = y+y-56
Y = 56
5x2 -6x = 56
5~-6x-56 = 0
(5x+ 14)(x-4) = 0
14
x=4--
, 5

Since, both the obtained values (x = 4, and - 1;) satisfies the given equation.

Hence, the solution set of the given equation is {4, _ I;}

Example 4. Solve ~x + 4 + ~x + 20 = 2F+li


Solution: Here, the given equation is ~x + 4 + ~x + 20 = 2 F+1i
Squaring both the sides, we get
(x + 4) + (x + 20) + 2 ~(x + 4)(x + 20) =4 (x+ 11)

=> 2 ~(x + 4) (x + 20) = 2x+20

=> ~(x + 4)(x + 20) = x+ 10


Again squaring, we get
(x+4)(x+20) = (x+ 1O)2=~+20x+ 100
=> ~+24x+SO = ~+20x+ 100
4x = 20
=> x =5
Clearly, x = 5, satisfy the given equation. Hence, x = 5 is the required root of the given
equation.

~~~~~~~I EXERCISE 1.3 ~I~~~~~~~


Solve the following equations:
1. (i)x(x+ l)(x+3)(x+4) = ISO (ii) (2x+ 3) (2x+ 5)(x-l)(x-2) = 30
(iii) (x-5)(x-7)(x+4)(x + 6) = 504 (iv) x(2x+ l)(x-2)(2x-3)=63
(v) (~-3x-1O)(~-5x-6)= 144 (vi) (x+ 2)(3x + 4)(3x + 7)(x+ 3) = 2600

2. (i) ~3x - 1 - F=l = 2 (ii) ~2x + S + F+5 = 7


(iii) ~x+4 +~x+20 = 2F+1i
(iv) F+1- F=l = ~4x - 1 (v) ~5x + 7 -- ~3x + 1 = ~x + 3
24 Remedial Mathematics

HINTS TO THE SELECTED PROBLEMS


1. (2x+3)(2x+5)(x-l)(x-2)=30
~ [(2x + 3)(x-l)[(2x+ 5)(x-2)] = 30
~ (:zx2+x-3)(2~+x-lO) = 30
~Let 2x2+x = y.
~ (y-3)(y-lO)=30
~ Y-13y+30 =30
~ y-13y = 0
~ y(y-13) = 0
~ y = 0 ot Y - 13 ~ 0
~ :zx2+x = 0 or 2xl+x-13=0

-1±)(1)2 -4x2x(-13)
~ x (2x + 1) = 0 or x = ----'--2-x-2----

1 - 1 ± .JI + 104 -I ± ft05


~ x =0 or = - - = -----'--
2' 4 4
2.(ii) .J2x+8+.Jx+5 =7.
squaring both sides.
(2x + 8) + (x + 5) + 2 .J(2x + 8) F+5 =49
3x + 13 + 2 .J(2x + 8)(x + 5) =49

3x-36 = - .J(2x + 8) (x + 5)
squaring again, we get
(3x-36i =4 (2x+ 8) (x+ 5)
~ 9 (x-12)2 =4 [:zx2 + 10x+ 8x+ 40]
~ ·9 (xl + 144-24x) =4[:zx2+18x+40]
~ 9xl + 144 x 9 - 24 x 9x = 8xl + 160 + 72x.
~ xl-144x+ 1136 = 0
on solving we get the required result.
2. (v) .J5x + 7 - .J3x + I = .Jx + 3
squaring both sides
(5x + 7) + (3x + 1) - 2.J5x + 7 .J3x + 1 = x + 3

~ 8x + 8 - 2 .J5x + 7 .J3x + 1 = x + 3
~ 7x + 5 = 2 . .J5x + 7 .J3x + 1
~ squaring both sides.
(7x+5i = 4 (5x+7)(3x+l)
Quadratic Equatiolls 25

=> 49X3 +25 +70x = 4 (15~ + 5x+2Ix+7)


=> 49.x2 + 25 + 70x = 60.x2 + lOOx + 28
=> Ilx 2 +30x+3 = 0
- 30 ± ~900 - 4 x II x 3
x=
2 x II

- 30 ± ~900 - 132
22
- 30 ± J768 - 30 ± 16 F3
22 22
-15±8F3
11

ANSWERS

I -1±OJiQ5
1. (/) - 6, 2, - 2 i .ftl (ii) 0, - -
2' 4

(iii) -7,-2,3,8 (iv)


_i 3 3±iJ47
2" 4

19 13±~-599
(v) - 3,2,2, 7 (vi) -- 2
3" 6
2. (i) 1,5 (ii) 4 (iii) 5
5 1
(iv) - (v) - -
4 11
(b) Removal of Common Factor Throughout in an Irrational Equation.

STEP KNOWLEDGE
In this method, we use the following steps.
Step 1. Factorize each given expression.
Step 2. Put common factor equal to zero and find one value ofx.
Step 3. Solve the remaining equation by the method discussed in (a).

~~~~~~~I SOLVED EXAMPLES ~I~~~~~~


Example 1: Solve ~x 2 - 16 - ~x 2 - 5x + 4 = x - 4

Solution: Here, the given equation is ~x 2 - 16 - ~x 2 - 5x + 4 = x - 4

=> ~(x - 4)(x + 4) - ~(x -I) (x - 4) = (x-4)


26 Remedial Mathematics

:::::> .J(x-4) [.Jx+4 -..r;=I-.Jx-4] =0

Now,either .Jx-4 =0:::::>x-4=0:::::>x=4

or[.Jx +4 -~ - .Jx- 4] = 0

:::::> .Jx+4-..r;=I = ~x-4


On squaring, we get.

4(~+3x-4) = (x+7)2=~+ 14x+49


4~+ 12x-16 = ~+ 14x+49
3~-12x-65 = 0

2±.J4+780 2±28 13
x= 6 =-6-=5'-3

Here, it is clear that x = 5, satisfy the given equation. Although x = - 1: does not

-13
satisfy the given equation, therefore x = -3- is an extraneous root.

Hence, the solutions set of the given equation is {4, 5}.

Example 2: Solve ~x2 + 2x - 3 + ~ = .J5 (x -1)

Solution: Here, the given equation is ~x 2 + 2x - 3 + ~(x2 - x) = ~5 (x -1)


:::::> ~(x -1)(x + 3) + Jx (x -1) = J5 (x -1)

:::::> JX-1[.Jx+3+Fx-J5] = 0

Now, either ~ = 0 :::::>x-l =0 :::::>x= I

or .Jx + 3 + Fx - J5 = 0:::::> J x + 3 +.Jx = J5


Again, squaring, we get
(x+3)+x+2.Jx F+3 = 5

:::::> 2.Jx(x+3) =2-2x


Again squaring 4x (x + 3) = (2 - 2x)2
:::::> 4~+ 12x = 4-8x+4~:::::>20x=4
4 1
Therefore, x = 20 ="5
Hence, the solution set of the given equation is {I. ~}

2
(c) Equation ofthe form ax + bx + c + p ~ax2 + bx + c = q
Qlladratic Equatiolls 27

STEP KNOWLEDGE
In order to solve such type of equation, we use the following steps.

Step 1. Assume ~ ax 2 + bx + c = y and obtain the quadratic equation in y.


Step 2. Solve the obtained equation for y.

Step 3. Finally obtain the ~alue of x by putting ~ax 2 + bx + C =Y

~~~~~~~I SOLVED EXAMPLES I~~~~~~~


Example 1: Solve x 2 - 4x -12 ~x2 - 4x + 19 =-51
Solution: Here, the given equation can be written as

(x 2 - 4x + 19) -12 ~x2 - 4x + 19 + (51-19) =0

~ (x 2 -4x+19)-12~x2 -4x+19 +32 =0

Put ~x 2 - 4x + 19 = y is (\), we get


;-12y+ 32 = 0, which is a quadratic equation iny.
Solving for y, we get,
Now, y = 4~ ~x 2 -
4x + 19 = 4
~ 2
x - 4x + 19 = 16 ~ ~ - 4x + 3 = 0
~ (x-l)(x-3) = O~x= 1,3
Also y = 8 ~ ~r-x-2---4-x-+-}-9 = 8
~ ~-4x+ 19 = 64
~ ~-4x-45 = 0
~ ~+5x-9x+45 = 0
~ x (x + 5) - 9 (x + 5) = 0
~ (x+ 5)(x-9) = 0
~ x = 9,-5
Hence, the solution set of the given equation is {I, 3, - 5, 9}

(d) Method ofIdentity : Equation of the form ~ ax 2 + bx + c + ~dx + ex + f = k


In order to solve such type of equation we proceed as the following example:

~~~~~~~I SOLVED EXAMPLES ~I~~~~~~


Example 1: Solve ~5x2 - 6x + 8 - ~5x2 - 6x -7 = 1
Solution: Here, the given equation is

~5x2 - 6x +·8 - ~5x2 - 6x - 7 = 1 ...(1)


28 Remedial Mathematics

Let ~5x2 - 6x + 8 = A and J5x 2 - 6x - 7 = B, then given equation reduces to


A -B = 1 ...(1)
Also A2_B2 = (5x 2-6x+8)-(5x2 -6x-7)= 15
=> (A-B)(A+B) = 15(\)
=> l.(A+B) = 15 ... (2) [Using(l)]
Solving (1) and (2), we get
2A = 16
=> A =8
=> J5x 2 -6x + 8 =8
=> 5~-6x+ 8 = 64
=> 5~-5x-56 = 0
6 ± ~36 + 1120 6±34
=> x= =--
10 10
14
=> x=4--
, 5

Hence, the solution set of the given equation is {4, _ I;}


Example 2: Solve ~2x2 - 3x - 5 - ~x2 - 3x + 4 =x + 3
Solution: Here, the given equation is J2x 2 - 3x - 5 - ~x 2 - 3x + 4 = (x + 3)

Let J2x 2 - 3x - 5 = A and Jx 2 - 3x + 4 = B, then given equation reduces to


A - B = x+3 ...(2)
Also, A2_B2 = (2x2-3x-5)-(x2-3x+4)=~-9
=> (A - B)(A + B) = x2 - 9
(x+3)(A+B) = (x-3)(x+ 3)[using (1)] ... (2)
A+B =x+3
2A=2x
A =x

Therefore, ~2x2 ---3x -


5 =x
On squaring, ~-3x-5,= x 2
=> ~-3x-5 = 0

3 ± ~9 + 20 3 ± 59
x=
2
=---2

· set 0 f th
Hence, t he so IutlOn '
e gIven . .IS {3 ± 2
equatIOn .J29}
Quadratic Eqnations 29

Solve tile following equations:

1. ~x2-5x+6+~x2-9 = ~2x2 -llx+15

2. ~x2 +2x-3 +~ = ~5(x-1)


3. ~2x2 - 5x - 2 - ~2x2 - 5x - 9 = 1

4. ~4x2 -7x-15 _~x2 -3x = ~x2 -9


3x - 2 I 2 (x + 1)2
5. - - + V2x - 5x + 3 = -'------'--
2 3

6. ~3x2 - 4x + 34 + ~3x2 - 4x - 11 = 9
7. ~x2 + ax + b - ~x + 9x + 6 = Jb +..Jc
8. x (x + 3) + 3 ~2x2 + 6x + 5 = 25

HINTS TO THE SELECTED PROBLEMS

1. ~x2 - 5x + 6 + ~x2 - 9 = ~2x2 -llx + 15


=> ~(x - 2)(x - 3) + ~(x - 3)(x + 3) = ~2x2 - 6x - 5x + 15
=> ~x - 3 [ ~x - 2 + ~x + 3] = ~2x (x - 3) - 5 (x - 3)

~x-3[~x,-2+~x+3J = ~(x-3)(2x-5)
=>~x-3[~x-2+~x+3-~2x-5J =0
either ~ x - 3 = 0 => x = 3.
or ~ x - 2 + ~ x + 3 - ~2x - 5
squaring both sides.
x-2+x+3+2~x-2 ~x+3 =2x-5
1+5 = -2~x-2 ~x+3

6= -2F=2F+3
=> 3 = - ~(x -2)(x + 3)
squaring again, we get
9 = (x-2)(x+ 3)
9 = ~-x-6
30 Remedial Mathematics

x2-x- 15 = 0

1 ± ~(1)2 + 4 x 1 x 15
x=
2xl

I±KI
x=
2
4. ~4x2 -7x-15 _~x2 -3x = ~
:::) ~4x2 -12x + 5x -15 - ~x (x - 3) = ~(x - 3)(x + 3)
:::) ~4x (x - 3) + 5(x - 3) - ~x (x - 3) = ~(x - 3)(x + 3)
:::) ~(x-3)(4x+5) -~x(x-3) = ~(x-3)(x+3)

=> ~x-3)[~4x+5)-J;-~x+3J =0
either ~x-3 = 0,x=3

or ~4x+5-J;-~x+3. =0
:::) ~4x+5 -J; = ~x+3
squaring both sides.
4x+5+x-2J; ~4x+5 = 2+3.

4x+5-3 = 2J; ~4x + 5


:::) 4x+2 = 2J; ~4x + 5
:::) 2x+l=J;~4x+5
:::)
2
4x + 1 +4x = x(4x-15)
:::) 4x2 + 1 + 4x = 4x2 + 5x
:::) 5x-4x = 1
:::) x =1

ANSWERS

2. 1, 1.
1 3 I±KI 9
. , 2 5 3. -2, -4

5
4. 3 5. 2, 1.
2 6. 3, - -
,.,
-'
7. O,a 8.2,-5
Type V: Reciprocal Equations
1
An equation which remains unchanged when x is changed to - is called reciprocal equation.
- x
Quadratic Equations 31

Remarks
1. The roots of a reciprocal equations occurs in pairs.

2. If a is a root of reciprocal equation, then 1..a also a root of the given equation.
To solve such type of reciprocal equation, we use the following steps :

STEP KNOWLEDGE
(A) For Even Degree (say, degree = 4)
(I) Divide both side by x?-
l I
(ii) Put x + - or x - - = y and solve for y.
x x
. 1 1
(iii) Finally obtained the value ofx by putting x + - = y or x - - = y.
x x
(B) For Odd Degree
(i) If the coefficient of the terms equidistant from the starting and end are equal in
magnitude as well as in sign. Then by inspection, we have that - 1 is a root and
then taking (x + I) as a common factor and get even degree equation.
(ii) If the coefficients of the terms equidistant from the starting and end aye equal in
magnitu~e but opposite in sign. Then, by inspection take x = 1 is awot'and take
(x - I) as a common factor and get even degree equation, which can be easily
solved.

~~~~~~~ISOLVED EXAMPLES I~~~~~~~


Example 1: Solve x4 - x 3 + 2J? - x + 1 =0
Solution: Here, the given equation is x4 -X3 + 2x? -x + 1 = 0
Divide throughout by x?-, we get
x2 _ x + 2 _1.. + _1_ = 0
x x2

...(1)

2
Put (x+';) =yi.e. x + x~ =1-2in(l),weget

1-2-y+2 = 0
=> l-y=o
=> y(y-I) = 0
y = 0,1
1
Now, if Y = o=>x+- =0
x
32 Remedial Mathematics

1
or y=l=>x+-=1
x
x2 +1 =x
~-x+1 =0

x =
l±i.J3
2
· set 0 f th
Hence, the so IutlOn '
e gIven . .IS {±l,. 1± 2i .J3}
equatIon

Example 1: Solve (x +'; r- %(x -.;) = 4, x ~O [UPTU B. Pharma 2004)

Solution: We have ( x + .; y-%( x - .;) = 4

=> [(x-.;y +4]-%(X-';) =4


2 3
Y +4-- Y =4
2

i-%Y = O=>Y(Y-%)=O
3
Y = 0 or Y - - =0
2
3
Y = 0 or y=-
2
Now y=O
x-! = 0
x
x2 -1 =0
x2 = 1
X =± 1.

3 1 3
Again y=-=>x--=-
2 x 2
2x2_2 = 3x
~ = 3x-2
=> 2x(x-2) + 1 (x-2) = 0
=> (x-2)(2x+ 1) = 0
=> x-2=00r2x+1=0
1
=> x = 2 or x = - -
2
Hence, The solution set of the given equation is {I, -I, 2, -~}.
Quadratic Equations 33

Example 2: Solve (x + .; f- 2 ( x - .; + 4 ) - /1 = 0

Solution: Here, the given equation is

(X+;-f -2(X-;-+4)-11 = 0 ... (1 )

Put x - ~ = Y i e. ( x + ;- f = (x - ;- y + 4=l +4

Then, equation(l) becomes ul+4)-2 (y+4)-11 = 0


or l-2y-15 = 0
=> (y-5)(y+3) =0
y = -3,5
1
Now, if y=5=>x--=5
x
=> .x2-5x-l = 0

x=
5±m
2
1
orif x = -3, then x - - =-3
x
.x2 + 3x-l = 0

x =
- 3± J9 + 4 = --"--
- 3 ± v'i3
2 2

· set 0 f th
Hence, the so1utlOn '
e given . .IS
equatIOn {5±m -3±v'i3}
2' 2 .

Solve the fol/owing equations:


1. 2x4 -x3 -11.x2-x+2=0 2. ~ -3.x2-3x+2 =0

3. 4x4 -4~ -7.x2-4x+4 = 0

5. 2x4_~+ 14.x2-9x+2 =0
4. (x + ;- r- x-;-
6(

6. x4+1-3(~+x)=2.x2
+ 1) - 5 = 0

7. x 4-4x3 -3.x2-4x+ 1 =0 8. x 5 - 4x4 + ~ + .x2 - 4x + 1 = 0

9. (x+;-f -%(x-;-)-4 =0
11. x 6 - x 5 + x4 -.x2 - x-I = 0
34 Remedial Mathematics

HINTS TO THE SELECTED PROBLEMS


1. 2x4-x3_11~-x+2=0
divide by~, we get

2x2 - x-II _l + ~2 =0
x x

~
2 (x2 + xI2) - ( x + ) - 11 = 0

2
=> 2 ( x + :2 + 2) - 4 - ( x + ~ ) - 11 = 0
=> 2(x2 +~J -(X+~)-I5 =0
Let (x+~) =y
21-y-I5 =0
21-6y+5y-15 =0
2y(y-3)+5(y-3) =0
(y-3)(2y+ 5) = 0

Y- 3 = 0 or 2y + 5 = 0
1 5
x + - = 3 or y = - -
x 2
~ + 1 = 3x or x + 1 = _1
x 2
2
x +1 5
x 2 - 3x + 1 = 0 or - - = --
x 2
+ 3 ± ~9 - 4 x 1 x l ?
X = 2x- + 2 = - 5x
2xI
tj ± 15
= -- or z.x2+5x+2 = 0
2
2~+4x+x+2 = 0
2x(x+ 12)+ 1(x+2) = 0
1
x=-2x=--
, 2
Quadratic Equations 35

~(X2+ X~-2}-6(X-~)-7=0
~ (x-~f -6(X-~)-7 =0
1
Let x-- =y
x
~ y-6y-7 = 0
6. x4+ 1-3 (x 3 +x)=al
x4+ l-al-3x3 -3x = 0
Divide by x?, we get.

x 2 + _I _ 2 - 3 x _1.. = 0
x2 X

~ (x2 + ~2 - 2) - 3 ( x + ~) = 0

~(x2+ ~2+2)-3(X+~)-4=0
~(x+~f -3(X+~)-4 = 0 ...(1)

~Let x+-1 =y
x
Then eqn (I) reduced to
y-3y-4 = 0
8. x 4x4 + x
5- 3+ x2- 4x + 1 = 0
putting x = 1 ~ (-1)5-4 (_1)4 + (- 1)3 + (_1)1 -4 x (-I) + 1
=-1-4-1+1+4+1=0
so (x + 1) is a factor of given equation.
(x+ l)(x4-5x3 +6x?-5x+ 1)= 0
(x+l) ( x2.-5x+6--+-2 =0 5 1)
x x

(X+l)[(X + ~2 )-5(X+~)+6] =0
2

~ (x+l)[(x+~r -2-5(X+~)+6l =0
~ (x+l)[(x+~f -5(X+~)+4l =0
~ Consider { x+~f - (x +~)
5 +4 = 0
36 Remedial Mathematics

1
Take x+- = y
x
y-5y+4 = 0
On solving we get required Answers.

ANSWERS

1. _1- -2 3±J5 2. -1 2 -
1
3.
1- 2 -3±R
2' , 2 ' '2 2" 4

4.
7±J53 1 ±J5
2 2
1
5. 1,2,-
2
6. 2 ± .)3, t (l ± -)'- 3) .

5 ±51 -±F3
, 1±.,F3 ,2 ±.)3 1
7. 8. -1, 9. -1--12
6 2 2 ' 2"

1_2 3 ±J5 1±.,F3


10. , , 2 11. +.
_I, 2

III NATURE OF ROOTS OF A QUADRATIC EQUATION


The roots of the quadratic equation ax 2 + bx + c = 0 are

-b ±~b2 -4ac
... (5)
2a
Here, the expression D = b2 - 4ac is called discriminant.
The nature of the roots, depend upon the value of D as given below:
(a) If b2 - 4ac ~ 0, then roots are real
(I) If b2 - 4ac > 0, then roots are real and distinct
(ii) If b2 - 4ac = 0, then the roots of the equation are real and equal.
-b+O -b
In this case, each root = ---- = -
2a 2a
(iii) Also if b2 - 4ac is a perfect square, then the roots are rational and in case it can't
be a perfect square, then the roots are irrational.

(b) Ifb 2 -4ac<Othen ~b2 -4ac is imaginary.


Therefore, the roots are imaginary and unequal.

III SYMMETRIC FUNCTION OF THE ROOTS


If a, p are the roots of the quadratic equation ax2 + bx + c = 0, then
b
a+p = - - (sum of the roots) ... (1)
a
c
and a p= - (product of the roots) ...(2)
a
Quadratic Equations 37

Therefore,

b ~b2 -4ac
a 2 _p2 = (a+p»(a-p)=-
a a
a 3 +p3 = (a+p)3-3aP(a+p)
b3 3bc -b(b 2 - 3ac)
= - 3 +- = -~--::--..:...
a a2 a 3
4
a +p4 = (a +p2)2_2a2p2
2
2 2
= (b - 2ac) _ 2c
a2 a2
The Sign of Expression (x- a) (x-b), (a < b)
Here, we have the following cases:
Case I (x-a)(x-b) =+ve
It is possible if either both factors are positive or both negative
i.e. if x-a> O,x-b>Oi.e.x>a,x>b
Therefore, x > b (:. a> b) ...(1)
Therefore, x <a ...(2)
From (l) and (2), we conclude that (x - a) (x - b) is positive if either x < a or x> b. In other
words, we mean that x does not lie between a and b (a < b).
CaseU (x-a)(x-b) = - ve
It is possible if one factor is positive and the other is negative.
Let x-a = +ve>O,xb=-ve<O
Therefore, x > a, x < b or a < x < b i. e. x lies between a and b (a < b)
or x-a = -ve<O,x-b=+ve>O
i.e. x < a and x> which is not possible.
Therefore, (x - a) (x - b) = Positive if x does not lie between a and b and is negative if
x lies between a and b.
For Example:
Consider the expression.
(x+3)(x-5) = [x-(-3)] (x-5)
a =-3,b=5=:::>a<b
It is positive if x does not lie between - 3 and 5 and is negative if x lies between - 3 and 5.
The Sign of Expression ax2 + bx + c
Here, we consider the following cases
38 Remedial Mathematics

Case I
Let the roots ofthe equation. ~ + bx + c = 0 (1) be imaginary.
Then we can write

a[(x+~)2 + 4ac-b
2
= ]
2a 2 4a
Now, since the roots of equation (1) are imaginary, therefore b2 -4ac < 0 i.e. 4ac-b2> O.
2
. ( b)2 4ac _ b .
Hence, the expressIOn x + - + 2 is positlve for all real values of x.
2a 4a
Therefore, ax2 + bx - c has same sign for all real value ofx.
Case II
Let the roots of the equations (1) are real and distinct, denoted by a and ~. Let a > ~ Then
we have the identity.
...(3)
If~ <x < a, thenx-a > 0 andx- ~ < 0 so that (x-a) (x-~) < O. It follows that the sign
ofax 2 - bx + c is opposite to that of a
Ifx> a or x <~, then (x-a) (x-~) > 0 since the factor (x-a) and (x-~) are either both
positive or both negative.
Hence, in this case the sign ofax2 + bx + c is the same as that of a.
Case ill
Let the roots a, b be equal. Then
ax2 + bx + c = (x - a)2
and (x - a)2 is positive for all real values of x and therefore ax 2 + bx + c has the same
sign as a.
Remark
• From, above three cases, we conclude that, for all real values of x, the expression
ax2 + bx + c has the same sign as a except when the roots ofthe equation ax2 + bx + c
= 0 are real and unequal, and x has a value lying between them.

III RELATION BETWEEN ROOTS AND COEFFICIENTS


Let us consider the quadratic equation
*
ax2 + bx + c = 0, a 0, a, b, c E R
To find the sum and product ofthe roots in terms ofthe coefficients a, b, and c.
Consider the quadratic equation
ax 2 +bx+c = O,(a*O) ... (1)
If a, ~ be the roots, then by the theory of equation, we have

-b+~b2-4ac -b-~b2-4ac
a = and ~ = --...!.----
2a 2a
Quadratic Equations 39

(i) The sum of the roots

=a+p=
2
J
- b + b - 4ac
+
J
- b + b 2 - 4ac b
=--
2a 2a a
(ii) The product of the roots

~ a~{b+J:: -4ac)( -b- J:: -4ac 1


(_b)2 -(Jb 2 -4ac)2 2 2
b - b + 4ac c
= -
2 2 a
4a 4a
b coefficient of x
Hence, we have sum of the roots = -- =- 2
a coefficentofx
c Constant term
and product of the roots = - = 2
a Coefficent of x

III FORMATION OF EQUATIONS


To find the equation whose roots are a, 13: Let the equation be
ax2 +bx+c = O,a:;eO ... (1)
Then by theory of equation,
b
Sum of roots a+ (3 =
a
c
Product of the roots a(3 = -
a
Now equation (1) can be written as
2 b c
x +-x+- = 0
a a
=> .x2-(a+(3)x+a(3 = 0 or x(x-a)-(3(x-a)=O
=> (x-a)(x-(3) =0
Remark
• Let S = a + (3, P = a(3, then the required equation is x 2 - Sx + P = O.

III COMMON ROOTS


(l) Condition for One Common Root
Consider the two quadratic equations, such that
ax 2 +bx+c = 0 ... (1)
and a'x2 + b'x + c' = 0 ... (2)
Let a be the common root then equations (1) and (2) gives
aa2 + ba+c = 0 ...(3)
a'a 2 + b'a + c' = 0 ... (4)
40 Remedial Mathematics

Solving (3) and (4), we get

a2 a
cd - e'a ab' - db ... (5)
be' - b'c =

Now taking first two members, we get \

a be' -b'e
, , :::>a= ...(6)
be' - b'e ea -ea cd -e'a
Taking last two members, we get
cd - e'a
ex = ab' - db
Now (5) and (6) gives.
be' - b'e ea' -e'a
cd - e'a ab' - db
:::> (ab' - d b) (be' c) = (cd - e'ai
which is the required condition for one common root.
(il) Condition for Both Roots Common
Here, the given equations are
ax2 +bx+e = 0 ...(1)
and • dx 2
+ b'x + e' = 0 ...(2)
Let ex, ~ be the common roots, then from (1), we have
b
ex+~ = - - ...(3)
a
e
and ex~ = - ...(4)
a
b'
From (2), we have ex+~ = -- ...(5)
a'

and ...(6)
Now (3) and (5) gives
b b' b b' a b
=--:::>-=-:::>-=- ...(7)
a d a a' - a' b'
Equation (4) and (5) gives
e e' a e
=-:::>-=- ...(8)
add e'
Combining (7) and (8), we have
a b e
d b' e'
which is the required condition for both common roots
Quadratic Equations 41

~~~~~~ISOLVED EXAMPLES I~~~~~~~


Example 1: !fa, 13 are the roots ofaX! + bx + c = O,findthefollowing:
1 1 ~ a
(a) ao. + b + a~ + b (b) ao. + b + a~ + b
(e) (ao. + br3 + (a~ + br3 (d) (ao. + br2 + (a~ + br2
Solution: Since a, ~ are the roots of d + bx + e = 0
e
Then, ao.2 + bo. + e = 0 => ao. + b + -
a

b e
also, o.+~ = --;o.~ = -
a a
1 1 a ~ 1
(a) - - + - - = ---- = --(o.+~)
ao. + b a~ + b e e e

= --1 (-
- b) b =-
e a ae

(b)
_~_+_o._ = _ o.~ _ o.~ = _~ . .:. =_~
ao. + b a~ +bee e a a
3 3
a + 13
(e) (ao.+~r3+(a~+br3 = - 3
e
3
1 3 b - 3abe
= - - [(a + 13) - 3a13 (a + 13)] = 3 3
c3 a e
0.2 + ~2
(d) (ao.+br2+(o.~+br2 = 2
e
1 2 b 2 - 2ae
= "2 [(a +~) - 2o.~] = 2 2
e a e
Example 2: !fa and 13 are the roots ofaX! + bx + e = O. Find the equation whose roofs are
as given below.
I I I 2 2 I I
(/) a + ~ , a +"i3 (ii) a +~ '-2 + 2"
a ~
Solution: Let a,~ be the roots ofthe given quadratic equation
ax2 +bx+e = 0
-b e
Then, we have a+~ =-ando.~ = -
a a
1 o.+~ a b (ae+b 2 )
Sum = - - + - - = - - - - = - -'-------'-
a +~ a~ b e be
42 Remedial Mathematics

1 o,+p 1 a
Product = a + P . ~ = a P = -;;-
Now, consider the equation
2
x 2 -xS + P = 0 => x 2 - x (-a c + b ) + -a = 0
bc c
=> bc . x 2 + (b 2 + ac)x+ ab = 0
2 2 1 1
(ii) S = a + P + -~ + 2"
o,~ P

_ (2 A2) (0,2 + p2) _ (b 2 - 2ac)2


p-o,+~. 22 - 22
ex, pac
2
Therefore, x - Sx + P = 0 gives
~c2~ - (b 2 - 2ac) (~+ c2 ) x + (b 2 - 2aci = 0
Example 3: If a, fJ are the roots o/x2 - p (x + J) - c = 0, show that
(a + J) (fJ + J) = J - c.
a 2 + 2a + 1 fJ2 + 2fJ + 1
Hence show that 2 2 + fJ2 2fJ = J
a + a+c + +c
Solution: Here, the given equation is
x 2 - P (x + 1) - c = 0
Therefore, 0,+ P = p, and o,p = - (p + c) ... (1)
Now (O,+l)(p+l) = O,p+(o'+p)+1
= -rt-c+p+ 1 = l-c ... (2)
0,2 + 20, + 1 p2 + 2P + 1
Also, ---::----+-'-::---'---
0,2 + 20, + C p2 + 2P + c
(0,+1)2 (P+l)2
---'--::---':--- + [using (1)]
(o'+l)2-0-c) (p+l)2-0-c)

a +1 P+1 (a + 1) - (P + 1)
= --+--= =1
O,-p p-O, O,-p
Example 4: If a be a root a/the equation 4x2 + 2x - J = 0, prove that
4 cJ - 3 a is the other root. (UPTU B. Pharma 20021
Solution: Given a be a root ofthe equation 4x2 + 2x - 1 = o.
Then 40,2+20,-1 = 0 ... (1)
Let p be the other root of the given equation
1 1
Then 0,+ P = - => p=---O,
2 2_
Quadratic: Equations 43

We have to show
3 2
Now4a -3a=a(4a +2a-I)- -(4a +2a-I)-a--
1 2 1
2 2
1 1
=a·O--O-a·-
2 2
1
= -a--=~.
2
Hence 4a3 - 3a is the other root of the gives equation.
Example 5: Two students solve an equation. In solving, one commits a mistake in constant
term andfind the roots 8 and 2. Other commits a mistake in the coefficient ofx andfind the
roots - 9 and - 1. Find the correct roots.
Solution: Let the correct equation be
x 2 +ax+b = 0 ...(1)
Roots found by first student are 8, 2 i. e. S = 10, P = 16
:. Equation is ~-IOx+ 16 = 0 ...(2)
Since, the committed mistake only in constant term :. a = - 10
Roots found by second student are - 9, -1, i. e. S = - 10, P = 9
Therefore equation is ~ + lOx + 9 = 0
Since the committed mistake is in the coefficient ofx. :. b = 9
Having found a = - 10, b = 9, the required equation is
x 2 -IOx+9 = 0 or (x-9)(x-l)=0
i.e. x 1,9
=

Hence, the correct roots are 1 and 9.


Example 6: If a, fl be the roots ofax2 + bx + c = 0 and y, 8those oflx2 + mx + n = 0, then
find the equation whose roots are ar + b8 and ao + flr
Solution: Here the given equations are
ax 2 +bx+c = 0 ...(1)
and ...(2)
bam n
Now, we have a+~ = -~,a~=~'Y+O=-T'YO=1

b2 - 2ac
Now, a2+~2 = (a+~)2-2a~= -""'7"-
2
a
2
.2 2 m - 2nl
and r +0 = P
Now, S = (ay+~o)(ao+By)=a(y+o)+~(y+o)
bm
=(a+ ~)(y+o)=-
al
44 Remedial Mathematics

and p = (ay+~o)(ao+~y)
= a 2yo + a~o2 + a~r + ~2yO
= (a2+~2yo+a~(r+02)

~ 2nl )
2
= T(b :2
2aC
) + ; (m2

nl (b 2 - 2ac) + ac (m 2 - 2nl)
2
a z2
b 2 nl + m 2ac - 4acnl
a2 12
Therefore the required equation is given by
x?-xS+ P = 0
2 2
2 (bm) b nl + m ac - 4acnl
=> x -x - + =0
al a 212
=> ~t2x? - xalbm + b2nl + m2ac - 4acnl = 0
Example 7: Jfp and q be the roots of2x2 - 6x + 3 = 0, find the value of
(p3 + q3) _ 3pq (p2 + q2) _ 3pq (p + q)
Solution: Since p and q are roots of2x? - 6x + 3 = 0
-6
Therefore, sum of the roots p + q = -2 =3
3
and product of the roots pq = 2"
Now, (p3 + q\_ 3pq (p2 + q2)- 3pq (p + q)
= (p + q)3_ 3pq (p + q)- 3pq [(P + q)2 - 2pq]- 3pq (p + q)

= 27-3.%(3)-3.%[9-2.%]-3.%.3

27 27
= 27 ---27 - - =-27
2 2
Example 8: Solve the equation x 2 + px + 45 = 0, given the squared differences ofits roots
is equal to 144.
Solution: Here, the equation x? + px + 45 = 0 ...(i)
Let a, ~ be its roots such that a > ~
.. a + ~ = - p, ex.~ = 45
From the given condition (ex. - ~)2 = 144
.. (ex. + ~)2-4ex.~ = 144
~ p2-4.45 = 144
=> p2 = 324
=> p = ± 18
Quadratic Equations 45

When p = 18, equation (1) becomes x 2 + 18x + 45 = 0


=> (x+3)(x+ 15) = 0
=> x =-3,-15
When p = - 18, equation (1) becomes
x 2-18x+45 = 0
=> (x-3)(x-15) = 0
=> x = 3,15
Hence, the roots of the given equation is 3 and 15 or - 3 and - 15.
1 1 1
Example 9: If the sum of the roots ofthe equation x + a + x + b = ~ is zero. Show that

the product of roots is -"21(a2+2


b ).
Solution: Here, the given equation is
1 1 1
--+-- =-
x+a x+b c
=> c(x+b)+c(x+a) = (x+a)(x+b)
=> a+~+a+w=~+~+fu+~
=>x2 + (a + b -2c)x + (ab - bc-ca) = 0
Given that, the sum of its roots is equal to zero.
_ (a + b - 2c) = 0
1
2c-a-b = 0
a+b
or c= - - ... (1)
2
ab -bc -ca
Product of roots = 1 =ab-c(a+b)

= ab_(a~b»)<a+b) = 2ab-<;+b)2

= -.!..[(a+b)2 -2ab] = _.!..(a 2 +b 2 )


2 2
Example 10: Ifa,j3and y, !Sbe the roots oftheequations~ + px-r = Oandx2 + px + r = 0
respectively, show that (a- y) (a- 5) = (fJ- y) (fJ- 5)
Solution: Since a, [3 are the roots of ~ + px - r = 0
.. a + [3 = - p and a[3 = - r
Now, since y, 1) are roots of x 2 +px + r = 0
.. a 2 +pa-r = 0 ...(1)
and [32 + p[3 - r = 0
Subtracting (2) from (1), we get
(a 2 -[32)+p(a-[3) = 0
a 2 + pa = [32+ p[3
46 Remedial Mathematics

(': p=-(y+o»

a 2-(y+o)a+yo = ~2_(y+O)~+y~ (by adding yo on both sides)


a -ya-oa+yo = ~2_~y_~0+Yo
2

a(a-y)-o(a-y) = ~(~-y)---o(~-y)
(a-y)(a-o) = (~-y)(~-o)
Example 11: If a, /3 are the roots of~ - 2x + 3 = O. Form an equation whose roots are
a+ 2. /3+ 2
Solution: Here, the given equation is
~-2x+3=0 (1)
Now since, a,~
are the roots of (I), therefore
a+~ = 2,a~=3
We are to fonn an equation whose roots a + 2, ~ + 2
S = (a+2)+(~+2)=(a+~)+4=2+4=6
and p = (a+2)(~+2)=a~+2(a+~)+4
= 3+2.2+4=3+4+4=11
Hence, the required equation is
x 2 -Sx+p=0
=> x 2 -6x+ll =0

1. Find the equation whose roots are

(/;\ 2 ~_5 (/'1') 4 5 (iii) - 5, - 3 (iv) 2 ± J3


J '2' 5'4
3 4
(v) 3 ± J5 (vi) 5 ± J7 (vii) 4'"3
2. If a,~ are roots of the equation ax2 + bx + c = 0 then find the value of
1 1 a3 ~3
(I) ~+ f32 (ii) T +~ (iii) a3~ + ~3a (iv) a 3 + ~3
(v) a 2 + ~2
3. Find the equation whose roots are a 2, ~2 where a,~ are the roots of the equation
~+x+ 1 =0.
1 1
4. Ifa,~ are the roots ofx2 -7x + 12 = O. Find the equation whose roots are a' 13 .
5. If a,~ are the roots ofthe equation ax 2 + bx + c = 0, then find the equation whose root~
are
a ~
(iv) 13' a
Quadratic Equations 47

6. If one of the root of x 2 + px + q = 0 is the square of the other, then show that
p3 -q(3p- I) + q2= 0
7. (i) If a be a root ofthe equation, 4x2 + 2x - I = 0, prove that 4a3 - 3a is the other root.
a
(ii) Form a quadratic equation whose roots are ~ a ± ~ a _ b

8. If a,~ be the root ofx2 - px + q = 0 and a' ,W be the roots of x 2 - p'x + q = 0, find the
value of
(a-a'i + (~-a'i + (a- W)2 + (~_ ~')2
~. If a and ~ are roots of ~ + px + I = 0 and y, 8 are the roots of ~ + qx + I = 0, show that
q2_p2 =(a-y) W-y) (a+ 8)(~ +8)
10. If the roots px2 + qx + 2 = 0 are reciprocals of each other, then
(a) p=O (b) p=-2 (c) q=O' (d) p=2
11. If the ratio of the roots of the equation, x 2 + px + q = be equal to ratio of the roots of
~ + Ix l- m = 0, then prove that p 2m = Pq.
12. If the ratio of the roots ofthe equation Ix 2 + nx + n = 0 be p : q, then prove that

H+~+f4 =0
13. (a) Find the value of p for which x + I is a factor of
x4 + (P-3)x 3 -(3p-5)~ + (2p-q)x+ 6.
Find the remaining factors for this value of p.
(b) If~- 3x+ 2 is a factor ofx4 - px 2 + q = 0, prove p= 5, q= 4.
14. Knowing that 2 and 3 are the roots of the equation 2x3 + mx2 - 13x + n = 0, determine
m and n and find the third root of the equation.
15. Find all the roots of the equation 4x4 - 24x3 + 57x2 + 18x- 45 = 0 ifone of them is
3+i16.

HINTS TO THE SELECTED PROBLEMS


3. ~ + x + 1 = 0 Given that it roots are a and ~.
So a+ ~ =-1
a~ = 1
we have to form quadratic equation whose roots are a 2 and ~2.
a2+~2 = (a+~i-a~
=(_1)2_2xl=1
a2~2 = (a~i=(li= 1
Required equation
x 2 - (a2 + ~2) X + a~ = 0
=> x 2 +1x+ 1 = 0
48 Remedial Mathematics

6. Given equation is 0.2 + px + q = O.


Let the roots are a. and 0.2 .
Then a. + 0. 2 = - P
~ a. (1 + a.) = - p ... (1 )
0.0.2 = q
~ 3
0. = q ...(2)
Cubing equation (I) we get
0. 3 (1 + 0.)3 = _ p3
0. 3 [I +0.3 +30.(1 +0.)] = _ p3
q[l+q+3(-p)] =_p3
~ q[l+q_3p)=_p3
~ p3_ q (3p_1)+q2 = O.

ANSWERS
1. (i) 2x2 + 5x - 25 = 0 (ii) 20x2 - 41x + 20 =0
(iii)x 2 + 8x + 15 = 0 (iv) x 2 -4x+ 1 =0
(v) x 2 -6x+4=0 (vi) x 2 -10x+18=0
(vii) 12x2 - 25x + 12 = 0

b 2 - 2ac (b 2 _ 2ac)2 _ 2a 2c 2
2. (i) (ii) 3
c2 a c
c (b 2 - 2ac) -b(b 2 -3ac) b2 - 2ac
(iii) (iv) (v)
a3 a3 a2
3. x 2 +x+l=0 4. 12x2 -7x+l=0
5. (i) a2x 2 + b (b 2 -3ac)x+ c 2 = 0 (ii) a 4x 2 - ca (b 2 - 2ac) x + c 4 = 0
(iii) a2x 2 - (b 2 - 2ac) x +c 2 = 0 (IV) acx2 - (b 2 - 2ac) x + ac = 0
(v) bcx 2 + (b 2 + ac)x+ ab = 0

7. (i) 4a 3 - 3a (ii) bx - 2aj;;; + a 2 = 0


8. 2 [p2 _ 2q + p,2 - 2a' - pp'] 10. d
13. (a) (x - 1). (x - 2), (x + 3)
5
14. m = - 5, n = 30, r = -'2 15. 3±iJ6,± J3
2

_ APPLICATION OF QUADRATIC EQUATIONS


Example 1: Divide 57 in to two parts such that their product in 782.
Solution: Let the required parts be x and (57 - x).
Then their product = x (57 - x).
.. x(57-x) = 782
~ ~-57x+782 = 0
Quadratic Equations 49

57 ± J(57)2 - 3128 = 52 ± ~3249 - 3128


x=
2 2
57 ± 51 57 ± 1-1
x=----=--
2 2
57 ± 11 68
x = --=-=34
2 2
57 -11 46
or x = ---=-=23
2 2
Hence the two parts are 34 and 23.
Example 2: The sum of the squares of two consecutive whole numbers is 6/. Find the
numbers.
Solution: Let the required consecutive whole numbers be x and x + 1.
x 2 +(x+ Ii = 61
=> ~+x2+ 1 +2x = 61
=> ~+2x-60 = 0
=>
=> (x + 6)(x - 5) = 0
=> x = -6, or x= 5
x is a whole number
x = 5
And required numbers are x and x + 1 as 5 and 5 + 1 = 6.
Example 3: The sum ofthe squares oftwo consecutive odd integer is 394. Find the integers.
Solution: Let the required integers be (2x + 1) and (2x + 3)
(2x+ Ii + (2x +3)2 = 394
=> ~+ 16x-384 = 0
=> x2+2x-48 = 0
=> ~+8x-6x-48=O
=> x (x + 8) - 6(x + 8) = 0
=> (x+8)(x-6) = 0
=> x + 8 = 0 or x - 6 = 0
=> x =-8 or x=6.
When x = - 8, the required integers are - 15 and - 13
when x = 6, the required integers are 13 and 15
Hence, the required integers are {-15, - 13} or {13, 15}.
8
Example 4: The sum of two natural numbers in 8. If the sum of their reciprocals in 15"
find the two numbers.
Solution: Let the numbers be x and (8 - x).
50 Remedial Mathematics

1 1 8
-+--
X 8-x 15
8-x+x 8
x (8 - x) 15
8 8
~8x - x 2 15

i.e, 120 = 64x - 8x2
8x2-64x+ 120 = 0
x2-8x+ 15 = 0
(x-5)(x-3) = 0
x = 5 or x=3
when x = 5, the Numbers are x and 8 - x = 5 and 3
when x = 3, The numbers are x and 8 - x = 3 and 5
:. Required numbers are 5 and 3.
Example 5: A number consists of two digits whose product in 18. When 27 in subtracted
from the number, the digits interchange their places. Find the numbers.
18
Solution: Let the tens digit be x. Then the units digit = -
x

Number formed = (lOX + 1:)


(lOX 1: + x )
Number formed on reversing the digits =

.. (lOx + I:) _27 (10 x1: + x)


=

162
=> 9x- --27 = 0
x
=> 9x2-27x-162 = 0
=> x2-3x-18 = 0
=> x2 - 6x + 3x - 18 = 0
=> x (x - 6) + 3 (x - 6) = 0
=> (x-6)(x+ 3) = 0
=> x = 6 or x = - 3.
x =6 [ Since a digit can never be negative]
18
Thus we have the tens digit = 6, the unit digit = (; = 3

Hence, the required number = 63.


Example 6: Two positive numbers are in the ratio 2:5. JfdifJerence between the squares of
these numbers is 189. Find the numbers.
Quadratic Equations 51

Solution: Let the numbers be 2x and 5x.


.. (5xi-(2x)2 = 189
=:> 25~-4~ = 189
=:> 21~ = 189
189
x2 = - =9
21
x = ±3
Since the required numbers are positive. :. x = 3.
And required numbers = 2x and 5x
= 2 x 3 and 5 x 3
=6andI5.
Example 7: One year ago, a man was 8 times as old as his son. Now his age is equal to the
square of his son s age. Find their present age.
Solution: Let the son's age one year ago be x years.
Then, the man's age one year ago = (8x) years.
Present age of the son = (x + 1) years.
Present age of the man = (8x + 1) years.
(8x+ 1) = (x+ 1)2
=:> ~-6x = 0
=:> x = 0 or x = 6
x =6 [Son's age can not be zero]
Present age of the son = (x + 1) years = 7 years.
and the present age of the man = (8x + 1) years = 49 years.
Example 8: The sides (in cm) of a right triangle are x - 1, x and x + 1. Find the sides of
triangle.
Solution: It is clear that the largest side x + 1 is hypotenuse of the right triangle.
According to Pythagoras Theorem, we have ~ + (x - 1i = (x + 1)2
=:> ~+~-2x+ 1 = x2+2x+ 1
x 2 -4x = 0
x+1
x(x-4) = 0 x
i.e., x = 0 or x=4 ;,
Since with x = 0 the triangle is not possible, hence x = 4
sides are x-I, x and x + 1 = 4 - 1,4 and 4 + 1 x-1
Fig. 1.1
i.e., 3 em, 4 cm and 5 cm.
Example 9: Rs 250. was divided equally among a certain number of children. If there
were 25 more children, each would have received 50 paise less. Find the number of
children.
Solution: Let the number of required children be x.
25000
The share of each = - - paise
x
52 Remedial Mathematics

25000J
It there were (x + 25) children, share of each = ( x + 25 paise.

25000 25000
.. - - - - - =50
x x+ 25

---- - I
=> x x+25 500
x+ 25-x
=> x (x + 25) 500
=> x (x + 25) = 12500
=> ~+25x-12500 = 0
=> (x+ 125)(x-100) = 0
=> x = -125 or x= 100
=> x = 100 [since number of children can not be negative]
Hence, the number of children = 100.
Example 10: The Hypotenuse ofright triangle is J m less then twice the shortest side.lfthe
third side is J m more than the shortest side, find the sides of the triangle.
Solution: Let the shortest side be x m.
.. Hypotenuse = (2x-I) m and third side = (x + l)m.
Applying Pythagoras Theorem, we get
(2x-li =x2 +(x+li
=> 4~-4x+ 1 = x 2 +x2 +2x+ 1 x
i.e., 2x2 -6x = 0
=> x 2 -3x = 0
=> x(x-3) = 0 x+1

=> x = 0 or x=3 Fig. 1.2


Since x = 0 makes the triangle impossible. There fore x = 3
and sides of the triangle are = x, 2x - 1 and x + I
= 3,2 x 3-1,and3+1
= 3m, 5 mand I m
Example 11: A passenger train takes 3 hours less for journey of 360 km if its speed is
Increased by 10 kmlhr. What is the usual speed?
Solution: Let the usual speed of the train be x km/hr
.
Tlmeta k en to cover 360 km atxkm/hr= -360 hr.
x
New speed = (x + 10) km/hr
360
Tune taken to hour 360 km at (x + 10) km/m = ) hr
(x + 10
360 360
- - - - =3
x x + 10
Quadratic Equations 53

x x + 10 20
x+lO-x
x (x +10) 120
=> x(x+ 10) = 1200
=> ~+ IOx-1200 = 0
=> .x2+40x-30x-1200 = 0

=> x(x+40)-30(x+40) = 0
=> (~+40)(x-30) = 0

=> x = -40 or x=30.


=> x=30 [ .: speed cannot be negative]
Hence, the usual speed of the train is 30 kmlhr.
Example 12: A plane left 30 minutes later than the scheduled time and in order to reach
its destination 1500 km awcry in time, it has to increase its speed by 250 kml from its usual
speed Find its usual speed
Solution: Let the usual speed to the plane = x km/hr
=> The increased speed of the plane = (x + 250) km/hr
1500
usual time taken by the plane to cover 1500 km = -- hrs
x
1500
and the new time taken to cover 1500 km = x + 250 hrs

From the given statement, it is clear that the new time taken is 30 minutes i.e., "21 hrs
less than the usual time.
1500 1500
x x+250 2
1500 (x + 250) -1500x
x (x + 250) 2
1500 x + 375000 -1500x
i.e.,
x 2 + 250 x 2

375000 1
=> 2 =-
x + 250x 2
On cross mUltiplying we get.x2 + 250x = 750000.
=> .x2+250x-750000 = 0
On factorising we get (x + 1000) (x-750) = 0
x = -10000rx-750
Rejecting the negative values of x, we get x = 750
The usual speed of the plane = 750km/hr
54 Remedial Mathematics

~~~~~~~I EXERCISE 1.8 ~I~~~~~~


1. Two numbers differ by 3 and their product in 504. Find the numbers.
2. The sum of two numbers in 18 and their product in 56. Find the numbers.
3
3. The sum of two numbers in 15 and the sum of their reciprocals in 10. Find the
numbers.
4. The sum of the squares ofthree consecutive positive integers is 50. Find the integers.
5. Find two consecutive even integers whose squares have the sum 340.
6. Find two consecutive positive odd integers whose squares have the sum 290.
7. A two digit numbers is 5 times the sum of its digits and is also equal to 5 more then
twice the product of its digits. Find the number.
8. A two digit number is such that product of the digits in 35. When 18 is added to this
number the digits interchange their places. Determine the number.
9. The sides (in cm ) of a right triangle containing the right angle are 5x and 3x - 1. If the
area of the triangle is 60 cm2, find the sides of the triangle.
10. The Hypotenuse of a right angle triangle is 6 meters more than thrice the shortest
side. If the third side is 2 meters less than the hypotenuse, find the sides of the
triangle.
11. The area ofa right angle triangle is 600 sq. cm. If the base of the triangle exceeds the
altitude by 10 cm, find the dimensions of the triangle.
12. If the perimeter of a rectangular plot is 68 m and its diagonal is 26 m, find its area.
13. A fast train takes 3 hours less than a slow train for ajourney of600 km. If the speed of
the slow train is 10 kmlhr less than that of the fast train, find the speeds of the two
trains.

HINTS TO THE ,SELECTED PROBLEMS


1. Let the required no. are x and x - 3
Their product = x (x - 3)
x(x-3) =504
On solving, we get x = 24 thenx- 3 = -21 and x =- 21 thenx- 3 =- 24
3. Let the required numbers be x and (15 - x)
1 1 3
Then -+-- =-
x15-x 10
on solving we get x = 10 and 5.
6. Let the required two consecutive positive odd integers be x and x + 2.
Then x2 + (x + 2)2 = 290
7. Let the numbers is (10 x + y)
Then (lOx+y) =5(x+y) ...(1)
and 10x+y =2xy+ 5 ...(2)
Quadratic Equations 55

From (1), we get 5 x=4y


4
x= -y
5

substituting x= sy
4
in (2), we get

(lOX 4;)+ Y = (2X 4; xy )+5


8Y-45y+25 =0
5
Y =5 or
8
since Y "# 5/8
Hence Y =5
Then x=4
9. It is clear from the figure,
1
-x5xx(3x-l) =60
2 5x
3x2 -x-24 =0
8
x= -- or-3
3
since, sides of the triangle can not be negative x = 3 Fig. 1.3
and sides =5xand3x-l
= 3 x 5 and 3 x 3 -1
= 15 cm and 6 cm.
Applying Pythagoras Theorem, we get
Ii = 152 +82 =289=(l7i
H = 17cm.
10. Let the length of the shortest side be x meters
Then, the hypotenuse = (2 x + 6) metres.
and, the third side = (2x+4)
By Pythagoras theorem we have
(2x+6i = x 2 + (2x+4i
=> x2- 8x-20 = 0
x = 10 or x=-2
x = 10 [since side ofa triangle can never negative]
Shortest side = 10 m
Hypotenuse = 2 x + 6 = 26 m
third side = 24 m
13. Let the speed of the train = x Kmlhr
and speed of the slow train = (x-l0) Kmlhr.
56 Remedial Mathematics

Time taken by fast train to cover 600 Km = 600 hrs


x
Time taken by slow train to cover 600 Km = 600 hrs
x-l0
600 _ 600 = 3
x-l0 x
600x - 600(x -10)
x(x-l0) =3
=> x'l-10x-2000 = 0
=> x = 50 or x=-40
Neglecting x = -40, we getx = 50 andx-lO = 50-10 =40
Speed to fast train = 50 Km/hr
speed of slow train = 40 KmIhr.

ANSWERS

1. (24 and21)or (-21 and-24) 2. 4 and 14 3. 10 and 5


5. 12 and 14 6. 11 and 13 7. 45 8. 57
9. 15 em, 8 em and 17 em 10. 10m,26mand24m
11. Altitude = 30 em, base = 40 em 12. 240m2
13. 50 kmlhr and 40 kmlhr.

OBJECTIVE EVALUATION

MULTIPLE CHOICE QUESTIONS


(Choose the appropriate answer)
1. Which of the following is linear equation
b
(a) ax2 + bx + C = 0 (b) ax+ - =0
x
(c) ax+2=0 (d) None of these
2. Which of the following are quadratic equation
1
(a) x-- =0 (b) x 3 + 3x2 + x-I = 0
x
(c) 2x+3=0 (d) None of these
3. The degree of the equation 2x - 3 = 0 is
(a) 1 (b) 2 (c) 3 (d) None ofthese
4. Solution of the equation 2x - 3 = 0 is
2 3
(a) - (b) - (c) 1 (d) 0
5 2
Quadratic Equations 57

5. The root of the quadratic equation 2.x2 - 3 x-I = 0 is


(a) 2 (b) 1 (c) 3 (d) None of these
,6. The roots of the equation 3x2 - 4x -7 = 0 is
7 3 7
(a) '30rl (b) '70r2 (c) '3 or - 1 (d) None of these

*
7. Ifax2 + bx + c = 0 (a 0) is a quadratic equation and b2 - 4 ac = 0 then the roots are
(a) real and distinct (b) real and equal (c) imaginary (d) None of these
8. Solution of the equation 16x2 = 25 is
3
(b) ±~
4
(a) ±- (c) 4
5 4
*
9. lithe discriminant ofax2 + bx + c = 0 (a 0) is greater then zero then root are
(a) real and distinct (b) real and equal (c) imaginary (d) None of these
10. The root of the equation x 2 + ax + b = 0 are equal if
(a) a2 =3b (b) a2 =4b (c) a2 =b (d) None of these
2
11. Solution of the equation x + 16x + 60 = 0 is
(a) -IOor6 (b) lOor-6 (c) -lOor-6 (d) None of these
12. The sum and product of the roots of the equation ax2 + bx + c = 0 is
be be be
(a) -, - (b) --, - - (c) - - - (d) None of these
a a a a a, a
13. The sum ofthe roots of the equation 6x 2 + x - 2 = 0 is
1 1 5
(a) "6 (b) -"6 (c) 4" (d) 6

14. If a, 13 are the roots of the equation d + bx + c (a * 0), then the value of a 3 + 13 3 is
3
3abc - c 3abc - b3 3abc - a 3
(a) (b) (c) (d) None of these
d
15. The equation whose roots are J3 and 3J3 is
(a) x 2 + 4.J3x + 9 = 0 (b) x 2 - 4.J3x + 9 = 0

(c) x2-4.J3x-9 =0 (d) none ofthese


16. The quadratic equation whose roots are 4 and 5 is
(a) x2-9x-20=0 (b) x2-9x+20-0
(c) ..z
+ 9x + 20 = 0 (d) none of these
17. lithe sum and product ofthe roots ofthe equation d - 5x + c = 0 is 10 then the value of a and
cis
1
(a) a=
1
2'c='5
1
(b) a = 5. c = '21 (c) a = -,C
2
=5 (d) None of these

18. The roots of the equation G>;2 + bx + c, a * 0 is


b+ Jb 2 -4ac -b± Jb 2 +4ac
(a) (b)
2a 2a

-b± Jb 2 -4ac
(c) (d) None of these
2a
58 Remedial Mathematics

19. The discriminant of the equation 5~ + I6x + 3 = 0 is


(a) 195 (b) 196 (c) 197 (d) none of these
20. The discriminant of the equation 3J7x2 + 4x - J7 = 0 is
(a) 98 (b) 99 (c) 100 (d) 1001

TRUE I FALSE
(Write T for True and F for False)
1. The solution of the linear equation 3x + 6 = 0 is - 2 (T/F)

2. The solution of the equation x - ~ = 0 is ± 1. (T/F)


x

3. The degree of the equation x - ~ - 3 = 0 is. (T/F)


2x

4. The sum of the roots of the equation 6x 2 + x - 20 = 0 is .! . (TIP)


6
5. The roots of the equation x 2 - 5x + 6 = 0 is 2,3. (T/F)

6 • ax + -b - 2 = O·IS a I'mear equatIOn.


. (T/F)
3x
. x2
7. x=-5Isarootof -+2x-1O =0 (T/F)
-5
8. The discriminant of (x - I) (x + 2) = 0 is 9. (T/F)
9. If the roots of the equations x 2 + 2x + ab = 0 are real and unequal then the equation.
x 2 - 2 (z + b) x + a 2 + b2 + 2c 2 = 0 has no real roots. (TIP)

10. The roots of J2x 2 - 5x - 3 = 0 will be equal. (T/F)


11. For a quadratic equation ax2 + bx + c = 0, a*,O it b2 - 4 ac < 0 the two distinct real roots will
exist. (T/F)

12. J2.x + ~ =.J3 is not a quadratic equations. (TIF)


x
13. The roots of the quadratic equation ax2 + bx + c = 0, a*,O can be found by using the quadratic
+ b ± ~b2 - 4ac . 2
formula , provided b - 4ac ~ O. (T/F)
2a
14. The equations (x - 1)3 = x 3 - 2x + 1 is not a quadratic equation. (T/F)

15. _1- + _1_ = 3 has two distinct real roots. (TIF)


x-I 2x+ 1
FILL IN THE BLANK
1. If x = 1 is a common root of the equations px 2 + px + 3 = 0 and x 2 + x + q = 0 the
pq=---

2. [;iscriminate for x + 3. - I = 0 is _ __
x
3. If the roots of the equations 4x 2 - Kx + 9 = 0 are not real this K should be _ __
4. The roots of a quadratic equations can be found by using the method of _ __
5. If a, p, are roots of the equations x2 - 2x + I = O. then a - Pis _ _ _ .
Quadratic Equations 59

ANSWERS

MULTIPLE CHOICE QUESTIONS


1. (c) 2. (a) 3. (b) 4. (b)
5. (b) 6. (c) 7. (b) 8. (b)
9. (a) 10. (b) H. (c) 12. (c)
13. (b) 14. (b) 15. (b) 16. (b)
17. (c) 18. (c) 19. (b) 20. (c)

TRUE/FALSE
1. True 2. True 3. False 4. False.
5. True 6. False 7. False 8. True.
9. True 10. False H. False 12. False.
13. False 14. False 15. True

FILL IN THE BLANK


1. 3 2. -7 3. -6J2 <K 6J2 4. Factorization
5. 0

REFRESHER

Do you know? After reading this chapter you should be able to learn the following
concepts:
• Equation ax2 + bx + c = 0 is a quadratic equation only when a:t. O.
• A real number a is real root of the quadratic equation ax2 + bx + c = 0 only when value
of aa 2 + ba + c is equal to zero
• Roots of the quadratic equation ax2 + bx + c = 0 (a:t. 0) are given by
-b±~b2 -4ac
x=
2a
• The quadratic equation ax2 + bx + c = 0, (a:t. 0) has no real rOQts of b2 -- 4ac < 0
• The discriminant of the quadratic equation ~ + bx + c = 0 is given by D = b2 - 4ac and
-b ± .fi5 -b + .fi5 -b - .fi5
the two roots are 2a i.e., 2a and 2a
• The roots will be real only when D ~ 0
• The quadratic equation ax 2 + bx + c = 0 have repeated roots it D = 0
• The quadratic equation ax 2 + bx + c = 0 have distinct roots if D > O.
Can we do? (Frequently Asked Questions)

1. Solve (2X+I)4 _IO(2X+I)2 +9 =0. [UPTUB.Pharma2005j


x-I x-I
60 Remedial Mathematics

[UPTU B.Pharma 2003]

3. If a. be a root of the equation 4~ + 2x + 1 = 0, prove that 4a.3 - 3a. is the other root.
[UPTU. B Pharma 2002]

4. Solve (x+~r -%(x-~) =4,x,tO, [UPTU B.Pharma 2004]

5. Divide 33 into two parts whose product is 342. [UPTU B.Pharma 2001]
6. Solve 7x+ 1 + 7 1- x = 50 [UPTU B. Pharma 2008]
4x+1 x
+1 5
7. If - - + - - - - =0 findthevalueofx [UPTU B. Pharma 2008]
x + 1 4x + 1 2 '

DOD
SIMULTANEOUS LINEAR EQUATION

• LINEAR EQUATION IN TWO VARIABLES


If a, b, and c are three real numbers with a"* 0 and x and yare two variables, then the equation
ofthe type ax + by + c = 0 or ax + by = c is called a linear equation in two variables.
For Example:
(i) 3x+ 5y-7 =0
(ii) 5x-8y= 15.

Solution of a Linear Equation


We say that x = a and y = ~ is a solution ofax + by + c = 0
if aa+b~+c =0.

Simultaneous Linear Equations in Two Variables


Two linear equations in two unknowns x and yare said to form a system of simultaneous
linear equations if each of them is satisfied by the same pair of values of x and y.
For Example:
The pair of linear equations 3x+ 2y = 7 and4x- 8y-2 = 0 in two variables x andy forms a
system of simultaneous linear equations.

• SOLUTION OF A GIVEN SYSTEM OF SIMULTANEOUS EQUATIONS


The values of x and y which satisfies each equation of the given system of linear equations
is called its solution.
For Example: Show that x = 5, y = 2 is a solution of the system oflinear equations
2x+3y = 16,x-2y=1.
Solution: The given equations are
2x+3y = 16 ...(1)
and x-2y =1 ... (2)
putting x = 5 andy = 2 ip(l) we get
IRS =2x5+3x2=16=RHS.
62 Remedial Mathematics

Putting x = 5 andy = 2 in (2), we get


LHS = 5 - 2 x 2 = 1 = RHS.
Thus x = 5 and y = 2 satisfy both (1) and (2)
Hence x = 5, y = 2 is a solution of the given system of equations

Consistent / Inconsistent Pair of Linear Equations


A pair oflinear equations in two variables which has a solution is called a consistent pair of
linear equations. A pair oflinear equations is two variables which has no solution is called an
inconsistent pair of linear equations.

Remarks
1. A system of simultaneous linear equations in two variables will have either
(I) only one solution
(il) no solution
(iii) an infinite number of solutions.
2. It must be taken into memory that no system of linear equations in two variables will
have only two solutions, only three solutions, only four solutions, etc. lnfact, if any
system of linear equations has two or more solution, it will always have an infinite
number of solutions .

• GRAPHICAL METHOD FOR SOLVING SIMULTANEOUS LINEAR


EQUATIONS

To solve the system of linear equations in two variables adopt the following steps.

STEP KNOWLEDGE
l. On the same graph paper, draw graph (straight line) for each given equation.
2. If the lines drawn intersect each other at a unique point; read the values of x and y
for this point. The values of x and y so obtained, gives the required solution of the
given system of equations.

Method
Let the given system of linear equations be
a/x + b\y + c\ =0 ... (1)
and a;X + bzY + c2 =0 I ••. (2)
We draw the graph of e~ch of: the 'given linear equations on the same graph paper.
Let the lines L\ and L2 represent these graphs.

Casel: When the lines L\ and L2 intersect at a point. Let the graph lines L\ and L2 intersect
at a point P (a, I)).
Then x = a, y = I) is the unique solution of the given system of equations.
Simultaneous Linear Equation 63

----~----~~------~---.x
o

Fig. 2.1
Case 2: When the lines L) and L2 are coincident.
In this case, the given system has infinitely many solutions.
y

x' -----+---------------
o x

y'
Fig. 2.2
Case 3: When the lines L) and L2 are parallel.
In this case, there is no common solution of the given equation, as shown in figure, i.e.,
the given system of equations has no solution. Thus in this case, the system of given
equations is inconsistent.
Y

x'
~L' L1

x
0

y'

Fig. 2.3
64 Remedial Mathematics

Remarks
• A system of two linear equations in x and y has
(;) a unique solution if the graph lines intersect at point.
(ii) infinitely many solutions if the two graph lines coincide.
(iii) no solution if the two graph lines are parallel.
• Straight lines as graphs oflinear equations alx + bly + c I = 0 and alx + bLY + c2 = 0
intersect each other at a point if
b.,
-a2al *-.
b 2
• Straight lines as graphs of linear equations alx + bly+ c I = 0 and a r + bLY + c 2 = 0
are parallel to each (i.e. do not intersect) if
b.,
-a2al -_ -*-.
b2 C2
cI

• Straight lines as graphs of Iinear equations a IX + b IY + c I = 0 and a r + bLY + c2 = 0


coincide (i.e. becomes single line) if.
al
-
b.,
cI
=-=-
a2 b2 c2

Here one equation is a constant multiple of the other equation.

~~~~~~I SOLVED EXAMPLES ~I~~~~~~


Example 1: Solve graphically the system ofequations:
x + 2y = 3, 4x + 3y = 2.

Solution: We have x + 2y = 3 => y = .!. (3 - x)


2
1
Now x = 1 => y = "2 (3 - 1) = 1.
1
x =3 => y="2 (3-3)=0.
Thus we have the following table:

x 3
y 1 o
Now, plot the point A (1,1) andB (3,0) on a graph paper. JoinAB and produce it on both
sides.
Now we have 4x+3y = 2
1
y = -;; (2-4x).
-'
Simultaneous Linear Equatioll 65

1
Now x =-1 y= 3" [2-4(-1)]=2.

1
x =2 y= 3" (2-4x2)]=-2.

Thus, we have the following table:

x -I 2
y 2 -2
Now, plot the point C (-I, 2) and D (2, - 2) on the same graph paper.

-5 -4 -3 -2

-3

-4

-5
y'

Fig. 2.4
Joint CD and produce it on both sides.
The two graph lines AB and CD intersect at the point C (-I, 2).
:. x = - 1, Y = 2 is the solution of given system of equation.
Example 2: Show graphically that the system ofequations:
3x - y = 2, 9x - 3y = 6.
has an infinite number of solutions.
Solutions: Graph 3x - y = 2.
3x-y =2 y=3x-2.
Now x =1 y=3 x l-2=1.
x =2 y=3 x2-2=4.
66 Remedial Mathematics

Thus, we have the following table:


x 2
y
Now plot the points A (1, 1) and B (2, 4) on the graph paper. Join AB and produce it on
both sides.
Graph of 9x - 3y = 6:
1
9x-3y = 6 ~ y= -(9x-6)
3
1
Now x =0 ~ Y = - (9 x 0 - 6) =-2
3
1
x =3 ~ y= 3(9 x 3-6)=7.
Thus, we have the following table
x 0 3
y -2 7.
Now plot the points C (0, - 2) and D (3, 7) on the same graph paper
y

D (3, 7)

x' ---+----f--+--I----f--+--++-+---+--+------i"-- x
-5 -4 -3 -2 4 5

Fig. 2.5
We find that both the points C and D lie on the line AB.
Since the graph lines AB and CD coincide, the given system has an infinite number of
solutions.
Simultaneous Linear Equation 67

Example 3: Solve the following system of linear equations graphically:


2x + y = 6, x - 2y + 2 = o.
Find the vertices of the triangle formed by the above two lines and the x - axis. Also,
find the area of the triangle.
Solution: 2x+y =6 ... ( I)
Y =6-2x
x 4 2
4 -2
Y
I 2

x-2y+2 =0 ... (2)


x+2
y=--
2
x o 4 2
y -3 2

The two straight lines intersect at A (2, 2).


y

-4

-5
y'

Fig. 2.6
:. x = 2,y = 2 is the solution. Vertices of the triangle are A (2,2), B (3,0) and C (-2, 0)
AL .1 BC BC = 5 units, AL = 2 units.

Area of 11 ABC = .!. x (5 x 2) = 5 square units.


2
Example 4: Show graphically that the system of linear equations
2x - 3xy = 5, 6y - 4x = 3.
has no solution.
68 Remedial Mathematics

Solution: Graphof2x-3y= 5.
2x-3y = 5

=> y = '3I (2x-5).


Now x = I
1
Y = '3 (2x 1-5)=-1.
x = 4

y = 'I3 (2 x 4 - 5) = - 1.
Thus, we have the following table
r-------r-------~----~
x 4
Y -1
Now plot the point A (1, - 1) and B (1, 4) on a graph paper.
Join AB and produce it on both sides. y

Graphof6y-4x=3
6y-4x = 3
I
=> y = '6 (3 + 4x).
x =0
I 1 ·o---+--+-~~+--+--+?~--+--+----
Y = '6 (3 + 4 x 0) = '2' x x

x =3
I 5
Y = - (3+4 x 3)=-.
6 2
Thus we have the following table:
yO
x o 3
Fig. 2.7
y 112 512

On the same paper, plot the points C (0, 112) and D (3,5/2). Join CD and produce it on
both sides. Then line CD is the graph of6y-4x = 3. Since the graph lines AB and CD of the
given equations are parallel, it follows ~hat the given system of equations has no solution.
Example 5: Which of the following pair of linear equation are consistent/in consistent?
U"e the algebraic conditions for ratios

~ !l and.:L.
b2 a2 0 c2
(i) 5x - y = 7, x - y + 1 = O.
(ii) 3x + 2y - 4 = 0, 3x + 2y + I = O.
(iii) 3x + 4y + 5 = 0, 6x + 8y + 10 = O.
Simultaneous Linear Equatioll 69

Solution: (I) 5x-y-7 = 0 ...(1)


and x-y+ 1 =0 ...(2)

~ ~;t;!!L
a2 b2
So, the pair of linear equations has unique solution and therefore, the equations are
consistent.
(ii) 3x+2y-4 =0 ... ( 1)
3x+2y+ 1 =0 ...(2)

~ =~=I·!!L=~=I·.:L=-4--4
a2 3 ' , b2 2 'C2 1 - .

~=!!L;t;.:L.
a2 b2 c2
So, the linear equations are in consistent because the pair of linear equations has no
solutions.
(iii) 3x+4y+5 =0 ...(1)
6x+ 8y+ 10 =0 ... (2)

~ = ~ = _. !!L = i = L Ct - 2. - ~
a2 6 2' b2 8 2' ~ - 10 - 2 .

~=!!L=.:L
a2 b2 C2
There are infinitely many solution for the given pair of lines. Hence, the equations are
consistent.
Example 6: Find the values ofkfor which the system ofequation.
x - 2y = 3, 3x + ky = 1.
has a unique solution.
Solution: The given system of equations are
x-2y =3, 3x+ky= 1.
These equations are ofthe form:
atx+bty+c t =0,a2x+bY'+c2 =0.
where at = -I, b t = -2, c t = -3 and a2 = 3, b2 = k, c2 = -I.
for a unique solution we must have

~
a2
This happens when k;t; - 6.
Thus, for all real values of k other than - 6; the given system of equations will have a
unique solution.
70 Remedial Mathematics

Example 7: Find the values ofkfor which the system ofequations


3x + Y = 1
(2k - 1)x + (k - 1)y = (2k + 1)
has no solution.
Solution: The given equations are
3x +y-l =0
(2k - 1) x + (k - 1) Y - (2k + 1) = O.
These equations are of the form.
atx + bty+ c t = 0, a2 x + b2Y+ c2 = O.
where at =3,b J =I,c]=1
a 2 = (2k-l), b2 = (k-l), c2 =-(2k+ 1).

for no solution, we must have ~ = !!L *.:L .


a2 b2 c2
3 1 -1
=--*
2k -1 k-l -(2k+l)'
3 I 1 1
= - - and - - * - -
2k - 1 k- 1 k - 1 2k + 1
¢:> (3k-3 =2k-l)and(2k+l*k-l)
¢:> k =2 and k*-2

Thus ~ = !!L
*.:L holds when k= 2.
a2 b2 c2
Hence, the given system of equation has no solution when k = 2.

~~~~~~~I EXERCISE 2.1 I~~~~~~~


Solve the following system of equations graphically.
1. x+y=3,2x+5y=12
2. 2x+y=3,2x-3y=7.
3. 2x+ 3y=4, 3x-y=-5.
4. 4x+3y=5,2y-x=7.
5. 3x+y+ 1 =0,2x-3y*8=0.
6. Solve the following system of linear equations graphically.
2x-3y-17=0,4x+y-13 =0.
Shade the region between the lines and the x-axis.
7. Solve the following system of linear equations grapJ,;r- l l -
2x+y-5 =0,x+y-3 =0.
fmd the points where th~ graph meet the x-axis.
8. Solve the following system of linear equations graphically
2x-y-4=0;x+y+ 1 =0,.
Simultaneous Lilleal" Equation 71

9. Solve the following system of linear equations graphically.


x-y+ I =0, 3x+2y-12=0.
Calculate the area bounded by these lines and the x-axis.
10. In each of tte following systems of equations, find whether it has a unique solution,
an infinite number of solution or no solution:
(I) 3x+5y=13,5x+3y=4.
(il) 2x-3y= 5, 6x-9y= 15.
(iii) 6x-lOy=3,3x-5y=7.

( iv) ~+.,[ =3 x-2y=2.


3 2 '
11. For what value of k, the system of equations
kx+2y= 5, 3x-4y= 10.
has (i) a unique solution (ii) no solution.
12. Find the value of k for which the following system of equations has no solution.
3x+ky= I,(2k-I)x+(k-l)y=(2k+ 1).

I ANSWERS I
1. x = I,y= 2 2. x = 2,y=-1 3. x=-I,y=2 4. x=-I,y=3.
5. x=-I,y=2. 6. x=4,y=-3. 7. x=2,y=I,P(0,5)andQ(O,3).
8. x = I, y = -2, P (2, 0) and Q (-I, 0) 9. x = 2, y = 3, 75 sq units.
10. (i) Unique (ii) Infinite (iii) No solution (IV) Unique.
3 3
11. (i) All real values except - - (ii) k = - -. 12. k = 2,
2 2

• ALGEBRAIC METHODS OF SOLVING A PAIR OF LINEAR


EQUATIONS IN TWO VARIABLES
It is difficult to ex'amine non-integer solutions of linear equations correctly by graphical
method. In such cases only approximate values of x and y can be obtained from the graph.
Algebraic methods provide exact and correct solution for each and every pair of linear
equations
a1x + b1y+ c 1 = 0 and azX+ by> + c2 = O.
The most commonly used algebraic methods of solving a pair of linear equations in two
variables are as under.
1. By substitution method.
2. By elimination method
3. By cross-multiplication method.
72 Remedial Mathematics

Substitution Method

STEP KNOWLEDGE
Suppose we are given two linear equations in x and y. To solve these we proceed using the
following steps:
Step 1. Express y in terms of x, taking one of the given equations.
Step 2. Substitute this value of y in terms of x in the other equation to solve it for x.
Step 3. Substitute the value of x in the relation taken in step (i) solve it for y.
Remark
• We may interchange the role of x and y in the above method.
Example 1: Solve 2x - y - 3 = 0, 4x - y- 5 = 0 by substitution method.
Solution: Given equations are
2x-y-3 =0 ... (1)
4x-y- 5 =0 ... (2)
from(l): y =2x-3 ... (3)
Substitute y from (3) in (2), we get
4x-(2x-3)-5 =0
4x - 2x - 3 - 5 = 0
2x=2=:.x=1
Substituting x = I in (3), we get
y =2-3 y=-l
The required solution in x = 1, y = - 1.
Elimination Method

STEP KNOWLEDGE
In this method, we eliminate one of the unknown variables and proceed using the following
steps.
Step 1. Multiply the given equ~tions by suitable numbers so as to make the coefficients of
one of the unknown variables numerically equal.
Step 2. Ifthe numerically equal cofficients are opposite in sign-then add the new equations.
Otherwise subtract them.
Step 3. The resulting equation is linear in one unknown variable. Solve it to get the value of
one of the unknown variables.
Step 4. Substitute this value in any of the given equations.
Step 5. Solve it to get the value of the other unknown variable.
Example 2: Solve lOx + 3y = 75
6x- 5y = 11
Solution: The given system of equations is
10x+3y =75 ... (1 )
Simultaneous Linear Equation 73

6x-5y=11 ... (2)


We multiply equation (1) by 3 and equation (2) by 5 so that the coefficients of x in both
the equations become equal and we get the following equations
30x+9y =225 ... (3)
30x-25y =55 ... (4)
On subtracting (4) from (3) we get
170
34y = 170 => y=-=5
34 .
Substituting y = 5 in (1) we get
lOx+3x5 =75 => 10x=60
60
=> x=1O=6.
Hence, the solution is x = 6,y = 5.
Example 3: Solve 2x + 3y - 9 = 0, 4x + 6y - 18 = 0 by substitution method.
Solution: 2x+ 3y-9 =0 ... (1)
4x+6y-18 =0 ... (2)
from(l). 3y =9-2x
9-2x
y=-- ... (3)
3
substitutingy from (3) in (2), we get.

=> 4x + 6 (
9
~2x ) - 18 = 0 => 4x+ 2 (9-2x)-18 = 0

4x-4x + 18 -18 =0 0=0


Which is a true statement.
Hence, the given pair of linear equations has infinitely mnay solutions. Let us find
solutions putting x = k (any real constant) is (3), we get
9-2x
y = -3-'
9-2x
Hence,x=k, y = -3-'
Example 4: Solve the following system oflinear equations:
J2 x - J3 y = 0, J5 x + J2 y = O.
Solution: The given system of equations is
J2 x- J3 y =0 ... (1 )
J5x+J2y=O ... (2)
Multiplying (1) by J2 and (2) by J3 , we get
J6 y
2x- =0 ...(3)
J15 x+ J6 y =0 ... (4)
74 Remedial Mathematics

Adding (3) and (4), we get


(2x + J1s) x =0 => x=O.
substituting x = 0 in (2), we get

J2 y =0 => y=O
Hence, the so lution in x = 0, y = O.
Example 5: Solve 78 x + 91 Y = 53, 65 x + 117 Y = 60 by elimination method.
Solution: 78x+9Iy=53 ... (1)
6x+ 117y =60 ... (2)
Multiplying (1) by 5 and (2) by 6.
We get 390x+455y =265 ... (3)
390x+702y =360 ...(4)
Substracting (3) from (4), we get
702y-455y =360 -265
95 5 5
247y =95 y- 247 -13 =>y-13
Substitutingy = 135 in (1) we get

5
78x+ 91x - =53
13
=> 78x+35 =53
18 3 3
=> x = - = - =>x= - .
78 13 13
3 5
Hence, x =13,y=13.

I I I I
Example 6: Solve---=-l, - + - =8.
2x y x 2y
I I
Solution: Taking - = u, and - = v, the given equation becomes
x y

~-v =-1 => u-2v=-2 ...(1)


2
u+ ~ =8 => 2u+v= 16 ... (2)
2
MUltiplying (2) by and adding the result with (I) we get
30
5u =30 => u= - 5 =6 .
substituting u = 6 in (I) we get
6-2v =-2 => 2v=8
=> v=u.
Simultaneolts Linear Equation 75

I
Now u =6 <=> - =6 • <=> x= -
x 6'
1 I
v =4 <=> - =4 <=> y=-.
y z
[
Hence, the solution is x =-
6' y"" -.
4
Method of Cross Multiplication
Theorem: The system a/two linear equations

has a unique solution, given by

Proof: The given equations are


a,x+b,y+c, = 0 ... (1 )
a,x+b;V'+c2 = 0 ... (2)
Multiplying equation (I) by b2 and (2) by b, and subtracting, we get
(a,b 2 -a2b,)x = (b,c 2 -b 2c,).

x = q C2 -b2c, [ .. ~;otll :=;,a b -a b ;otO]


a, b2 -a2 b, · a2 b2 '22'

Multiplying equation (2) by al' (I) by a 2 and subtracting, we get


(a 1b2 -a2b 1)y = (c,a 2 -c2a 1)·
C1 a2 -c2 a1
y = [ .,' (a,b 2 -a2b,);otO].
a1 b2 -a2 b,
Hence, a unique solution exists, which is given by

This can be written as


x y

Remark
• The following diagram helps in remembering the above solution.
x
~~~
b
b'Xc'Xa'X '
y 1

b2 c2 a2 b2
76 Remedial Mathematics

Numbers with downward arrows are mUltiplied fIrst and from this product, the product
of numbers with upward arrows is subtracted.

~~~~~~~~I SOLVED EXAMPLES


Example 1: Solve the system ofequations 2x + 3y = J 7, 3x -2y = 6 by the method ofcross
multiplication.
Solution: The given equations may be written as
2x+3y-17=O ... (1)
3x-2y-6 = 0 ...(2)
By cross multiplication, we have:
x y 1
~~~
3 -17 2 3

x
-2XX X2 y
3

{3 x (-6)-(-2) x (-17)} {(-17) x 3 -(-6) x 2}


1
{2 x (-2)-3 x 3}
x -.::....y-=--
=> -51+12 -4-9·
-18-34
x y
-=-=-
-52 -39 -13
-52 -39
=> x = -13 =4,y= -13 =3.
Hence x = 4, y = 3 is the required solution.
Example 2: Solve 3x - y - 2 = 0; 2x + Y - 8 = 0 by method ofcross multiplication.
Solution: 3x+y-2 = 0 ...(1)
2x + Y - 8 = 0 ... (2)
are the given equations.
By cross multiplication method
x Y
-t -2 -2 3 3 -t
t X -8 8X 2 2X t
X Y 1
=
{( -1)(-8) - (1)( -2)} {(-2)(2) - (-8)(3)} {(3)(1) - (2)( -I)}
x
=> --
8+2
y
-4+ 24
=--
3+2
Simultaneous Linear Equatioll 77

x =L=.!..
10 20 5
X I y 1
= - and - =-
10 5 20 5
10 20
x = - andy=-
2 2
=> x=5andy=4.
Hence x= 5,y=4 is the required solution.
x y x y
Example3:Solve -+-b =a+b; 2"+2"=2.
a a b
Solution: The given equations may be written as
1 1
-x+-y-(a+b) =0 ...(1)
a b
1 1
-x+-y-2
2
=0 ... (2)
a b2
By cross multiplication
x y x

~ X-(a+b)
-(a + b) X a
1 1

~X
b
I
Il -2
-2 -;; -;;
1
li
x y

[-(aa;b) +~] [a~2 - )b]


y
=--=--
a-b a-b

(a-b) a 2b 2 2
and y = --
2
x - - =b.
a (a-b)
Hence, x = ~, and y = b2 in the required solution.
a b ab 2 a 2b
Example4: So/ve---=O, - + - =(cI+b2);wherex~O,y~O.
. x y x Y
1 1
Solution: Taking - = u and - = v the given equations become.
x y
au-ab *0 =0 ... (1)
ab 2u + ~bv- (~+ b2) =0 ... (2)
78 Remedial Mathematics

By cross multiplication, we have


u v
o 0 a -b
_(a 2 +b 2 ) ab 2 ab 2 a 2 b 2
u v
[b(a 2 + b 2 )-0] [0 + a(a 2 + b 2 )] a 3 b + ab 3
2
2
u = b(a + b ) a(a 2 +_
v= ----0.-_ b 2_'_)
a 3 b + ab 3 ' a 3 b + ab 3 .
1 1 1 1
=> u=- v=- =>-=- -=-
a' b· x a' y b
=> x =a,y=b.
Hence, x = a, y = b is the solution. Hence, x = 4, Y = 3 is the required solution.

Solve each of the following systems of equations by using the method of cross multiplication.
1. 2x - y - 3 = ° 2. 3x - 2y + 3 = °
4x+y-3 =0 4x+3y-47=0.
3. 2x+y=35 4. 3x-5y+25 =0
3x+4y=65 2x+y+ 10=0.
5. 4x-7y+28=0
5y-7x+9=0.
2 3 5 4
6. Solve - + - = 13 and ---=-2, wherex*O,y *0.
x y x y

7. ax+by=(a-b) 8. ~= l.
a b
bx-ay= (a + b) ax + by= if + b2 .
1 1
9. ~ + l. = (0 + b) 10. -+ - =7
a b x y
2 3
2.+1::..=2 -+ - = 17. (x* O,y*O).
a2 b2 • x y

ANSWERS
1. x=l,y=-l 2. x=5,y=9 3. x= 15,y= 5
125 -89 I 1
4. x= 41,y=TI 5. x=7,y= 8. 6.x=- v=-.
2 '. 3
7. x= l,y=-l 8. x = a,y = b 9. x=a 2,y=b2
I I
10. x= 4'y= 3".
Simultaneolls Linear Equation 79

HINTS TO THE SELECTED PROBLEMS

5. Given equations are:


4x-7y+28 = 0 ... (1 )
-7x+5y+9=0 ... (2)
By cross multiplication, we have
x y
-7 x 9-5 x 28 28 x (-7) - 9 x 4 4 x 5 - ( - 7)( -7)
x y =--
=>
-63 -140 -196-36 20-49·
x =--L=_I_
=>
-203 -232 -29
-203 -232
=> x = --=7andy= - - =8.
-29 -29
Hence, x = 7, y = 8 is the required solution.
1 1
6. Taking - = uand - =v.
x y
We get 2u+3v-13 = 0 ...(1)
5u-4v+2 = 0 ... (2)
8. The given equations are
bx-ay = 0
ax+by-(el+b 2) = o.
9. The given equations are
bx+ ay-ab (a+ b) = 0
b2 X + ely - 2el b2 = O.
1
10. Put = u and - =v.
x y

EXERCISE 2.3 ~~~~~~~~~


Solve
1. x-y=3 2. 2x+ 3y=0
3x-2y= 10 3x+4y=5.
3. 2x+y=7 4. l1x+ 15y+23=0
4x-3y=-1 7x-2y-20=0
5. 4x-3y=8 6. 0.4x-1.5y=6.5
29
6x-y= - O.3x + O.2y = 0.9
3
8
7.2x+5y=- 8. 31x+23y=39
3
5
3x-2y== -. 23x+31y= 15.
6
80 Remedial Mathematics

9 4
9. ax+ by=a- b 10. - - - = 8.
x Y
13 7
bx-ay= a+ b. - + - = 101, (x;toO,y;t:O)
x y
I I
It. -+-=3 12. 4x+ 6y= 3xy
7x 6y
I I
- - - =5 (x;toO,y;toO). 8x+9y=5xy.
2x 3y
x+y x-y
13. 3(x+3y)=11xy 14. --=2, - - =6, (x;toO,y;toO)
xy xy
3 (2x+y) = 7xy
12
15. - - - + - - - -
2(2x + 3y) 7(3x - 2y) 2
7 4
- - + - - =2 where (2x+ 3y);toO, (3x-2y);toO.
2x+3y 3x-2y

I ANSWERS
1. x =4,y= I 2. x = 15, Y = - 10. 3. x= 2,y= 3
3 2
4. x=2,y=-3. 5. x= 2,y=-3 6. x=5,y=-3.

1 1
7. x= -,y=-. 8. x= 2,y=-1 9. x=l,y=-1
2 3
1 1 1 1
10. x= -,y=-.
4 7 11. x= 14'y = 6' 12. (x=O,y=O)or(x=3,y=4)

13. (x=O,y=O)(x= l,y=3/2)

15. x = 2, Y = 1.

HINTS TO THE SELECTED PROBLEMS

3. Wehave(l)y=(7-2x)
substitutingy = (7 - 2x) in (2), we get
4x-3(7 -2x) = -1
=> 4x-21 +6x =-1
=> lOx = 20 => x=2
from (1): 2 x 2+y=7 y=3.
4. The given equations are l1x + 15 y = - 23 ... (1 )
7x-2y = 20 ... (2)
Multiplying (1) by 2 and (2) by 15 we get
22x+30y = -46 ...(3)
105x-30y = 300 ...(4)
Simultaneous Linear Equation 81

Adding (3) and (4), we get


254
127x = 254 => x= 127 =2.
Substituting x = 2 is (1) we get
11 x2+ 15y = -23
=> 15y = -45,y=-3.
1 1
10. put - =u, - =v.
x Y
1 1
11. put - =u, - =v.
x Y
12. Divide these equations by xy.
15. Let __1_ =uand __1_ =v.
2x+3y 3x+2y

• WORD PROBLEMS ON SIMULTANEOUS LINEAR EQUATIONS

Example 1: 7 audio cassettes and 3 video cassettes cost Rs 1395, while 5 audio cassetts
and 4 video cassettes cost Rs 1665. Find the cost of an audio cassette and that of a video
cassette.
Solution: Let the cost of each audio cassette be Rs x and that of each video cassette be Rs
y. Then
7x+3y = 1395 ...(1)
5x+4y = 1665 ...(2)
Multiplying (1) by 4 and (2) by 3 and subtracting, we get
13x = 585 => x=45.
substituting x = 45 in (1) we get
(7 x 45) + 3y = 1395
=> 3y = (1395-315)= 1080
y = 360
cost of 1 audio cassette = Rs 45 and
cost of 1 video cassette = Rs 360.
Example 2: Five years ago, a man was seven times as old as his son, andfive years hence,
the man s age will be three times his son s age. Find their present ages.
Solution: Let the present ages of the man and his son be x years and y years respectively.
The man's age 5 years ago = (x-5) years.
The son's age 5 years ago = (y-5) years.
.. (x-5) = 7 (y-5) => x-7y=-30. ... (1)
The man's age 5 years hence = (x + 5) years.
The son's age 5 years hence = (y + 5) years.
(x + 5) = 3 (y + 5)
82 Remedial Mathematics

=:> x-3y = 10 ...(2)


on subtracting (1) from (2)
4y =40 =:> y= 10.
puttingy = lOin (2), we get
x-3 x 10 = 10 =:> x =40.
:. the man's present age = 40 years.
:. the son's present age = 10 years.
Example 3: Four years ago mother was four times as old as her daughter. Six years later,
the mother will be two and half times as old as her daughter, form the pair of linear
equations for the situation and determine the present ages of mother and her daughter in
years, solving the linear equations by substitution method
Solution: Present age of mother = x years.
Present age of daughter = y years.
Four years ago.
The age of mother = (x - 4) years.
The age of daughter= (y - 4) years.
(x-4) = 4 (y-4)
=:> x-4y+ 12 = 0 ...(1)
Six years later:
The age of mother = (x + 6) years.
The age of daughter = (y + 6) years.

(x + 6) = 2 (y + 6)
2
2x-5y-18 =0 ... (2)
from (1) x =4y-12. ... (3)
Substituting x from (3) in (2), we get
2(4y-12)-15y-18 =0
or 8y-24-5y-18 =0
or 3y =42
or y = 14
substitutingy = 14 in (2) we get
x=4 x 14-12=:>x=44
Therefore age to mother = 44 years and the age of daughter = 14 years.
Example 4: A two digit number is 4 more than 6 times the sum of its digits. If 18 is sub-
tracted from the number, the digits are reversed. Find the number.
Solution: Let the tens digit of the required number be x and let its units digit be y.
Then the number = (lOx + y).
10x+y =6(x+y)+4.
=:> 4x-5y =4 ...(1)
Simultaneous Linear Equation 83

Number formed on reversing the digits = (lOy + x).


.. IOx+y-18 = lOy+x
=> 9 (x-y) =18 => x-y=2. ... (2)
Multiplying (2) by 5 and subtracting (1) from the result.
We get x = 6.
Substituting x = 6 in (2), we get
y = (6-2)=4
Thus x = 6 and y=4.
tens digit = 6 and units digit = 4
Hence, the required number = 64.
Example 5: Seven times a two digit number is equal to four times the number obtained by
reversing the order of its digits. If the difference the digits is 3, find the number.
Solution: Let the tens and units digits of the required number be x andy respectively.
Then, the number = 10 x + y. The number obtained reversing the digits = (10 y + x).
.. 7(10x+y) =4(10y+x)
=> 33 (2x-y) = 0 => 2x-y= 0
=> y=2x ... (1)
Thus, unit digit = 2 x tens digit (unit digit) > (tens digit) so y > x
y-x =3 ...(2)
using (1) is (2), we get
(2x-x) =3,x=3
On substituting x = 3 in (I), we gety = 2 x 3 = 6.
Hence, the required number = 36.
Example 6: The sum of the present ages of Kamal and his mothers is 89 years. After 11
years, mother s will be Kamal s age. Find their present ages. rUPTU B. Pharma 20011
Solution: Let the present age of Kamal be x years and that of his mother be y years. Then
according to the question, we have
x+y =89 ...(1 )
and (y + 11) = 2 (x + 11) or 2x - y = - 11 ...(2)
Adding (1) and (2) we get 3x =78
x =26.
putting x = 26 in (1), we get y = 89 -26 = 63.
Hence, Kamal's present age is 26 years and his mother's present age is 63 years.
Example 7: In a given fraction, if the number is multiplied by 3 and the denominator is

reduced by 1, we get ~, but if the numerator is increased by 12 and the denominator is


2
1
multiplied by 3, we get 3" find the fraction. IUPTU B. Pharma 2007j

Solution: Let the fraction be ~. Then according to question,


y
84 Remedial Mathematics

3x 3
=> 6x=3y-3
y-l 2
=> 6x-3y =-3
=> 2x-3y =-1 ...(1)
x + 12 1
and - - = - => 3x+36=3y
3y 3
=> 3x-3y = -36
=> x-y=-12 ...(2)
Subtracting (2) from (1), we get x = 11. putting x = 11 in (1), we get
2 x 11 - y = - 1 => Y = - 1- 22
=> Y = 23.
11
Hence, the required fraction is 23 .

Example 8: If the numerator of a fraction is multiplied by 2 and its denominator is in-


creased by I, it becomes I . However, if the numerator is increased by 4 and denominator
is multiplied by 2, then the ratio of the numerator and denominator is I : 2. from a pair of
linear equations for the problem and solve by substitution method and hence find the
fraction.
x
Solution: Let the given fraction be - . According to the given conditions.
y
2x x +4 1
= 1 and - - =-.
y +1 2y 2
=> 2x = y+ 1 and x+4=y.
Thus we get the required pair of linear equations:
2x-y-1 = 0 ... (1)
x-y+4 = o. ... (2)
from (1) Y = 2x-l ...(3)
substitutingy from (3) is (2), we get
x-(2x-I)+4 =0 => x=5.
substituting x = 5 in (3), we get
y =10-1=9.
5
Hence, the required fraction is "9'

~~~~~~~I EXERCISE 2.4~1~~~~~~~


1. Find two numbers such that the sum of twice the first and thrice the second is 92; and
four times the first exceeds seven times the second by 2.
2. If2 is added to each of two given numbers, their ratio becomes I: 2. However, if 4 is
subtracted from each of the given numbers, the ratio becomes 5 : II. Find the numbers.
Simultaneous Linear Equation 85

3. The monthly incomes of A and B are in the ratio 8 : 7 and their expenditures are in the
ratio 19 : 16. If each saves Rs 2500 per month, find the monthly income of each.
4. A mother is three times as old as her daughter five years later, the mother will be two
and a halftimes as old as her daughter, find the age of the daughter and that of her
mother in years.

5. A fraction becomes .!. if 5 is subtracted from its numerator and 3 is subtracted from its
2
denominator. However, if we divide the numerator by 2 and add 7 to the denominator,

the fraction becomes .!.. Determine the fraction.


4
6. Of the numbers, one number is greater than thrice the other number by 2 and 4 times
the smaller number exceeds the larger number by 5. Find the numbers.
7. The sum ofthe digits of a two digit number is 93. The number obtained by interchang-
ing the digits of the given number exceeds that number by 27, find the number.
8. Harsh purchased 4 chairs and 3 tables for Rs 1650. From the same place and at the
same rates_Sunit purchased 3 chair~ and 2 tables for Rs 1150. Find the cost per chair
and per table.
9. The sum of the digits of a two digit number is 12. Ifthe digits are reversed, the new
number is 12 less than twice the original number. Find the original number.
10. If 1 is added to both the numerator and the denominator of again fraction, it becomes

~. If however, 5 is subtracted from both the numerator and the denominator, the
fraction becomes 112. Find the fraction.

HINTS TO THE SELECTED PROBLEMS

3. Let the monthly income of A and B be Rs 8x and Rs 7x respectively and let their
expenditures be Rs 19y and Rs 16y respectively.
Then A's monthly saving = Rs (8x-19y).
and B's monthly saving = Rs (7x - 16y).
.. 8x-19y =2500 ... (1)
7x-16y =2500 ...(2)
6. Let the larger number = x and the smaller number = y.
Then according to the gives conditions, we have
(largernumber)-3 x (smallernumber) = 2
and 4 x (small number) - (larger number) = 5
x-3y =2 ...(1)
-x+4y =5 ...(2)
Adding (1) and (2) we gety = 7
substituting y = 7 in (i) we get
x-3 x 7 =2 ~ x=23.
the larger number = 23 and the smaller number = 7.
86 Remedial Mathematics

9. Let the tens digit and units digits be x and y respectively.


Then x+y =12 ... (1)
original number = lOx + y.
Number obtained on reversing the digits = (lOy + x).
lOy+x =2(lOx+y)-12
19x-8y =12 ... (2)

10. Let the required fraction be .::.. The according to the given condition, we have
y
x +1 4 x-5 1
- - = - and - - = -.
y +1 5 y-5 2
5 (x + 1) = 4 (y + 1) and 2 (x - 5) = (y - 5).
-5x-4y =-1 ... (1)
2x-y =5 ...(2)

ANSWERS

1. 25, 14 2. 34, 70 3. Rs 1200 and Rs 10500


4. Age of daughter = 15 years, Age of mother = 45 years.

3
5. 6. 23 and 7. 7. 58
7
8. cost of chair = Rs 150, costoftable=Rs350.
7
9. 48. 10. '9'

OBJECTIVE EVALUATION

MULTIPLE CHOICE QUESTIONS


. Choose the most appropriate Answers:
1. Solution of3x- 4y= I, 4x- 3y-6 = 0 is
(a) x=3,y=2 (b) x=2,y=3
(c) x=2,y=2 (d) x=3,y=3
2. The value ofk for which x - 2y = 3, Icy + 3x = I represent paralleilines is
(a) k= 6, (b) k=-6,
(c) k-ci' q, (d) k *-6.
3. The system.of linear equations 21'. - 5 = 3y and 4x - 6y = 3 represents a
(a) Intersecting lines. (b) Paralieilines.
(c-) Coincident lines. (d) None of these.
:Simultaneous Linear Equatioll 87

4. The system of linear equations 3x + 2y - 4 = 0 and ax -y - 3 = 0, will represent intersecting


lines if
(a) a =-3/2 (b) a *- 3/2
2
(c) a=-2/3 (d) a*- -.
3
5. If the system of equations 2" + 3y = 7 and (a + b) x + (2a - b) y = 21 has infinitely many
solutions then
(a) a = 1, b = 5 (b) a=5,b=1
(c) a = -I, b = 5 (d) a=5,b=-1.
6. The system of linear equations ax + by + e = 0 and xd + ey + f = 0 will represent coincident
lines if
(a) ae = bd and bf= ec. (b) ae = bd and bi ee. *
(c) ab=deandbc=ef (d) ab = de and be ef *
7. Equation of line which is prallel to J2 x - fj y = 5 is
(a) J8x-2 fjy=5 (b) J8x-2 fjy= I
(a) -J8x+2.Jjy=5 (d) -J8x+2 fjy=l.
8. The system of linear equations b = 3 (y - 3) and 6x - 9y = 5 represents a
(a) parallel line (b) coincident lines
(a) intersecting lines (d) none of these.

9. Equation of a line which is parallel to 3. x + 3y + I = 0 is


3
2 2
(a) - x+ 3y+ 1 = 0 (b) - x-3y+5=0
3 3
3 3 3 -
(c) -x+ 3y+ 1 = 0 (d) -x-y+)=O
2 2
10. Value of k for which 2 (k - 1) x + Y = I, 3x - y = I represents parallel lines is
(a) k* 5/2 (b) k= 5/2
(c) k= 112 (d) k*-1I2.

ANSWERS
1. (a) 2. (b) 3. (a) 4. (b) 5. (b) 6. (a) 7. (a) 7. (b)
8. (a) 9. (a) 10. (c)

FILL IN THE BLANKS


1. The lines b - 3y = 9 and kx - 9y = 18 will be parallel if k is - - -
2. Value of k for which
x - 2y = 5, 3x + ky + 15 = 0 is incoincident is - - -
3. The value ofp for which the syStem of equations 3x + 5y = 0 and px + lOy = 0 has a non zero
solution is - - -
4. The value of k for which the system of equations kx - 2 = y and - 2y 3 = 6x is consistent with
unique solution is - - -
88 Remedial Mathematics

5. Solution of.J2 x- ../5y= 0,2 J3 x- J7y=O i s - - - .


6. ax + by = c, Ix + my = n has a unique solution. i f - - -

7. The equation ~x- 5y+ 1 = 0 and ~ x + 3 = 5yare---


3 2
8. The system of equations
4x + py + 8 = 0 and 2x + 2y + 2 = 0 will have a unique solution i f - - -

ANSWERS
1. k= 6 2. k= 6. 3. 6*p. 4. k* 6.
5. x= O,y= 0 6. am * bl. 7. intersecting 8. p*4.

TRUE/FALSE
State whether given statement is true or false.
1. The lines 2x - 3y = 9 and 8 - 2y = x are parallel. (T/F)

2. The lines 5x - 1= 2y and y = - .!. + ~ x are c~incident. (T/F)


2 2
3. Equation ofline which is coincident to 5 (x - y) = 3 is lOx - 10 y - 3 = O. (T/F)
4. The lines 2 (x - 3) = 5y and 4x -1 = lOy represents intersecting lines. (TIF)
5. If 2x - 3y = 7 and (a + b)x - (a + b - 3)y = - 4a + b represent coincident lines then
a+5b=0. (T/F)

6. The solution of 2x - '"3y = 5 and ~ - 3y = 1 is x = 4, y = 1. (T/F)


4

ANSWERS

1. False. 2. True. 3. False 4. False.


5. False. 6. False.

REFRESHER

Do you know? After reading this chapter you should be able to know the following
concepts:
• The equation ofthe type ax + by + c = 0 or ax + by = c is called a linear equation in two
variables.
• A pair of linear equations in two variables which has a solution is called a consistent
pair oflinear equations.
• A pair oflinear equations in two variables which has no solution is called an inconsis-
tent pair of linear equations.
• A system of two linear equations in x and y has
(I) a unique solution if the graph lines intersect at a point.
(il) Infinitely many solutions if the two graph lines coincide.
(iii) No solution if the two graph lines are parallel.
Simultaneous Linear Equation 89

• Straight lines as graph of linear equations alx + bly + c l = 0 and ai' + b2y + c2 = 0
intersect each other at a point if
at
-*--
q
a2 b2
• Straight lines as graph of linear equations atx+ bty+ c l = 0 and ai' + b2Y+ c 2 = 0 and
parallel to each other if
al _ q Ct
---*--.
a2 b2 c2
• Straight lines as graph of linear equations atx + bty + c t = 0 and ai' + b2y + c2 = 0
coincide if

a2 b2 C2
Can we do? (Frequently Asked Questions)
1. Calculate the value of x and y when
4x-5y=3 andxy=2. [UPTU B. Pharma 2001]
2. In a given fraction, if the numerator is multiplied by 3 and the dinominator is reduced

by I, we get ~. But if the numerator is increased by 12 and the denominator is


2
1
multiplied by 3, we get "3' Find the fraction. [UPTU B. Pharma 2007]

3. The sum of the present ages of Kamal and his mother is 89 years. After 11 years,
mother's age will be twice Kamal's age. Find their present ages.
[UPTU B. Pharma 2007]
4. Solve the following system of linear equations by using the method of cross
multiplication.
x Y
-+- =(a+b)
a b

""::"'+L
2 2
=2
a b .
5. Show Graphically that the system of linear equations
2x-3y = 5, 6y-4x= 3.
has no solution.

DDD
DETERMINANTS

III INTRODUCTION
Consider two homogeneous linear equations
alx+b,y =0,
a2!+b7Y =0;
Multiplying the first equation by b2 , the second by bI' subtracting and dividing by x, we
obtained
a l b2 - a2b, =0
This result is sometimes written as

I: ~I =0
and the expression on the left is called the determinant.
A determinant also is an arrangement of numbers in rows and columns but it always has
a square form and can be reduced to a single value. Therefore, a determinant is distinct from
matrix in the sense that the determinant is always in square shape and it has a numerical
value. The arrangement of the numbers of a determinant is enclosed within two vertical
parallel lines.
Order of a Determinant
The determinant of a square matrix of order n is known as determinant of order n.

DETERNDNANTOFORDERTWO

Let all' a 12 , a21' a22 be any four number (real or complex). Then

IA 1= la,1 bl2l
a21 ~2
represent the number all a22 - a21 a l2 and is called a determinant of order two.
For example

IA I = I~ ~71 =(5)(-7)-(3)(2)
=-35-6=-41
Determinants 91

• DETERMINANT OF ORDER THREE

aIJ a12 al3


Let IA I = a21 a22 a23

a31 a32 a33

is called a determinant of order 3 and its value can be obtained as follows:

IAI =alll::~ :::I-a 12


I: : :::I+ I: : a13
::~I
=a ll (a22a33-a32a23)-a 12 (a2Ia33-a31a23)+ a 13 (a21a32-a3Ia22)
2 3 5
For example, IA I = -I 2 3
4 -2
2 1
=21 31_ 31- 31+41-1 21
-2 I 4 I 4-2
=2(2 +6)-3(-1-12)+4(2-8)
= 16+39-24=31
Remarks
• The value of a determinant is not changed if it is expanded a!~ng any row or column.
• When no reference of the corresponding matrix is needed, we may denote a determi-
nant by D.
• The determinant of a square zero matrix is zero.

~~~~~~~I SOLVED EXAMPLES ~I~~~~~~


cosa -sinal
Example 1: Find the value ofl A I ifA is given by .
Isma cosa

Solution: IA I _leo.
-
sma
sa -sinal
cosa
= cos2 ex - (- sin 2 ex) = cos 2 ex + sin 2 ex = 1.

Example 2: Find the value of J I


(J)
(J)
-(J)
I.

Solution: IA 1= II
(J)
(J)
-(J)
I =-ro-{02=_(ro+ro2)=(-I)= 1.
Example 3: Solve for x:

x 31 = 155 -413
I5 2x
92 Remedial Mathematics

Solution: We have
I~ :xl I~ ~41 =

=> 2~ - IS = IS + 20
=> 2x2=SO => x 2 =2S=> x=±S.
a h g
Example 4: Find the value of h b f.
g f e
a h g
Solution: Let /1 = h b f
g f e
We expand /1 along first row, we get

/1=a(-1)21; ~1+h(-l)31; ~1+g(-1)41;;1


= a (be- f2)-h (he -fg) + g (hf- bg)
= abc - af2 - eh 2 + fgh + fgh + fgh - bi
= abc + 2fgh - a/- bi - eh2
o 1 sec 8
Example 5: Find the value of tan8 -seeS tan8
101
o sec8
Solution: Let/1= tanS -secS tanS
o
expand along Rl

/1=0+1(-1) 3 1tan 8 tan 81 +secS(-I) 4 \tan 8 - sec 8\


1 1 1 0
= - (tan 8 - tan 8) + sec S (0 + sec S) = sec S (0 + sec S) = sec 2 S.

__ CO-FACTORS AND MINORS OF ANELEMENT

If in the expansion of a determinant I aijl, all the containing aij as a factor, are collected and
their sum a is denoted by aijAi; then the factor Aij is called the co-factor of the element a , ..
Hence, in a determinant of order n 1J

n
laijl =ai1AI1+ai2A,2+····+a,~in= 'L,aijAij
j-l
Now, let ~J be the (n-l) x (n -1) sub-matrix ofl a y In xn obtained by deleting the ith row
andjth column. Then IMijl is called the minor of the element aij the determinant layl of order n.
Thus we can express the determinant as a linear combination ofthe minors of the elements of
any row or any column.
Determinants 93

Remark
• (- 1)1 +j is 1 or -1 according as t + j is even or odd
:. Ai) and Mi} coincides if i +j is even and if i + j is odd then we have Ai} = -Mi)'

~~~~~~~I SOLVED EXAMPLES ~I~~~~~~

Example 1: Find the minors and cofactors ofelements of the determinant I~ ~ I


Solution: Minor of the element all is Mil = 171 = 7
Minor of the element a l2 is MI2 =3
Minor of the element a 21 is M21 =-2
Minor of the elem~nt a 22 is M22 =5
Hence, A II = (-1) I + I Mil =7
Al2 = (_1)1+2 MI2 =-3
A21 =(-li+ M21
1=2
2
A22 = (-li+ M22 =5
Example 2: Find all the minors and cofactors ofthe elements infollowing determinants
4.3 J
J 3 2
2 J 5
Solution: Here all = 4a l2 = 3al3 = 1
a 21 = la22 = 3a23 = 2
a 31 = - 2a32 = 1a 33 = 5

Mil = I~ ~I = 15-2= 13 M l2 = I~ ~I =5-4= 1


Ml3 = I~ ~I = 1 - 6 = - 5 M21 = I~ ~I = 15 -1 = 14
M22 = I~ ~I = 20- 2 = 18 M23 = I~ ~I =4-6=-2
M31 = I~ ~I =6-3 =3 M32 = I~ 211 =8-1=7
'

M33 =I~ ~1=12-3=9.


The co-factors are
All =(_1)1+1 Mil = 1 x 13 = 13 Al2 =(-1) 1+2 Ml2=-lx 1 =-1
Al3 =(_1)1+3 Ml3 = Ix (-5) =-5 2+1
A21 = (-1 ) M21 =-lx 14 =-14
A22 =(_1)2+2 M 22 = Ix 18= 18 A 23 =(_1)2+3 M23=-lx (-2)=2
A31 =(-1)3+IM31 =l x 3=3 A32 = (_1)3+2 M32 =-1 x 7 =-7
A33 =(-1)3+3M33 =l x 9=9.
94 Remedial Mathematics

Example 3: Find the minor and co-factors ofelements ofthe following determinant
2 -3 5
6 0 4
1 5 -7
Solution: We have
0
M1·1 -- 5 41 = 0-20 =-20 All =-20
1 -7

M12 = I~ 41 =-42-4=-46
-7

Ml3 = I~ ~I =30-0=30 Al3 =30

3
M2l 51 =21-25 =-4
= 1-5 -7 A2l =4

M22 = I~ 51 =-14-5 =-19


-7
A22 =-19

M 23 = I~ -31
5 =10+3=13 A 23 =-13

3
M31 = 1-0 :1 =-12-0=-12 A3l =-12

M32 = I~ :1 = 8-30= -22 A32 = 22

M33 =I~ -31


0=0+18=18 A33 =-18.

Example 4: Write the co-factors ofelements ofthe second row ofthe following determinants
and hence evaluate them
1 a be
1 b ca
1 c ab

a be
Solution: Let ~ = 1 b ca
1 c ab

A 21 =(-l)2+1l ac
2
A22 = C-l + 11iab
1
=ab-bc bel
Determinants 95

• PROPERTIES OF DETERMINANTS

Theorem 1: The value of a determinant does not change when rows and columns are
interchanged.
al bl cI
Proof: Let I A I= a2 b2 c2 be a determinant of order three.
a3 ~ c3
Expanding I A I along the first row, we get
IA I = a l (b 2c 3 - b 3c 2) - b l (a 2c 3 - a 3c 2) + c I (a 2b 3 - a 3b 2)
= a l (b 2c 3- b3c2) - a 2 (b l c3 - a3c l ) + a 3 (b l c2 - b 2c l )
(by rearrangement of terms)

CI c3 C2
Hence, the theorem is proved.
Theorem 2: Ifany two rows [or columnsJ ofa determinant are interchanged, the sign ofthe
determinant is changed.
al bl cI
Proof: Let I A I~ lb._ j/2., c2 be a detenninant of order three.
t.
I I
a3 bJ '

Expanding A along the first row, we get


c3

IA I = a l (b 2c 3 - b3c2) - b l (a2c 3 - a 3c 2) + c I (a2b3 - a 3b2)


= - {a 3 (b 2c l - b l c2) - b3 (a2c I - a l c2) + c 3 (a2b l - a l b2)}
(by rearrangement ofterms)
al a2 a3
=- q b2 ~ =(-I)IA I

CI c2 c3
Theorem 3: If two rows or two columns ofthe determinant are identical, then the value of
the determinant vanishes, i.e.,
al bl cI

I A 1= a2 b2 c2 =o.
al bl cI
96 Remedial Mathematics

Proof: We have I A I is a determinant of order 3 whose first and third row are identical. If we
interchange the two identical rows, then obviously there will be no change in the value of
1A I. But by theorem 2, the value ofA ismu'ltiplied by-l if we interchange two rows. Therefore,
we get
IAI=-IAI
21 A = 0 or A = 0
1 I 1

Theorem 4: Ifall the elements ofany row, or any column, ofa determinant are multiplied by
the same number then the determinant is multiplied by that number.
all a12
a21 a22
Proof: Let IAI = be a determinant of order n

ani a n2
mall al2
ma21 a22
We have

manl a n2
(where Ail' A i2 ...A m be the cofactor of elements ail' a i2 , ... am of ith row ofl A I)
Theorem 5: If in the determinant, the elements of a row are added in and m times the
corresponding elements ofthe another rows (or column), the value ofthe determinant does
not change in particular,
al + mb.. + nCI b.. cI al b.. cI
a2 + mb2 + nC2 b2 c2 a2 b2 c2

a3 +m~ +nc3 ~ c3 a3 b3 c3
Proof: We have
al +mbl +ncI b.. cl al bl cI mb.. bl cI nCI b.. cI
a2 +m~ +nc2 b2 c2 a2 b2 c2 + mb2 b2 c2 + nC2 b2 c2
a3 +m~ +nc3 ~ c3 a3 b3 c3 m~ b3 c3 nC3 ~ c3

al bl cI b.. b.. ci cI b.. cI


a2 b2 c2 +m b2 b2 c2 +n c2 b2 c2

a3 b3 c3 ~ b3 c3 c3 ~ c3
(By theorem 4)
al b.. cI
a2 b2 c2 +m (O)+n(O) (By theorem 3)
a3 b3 c3

al b.. cI

a2 b2 c2'
a3 b3 C3
Determinants 97

~~~~~~~I SOLVED EXAMPLES I~~~~~~~


Example 1: Evaluate the following determinant

I~ -:1
Solution: We have IA I= I! ~21 = 3 x 5 - 4 x (-2) = 15 + 8 = 23

Example 2: Find the value ofthe determinant ofthe matrix

A ~ [~ ~l ;

123
Solution: We have IAI=231
312
On expanding the detenninant along the first row, we get

= 1 I~ ~I-21~ ~I + 31~ ~I
= 1. (6-1)-2. (4-3) + 3. (2 -9) =-18
4 J 4
Example 3: Evaluate the determinant of 0 J O.
J 2 J
4 4
Solution: We have I A 1= 0 0
1 2
On expanding the detenninants along first column, we get

=41~ ~I-ol~ ~I + ll~ ~I


=4(1-0)-0+ I (0-4)
=4-4=0.
Example 4: Show that:
J x Y
0 cosx siny = cos (x + y)
0 sinx cosy
I x y
Solution: We have 0 cos x siny
0 sinx cosy
98 Remedial Mathematics

On expanding the determinant along first column, we get

= 1 cosx sinyl -0 I x y I+ 0 x I
Isinx cosy sinx siny cosx
= cos x cos y - sin x sin y = cos (x + y)
I I I
Example 5: Show that I l+x I =xy
I I l+y
I 1 .
Solution: We have L.H.S. = I +x
1+ y
Applying C2 - C1 and C3 - C] in the given determinant, we get
1 0 0
= 1 x 0
lOy
On expanding the determinant along the first row, we get

=ll~ ~I-ol; ~I-ol; ~I =xy=R.H.S.


Example 6: Without expanding, show that
b-e e-a a-b
e-a a-b b-e = 0
a-b b-e e-a
Solution: We have
b-e e-a a-b 0 e-a a-b
e-a a-b b-e 0 a-b b-c
a-b b-e e-a 0 b-e c-a
(Operating C1 ~ C1 + C2 + C3 , we get) = 0
Example 7: Without expanding, show that
b 2 e2 be b+e
2 2
e a ea e+a =0
2 2
a b ab a+b
Solution: Consider
b2 c 2 be b + e b2 e2 be b+e
2 2 abc 2 2
e a ea e + a e a ea e+a
abc
a2 b 2 ab a + b a2 b 2 ab a+b
(Multiplying RJ by a, R2 by band R3 by c)
Determinants 99

ab 2 c 2 abc ab + ca
= _1_ bc 2 a2 abc bc + ab (Take abc out from C I and C2 )
abc 2 2
ca b abc ca + bc
bc ab+ca
abc.abc
ca bc+ab
abc
ca ca+bc
bc ab +bc +ca
= abc ca ab +bc +ab
cb ab +ca+bc
bc
= abc (ab + bc + ca) C{:J

cb
= abc (ab + bc + ca) x 0 = 0
x+l x+2 x+a
Example 8: Ifa, b, c are in A.P. prove that x + 2 x +3 x +b = 0
x+3 x+4 x+c
Solution: Given a, b, care inA.P. therefore a + c = 2b
=>a+c-2b=0
Operating Rl ~ Rl + R3 - 2R2 , we get
x+ 1 x+2 x+a 0 0 a+c-2b
x+2 x+3 x+b x+2 x+3 x+b
x+3 x+4 x+c x+3 x+4 x+c
0 0 0
=x+2 x+3 x+b =0
x+3 x+4 x+c
Example 9: Prove that
a b c 1 1 J
2 2 2
a b c = abc a b c = abc (a - b) (b - c) (c - a)
3 3 3 2 2 2
a b c a b c
a b c 1
2 2
Solution: We have I A 1= a b c 2 = abc a b c
a3 b 3 c3 a2 b 2
c2

1
Now again IAI = abc a b c
2
a 2
h c2
100 Remedial Mathematics

Applying C2 - C t and C3 - C t , we get


o 0
= abc a b- a c- a
a2 b2 _ a2 c2 _ a2
On expanding along the first row, we get

= abc 1 ~
- a2
b -a
~
- a 21
c-a
= abc [(b - a) (c 2 -~) - (b 2 - a2) (c - a)]
= abc [(b -a)(c-a) {(c + a) - (b + a)}]
= abc (b-a) (c-a) (c+ a- b-a)
= abc (a- b)(b - c)(c-a)
Example 10: Prove that
a + b + 2c a b
c b+c + 2a b = 2 (a + b + c/
c a c + a + 2b
a +b +2c a b
Solution: Let IAI = c b +c + 2a b
c a c + a +2b
Adding C2 and C3 in Cl' we get
2(a +b +c) a b
=2(a+b+c) b+c+2a b
2(a +b +c) a c+a+2b
a b
=2(a+b+c) 1 b+c+2a _ b
1 a c + a + 2b
Applying (R2 - R t ) and (R3 - R t ), we get
1 a b
=2(a+b+c) 0 b+c+a 0
o O· c+a+b
On expanding the determinant along the first column, we get

=2(a+b+C)\b+c+a 0 \
o a + b+'c
=2(a+b+c)(a+b+ci
=2(a+b+d
Determinallls 101

Example 11: Prove that

( 1 1 1)
1+a 1 1
1 1+b 1 = abc 1 + - + - +- [VPTV B. Pharma 2000, 06J
abc
1 1 1+e
Solution: Operating C]~ C] - C3 and C2 ~ C2- C3 , we get
I+a I I a 0
l+b = 0 b
1 l+e -e -e l+e
=a[b.(l +e)-(-e).l]+ 1 [0. (-c)-(-c)b]
= a (b + be + c) + be

= abc + be + ea + ab = abc ( 1 + -1 + -1 + -1 )
abc
Example 12: Prove that
a-b-e 2a 2a
2b b-e-a 2b =(a+b+el
2e 2e e-a-b
Solution: Operating R] ~ R] + R2 + R3, we get
a- b- e 2a 2a a + b +e a + b + e a + b + e
2b b-e-a 2b 2b b-e-a 2b
2e 2e e-a-b 2e 2e e-a-b
[Take (a + b + c) out from Rd

=(a+b+e) 2b b-e-a 2b
2e 2e e-a-b
(Operate C2 ~ C2 - C] and C3 ~ C3 - C])
o 0
=(a+b+e) 2b -b-e-a o (expand by R])
2e 0 a-b-e
=(a+b+e) 1 (-a-b-e)(-a-b-e)=(a+b+e)3
Example 13: Without expanding the determinant, show that
0 b -e
-b 0 a =0. [UP TV B. Pharma 20011
e -a 0
0 b -e
Solution: Let.1 = -b 0 a
e -a 0
102 Remedial Mathematics

By changing columns into rows:


0 -b e 0 b -e
A = b 0 -a =(_1)3 -b 0 a
-e a 0 e -a 0
(taking (-I) Common from each column)
= (_1)3 A =-A.
2 A =OorA=O.
Example 14: Without expanding the determinant, show that

I a be I a a2
I b ca I b b2 and evaluate it.
I e ab I e c2
[UP TU B. Pharma 2001,20081
1 a be
Solution: Let A = 1 b ea
I c ab
Multiplying the 1st, 2nd and 3rd rows by a, b, c respectively. We get

a a2 abc a a2
I abc b b2
A=- b b2 bca =
abc abc
c e2 abc c e2
Taking abc common from 3rd column

a I a2
b b 2 applying C2 ~ C3
C 1 e2

1 a a2
b b 2 applying C 1 ~ C2.
I c e2

a a2
b-a b 2 _a 2
A = 0 b-a b 2 _a 2
e-a c2 _a 2
0 e-a e 2 _a 2
on expanding the determinant along C 1

= (b-a)(e-a) 1 b + al
11 e + a
Delerminants 103

taking (b - a) common from R\ and (c - a) common from R2


= (b -a)(c-a) [c + a- (b + a)] = (b -a)(c-a) (c- b)
= (a- b}(b -c)(c-a).
Example 15: Without expanding the determinant show that (a + b + c) is a factor oj
[ollowing determinant.
abc
A= b c a [UPTU B. Pharma 20031
cab
If a, b, and c are positive and unequal, show that the value of A is always negative.
Solution: Applying C\ ~ C\+ C 3, we get
a+b+c b c b c
~ = a+b+c c a =(a+b+c) 1 c a
a+b+c a b a b
b c
=(a+b+c) 0 c-b a-c
o a-b b-c
Applying R2 ~ R2 - R\, R3 ~ R3 - R\

=(a+b+c) IC-b a-cl


a-b b-c
= (a+ b + c) {-(b-d-(a-b)(a-c)}
= (a + b + c) (- ~ - b2 - c2 + ab + bc + ca)
Thus (a + b + c) is 0 factor of~. Now we shall prove the next part.
we have ~ = (a + b + c) (-<1 _b 2-c2 + ab + bc + ca)

= -.!.. (a+b+c)(2~+2b2+2c2-2ab-2bc-2ca)
2
= -.!.. (a+b+c)[(a-bi+(b-cl+(c-ai]
2
< 0, since a, b, c are positive and unequal. Therefore, the gives determinant is always
negative if a, b, c are positive and unequal.
Example 16: Show that
a b c
a-b b-c c-a
b+c c+a a+b
Solution: Operating R2 ~ R2 - R\ and R3 ~ R3 + R\, we get
abc abc
a-b b-c c-a -b -a -c
b+c c+a a+b a+b+c a+b+c a+b+c
104 Remedial Mathematics

[Take (a+ b + c) out from R3 and (-1) fromR 2)


abc
=-(a+b+e). b e a (expandbyR 3)
111
=-(a+ b + c). [1. (ab -e2)-I(~-be) + 1. (ea-b 2)]
= - (a + b + c). (ab + be + ea - a2 _b 2 - e2)
=-(a+ b + c). (~+ b2 + e2 -ab-be-ea)
= a3 + b3 + e3 - 3abe
Example 17: Find the value o/x if
3+x 5 2
1 7+x 6 =0
2 5 3+x
3+x 5 2
Solution: We have 7+x 6 =0
2 5 3+x
Applying (R 1- R3), we get
l+x 0 -1-x
1 7+x 6 =0
2 5 3+x
Applying C3 -+ C3 + Cl' we get
l+x 0 0
7 +x 7 =0
2 5 5+x
On expanding the determinant along the first row, we get

(l+X)i +
7 X
71 =0
5 5+x
(l +x)[(7+x)(5 +x)-35] =0
or (1 +x)(;+ 12x) =0
x(1 + x)(x+ 12) =0
x=O,-I, -12.
Example 18: Evaluate:
3 2 1 4
15 29 2 14
IA I 16 19 3 17
23 39 8 38
Determinants 105

Solution: Applying C 1 ~ C J - 3C2 , C2 ~ C2 - 3C3 , C4 ~ C4 -4C3 , we get


o 0 1 0
9 25 2 6
IAI = 7 13 3 5
9 23 8 6
On expanding the determinant along first row, we get
6 9 25
=17 13 5
9 23 6
Applying R) ~ R 1- R3, we get
020
=17 13 5
9 23 6
On expanding the determinant along the first row, we get

=-21: !I =-2(42-45)=6

Example 19: Usingproperties ofdeterminants, solve thefollowingdeterminantforx.


a+x a-x a-x
a-x a+x a-x
a-x a-x a+x
a+x a-x a-x
Solution: Given a-x a+x a-x =0
a-x a-x a+x
3a-x a-x a-x
3a-x a+x a-x =0
3a-x a-x a+x
a-x a-x
=> (3a-x) 1 a+x a-x =0
a-x a+x
1 a-x a-x
(3 a-x) 0 2x 0 =0
0 0 2x
2X
=> (3a-x).1. 0 2°xl =0
1

=> (3a-x). (4~-0) =0


106 Remedial Mathematics

=> 4x2 (3a-x) =0


2
=> X = 0 or 3a-x=0
=> x =0,0,3a
Hence, the values of x are 0, 0, 3a.
Example 20: Using properties of determinants, prove that

I~ ~ ~ = (a- fJ) (fJ- r) (y- a)


Il3y ya al3
Solution: Operate C 2 -+ C2 - C j and C3 -+ C3 - C j , we get
o 0
a 13 y = a l3-a y-a
l3y ya al3 l3y y(a-l3) l3(a-y)
(Take (a - ~) out from C2 and (y - a) out from C3 ]
1 0 o
=(a-l3)(y-a) a -1 (expand by C j )
l3y Y -13

~131
-1
=(a-l3)(y-a).l. y
1
= (a-l3) (y-a) (13 -y)
= (a-l3) (13 -y) (y-a)
Example 21: Prove that
a2 + 1 ab ae
ab 2
b +1 be = 1 + ~ + b2 + e 2
ae be e2 + 1

a2 + 1 ab ae
2
Solution: We have 1 A 1= ab b +1 be
2
ae be e +1
nd
Now multiply the column 1st, 2 and 3rd by a, b-and c respectively, we get
a(a 2 + 1) ab 2 ac 2

IA 1= 1
abc
a2 b b(b 2 + 1)
2
(c(c + 1)
To take a, b, c common from 1st, 2 nd and 3rd rows respectively, we get
a2 + 1 b2 c2
abc
=-
abe
Determillallts 107

Now apply C, ~ C, + C2 + C3 , we get

a 2 + b2 +c 2 + 1 b2 c2
2 2
a + b + c 2 + I b2 + I c2
a 2 + b2 + c 2 + I b2 2
c +1

b2 c2
= (a2 + b2 + c2 + 1) 1 b 2 + 1 c2
b2 2
c +1
Now applying R2 ~ R2 - R, and R3 ~ R3 - R" we get

= (a2 + b 2 + c2 + 1) I~ ~I
= a2 + b 2 + c2 + 1
= a2 + b 2 + c2 + 1
b+c c+a a+b abc
Example 22: Prove that q + r r+p p +q = 2 P q r
y+z z+x x+y x y z
Solution: We have
b+c c+a a+b
L.H.S. = q + r r + p p +q
y+z z+x x+y
Applying C, ~ C, +C 2 -2C3 , we get
2c c + a a + b c c+a a +b
= 2r r + p P+q =2 r r+p P+q
2z z +x x +y z z+x x+y
Now applying C2 ~ C2 - C 1, we get
c a a+b
=2 r p p+q
z x x+y
Applying C 3 ~ C 3 - C2 , we get
cab abc
=2 r p q =2 P q r (byInterchangingthecolumns)=R.H.S.
z x y x y z
X x2 1+ x3
Example 23: If x,y, z are all different and y y2 1+ l =0
z z2 1 + z3
show that xyz = - 1.
108 Remedial Mathematics

x X
2 1+ x 3
Solution: Given y y2 I+i =0
z z2 1+ z3

x x2 x x2 x3
Y y2 1+ y i y3 =0

z z2 Z z2 z3

[Take x, y, z out from R J, R2 and R3 respectively from the second determinant]

=> Y y2 + xyz I y i = 0

I x x2
=> y i (1 +xyz) =0
I z z2

=> (x-y). (y-z). (z-x). (l+xyz) =0


=> (1 +xyz) =0
(Because x, y, z are distinct, so x - y ¢ 0, Y - z ¢ 0, z - x ¢ 0).
=> xyz =-1
Example 24: Evaluate the value of x for which
4x 6x + 2 8x + 1
6x + 2 9x + 3 12x = 0
8x+l 12x 16x+2
4x 6x +2 8x + 1
Solution: We have 6x + 2 9x + 3 12x
=0
8x + I 12x 16x + 2

Applying ( C2 ~ C2 -%CJ ) and C3 ~ C3 - 2C 1, we get

4x 2 I
6x + 2 0 -4 =0
8x + 1 -3/2 0
Now appl?,ing R2 ~ R2 + 4R J
4x 2
=> 22x+2 8 0 =0
8x+ 1 -3/2 0
Determinants 109

On expanding the determinants along 3rd column, we get

II22X + 2 8
8x + 1 -3/2
=0 I
=> -33x-3 -64x-8 =0
-II
or -97x= 11 orx =-
97
Example 25: Without expanding show that the value ofthe determinant given below is zero
sin a. cos a. sin (a. + 0)
sin J3 cos J3 sin(J3 + 0)
sin y cos y sin(y + 0)
sin a. cos a. sin(a. + 0)
Solution: Let ~ = sin J3 cos J3 sin(J3 + 0)
sin y cos y sin( y + 0)
Using sin (A + B) = sin A cos B + cos A sin B
sin a. cos a. sin a. cos 0 + cos a. sin 0
~ = sin J3 cos J3 sin J3 cos 0 + cos J3 sin 0
sin y cosy sin ycoso + cosysino
sina. cosa. 0
sinJ3 cosJ3 0 UsingC3~C3(coso)Cl-(sino)C2=0
siny cosy 0
Example 26: Show that
(b+c)2 a 2 be
(c + ai b2 ca = (cl + b2 + c2) (a + b + c) (b - c) (c - a) (a-I
(a + b)2 c2 ab

(b + ci a2 be
Solution: Let (c + a)2 b 2 ca
(a+bi c2 ab
110 Remedial Mathematics

Operating R2 ~ R2 - R\ and R3 ~ R3 - R2
a2 be
2 2 2
=(c?+b +e2 ) 0 b _a (ea-be)
2 2
o e -a (ab-be)
2
a be
2 2
=rc?+b +e )(b-a)(e-a) 0 b+a -c
o e+ a -b

c?
2 2
=(c?+b +e )(b-a)(e-a) 0 b+a
o e-a

a2 be
2 2
= (c? + b +e )(b-a)(e-a)(e-b) 0 b + a -e
o
Expanding along first column, we get
A= (c?+ b2 + e2 ) (b-a) (e-a) (e-b) (a+ b + e)
Example 27: Show that
a+b b+e e+a abe
b+e e+a a+b =2 b e a
e+a a+b b+e e a b

a+b b+e e+a


Solution: Let A = b+e e+a a+b
e+a a+b b+e
Applying C\ ~ C\+ C2 + C3, we get
2(a +b + e) b+e e+a
= 2(a +b + e)
e +a a +b
2(a+b+a) a+b b+e
a +b +e -a -b
=2 a+b+e -b -e .fpplyingC2~C2-C\,C3~C3-CI
a+b+e -e -a
a+b+e a b
We get =2(-1)(-1) a+b+e b e
a+b+e e a
Determinants 111

Applying C 1 ~ C 1- Cr C3, we get


cab
=2 a b c
b c a
a c b abc
= 2 b a c (C 1 ~ C 2 ) = 2 b c a
c b a c b a
Example 28: Ifa, b, c (all positive) are the pth, lh and /h terms respectively ofa geometric
progression, show that
loga p I
10gb q I = 0
logc r I
Solution: Consider the terms of GP. which are A, AR, AR2 ,....
a =T =AW'-l
p
b =T =ARq-l
q
c = Tr =ARr - 1

loga p logARP-l p
Consider log b q logARq-l q
logc r logAR r - 1 r 1

logA+(p-l)logR p
log A + (q -1) log R q 1
log A + (r -1) log R r 1

logA p 1 (p-I)logR P
logA q 1 + (q-l)logR q 1
logA r (r-l)logR r 1
p p-l p
=logA q 1 + log R q -1 q
r 1 r-l r 1

p P
=logAxO+logR q q 1 =O+logRxO=O
r r I
112 Remedial Mathematics

Evaluate the following determinants (1 to 7):


1 8
1. 2 2.1-2 31 3.lcose -sinel
4 -9 sine cose
4 2
I 0 6 23 12 11
4. IX2 -x + I X-II 5. 3 4 IS 6. 36 10 26
x+l x +I
5 6 21 63 26 37
3 -4
7. 3 2 5
I 3
write the minor and co-factor of each element of the following determinants and also
evaluate the determinants in each case (8 to 11):
3 -2

8·1~ -~Ol 9. 4 -5
3 5
6
2
0 0 04
0 0 11. 3 5 -I
0 0 0 2
x+l x+2 x+4 1 a be
12. Evaluate x-l;5 x+6 x+8 13. Evaluate I b ea
x+7 x+lO x+14 e ab
x+A. x x b+e a a
14. Evaluate x x+A. x 15. Evaluate b e+a b
x x x+A. e e a+b

16. Prove that


y i = (x-y) (x-y)(z-x)
1 z z2

_a 2 ab ae
17. Prove that ba _b 2 be = 4ifb 2e2
ac be -c2

x y2 yz
18. Prove that y y2 zx = (x- y) (y-z) (z -x) (xy + yz + zx)
z z2 xy
Determinants 113

19. Using properties of detenninants, prove that


y+z x y
z+x z x =(x+y+z)(x-zi
x+y y z
20. Using properties of detenninants, prove that
a-b-c 2a 2a
2b b-c-a 2b =(a+b+c)3
2c 2c c-a-b
21. Solve the following detenninants
x-2 2x-3 3x-4
x-4 2x-9 3x-I6 = 0 [UP TU B. Pharma 2007)
x-8 2x-27 3x-64
22. Prove that using properties of detenninants
1 + a2 _b 2 2ab -2b
2ab
2b
23. Prove that

X x 2 I+px 3
y i 1 + pi =(1 +pxyz)(x-y) (y-z)(z-x)
z z2 1 + pz3

24. Prove that using properties of detenninants


3a -a+b -a +c
-b+a 3b -b+c =3(a+b+c)(ab+bc+ca)
-c + a -c +b 3c
25. Prove that
sina cosa cos(a + 0)
sin f3 cos f3 co'S(f3 + 0) = 0
sin y cosy cos(y + 0)

HINTS TO THE SELECTED PROBLEMS

(1). 11/ 2 81 = -I x 2 - 8 x 4 = 1 - 32 = - 31.


4 2 2

cose -sinel 2 2
3. We have = cos e + sin e = 1.
I
sine cose
114 Remedial Mathematics

4. On expanding, we get => (x2 -x + 1) (x-I) - (x-I) (x + I )


2
= (x-l)(x -x+ l-x-l)
=(x-l)(x2 -2x)
=x3 _2x 2 _x 2 + 2x
1 A 1 = x 3 - 3x2 + 2x
I 0 6 I 0 3.2 0 2
5.IAI= 3 4 15 = 3 4 3.5 =3 3 4 5
5 6 21 5 6 3.7 5 6 7
o 2 0 2
=33 2.2 5 =6 3 2 5 =6[(14-15)+2(9-10)]=-18.
5 2.3 7 5 3 7

8.1~ -~Ol
Minor ofthe element all is MIl = 131 = 3.
Minor ofthe element a l2 is MI2 = O.
Minor of the element a 21 is M21 = -10.
Minor of the element a22 is M22 = 5.
Hence cofactors are as All = (_1)1+1 M II = 3
AI2 =(_1)1+2 M =0
12
1
A21 =(-li+ M 21 = 10
A22 =(-li+2 M 22=5

IA 1 = I~ -~Ol = 15-0= 15.


9. Minor of all = 5
-5
1
~I =-40 Minor of a l2 = I; ~I =-10

Minor of a l3 = I; -51
5 =35 Mmor
.
3
of a 21 = 15 -21
2 = 16

Minor of a 22 = 31 -21
2 =8 Minor of a 23 = I~ ~I =-4
1
Minor of a 31 = 3 -21 =8 Minor of a 32 = I~ -21
6 = 14
1-5 6

Minorofa33 = I: ~51 =-17

Now Cofactors are:


A II =(-I)I+! Mll =-40
A 12= (_1)1+2 M 12 = 10
A 13= (_1)1+3 M13 =35
Determinants 115

A2! = (-li+! M2! = 16


2
A22 = (-li+ M22 = 8
A 23 = (-I )2+3 M 23 = 4
12. Applying the following operations and then expanding R3 -+ R3 - R I , R2 -+ R2 - R]
and C3 -+ C3 - C I , C2 -+ C2 - C I .
13. Applying R2 -+ R2 - RI and R3 -+ R3 - R]. And expanding along all' we get the
required result.
14. Applying RI -+ RI + R2 + R 3·
Then C2 -+ C2 - C I and C3 -+ C3 - C] and expanding, we get the required result.
17. Taking a, b, c common from the fIrst, second and third columns respectively, we get
-a a a -1
2 2
A = abc b -b b = ifb c 1 -I
c c -c 1 1 -I
Taking a,b, c common from 1st, 2 nd and 3rd row respectively.
I'
o 2 applying R2 -+ R2 + R 2, R3 -+ R3 + R]
2 0
=ifb c (_I)(-4)=4ifb 2c2.
2 2
18. MUltiplying the fIrst, second and third rows of the determinant on the L.H.S. by x,
y and z respectively. We get,
x 2 x 3 xyz
1
xyz
i l xyz
z2 z3 xyz

x2 x3
= xyz
i l x2 i z2
xyz
z2 z3 x3 l z
3

Applying C2 -+ C2 - CI' C3 -+ C3 - CI' to the determinant. We get


1 0 0
x2 y2 _x 2 z2 _x 2
3
X Y 3 -x3 3
z-x 3

(y - x )(y + x) (z - x )( z + x) I
I
= (y-x)(i +xy+x2) (z-x)(z2 +zx+x2)

y+x
= CJ.;-x) (z-x) 2 2
IY +xy+x
116 Remedial Mathematics

[Taking (y-x) common from the first column and (z -x) from the second column]
Now Applying C2 ~ C2 - C?I
y+x
We get =:> (y-x)(z-x) 2 2
z-y 1

1
Y +xy+x (z2 - y2) + zx-xy

y+x z-y 1
= (y-x)(z-x) 2 2
Y + xy + x
1
(z - y)(x + y + z)

= (y-x)(z-x)(z-y) 12 y+x 2 1 1
y +xy+x x+ y+z
= (x-y) (y-z) (z-x) (xy+ yz+ zx).
20. Applying RI ~ R] + R2 + R3
C2 ~C2-C]
C3 ~ C2 - C I we get the required result.
21. Applying R2 ~ R2 - Rl' R3 ~ R3 - R I, the given equation becomes
x-2 2x-3 3x-4
-2 -6 -12 =0.
-6 -24 -60
x-2 2x-3 3x-4
or 3 6 =0.
4 10
Expanding the determinant along the first row, the above equation becomes:
(x-2) [30-24] -{2x-3) [10-6] + (3x-4)(4 -3) = 0
6x-12-8x+ 12 +3x-4 =0, x=4
x x 2 1 + px3 X x2 x x 2 px3
1 + py3 = Y 2 3
23. Y i i 1+ Y Y ;Py .
z2 1 + pz3 z2 2 pz3
Z Z 1 Z Z

X x2 x px 2
Y i 1 +xyz 1 y pi
z z2 z pz2

X x2 1 X x2
= y i l+pxyzl Y y2
z z2 1 z z2
Determinants 117

x X2

=(1 +pxyz) I y i
I z Z2

= (1 + pxyZ)(X-Y) (y-z)(z-x).
sin a cos a cos( a + 0) sin a cos a cos a cos 0 - sin a sin 0
25. sin ~ cos ~ cos(~+ 0) = sin ~ cos ~ cos ~ cos 0 - sin ~ sin 0
siny cosy cos(y+o) siny cosy cosycoso-sinysino
Applying C3 ~ C3 + (sin 0) CI-(cos 0) C2 . We get
sina cosa °
sin~ cos~ =0.
siny cosy °
°
ANSWERS
1. -31 2. 6 3. 1
5. -18 6.
8. Mll =3,MI2 =O,M21 =-IO,M22 =5
° 7. 49

All =3,A I2 =0,A 21 = IO,A 22 =5, 15


9. MIl =-40,MI2=-lO,Ml3=35,M21 = I6,M22 =8,
M23 =-4, M31 = 8. M32 = 14, M33 =-17
All =-40,A 12 = lO,A l3 =35,A 21 =-16,A 22 =8,A 23 =4
A31 =8,A 32 =-I4,A 33 =-17;-80
10. MIl = 1, M12 =0, M13 =O,M21 =0,M22 = I,M23 =0,
M31 =0, M32=O, M33= 1
All = I,AI2=0,A13=0,A21 =O,A 22 = I,A 23 =0
A31 =0,A 32 =0,A 33 = 1; 1
11. Mil = II, M I2 =6,M13 =3, M21 =-4,M22 =2, M23 = 1,
M3l =-20,M32 =-13,M33 =5
All = II,AI2=--6,A13=3,A21 =4,A 22 =2,A 23 =-I,
A31 =20,A 32 = I3,A 33 = 5; 23
12. -24 13. (a-b)(b-c)(c-a)
14. 1..2 (3x + Iv) 15. 4 abc 21. x=4.

• CRAMER'S RULE

Consider the system of linear equations


alx+bly+clz =dl
r
a + b'])l + c 2z = d2
118 Remedial Mathematics

a + bY' + C3Z = d3
r ...(1)
We define A = determinant coefficients
al q Cl

=a2 ~ C2'
~ ~ c3
Now we defme Ax which is obtained by suppressing the column of coeffic;ients of x and
replacing it by the column of constant terms d1, d2, d3 on right hand side
d1 "q cl

L\ = d2 ~ c2

d3 ~ C3
Similarly, we obtained
a) d1 cl
lly = a2 d2 c2 and Az = a2 b2 d2
a3 d3 c3 a3 ~ d3
Now
Case I: If A * 0 solution of system (1) is given by
n A A
x= ; ,y= ; ,z= ;
and system is called consistent.
*
Case II: A = 0 but at least one of Ax' Ay ' Az 0, then the system does not posses any
common solution and system is called inconsistent.
Case III: A = 0, also Ax = Ay = Az = 0 and at least one cofactor of A * 0, then system has
infinitely many solution and the system then be solved by elimination method.
Elimination of one unknowIY from three equations gives anyone equations in two
unknowns therefore two unknowns can be found in terms of the other, we give this unknown
an arbitrary value.
If A = Ax.=·~ = Az = 0 and all cofactor of A, Ax, Ay and !Yz are zero then system is
equivalent to only one equation in three unknowns and then we give any two unknowns
arbitrary values and find the remaining unknown in terms of three constants.

~~~~~~~I SOLVED EXAMPLES ~~~~~~~ I


Example 1: Using the Cramer s rule, solve the following system ofequations
x + y- 4 = 0, 2x - 3y - 8 = O.
Solution: The given equation is
x+y-4 =0 ... (1)
2x-3y-8 =0. ...(2)

Here, A=jllj=_5*0
2 -3
Determinants 119

11 =-15
-3

~I =-5.
:. By Cramer's rule
A A
x= ---.!.=3,y= -L == l.
A A
Example 2: Show that the system 0/equations x + y - 2 = O. 2x + 3v - 5 = 0, 4~ - Y- 3 = 0
is consistent. Find the solution using Cramer 50 rule.
Solution: The given system of equation is
x+y-2 =0
2x+3y-;-5 =0
4x-y-3 =0
is consistent (Le., have common solution), ifthe determinant
1 -2 0 o
A"= 2 3 -5 =Oi.e., 2 I -I =0.
4 -I -3 4 -5 5
:. A" = 5 - 5 = 0, hence the system is consistent, so it is sufficient to solve any two
equations by Cramer's rule.
Let us consider equation (1) and (2)

A=I~ ~I=I(*O)
Ax=l~ ~I =6-5=1
Ay = I~ ~I =5-4= 1
A A
x=~=ly=-L=1
A A
Hence the required solution is given by x = y = 1.
Example 3: Solve the/allowing by Cramer 50 rule
x+y+z =6
x-y+z=2
3x + 2y - 4z = - 5.

I-I 100
Solution:WehaveA= -1 = 1 -2 0 =14*0
3 2 -4 3 -1 7
120 Remedial Mathematics

6 1 6
Ax = 2 -1 1 = -4 -2 o = 14
-5 2 4 19 6 0
6 6
Ay = 1 2 = 0 -4 0 =28
3 -5 -4 0 -23 -7
6 1 6
A = 1 -1 2 = 0 -2 -4 =42
3 2 -5 0 -1 -23
Hence, by Cramer's rule
A A A
x=-L=I---L=2=yz=-% =3
A 'A 'A'
Hence, the solution is given by x = l,y = 2, Z = 3.
Example 4: Solve the following system equations with the help ofCramer srule.
3x - 4y + 5z = -6, x + Y - 2z = - 1, 2x + 3y + Z = 5. [UPTU B. Pharma 20041
-4 5
3
Solution:LetA= 1 1 -2 =3(1+6)+4(1+4)+5(3-2)=46:;1:0.
2 3
since A :;I: 0, therefore the given system has a unique solution given by
x y z 1
-=-=-=-
Ax Ax Az A
-6 -4 5
Now Ax = -1 1 -2 by RI ~RI +4R2,R3 ~R3-3 R2·
5 3 1
-10 0 -3
-1 -2 = I-~O -31
7 =-70+24=-46.
8 0 7
3 -6 5
Ay= 1 -1 -2 byRI~RI-3R2,R3~R3-2R2
2 5 1

o -3 II 3 III
= 1 -I -2 =- 7 5 =92
1-
o 7 5
Determinants 121

3 -4 -6
~
z = 1 -1 by R\ ~R\-3R2' R3 ~ R3 -2 R2
2 3 5
0 -7 -3
1 -1 =_/~7 -3/
7 =46
0 7
The solution of the given system is
~ -46 ~ 92 ~z 46
x= -L = - =-I,y= ~= - =2andz= - = - = 1
~ 46 ~ 46 ~ 46
Hence, the required solution is x = - 1, y = 2, z = 1.
Example 5: Solve using Cramer srule
x + y = 5, Y + Z = 3, Z + x = 4. [UPTU B. Pharma 2001, 07]
1 1 0
Solution:Let~= 0 I 1 =1(1-0)-1(0-1)=1+1=2
1 0
since ~ * 0, therefore the given systems has a unique solution given by

~ = L=--=-=_
~x ~y ~z ~

5 1 0
Now ~=3 1 =6.
4 0 1
5 0
~y = 0 3 1 =4
1 4
5
~z= 0 3 =2.
1 0 4
The solution of the given system is
~ 6 ~y 4 ~ 2
x= ~= - =3,y= - ' = - =2,z= --!...= -= 1.
~ 2 ~ 2 ~ 2
Example 6: Solve the following by using Cramer s rule
x - 2y + 3z = 2, 2x - 3z = 3, x + y + Z = 6. [UPTU B. Pharma 2002)
-2 3 0 0
Solution: Let ~ = 2 0 -3 = 2 4 -9 byR2+2RpR3-3R\
1 1 3 -2

= 4 -9/
/3 -2 =-8+27=19*0
122 Remedial Mathematics

since A *- 0, therefore the given system has a unique solution given by


x y z
- =-=-=
Ax Ay Az A
2 -2 3 2 0 5
Now Ax = 3 0 -3 = 3 3 o byR2 +R l ,R3 +R l
6 1 1 6 7 7

=21~ ~I +0+51! ~I =57.


2 3 1 0 0
A = 2 3 -3 = 2 -1 -9 by R2 - 2Rl' R3 - 3R l
Y
1 6 1 1 4 -2

=1 1-1 -9/ =38.


4 -2
1 -2 2 1 0 0
Az = 2 0 3 = 2 4 -1 by R2 + 2R l , R3 -2Rl
1 6 3 4

=11; ~11=16+3=19.
The solution of the given system is
Ax 57 Ay 38 A 19
x = ~ = 19 = 3, y = A = 19 = 1and z = ~z = 19 = 1.
Hence, the required solution isx = 3,y = 2, z = 1.
Example 7: Find the value of Afor which the system ofequations x + y - 2z = 0, 2x - 3xy +
z = 0, x - 5y + 4z = A are consistent andfind the solutions for all such values of A.
Solution: The given system of equations is
x-5y+4z =')... ... (1)
x+y-2z =0 ... (2)
2x-3y+z =0. ... (3)
1 -5 4 1-5 4
A = 1 -2 = 0 6 -6 =0.
2 -3 o 7 -7
Hence, system is consistent only when
A.x =Ay=Az=O.
')... -5 4
Now A.x 0 1 -2 =-5')...=0=>')...=0.
o -3
Determinants 123

For A. = 0, clearly ~ = ~z = o.
:. System is consistent if A. = 0, then on eliminating x from (1), (2) and (1), (3), we have
6y-6z=0,y-z=0
and 7y-7z= 0 ory=z.
Lety=z= k E R, then from (1), we have x = 5k-4k= k.
Hence, solution is given by x = y = z = k E R.
Example 8: Solve the equations by Cramer 50 rule
4 3
- - + - - =-1.
x+5 y+7
6 6
- - + - - =-5.
x+5 y+7
Solution: The given system of equations is
4 3
--+--=-1
x+5 y+7
6 6
- - + - - =-5.
x+5 y+7
I 1
Now putting - - = a,-- = b, the equation becomes
x+5 y+7
4a+3b =-1
6a-6b =-5

~ = 146 31 = - 42 :;t 0
-6

~
a
-1
= 5 31
-6
=21
'
~b = 146 -11=_14.
-5
1

So by Cramer's rule

a = ~a = ~ =.!.. b = ~b =-14 =.!..


~ --42 2' ~ --42 3
x+5 =-2,y+7=3·
or x =-7,y=-4.
Hence, the solution isx=-7,y=-4.
Example 9: Using Cramer 50 rule solve the following equations:
x + 2y + 3z = 6
2x + 4y + z =17
3x + 2y + 9z =2
Solution: We have
2 3
~= 2 4 =-20
3 4 9
124 Remedial Mathematics

62 3
~x = 17 4 I =-20
2 2 9
6 3
L\ = 2 17 I =-80
3 2 9
2 6
~ = 2 4 17 =-20
z
3 2 2
Then by Cramer's rule, we have
~
-20
x =~=-=I
~ -20
~y -80
y = ~= -20 =4
~z 20
z =-=-=-1
~ -20

1. (a) Using Cramer's rule, solve the following equations


... x+y+z=6
2x+y-z=1
x+y-2z =-3.
(b) x+y+z=6
x-y+z=2
3x+2y-9z =-5
2. Find the value of k if the following equation are consistent:
x+y-3 =0
(1 +k)x+(2+k)y-8 =0
x-(l + k)y+ (2 + k) =0.
3. Find the value of k if the system of equations
(k+ 1)3 x +(k+2)3 y =(k+3)3
(k+ l)x+(k+2)y =(k+3)
x + y = I : is consistent.
4. If the system of equations
x + 2y = 5, 2x - y = 5, x + 3y = 6 is consistent, solve it.
S. Solve the following by Cramer's rule
x+y+z =11
2x-6y-z =0
3x+4y+2z =0.
Determinants 125

6. Show that the system of equation


3x-y+4z =3
x+2y-3z =-2
6x+5y+Az =-3
has at least one solution for any real numbers f.... Find the set of solution if f... = - 5.
7. Using Cramer's rule to solve the following system of linear equations
2x-3y+z =7
2x+y+z=l
4y+3z =-11

1 ANsWERS

9 21
1. (a)x= 1,y=2,z=3 (b) x= 4,y=2,z= 12
2. k= 1 or-5/3 3. k=- 2 5. x=-8,y=-7,z=26.

REFRESHER ~)~~~~~~~
Do you know? After reading this chapter you should be able to learn the following
concepts:
• The value of a determinant does not change when rows and columns are interchanged.
• If any two rows (or columns) of a determinant are inter changed, the sign of the
determinant in changed.
• If two rows or two columns of the determinant are identical, then the value of the
determinant are vanishes.
• If all the elements of any row, or any column, of a determinant are multiplied by the
same number, then the determinant is multiplied by that number.
• Ifin a determinant each element in any row (or column) consists of two terms, then the
determinant can be expressed as the sum of two other determinant.
• If in a determinant, the elements of a row are added in and n times the corresponding
elements of the another rows (or columns) the value of determinant does not change is
particular

a2 + mb2 + nC2 b2 c2 a2 b2 C2
a3 + mb:, + nC3 b:, c3 a3 b2 c3
Can we do? (Frequently Asked Questions)
1. Without expanding the determinant. show that
o b -c
-b 0 a =0. [UPTU B. Phafma 2002)
c -a 0
126 Remedial Mathematics

2. Without expanding the determinant, show that

a be a c?
b ca b b2 and evaluate it. [UPTU B. Pharma 2001]
cab l c c2
3. Prove that
l+a
A= l+b
1 1+ c

( 1 1 I)
= abc 1 + - + - + -
abc
= abc + ab + bc + ca. [UPTU B. Pharma 2004,06]

4. Without expanding the determinant show that (a + b + c) is 0 factor of the following


determinant
abc
As: b ca.
cab
If a, b, c are positive and unequal. Show that the value of A is always negative .
. [UPTU B. Pharma 2003]
x-2 2x-3 3x-4
5. Solvetheequations x-4 2x-9 3x-16 =0. [UPTU B. Pharma 2007]
x-8 2x-27 3x-64

a cJ
6. Prove that b' b3 =(a-b)(b-c)(c-a)(a+b+c). [UPTU B. Pharma 2005]
3
1 c c
7. Solve by Cramer's rule
x-2y=4, -3x+ y=-7. [UPTU B. Pharma 2001]
8. Solve using Cramer's rule
x+y=5,y+ z= 3,z+x=4. [UPTU B. Pharma 2001, 07]
9. Solve the following by using Cramer's rule.
x - 2y + 3z = 2, 2x - 3z = 3, x +Y + Z = 6. [UPTU B. Pharma 2002]
10. Solvb the following system oflinear equations with the help of Cramer's rule.
3x-4y + 5y = 6, x + y- 2z =-1, 2x + 3y + z= 5. [UPTU B. Pharma 2004]
11. Prove that
a
b b2 = (a-b)(b-c)(c-a) [UPTU B. Pharma 2008]
c
DOD
MATRICES

• INTRODUCTION
'Matrices' is a powerful tool of modem mathematices. The study of 'Matrices ' is essential in
almost every important branch of science like mathematics and physics.
The word 'matrix' was used by J.J. Sylvester in 1850 and developed by 'Arthur Caylay'
in 1858.

Definition
A set having mn numbers either real or complex, arranged in the form ofrectangular array
in which there are m rows and n columns. This rectangular arrangement is called a matrix
oforder m x n which is denoted by [aij]m x 11' where i = 1,2,3, ... m andj = 1,2,3, ... n and a
matrix of order m x n is usually written as
all al2 a]3 al n
a21 a22 a23 a2n
[aij]m x n a31 a32 a33 a3n

amI a m2 am3 amn mxn

• TYPES OF MATRICES

(i) Null matrix (zero matrix): A matrix oforder m x n is called a null matrix ifit contain
all mn elements as zero. It is denoted by 0 and usually written as
o 0 0 0
o 0 0 0
o 000 o

o 0 0 0
(ii) Square matrix: A matrix in which number of rows equal to number ofcolumns,Js
called a square matrix.
128 Remedial Mathematics

For example,

This matrix A is a square matrix of order 3 x 3.


(iii) Unit matrix: A square matrix of order n x n having all non-diagonal elements equal to
zero and each of the diagonal element equal to 1 is called a unit matrix. It is denoted by
In and usually written as
000
o 1 o 0
o 0 o

000 nxn
The unit matrix is also known as identity matrix.
(iv) Row matrix: A matrix having only one row and n columns is called a row matrix of
order I x n.
For example, A = raJ 1 a 12 a l3 ... a1n]1 x n'
(v) Column matrix: A matrix having m rows and only one column is called column matrix
of order m x 1.

For example A

Triangular, Diagonal and Scalar Matrices


(i) tJ pper triangular matrix: A matrix of order n x n is called an upper triangular matrix if
it contains all itlelements below the diagonal elements equal to zero, i.e .. , A = [aij] nxn
if aij = 0 for i >j, then A is upper triangular matrix.

For example, A = [~ll :~: :~:l


o 0 a33 3x3

(Ii) Lower triangular matrix: A matrix of order n x n is called a lower triangular matrix if it
contains its all elements above the diagonal elements equal to zero. Suppose
A = [a ymQ
..] .. = 0 for i < j, then A is called lower triangular matrix.
and if a U .

For example, A = [:~: a:2 ~1


a31 a32 a33 3x3
Matrices 129

(iii) Diagonal matrix: A matrix of order n x n is called a diagonal matrix ifit contains all its
off- diagonal elements equal to zero. Suppose A = [aU]nxnand if aU = 0 for all i :;tj,then
A is called diagonal matrix, it is denoted by
Diag.. [all a22 ··· ann]·
(iv) Scalar matrix: A diagonal matrix whose diagonal elements are all equal but not equal

to 1, is called a scalar matrix. For example A = 0 k


k 0 0]0 , and k:;t 1.
[
k 3x3 o 0
(v) Idempotent Matrix: A matrix such that A2 = A is called idempotent matrix.

For example, A =
2
-1
-2
3
4]
4 :::>A 2 =A·A= -1
[2 -2 3 ; ] =A.
[
1 -2 -3 1 -2 -3
(vi) Periodic matrix: A matrix A is called a periodic matrix if Ak+ I = A, where k is a +ve
integer if k is the least +ve integer for which A k + I = A, then K is said to be the period
of A . Ifwe choose K = 1, we get A2 = A and we call it to be idempotent matrix.
(vii) Nilpotent matrix: A matrix A is c~lled a nilpotent matrix ifAk = 0 (null matrix) where K
is a +ve integer. Jfhowever K is the least +ve integer for which AK = 0, then K is the
index ofthe nilpotent matrix, where 0 is the null matrix.
2
For example, A = ab b ]
[ _a 2 -ab
(viii) Involuntary matrix: A matrix is called an involuntary matrix ifA2 = I (identity matrix).
Since P = I always.
:. Unit matrix is an involuntary matrix.

DETERMINANT OF SQUARE MATRIX


Let A be a square matrix. Then the determinant, which is formed by the elements of a matrix
A , is usually denoted by I A I.

For example, if A =
[a"a21
al2

a22
~3]
a23
a31 a32 a33 3 x 3

all al2 al3


Then, IAI a2l a22 a23
a3l a32 a33

• SINGULAR AND NON - SINGULAR MATRIX

Definition: A matrix A , whose determinant is zero, is called singular, otherwise non - singular.
130 Remedial Mathematics

• SUB- MATRIX OF A MATRIX


Definition: LetA be a matrix of order m Xn, then a matrix which is obtained by leaving some
rows and columns from the given matrix A, is called a submatrix of matrix A .
For example:

Let A =
[all a21
al2

a22
al3
a23
~4]
a24
a31 a32 a33 a34 3x4

a12
Then the matrix B = [all al3 ]
a31 a32 a33 2x 3

is a submatrix of A , which is obtained by leaving second row and fourth column. If the given
matrix A is a square matrix, then a square submatrix of the given matrix is called principal
submatrix.

• MINORS OF A MATRIX
Definition: LetA be a matrix of order m x n, then the determinant of every square submatrix
of A is called a minor of A .

• TRANSPOSE OF MATRIX
Definition: Consider a matrix A = [alj ] m x n" Then a matrix which is obtained by interchanging
the rows and columns of A is called the transpose of A. It is denoted by A' or AT.

For example
A = [:~:
Then the transpose of A is

all a21]
A' = al2 a22
[
al3 a23 3x2

Remark
• The transpose of the transpose of a matrix is the matrix itself i. e ., (A ')' = A

• SYMMETRIC AND SKEW- SYMMETRIC MATRICES


Symmetric Matrix: Amatrix 'A'is said to be asymmetric matrix A I =A i.e. the transpose ofa
matrix is equal to the matrix itself.
Matrices 131

Skew symmetric matrix: A matrix 'A' is said to be a skew-symmetric matrix if A' = - A

For example: A = [~2 ~


-3 -4 0
:]
Remark
• The diagonal elements ofa skew symmetric matrix are all zero.

• COMPLEX MATRIX

Definition: A matrix 'A' is said to be complex matrix, if some of its elements are complex
numbers.

For example A =
[1-3i+ 2i 3i
2+3i 1+ i
7]
Conjugate of Complex Matrix
Definition: Let A be a complex matrix, then a matrix which is obtained by replacing all the
complex element~ ofA by their conjugate complex number, is called conjugate of a matrix and
it is denoted by A.

[~+2i
~+il
3i
For example: If A =
2+4i

[~-2i
~-J.
-3i
Then A =
2-4i

Transpose Conjugate of a Matrix


Definition: The transpose ofthe conjugate of a matrix is called the transpose conjugate of a
matrix. It is denoted by AO That is,
AS = (.4)'

Hermitian And Skew-Hermitian Matrices


(i) Hermitian matrix: A matrix 'A' is said to be Hermitian ifAo =A.

For example =[33-i]


A 3+i 4
(ii) Skew Hermitian matrix: A matrix 'A' is said to be skew-Hermitian if AO =-A.

For example A = 2i
[ -5 -i
5]
Remarks
• The diagonal elements of a Hermitian matrix are necessarily real.
• The diagonal elements ofa skew-Hermitian matrix are either purely imaginary or zero.
132 Remedial Mathematics

• ALGEBRA OF MATRICES
(i) Addition of Matrices. Let A and B be two matrices of m x n types. Then the sum of A
and B i.e., (A +B) is defined to be the matrix of the type m x n obtained by adding the
corresponding elements of A and B.
Let A = [aij] m x nand B = [bij] m x n' Then A + B =:[aij + bij] mx n'
Thus A + B is also a matrix of type m x n .
a12 l

Thus, if
[ all
A = a21
a22 a2n
a ·1
ami am2 amn mxn

[~I ~. 1
bl2
b22 b2n
and B = b21

bml bm2 bmn mxn


all +ql a12+q2 al n+qn
a21 +b21 a22+ b22 a2n+ b2n
Then A+B

ami +bml am2+ bm2 amn+bmn


mxn
(ii) Subtraction of two matrices. If A and B be two matrices of m x n types, then the
subtraction of two matrix denoted by A - B is also a matrix of m x n.
Let A = [aij] m x nand B =[bij] m x n
then A-B =[aij-bij]mxn
Remark
• Two matrices should be of same order for the process of addition and subtraction.

~~~~~~I SOLVED EXAMPLES I~~~~~~

~11x3
3 -1]
Example 1: If A = [2 andB= [2 -1
-3 -1 4 2x3 7 2
3 -1 -1 + 3]
Then A+B= [2 +2
-3 +7 -1+2 4-1 2x3

A+B=[~ ~lx3
2
Matrices 133

~ [~ -I] [-3
Example 2: ifA 7
3 3x2
andB=-1
1 ~L
Then A+B= 3-1
[5-3 -1+3] 7-4 - [22 32]
2+1 3+1 3x2 3 4 3x2

q c, ] h3 c3 ]
Example 3: ifA = [a, andB= [a3
a2 b2 c2 2x3 b
a4 4 c4 2x3

[a l -a3 bl -b3 C, -c3 ]


then A-B =
a2 -a4 b2 -b4 c2 -C4 2 x3

Example 4: ifA = [~ 5 -1]


-3
4 andB= [-1
4
2 -3]
1 2

~lx3"
_ [2+1 5+2 -1+3] _ [3 3
then A-B - 7 - 4 -3-1 4 -2 2x3 - 3
-4

• PROPERTIES OF MATRICES ADDITION


Theorem 1: Addition ofMatrices is commutative, i. e.
ifA and B be two matrices ofm x n types then A + B = B + A .
Proof: Let A = [aij] m x nand B = [bij] m x n
then A + B = [aij + bij] m x n
= [bij + aij] m x n
= [bij] m x n + [aij] In x n
[by definition of addition of two matrices]
A +B =B+A.
Theorem 2: Addition of Matrices is associative.
IfA, Band C be three matrices of m x n type, then
(A + B) + C = A + (B + C).
Proof: Let A = [aij] m xn B = [bij]m x~ and C = [cij] In xn
Then (A + B) + C = ([aij] In x n + [bij] 11/ x J + [cij] m x n
= (aij + bij) m x n + [cij]m x n
= [(aij +bij) + cij] In x n
= (aij) m x n + ([bij + cij] ) In x n
(A + B) + C = A + (B + C)
134 Remedial Mathematics

Theorem 3: Existence ofadditive identity i . e. ifA = [a;) m x Yi is the given matrix and 0 be
the m x n null matrix then
A +0 =A =O+A.
Proof: LetA = [au] m x n and 0 = [0] m x n
then A + 0 = [au] m xn + [0] m xn = [aij + o]m xn = [aij]m xn =A
0+ A = [o]m x n + [aU] m x n = [0 + aij] m x n = [alj]m x n = A
Hence A + 0 =A = 0 +A .
Thus the null matrix 0 of m x n type acts as the identity element for addition in the set of all
m x n matrices.
Theorem 4: Existence of additive inverse: If A = [aijl m x n be a matrix then there exist
another matrix B of m x n type such that A + B = 0 = B + A then the matrix B is called the
additive inverse of the matrix A or the negative of A .
Theorem 5: Cancellation law hold in case ofmatrices addition, i.e., if A, Band C be three
matrices of m x n type such that
A + B = A + C then B = C .
Proof: We have A + B =A + C=>-A + (A + B) =-A + (A + C) adding-A both side. Then,
by associative law of addition, we have
(-A + A)+ B = (-A + A)+C
O+B=O+C
B=C

SOLVED EXAMPLES

Examplel:ifA=[~ ;lB=[-~ ~JFindA+B.


Solution: We have

A+B = [~ ~] + [-~ _~] [~=~ ~:~] [~


= = :]

Example 2: if A = [2 3 1] andB = [1 2 -1] find 3A -4B.


o -1 5 0 -1 3
1
Solution: Wehave3A-4B =3 [2 3 1]_4[1 2 -3 ]
o -1 5 0-1

= [~ -~ l~]-[~ -48 -4]


12
6-4
= [ 0-0
9-8 3-(-4)] = [2
-3-(-4) 15-12 0 1 3
1 7]
Example 3: If A ~[~ ~ -~l and B = [~ ~ -~l· Find the matrix C such that
-/ / -2 0 -/
A +2C=B.
Matrices 135

Solution: Given A + 2C = B or 2C = B - A

Now, 2C = [~ -!]-[~ ~ -~]


1
-2 0 -1 1 -1 1

=[~=~ ;=~ -24-_(~3)]=[_~-1~]


-2-1 0-(-1) -1-1 -3 -2

~[_~ ~] = [_5 /2 ~~~2 1~2].


1
C = -1
-3 -2 -3/2 1/2 -1
Example 4: Find,the additive inverse o/the matrix

A = [~ =~ -~ ~]
J 287
Solution: The additive inverse of matrix A is the matrix each of whose elements is tlie
negative of the corresponding element ofA. Hence, if we denote the additive inverse of A by
- A then we have

-2 3 1-I]
-A =
[
-3
-I -2
I -2 -2
-8 -7
Example 5: Solve the/ollowing equations/or A and B,

2A-B = [~ -33 20] and2B +A = [-J4 J


4 ~l
~]
-3
.
Solution: GIven 2A - B = [33
3
Multiplying both sides by 2, we get

~] =[: ~J
-3 -6
4A-2B = 2G ... (1)
3 6

Also given that 2B + A = [4 I


-1 4
adding equation (I) and (2), we get
-!J ... (2)

I
5A =[: -6 0] + [ 4
6 4 -1 4 -!]
= [6+4 -6+1 0+5] =[105 ~]
-5
6-1 6+4 4-4 10

=.5!.fO5 ~] =[~ ~]
-5 -1
A
10 2
136 Remedial Mathematics

Now substituting the value of A in equation (2), we get

2B = [_~ ~ _!]-[~ -~ ~]
= [4-2 1-(-1) 5-1] = [2 2 -44]
-1-1 4-2 -4-0 -2 2

B - [_~ ~ _~]
sec 2 e sin
2
e ] andB= [-tan 2 e cos
2
e]
. Find A +B
Example 6:.if A = [ 1/3 2
cosec 2e 2;'3 -cot e
sec2 e sin 2 e ]+[-tan 2 e cos 2 e ]
Solution:A+B= [ 113
cosec 2e 2/3 -cot 2 e
2 2 2 2
= [sec e-tan e sin e+cos e] = [1 11]
1/3+2/3 cosec 2e-cot 2 e 1

Example 7: .if A = [~ ; :] and B = [~ ~ ~] thenfind 3A - 4B .


456 001

Solution: We have 3A - 4B = 3 [~ ~ ~1-4 [~ ~ ~l


456 001

= [~ ! 1~1-[~ ~ ~l
12 15 18 0 0 4

1~]
0-4 3-0
= 6-0 9-4
[
12-0 15-0 14

• MULTIPLICATION OF MATRICES
Let A = [aij] m x nand B = [bjk] n x p be two matrices such that the number of columns in A is
equal to the number of rows in B then the product of A and B denoted by A 13 is defined as
a matrix C = [c ik] mx p where cik = '£aij bjk or
The product AB is defined as the matrix whose element in the ith row and! kth column is
ail b1k + ai2 b2k + ai3 b3k ... + ain bnk' thus we conclude that:
if A is an m x n matrix and B is an n x k matrix then the product matrix AB, is an m x k matrix.
In the product AB, the matrix A is called the pre-factor and the matrix, B is called the post-
factor. Also we saythatthe matrix A has been post -multiplied by the matrix B and the matrix
Matrices 137

B has Deen pre multiplied by the matrix A. The product in both the above cases AB and BA
mayor may not exist and maybe equal or different. The productAB can be calculated only if
the number of columns in A is equal to the number of rows in B.
Remarks
• If AB = BA, then the matrices A and B are called commutative and if AB =-BA then the
matrices A and B are called anticommutative.
• The product oftwo non- zero matrices may be a zero matrix.
• The product of matrices generally does not obey the law of cancellation.
Theorem 1: Let A and B are symmetric matrices, show that AB is symmetric if and only if
AB=BA.
Proof: It is given A and B are symmetric matrices, therefore
A ' = A and B '= B ... (1)
Let us first suppose AB =BA ...(2)
To prove AB is symmetric.
We know that (AB) , = B' A' (R,eversallaw)
=B.A [using (1)]
=AB [using (2)]
(AB)' =AB
Hence, AB is symmetric.
Conversely, Let AB is symmetric, i. e.,
(AB)' =AB
Consider L.H.S. = (AB)'
= B' A'
=BA [using (1)]
Hence, AB = BA

Examplel:lfA ~[~ 3] I andB= 0


[1 0 2]
1 2.FindBA,canwefindAB?
4 0 2 3
Solution: We cannot finds AB since the number of columns of A is not equal to the number
of rows of B. i.e., column of A is 2 and rows of B is 3. They are not equal.
Since the matrix B has 3 columns and matrix A has 3 rows in BA, so product BA is defined.

I 0 2] [I 3]
Now, BA=012
[o 2 3 33
x
x21
0 4 3 x2

_[1.1 + 0.2 + 2.0 1.3 + 0.1 + 2.4] [I 11]


~4
- 0.1+1.2+2.0 0.3+1.1+2.4 = 2
0.1+2.2+3.0 0.3+2.1+3.4 3x2 4 3x2
138 Remedial Mathematics

Example 2: IfA = [
-4
-2 3
J
2 5
] and B = 4
[
2
2
n Find AB and shaw that A B '" BA.

Solution: We have AB ~ [~ -~ ~l'l~ :]


= [1.2+(-2).4+3.2 1.3+(-2).5+3.1]
-4.2+2.4+5.2 -4.3+2.5+5.1

= L~ ~]
Now
BA ~ [~ l [-~ -2 3]
2.1 + 3(-4)
2 5

2( -2) + 3(2) 2(3) +3(5)]


= 4.1 + 5(-4) 4(-2) + 5(2) 4(3)+ 5(5)
[
2.1+1(-4) 2(-2)+1(2) 2(3) + 1(5)
-10 2
=
21]
-16 2 37
[
-2 -2 11
Hence, AB ~ BA.

Example 3: IfA = [~ -;
oJ 2]2. Find AB and show that AB ~ BA.
-3 J 2 0
Solution: Since A and B both are 3 x 3 type square matrices, therefore

AB = [~ -~ -~] [~ ~ ~] x
-3 2 1 2 0
1.1 + (-2).0 + 3.1 1.0 + (-2).1 + 3.2 1.2 + (-2).2 + 3.0]
2.1+3.0+(-1).1 2.0+3.1+(-1).2 2.2+3.2+(-1).0
[
-3.1+1.0+2.1 -3.0+1.1+2.2 -3.2+ 1.2 +2.0

= [~ ~ ~~]
-1 5 -4
Matrices 139

Now BA = [~ ~ ~] [~ -~ -~]
x
1 2 0 -3 2

_ [1.1+0.2+2.(-3) 1.(-2)+0.3+2.1 1.3 + 0(-1) + 2.2]


- 0.1+1.2+2.(-3) 0.(-2)+1.3+2.1 0.3+1.(-1)+2.2
1.1 + 2.2 + 0.(-3) 1.(-2) + 2.3 + 0.1 1.3+2.(-1)+0.2

= [~ ~ ~]
5 4 1
HenceAB 7f!:BA

Example 4, ifA U
~ ~ ~] and B ~ H~ ~lt~n prow that AB~ BA

Solution: WehaveAB= [~ ~
-1 1
2.1+3(-1)+4.0 2.3+3:2'+4.0 2.0+3.1+4.2]
= 1.1+2(-1)+3.0 1.3+2.2+3.0 .1.0+2.1+3.2
[
(-1).1 + 1.(-1) + 2.0 (-1).3+1.2+2.0 (-1).0+1.1+2.2
12
[ -I
= -1 7
-2 -1
I:] ... (1)

BA~H
3
Now 2 0][
1 x 21 23 41
3
0 2 -1 2

[ 1.2+3.1+0.(-1) 1.3+3.2+0.1 1.4+3.3+02]


= (-1).2+2.1+2.1+1(-1) (-1) . 3 + 22 + 1.1 (-1).4+2.3+1.2
0.2+0.1+2.(-1) 0.3+0.2+2.1 0.4 + 0.3 + 2.2

[ 2+3+0
"" -2+2-1
3+6+0
-3+4+1
4+9+0]
-4+6+2 =
[5 I~]
-1
9
2 ...(2)
0+0-2 0+0+2 0+0+4 -2 2
From (1) and (2) , we conclude thatAB7f!: BA.

EnmpleS, if A ~ [ ~ 1 3] 2 6. ShowthatA 2 = 0
-J -J -3
140 Remedial Mathematics

-Solution: We have

A2 =A xA =[ ~. 2
-1 -1
1.1+l.2+3(-1) 1.1 + l.2 + 3( -1) 1.3+l.6+3(-3) ]
= 2.l + 2.2 + 6( -1) 2.1+2.2+6(-1) 2.3+2.6+6(-3)
[
-1.1 + -l.2 + -3( -1) -1.1-1.2 - 3(-1) -1.3 -l.6 - 3(-3)

= [~o ~0 ~] ~ 0, where 0 is 3 , 3 null matrix.


Hence, A2 = 0
Example 6: Find the product ofthe following matrices
2
o c -b a
A = [-c 0 a] and B = ab
[
b -a 0 ac
Solution: We have

0 c
AB=-c?
[
b -a
2
o.a 2 +c.ab+(-b).ac O.ab+c.b 2 -b.(bc) O.ac + c.(bc) + (-b).C j
2 2 2
= (-c).a + O.ab + a.ac -c(ab)+O.b +a.(bc) -c.(ac) + O(bc) + a.c
[ 2
b.a +(-a).ab+O.(ac) b(ab)+(-a).b +O.(bc) b(ac) + (-a)bc + O.c 2
2

~[~ ~ n
Example 7: Prove that the product oftwo matrices

cos8sin8] and [ cos ~ cos~sin~]


2 2
cos 8
[ cos8sin8 sin 2 8 cos~sin~ sin2~
7t
is zero when (J and ¢ differ by an odd multiple of "'2
Solution: The required product
cos 8 sin 8] x [ cos ~ cos ~ sin~]
2
cos2 8
[ cos8sin 8 2
sin 8 cos~sin~ sin 2 ~
Matrices 141

[COS 9cos ~+cos 9sin 9cos ~sin ~COS2 9cos ~sin ~ +cos 9sin 9sin2 ~]
2 2
=

cos 9sin 9cos 2 ~+ sin 2 9cos ~sin ~cos 9sin 9cos ~sin ~+sin 2 9sin2 ~
= [cos 9cos ~(cos 9 cos ~ + sin 9sin ~)cos 8 sin ~(cos8 cos ~ + sin 8 sin~]
sin 9cos ~(cos9 cos ~ + sin 9sin ~sin 9sin ~(cos 9cos ~ + sin 9sin ~
= [cos 9 cos ~ cos(8 - ~) cos 9 sin ~ cos(8 - ~)]
sin 8cos ~cos(8 -~)sin 8sin ~cos(8 -~)

Now if8 - ~= an odd multiple of '21t .


Then cos (9 - ~) >= 0 and consequently the above product is zero .

~ ~ ~]
a h
Example 8: IfA = [xyz}, B = h b ] and C [ be three maMces. thenfind ABC
[g f

Solution: We have AB ~ [xyzl' [; h


b
g]
f
f e
= [x. a + y.h +z.g x.h + y.b + z.J x.g +y.J + z.e]

Now ABC ~[x.a + y.h +zg'" + yb + ifxg+yf+ zc] x[~]


= [x(ax + by +gz) + y(hx + by + fz) + Z (g:x +fy + ez)]
= [ax 2 + by + e~ + 2hxy + 2gzx + 2fyz]

Example 9: JfA = [~ 2 2] 1 2 ,show that A2 - 4A -51 = 0


2 1
Solution: We have
142 Remedial Mathematics

9-4-5 8-8+0
8-8+0]
= 8-8+0 9-4-5 8-8+0
[
8-8+0 8-8+0 9-4-5

~ [~ ~ ~]
Hence, A2 - 4A - 51 = 0
Example 10: Find the value o/x, y, z in the/ollowing equation

[~ 2 3]
Y
[X] [lX+2. +3.Z]
SoloUon: We have 1 2 x Y = 3x+1.y+2.z
31 Z 2x+3.y+1.z

and
4 -2]
o -6 x [~] [4.2+(-2).1] [6]
= 0.2+(-6).1 = -6
[
-1 2 -1.2+2.1 0
With the help of (1) and (2) the given equation reduces to

[;::;:~:] [-!].
2x+3y+z
=
0
On comparing the corresponding elements on both sides, we get
x+2y+3z = 6
3x+y+2z =-6
and 2x+3y+z = 0

Solving these, we get ; : ;4}


z=2

Example 11: IfA = [4-1 2]1 Find (A - 21) (A - 31).


Solution: We have A _ 21 =;0 [ 4 2] _ 2 [1 0]
-1 1 0 1

= [~1 ~]-[~ -~]=[~1 ~1]


Also
Matrices 143

(A - 21) (A -31) = [_~ _~] [_~ _~] = [~ ~] =0


Hence (A - 21) (A - 31) = O.

Example 12:.lf I = [~ ~] C = [~ ~] show that (aI + bc)3 = a3I +3clbc.


Solution:We have aI + bc = a [~ ~] + b[~ ~]
=[~ ~]+[~ ~]=[~ ~]
2
(aI +bci = [a b] [a b] = [a 2ab]
OaOa 0 a2
2 3 2
a 2ab] [a . b] [a 3a b]
(aI + bc)3 = [0 a2 0 a = 0 a3

Verify that (AB),= B'A' where A' and B' are the transpose ofA and B respectively. I
[RGPV B. Pharma 2001)

Solution: We have AB = [~ ~ ~1] [~ 0 ~]


4 5 0 0 3

= [~ ~ ~]
14 5 0

L. H.. S = (AB)' =
[5 3 14]1 2 5
-3 6 0

[1 0 0] [1 2
Now B = 2
o
1 0
1 3
~B'= 0
0 0 :]
144 Remedial Mathematics

and
A{ 2
5
-I]
0
[ 1
o 2 ::::::>A '= 2 0
3

-1 2 ~]
B'A' =
[5I 32 14]5
-3 6 0
L. H.S = RH.S.

Example 14: IfA = [~ =~] show that Ak = [l+/k 1~4;k] where k is any positive integer.
Solution: We have

A 2 = [3 -4] [3 -4]
I -I I -I
= [5 -8] [1
2 -3
= + 2.2 -4.2]
2 1-2.2
... (1 )

A3=[~ =~][~ =~]=G ~:]=[1+32.3 1~~~3]


Thus the result is true for k = 2,3.
1+2k -4k] then
Now assume that result is true for integer k i.e Ak = [
k. 1-2k

A k+/ -_ A·A
k [1+2k
k
-4k
1-2k
][3 -4]
I -I

=[O+2k).3-4k.1 (1+2k).-4+ C-4k)C-I)] = [3+2k -4Ck-1)]


k.3+(l-2k).1 k.(-4) + (l-2k).(-I) k+1 -1-2k

= [I + 2(k + 1) -4(k + 1) ]
k +1 1- 2(k + 1)
Hence, the result is true for Ak . Then it is also true for Ak+ / . Hence by induction the
required result follows.

Example 15: IfA = [ ; ~l thenprovethatA 2 -6A +51=

Where 1= [~ ~l [UPTU B. Pharma 2002)

Solution: We have A2 = A.A = [~


Now A2 -6A +51 =
[1918 6]7 + [-24
-18

= [~ ~]
Matrices 145

Example 16: Find the value o/x such that

[IIxJ [~ ~ ~][}o [UPTU B. Pharma 2001, 061

Solution: The given matrix is

[I Ix] [~ ; m:] = 0

=> [1+0+2x 0+2+x 2+1+0{] =0

[I +2x2+X{] =0
=> [1 +2x+2+x+3] =0 =>3x+6=0
x=-2

Enmplel7'ifA = [~ :]andB = [-; =n FindD = [; :] ,"chthalA + B-D = 0

Solution: We have A + B - D = 0 or D = A+B

1 2] [-3 -2] [1-3 2-2]


or
[
D = 3 4 +
5 6 4
1 -5
3
= 3 + 1 4-5
5+4 6+3

D
-2 0] [P q] = [-2: - ~0] .
= [: -~ or ; :

Hence P = -2 ,q=O ,r=4 ,s=-I, t=9, u=9.

Example: 18: IfA = [0


-I 0
I] . Find number a, b, so that (al +bA / = A.

Solution: We have al + bA - a [1
- °1 + °
0] b [0 1 ]
-1

=[ ~ : ] +[ ~b ~] = [:b ~]
146 Remedial Mathematics

(aJ +bA/ = [a b] [a b]
-b a x -b a

a 2 _b 2
= [ -2ab
... If(aJ + bA)2 = A then, we have
2
a _b2 2ab 1 [0 1]
[ -2ab a 2 _b 2 =-1 ° .
Equating the corresponding elements, we have
~ a2-b 2 =0,2ab =1
a =b = [1I.J2].

Example 19: A = [~ =;1 B = [: ~l] and(A +Bi =A2 + B2. Find a and b.

Solution: We have
-I] [1-2 -1+1] [-1 0]
-1
=
2-2 -2+1
=
0-1

a +b a-I]
2
and
~I] = [ab-b b+ I
A2+B2 =[-1° 2
0]+[a +b a-I]
-I ab - b b + I
=[a 2ab+b-l
-b
ab-I]

Also
A + G=;]+[: ~I] -G:: =;:;]=G::
B -

(A+ Bi = [ 2+b
1+a 0] [I+a
-2 x 2+b
0] [
-2
(1+ai +0
(2+b)(1+a)-2(2+b)

(1+a)2 40] ...(2)


= [ (2+b)(a-l)
Now given that (A + B/ = A2 + 8 2.
Hence from (I) and (2) , we get

(1+a)2
[ (2+b)(a-l) 4 =
0] 2
[a +b-l
ab-b
or a -I °
= and b = 4 .
Hence a=l,b=4.
Matrices 147

~~~~~~~I EXERCISE 4.1 ~I~~~~~~~

1. If A = [20 3 l]andB=[1 2 -I]. Find2A-3B.


-1 5 0 -I 3

2. If A = [~ ~ ~]andB=[~ ~ ~].Find3A-2B.
689 570

3. If A = [~ ~]
4 1
andB = [~ ~ ~l'
2 3 0
FindBA.

1
4. Find the product ofthe matrices A = [; ~7 ~ ~8] , [~3 ~l'
B=
2 4 -3 1
3 1

I -3 2] [1 4 ~ ~] [~ ~2 =; -o-~]
[
5. IfA = 2 1 -3, B = 2 1
4 -3 -1 1-2 1 2
andC, =
2 -5 -1
Showthat

AB =AC.

6. If A = [ 1
-1
-1] 1 ShowthatA 2 =2A andA 3 =4A.

7. If A = [~ ~lB= [~ ~lc= [~ ~lShowthatA(B+C)=AB+AC.


8. IfI= [~ ~] andE= [~ ~] 3
provethat(aI+bE)3=a I+3a 2 bE.

9. [fA ~ ~l ~lFindA2-5A
[; +61.

cosh u sinh u] [COSh nu sinh nu ]


10. If A = . then show that An = . , where n in any
[ smh u cosh u smh nu cosh nu
positive integer.
11. Show that multiplication of matrices is not commutative.
148 Remedial Mathematics

HINTS TO THE SELECTED PROBLEMS

cosh U sinh U]
to. Let A =
[ sinhu coshu

A2=A.A = [COShU SinhU][COShU SinhU] = [COSh2U Sinh2U] ... (1)


sinh U cosh U sinh U cosh U sinh 2u cosh 2u
In equation (1) and given value of A. Let us assume that the
An = [COSh na sinh na]
sinh na cosh na
Now to show A n+ 1 = An.A ...(2)
= [COSh na sinh na] [COSh a sinh a]
sinh na cosh na sinh a cosh a
= [COSh(n + I)a sinh(n + I)a]
sinh(n + I)a cosh(n + I)a
i.e., equation (2) holds forn+ 1 ifittrue fonn.
Hence by mathematical Induction, we have
An = [COSh na sinh na]
sinh no. cish no.

11. Let us take A ~ [ ~


~]
0
-23 -I3] andB= [I0 1
-3 ] 2 ] 2

AB~p -23 -I3]['0


~l
0
1
-3 2 1] 2
[1.1 + (-2).0 + 3.1 1.0+(-2).1+3.2 1.2 + (-2).2 + 3.0]
2.1+3.0+(-1).1 2.0+3.1+(-1).2 2.2 + 3.2 + (-1).0
-3.1+1.0+2.1 -3.0+1.1+2.2 -3.2 + 1.2 + 2.0

[~ ~ ~~l
-1 5 -4

Now BA = [0]1 ~ ~l [~ ~2 ~]l = [~ ~ ~l


2 0
x
-3 2 5 4 ]
Hence AB ;If BA
Clearly, the ~ultiplication of matrix is not commutative.
Matrices 149

'ANSWERS

6 3] [~O
Co 10
~]
[ -I
1. 5] 2. ~6 15 11 3.
1
10 27 11

4. AB~ [: : ] BA j, undefined 9. [ ~I
-1
-1 -~O -3]
-8 -8 -5 4

• ADJOINT OF A MATRIX

Definition: Let A = [aij] n x n be a square matrix of order n x n. Then the adjoint ofA is a
matrix of the same order n x n which is obtained by the transposing of a matrix whose
elements are cofactor ofthe element ofA in the determinant A . That is ifB = [A ti] n x n where
Aij are the cofactors ofthe elements aij in the determinant IAI. Then B'is called the adjoint
ofA. It is denoted by ad) A.
Remark
• Sometimes the adjoint of a matrix is also called the adjugate of the matrix.
Theorem 1: IfA is a square matrix oforder n then
A. (adj A) = (adj A) .A = IAl.ln
where I is the unit matrix ofthe same order as A.
"'-
Proof: We have the (i,j) th element of the product A (adj. A) = Product of the ith row of A and
jth column of adj A I

= aliAjl +a2i Aj2 · .. +aniA jn


= 0, when i;f:j
= IAI when i = j.
Thus, is the product, only the diagonal elements exist and each is equal to IAI while all
other elements are zero , so that
IAI 0 0 o

A .(adjA) =
o
0
IAI
0
0
IAI
o
o ".
o 0 o IAI

or
A.(~AHA{t : . ~.l
... I
= IAI .1

Similarly, (ad) A). A = IAI . I.


Hence the theorem is proved.
150 Remedial Mathematics

~~~~~~~I SOLVED EXAMPLES ~I~~~~~~


Example 1: Find the adjoint ofthe matrix

A =
: [~
:]_
9 10 12
Solution: For the given matrix A, we have

AJI= [170 1~] =4; A12=-[~ 1~] 7


[~ 10] =-13;
=12;A J3 =

A21 =- [1~ 1:] = G1:] =-24; [~ 1~] =8;


16;A 22 = A23=-

A
31
=[27 4]8 =-12- =_[15 4]8 =12- =[15 72] =-3
'32
A
'33
A

Therefore the matrix B formed by the cofactors of the elements of IAI is:

B =
8
[1: ~;4 ~~3]
-13 -3
Nowadj A = transpose ofthe matrix B

= [1:-12 ~;412
• INVERSE OR RECIPROCAL OF MATRIX
Definition: Let A be a square matrix of order n x n and there exists a square matrix ofthe
same order such that AB = BA = 1, where In is a unit matrix oforder n x n _ Then the matrix
B is called the inverse of a matrix A_

Remarks
• A matrix 'A' is invertible ifit is non - singular.
-1_adj.A I I
• A -lA!' A :;1;0
Theorem 1: The inverse ofa matrix is unique. •
Proof: Ifpossible, let Band C be two inverses of the matrix A, then by definition
AB =BA =1 ...(1)
and AC =CA =/. ... (2)
From (l) and (2) , we get AB = AC , each being equal to I
or B(AB) = B(AC) or (BA) B = (BA) C
or IB =IC
or B=C.
Hence, the inverse of a matrix, if exists is unique.
Matrices 151

Theorem 2: A square matrix A has an inverse if and only ifA is non - singular
Proof: The condition is necessary; LetB be the inverse of the matrix A, thenAB = I
IAIIBI = III =1
Therefore, IAI ¢O
The condition is sufficient: Let IAI ¢ 0, we assume that
B = adjA
IAI
(adjA)
AB =AliAif

IAII
=TAT1 (A ad]. A) =,lAI =I .
Similarly BA =1
AB =BA=I
Hence, A has an inverse.
Theorem 3: IfA and B are two non - singular matrices of the same order, then AB is also
non - singular and (AB ri = S-i A-i
Proof: Let A -i and S-I exist. Since A and B are non - singular, therefore,
(AB) (B- 1 A-I) = A (BS-I) A-I, (By associative law)
= AlA-I (By inverse property)
=AA- I =1
Similarly, (S-I A-I) (AB) = I
(S-I A-I) (AB) = (AB)(S-I A-I)=I
i.e., S-I A-I is tIl1e inverse of AB or (AB)-I = S-I A-I and as such AB is also non - singular
Theorem 4: IfA is a non - singular matrix, then (A-ir i =A.
Proof: Let A-I be the given matrix instead of A, then
(A-I)A =A(A-I)=l.
This show that A is the unique inverse of A-I
i.e., A =(A-irl
Theorem 5: The inverse of the transpose ofa matrix A is the transpose ofthe inverse ofA,
i.e., (AT i = (A-i) '.
Proof: We have AA-i = A-i A
.. (AA- I ), =f=(A-IA),or(A-I),A'=I=A'(A- I ),
Hence, (A"I)' is the inverse of A'
(~.4')-1 = (A-I),

• ORTHOGONAL AND UNITARY MATRICES

(A) Orthogonal Matrix


Definition: A matrix A is called an orthogonal matrix. IfAA '=1 = A 'A, where I is an identio
matrix .
152 Remedial Mathematics

Theorem 1: ifA is an orthogonal matrix, then A-I is also an orthogonal matrix.


Proof: By definition, if A is orthogonal, then
AA'=A'A =1
or (AAT I =(A'Arl =]' [:.1' =1]
or (A- 1),A-1 =A-1 (A- 1),=1.
Hence A-I is orthogonal.
i.e., Inverse of an orthogonal matrix is also orthogonal.
Theorem 2: IfA and Bare n- square orthogonal matrices then AB and BA are orthogonal
matrices.
Proof: Since A and B are orthogonal matrices, we have
AA' =1andBB' =1 ... (1)
Now (AB)(AB)' = (AB)(B'A')
=A (BBJA' by associative law
=AIA' since BB'= 1 = AA'
Hence AB is an orthogonal matrix. Similarly we can show that BA is also orthogonal.

(8) Unitary Matrix


Definition: A square matrix A is said to be unitary if AB A = 1 where 1 is an identity matrix
and A Bis the transposed conjugate ofA . The elements ofA are complex numbers.
SinceAs=IAland IAAsl = IAIIA S I,thereforeifAA s =1,wehave IAI IAI = I.
Thus determinant of a unitary matrix is of unit modulus.
For a matrix to be unitary, it must be non- singular.
Hence AA s =/::::;.AsA = 1
i. e., AA s =1=A sA.

~~~~~~~I SOLVED EXAMPLES ~I~~~~~~

Example 1: find the adjoint ofthe matrix A = [1 2]


3 -5
and verify the theorem

A(adj A) = (adjA)A = IAIl.

Solution: We have IAI = [1 2] = 1(-5) - (3) 2 =- 5 - 6 =- 11 .


3 -5
The cofactors of the elements of the first row of IAI are - 5 , - 3 respectively. The
cofactors ofthe elements of second row of IAI are - 2 ,1 respectively. Therefore the matrix B
formed by the cofactors of the elements of IAI is

B = [-5 -3]
-2 1

adj A = transpose of the matrix B = [=~ ~2]


Matrices 153

Now A(adj A) = [1 2] [-5 -2]


3 -5 -3 1

= [-5-6 -2+2] = [-II 0]


-15+15 -6-5 0 -11

=(-11) [~ ~] = IAI I
Also (adj A)A = [=~ ~2] [~ ~5]
= [-5-6 -10+10]
-3+3 -6-5

= [ -~ 1 _~ 1] = (-11) [~ ~ ]
HenceA (adj A) = (adj A)A = IAII.
Example 2: Find the adjoint a/the matrix B

A~[~ iJ ~l
Solution: For the given matrix A, we have

All =1-12 -313 =3, A 12 =_1 21 3 =-9 '


-31
21=_5 '
-1
A 21 =_1 -11 311 =-4 '
1
11
3
= I
'
A 23 =_1 2 11
-1 =3 '
A31 =I~ ~31 =-5, A
32
= _III 11
-3 =4 '
A33 =I~ ~I =1.
Therefore the matrix B formed by the cofactors ofthe elements of A is

B= [~4
-5
-9 ~5l
4 1

Adj A = B'= [~9


-5 : ~l
154 Remedial Mathematics

Example 3: ifA = [~ ~ ~] ,find A2 - 2A + Acij A .


243

Solution: We have A2 = [~ ~ ~] [~ ~ ~]
x
24324 3

[1+0+6
= 0+0+0
2+10+12
0+25+0 0+0+0
3+0+9]
2+0+6 4+20+12 6+0+9

= 0 25
[7 24 12]
0 ...(1)
8 36 15

Also
All = I~ ~I = 15, AI2 =-I~ ~I =0.
AI3 =I~ !I =- 10 A21 =-I~ 31 =6
3 '

=I~
1
A22 31 =-3
3 '
A 23 =_1 2 21 =0
4 '

A31 =I~ o31 =-15


' A32 =-I~ o31 =0 '
A33 = I~ ~I =5.
.,
[ 15
B= 6
-15 0
0
-3
-n
.. AdjA =B'=
[150 -36 -15]
0 ...(2)
-10 o 5

., A'-2A+AdjA ~ [~
24
25
12]
o -2 [I0 2 3] [15 6 -15]
5 0 + 0 -3 0
36 15 2 4 3 -to o 5
(Using(l) and (2»

4 6] [15
10 0 + 0
8 6 -to
Matrices ISS

7 -2 + 15 24-4+6 12-6-15]
= 0-0-0 25-10-3 0-0-0
[
8-4-10 36-8+0 15-6+5

=
20
0
28
12
-9]
0
[
-6 28 14

Example 4: Find the inverse ofA = [~ :

3 5
Solution: For the given matrix A ,we have

A
11
=14 51=_1
5 6 ' A 12 =_1 23 51 =3
6 '

AJ3 = I~ ;1 =-2, A 21 =_1 25 31 =3


6 '

A22 = I~ !I =- 3, A 23 =_1 31 21 =-1


5 '

A31 = I! 31 =-2
5 '
A 32 =_1 21 31 = 1
5 '

A33 = I~ !I =0.

3
.. B= [-I3 -3 -2]
1 .
-2 0
3
.. [-I
AdjA =B'= 3 -3 -2]
1 .
-2 0

[I 2
Also IAI = 2 4
3 5 :]
= 1(24-25)-2(12-15)+3(10-12)
=-1+6-6
=-1.

Now
A-I ~ 01A {~ -~ -J [~ ~: ~I]
156 Remedial Mathematics

Example 5: ifA =
[
9
5
6
7
-J
8
n Findadjaint AandA-/ [RGPV B. Pharma 20021

Solution: The given matrix is A = [~ ~1 ~l


682

IAI = [~ ~1 ~] =9(-2-32)-7(10 -24) +3(40+6)


682
=-70.
as IAI :#: 0 so A-I will exist. The cofactors of the given matrix are
We have
All =-34 A21 = 10 A31 =31
AI2 = 14 A22 =0 A32 =21
AJ3 = 46 A23 = 30 A33=-44.

Adj A =
[-34
10
10
0 30
46] = [-34
10
10
0 21
31 ]
31 21 -44 46 30 -44

.. A-I = a~~
1
=_ 70
[-34
10
10
0 21
31 ]
46 30 -44

1 [34 -10 -31]


= --- -10 0 -21
70
-46 -30 44

Example 6, Find the in"",e afthe matrix [~ ; :J [RGPV B. pharma 2003]

Solution: The given matrix is

A= [~ ! ~]
2 7 11

IAI = 1(44-35)-2(33+10)+3(21-8)
=2.
as IAI :#:, so A-I exists.
Matrices 157

The cofactors ofthe given matrix A are:


All = 9 A21 =-1 A31 =-2
AI2 =-23 A 22 =5 A32=4
Al3 = 13 A 23 =-3 A33=-2

AdjA = [~1
-2
-:3 ~~11
4 -2
= [-~3 ~I ~21
13-3 -2

A-I = AI~IA ~ [-~3 ~I


=
~l
13 -3 -2

Example 7: Find the inverse ofthe matrix A =

Solution: We have IAI = 1(6-1)-2(4-3)+3(2-9)


[~ ~
3 1 ~l [UPTU B. Pharma 2001)

= 5 -2-21 =-18
Since IAI :t; 0, therefore £1 exists.
Let Ai} be the cofactor of ai} in IAI, Then we have
All =5 A21=-1 A31=-7
AI2=-1 A22=-7 A 32 =5
Al3=-7 A 23 =5 A33=-1

Now adjA = [~1 =~ ~71 [~1 =~ ~71 =


-7 5 -1 -7 5 -1

A-I =1~ladjA=-I~ [~1 =~ ~71 -7 5 -1

Example 8: Find the inverse ofthe matrix A = [; ~1 ~11 [U PTU B. Pharma 2001)
2 3 -1
Solution: We have IAI = (1 +3)-2(-1 +2) +5 (3 +2)
= 4-2+25 =27
Since IAI:t; 0, therefore A-I exists.

Let Ai} be the cofactor of ai} in IAI , then we have


. A I1 =4, AI2=-I, A 13 =5,
A21 = 17 A22=-ll, A23= 1
A 31 =3, A 32 =+1, A33=-3
158 Re~dilll Mathematics

Now adjA ~ [I;


-1
-11 T[4
1
-3
=-1
5
17
-11
1 JJ
~ I~I adj ~ 12~1 [ ~I
17
A-I A -II
1 JJ
Example " Find the invem ofthe mo"ix A ~ [~ 33 4]1 [UPTU B. Pharma 2002J
2 4

Solution: We have IAI =2(12-2)-3(16-1)+4(8-3)


= 20-45 +20 =-5.
Since IAI * 0, Therefore A-I exists.
Let Aij be the cofactor of aij in IAI
Then we have

Now

Example 10, ifA " [~ ~ ~], using A' - 4A - 51 ~ 0, ond hence find A-I

[UPTU B. Pharma 20003-2008)

Solution: We have
[~1 22 2~]
Matrices 159

2
A -4A-5/= [:
889221001
! :]-4[~ 2~]-5[~ ~ ~]
9 4 5 8-8-0
8-8-0]
= 8=8=0 9-4-5 8-8-0
[
8-8-0 8-8-0 9-4-5

Now we have to find A-I


~ [~ ~ n
Multiplying both sides of A2 - 4A - 51 = 0 by A-I, we get
A-I (A 2 -4A -51) = O.
A-I A -4A- IA-5A- I 1=0
2

or 5A-
1
~A-41~ [~ :

= [-; ~3 ~]
2 2 -3

~J
-3 2
-I 1
A = 5[ ~ ~3

~
2 -J
Example 11: Show that the matrix A = - J 2 ] satisfi., the equation
[
J -J
A3 - 6A 2 + 9A - 41 = O. Hencejind A-I. [UPTU B. pharma 20041

2-I I]
Solution: We have A = -I
[ 1
2 -I
-I 2

A2 = A.A= -I
[
1
2 -I
2
-I
~I] [~I ~I ~I] [~5
2 1 -I 2
=
5
-: ~5]
-5 6 .

~I ~Il [~5 ~ 51 [22 -21


-5 = -21 22 21]
-21
-I 2 5 -5 6 21 -21 22
160 Remedial Mathematics

Now A 3 -6A 2 +9A-41


22 -21
= -21 22 21; -6-5
-21 [ 6
[
21 -21 22 5

0 0 0]
= 0 0 0 =0.
[ 000
Now we have to find A-I
Multiplying both sides of A 3_ 6A 2 + 9A - 41 = 0 by A-I, we get
A-I (A 3 -6A 2 +9A-,4J) = 0
£1 A 3-6A- I A 2 +9A- I A-4A-I 1 = 0
A 2 -6A +91-4A- = 0
4A- I = A2-6A +91.

-[~ ~]-f' ~}9[~ ~]


-5 -1 0
6 2 1
-5 -1 0
1

-[: 3
:']
A-I - ±[: 3 ' -I] 1
-1 1 3

1 2 1]
Example 12: Given that A = 3 2 3 ,compute (i) det A (ii) Adj A (iii) A-I.
[112
Solution:We have

(i) IAI = [~ ~ ~]
I I 2
=1 (4-3)-2(6-3)+ I (3-2)
=1-6+1
=-4.
(ii) Now the cofactors of the elements of the first row of IAI are

I~ ~I'I~ :21'1~ ~1'i.e.,arel,-3,t.respectivelY.


Matrices 161

The cofactors of the elements of the second row of IAI are

-I~ ~I' I~ ~I' -I~ ~I i. e., are - 3 , 1 , 1 respectively.


The cofactors of the elements of the third row of IAI are

I~ ~I' -I~ ~I'I~ ~I i. e., are 4 , 0 , - 4 respectively.

~ [~ ~l
-3
B
0

~3 ~l
-3
AdjA = B'=
[
-4
1 3
-- - -1

[, 4 4

l
-3
.... Adj A 1 3 1
(m) The mverse of A = lAI
= - "4 ~3 1
4
--
4
0
I -4

il
4 4

Example 12: IfA = 0


[0 01 ,how thot A-I ~ A.
1 0

So'ution. W,have IAI ~ [: o1 0'] = [~ ~] =-1.


. 1 o 0
Now for the given matrix A, we have

All
[~ o0] =0' AJ2 =- [~ o0] =0
'
AI3
[~ o1] =-1 ' A21 =- [~ o1] =0 '
A22
[~ o1] =-1 ' A23 =-[~ o0] =0
'
A31
[~ o1] =-1 ' A32 = -[~ o1] =0 '
A33 [~ ~] =0.
162 Remedial Mathematics

Therefore, the matrix B formed the cofactor of the elements of IAI is

0 0 -1]
B = 0 -1 0 .
[
-I 0 0
Acij A = B' = transpose of the matrix B

= [~ ~l ~l].
-1 0 0

£1 = AcijA = _1_ [ ~ ~1 ~1]=[~ ~ ~]=A


IAI (-1)_1
o 0 1 0 0

~ ~2]
1
. 1
Example 13: Show that the matrzx A ="3 2 is orthogonal.
[
-2 2 -1

Solution: We have A = ~ 2
1 21 -22] .
[
-2 2 -1
1 2
Then A' = ~ 2 1
[
2 -2

We have AA'=.!.X.!.[~-2
3 3

9 0
= io [
0 9
0
Hence, the matrix A is orthogonal.

Example 14: Show that the matrix.


cose Sine]
[ -sme cose is orthogonal.
cose Sine] .
Solution:LetA = [
-sine cose

A' = [cose -Sine]


Then
sine cose
Matrices 163

cosO sinO] [COSO -sinO]


We have AA'= [ -sinO cosO sinO cosO
2 2
cos 0+sin 0 cososino-sinocosoj
I [ sin Ocos O-cos Osin 0 sin 2 O+cos 2 0

=[~ ~]=I.
Hence, the condition of orthogonality is satisfied.
Therefore, the given matrix is orthogonal.

1 +i -1+i]
2 . .
Example 15: Prove that the matrix 2. . IS umtary.
[ 1+ I I-I
2 2
I+i -1 +i]
'Solution: LetA = 2 2
1+; 1-;
. [-
2 2

Then

Now
AA=[ 1;; l;i][I;i -12+i]
9
-1-; l+i l+i l-i
- - -- -- --
2 2 2 2

Hence, the matrix A is unitary .


164 Remedial Mathematics

~ [~ ~]
0
1. Find the adjoint of matrix A 1
2

~ [i
2. Find the adj oint of the matrix A 2
-1 ~l
[~
3. Find the inverse ofthe matrix A =
~l
~
n
0
4. Find the inverne of the matrix A [: 1
0

~l
-2
5. Find the inverse ofthe matrix A = [ 22 3
-1 -1
2 2
3 3 3
2 2
6. Show that the matrix is orthogonal.
3 3 3
2 2 1
3 3 3
. A =.fi
7. Show that the matnx 1 [1 . i ] is unitary.
2 -/ -1

8. Find the inve"e of the matrix A ~ [~ ~ :]

9. Find the inverse ofthe matrix A , where

A ~ [~ =: ~l
10. Find the inverne of A ~ ~ ~l
[; [RGPV B. Pharma 2004]

11. Find the inverne of the matrix A ~ [! : ~1 [RGPV B. Pharma 20011


Matrices 165

I ANSWERS
[~2 ~2] [~9 ~]
4 -4
1. -5
-2
2.
-5
1
3
3.
[ ~7 ~2]

[~I-1 ~] ~ [-5 -~6]


[~I
0 2
4.
0
5. 0
10 5
2
0 10
8. 0
-1 7]
[~2 ~] ~[~
-1 2
9.
-2 3
3
10.
-3
2
7]
11. __I[~
67 11
-37
8 -1
18 -19
13]

• SOLUTION OF EQUATIONS USING INVERSE OF A MATRIX

In this section, we shall express the system of linear equations as matrix equation and solve
them using inverse ofthe coefficient matrix.
Consider the system of equations.
alx+bly+ clz = dl
azx + b2Y+ c2z = d2
a 3x + bJY + c 3z = d3 .

Lcl A = [:: f : ].
Then, the system of equations can be expressed in the form
X= [;]. B= [~:]

[~ ~
a3
.
v3
: : ] [ : ] = [::]
C3 Z d3
i.e., AX=B.
If A is non-singular matrix, then its inverse exists.
Hence, we have
A-I (AX) = £1 B
X=A-1B
This matrix equation provides solution for the variables x, y and z .
166 Remedial Mathematics

ISOLVED EXAMPLES I
Example 1: Solvefor x andy by inverting the matrix in the following

IUPTU B. Pharma 20011

Solution: We have IAI = 6 - 1 = 5 :# 0 . Therefore A-I exists.


Cofactors ofthe elements ofthe first row of IAI are 3 ,-I .
Cofactors of the elements of the second row of IAI are -I ,2.

Now adj A = [ 3 -I] 3 -I].


-I 2
T =[
-I 2

.. A-I = I~I adjA= ~[~I ~Il


I
X=A- B= ~[~1 ~IJ [~J = GJ
or
[;J = [~J
Hence x = l,y=2
Example 2: Solve using matrix method, the following equations
x+y=O, y+z=l, z+x=3, [UPTUB.Pharma2005]
Solution: The given system of equations can be written is matrix form as
AX=B.

where

We have IAI = 1 (1-0)-1(0-1)= 1 + 1 =2:#0.


Let Aij be the cofactor of aij is IAI . Then we have
A\1 = 1, A21 = -1, A31 = 1
A12=1 A22=1 A32=-1
A\3 = - 1 A 23 = 1 A33 = 1.

Now adjA = [~1 : ~llT = [: -1 ~ll


1 -1 1 -I 1

A-I ~ I~I adj A ~~ [I -I ~Il


Matrices 167'

We have X=A- B=
1
~[~
-1
Hence, x = 1, y=-I,z=2.

Example 3: IfA ~ [~ ~1 ~ 1 ,FindA- 1 .


1 -1
Hence solve the equations
x + y + z = 6, x - y + z = 2, 2x + Y - z = 1.
Solution: We have
IAI =1(1-1)-1(-1-2)+1(1+1)=0+3+3=6.
Since IAI :1= 0, therefore A is invertible.
Let Aij be the cofactor of aij in IAI . Then we have
A
11
= (_1)1 + 1 1-1
1
II I-I
-I
= = 0.

11
A =(-1)1+21 1=3.
12 2 -1
A13 =3, A21 =2, A 22 =-3, A 23 = I, A31 =2,
A32 = 0, A33 =-2.

Now adj A = [~ ~3 ~ 1= [~ ~3 ~ 1
2 ° -2 3 -2

A-I ~ I~I adj A ~ ~[: ~3 ~J


Writing the given system as a single matrix equation, AX = B. we get

~[~ ~I ]l[~l
~3 ~J [~l
or

Hence, x = 1, y=2,z=3.
168 Remedial Mathematics

Example 4: Solve the following equations by matrix method:


2x-y+3z=l, x+2y-z=2, 5y-5z=3, [UPTUB.Pharma2003,06]
Solution: The given system can be written as a single matrix equation
AX=B.

where

We have IAI = 2(- 10 + 5) + 1(-5 + 0)+ 3(5 -0)=-10-5 + 15 =0.


Since IAI = 0. Therefore, to judge whether the given system is consistent or inconsistent,
we shall calculate (adj A) B.
Let Aij be the cofactor of aii' then we have
All =-5, A 12 =5, A 13 =5, A 21 =0, A22=-10,
A 23 =-10, A31 =-5, A 32 =5, A 33 =5.

Now adjA~ [~~ ++H: ~:~ -!l


. (adjA)B ~ [: ~:: :ml {:~~E::H~l
Since IAI = 0, (adj A) B = 0, therefore the system of linear equations is consistent and
possesses infinite number of solutions.
Taking Z = K where K is real
Substituting for z in the first and second equations of the given system, we get
2x-y= 1-3k, x+2y=2+k.
Writing these equations as a single matrix equation AX = B, we get

[~ ~1] [;] = [~-:: l


Here IAI =4+1=5.
Cofactors of the elements of the first row of IAI are 2, - I.
Cofactors of the element of the second row of IAI are 1,2.

Now adj A = [21 _I]T = [ 2


2 -I 2
I]
A-I=~(adjA)=.!.[2
IAI 5 -1
I]
2

We have _I 1 [2 1][1-3k] 1 [2-6k+2+k] I [4-5k]


X=A B="5 -1 2 2+k ="5 -1+3k+4+2k ="5 3+5k .

1 1
=> x = - [4-5kl, y= - [3+5kl,
5 5
Matrices 169

1 1
Since x = - [4 - Sk], y = - [3 + Sk], z = k, k is real, also satisfy the third equation of
S S
the given system for all real values of x.
1
Therefore, x = - [4 - Sk]
S
1
Y = -S [3+Sk] '
z=k
form infinite solutions of the given system.
Example 5: Solve the following system ofequations by matrix method:
x + 2y + z = 4
2x + Y = 3
x +z = 2 [RGPV B. Pharma 2004]
Solution: The given systems is equivalent to AX = B :. A-I B.

Where

Now IAI ~ [~ : ~] ~ 10-0)-2(2-0)+ 1(0-1)~ 1-4-\ ~-4


The cofactors of A are
All = 1 A21 =-2 A31 =-1
A22 =0 A32 =2
A 23 =2 A33 =-3

Adj A = [~2 ~ ~1]1 [~2 ~2 ~1].


=
-1 2 -3 -1 2 -3

Then A
-I 1
=-"4-2
1
-2o -1]2
[ -1 2 -3

x=l,y=l, z=l.
170 Remedial Mathematics

Example 6: Using matrices solve the equations


2x-y + 3z =9
x+y+z=6 [RGPV B. Pharma 2001)
x-y+z=2
Solution: Given system is equivalent to AX = B.

where

X=A- I B.

~ ~2 (-1-1)~4-6~-2.
-1

(A) [: 1 :] (1+ 1)+ 1 (1-1)+3


-1
The cofactors ofthe elements of IAI are
All = 2 A21 =-2 A31 =-4
AI2 = 0 A22 =-1 A32 = 1
A\3 =-2 A 23 = 1 A33 =3

-2) [2
AdjA+~-4 ~]
0 -2
-1 1 = 0 -1
3 -2 1

~ ~~IA ~ ~IP-2 ~]
-2
A-I -1

Then X=A- I B

~~~~~~~I EXERCISE 4.31~~~~~~~~


Solve the following system of equations by matrix method
1. 5x+2y=4 2. 2x-y=-2
7x+3y=5 3x+4y=3
3. 4x-3y=3 4. 2x+5y= 1
3x - 5y = 7 3x + 2y = 7
Matrices 171

5. 2x+y+z= 1 6. x-y+z=4
x-2y-z= 3/2 2x+y-3z=O
5y-5z=9 x+y+z=O
7. 2x+3y+5z=5 8. x-y+2z=7
x-2y+z=-4 3x+4y-5z=-5
3x-y-2z=3 2x-y+3z= 12.
9. Solve the system of equations using matrix method
x+y+2z=4
x+3z= 5
2y+3x= 7 [RGPV B. Pharma 2001]
10. Solve2x+3y+z=9
x+2y+3z=6
3x+y+2z=8.
By finding inverse of coefficient matrix. [RGPV B. Pharma 20011

HINTS TO THE SELECTED PROBLEMS

4. The system of equations can be written in the form AX = B, where

Now IAI = -11 ,.to.


All = 2,A I2 =-3,A 21 =-5,A 22 =2.

Hence A-I = - /1 [~3 ~]


X=A -I B=--
II -3
1[2
= - 1\ [~313] = [~1]
x = 3,y=-1.
8. IAI =-17,.tO.
All =-IA I2 =-8A 13 =-1O
A21 = -5A 22 =-6A 23 = 1
A31 =-IA 32 =9A 33 =7.

Hence, A-
1
~ ~ ~~o ~ ~ll
- 1 [
172 Remedial Mathematics

-I
X = [IB=_ I~ -8
[
-10

we have x =1,y=2,z=3.

I ANSWERS
5 12 6 -19
1. x=2,y=-3 2. x=-ll ,y= II 3. x=-ll ,y= I t
1 3
4. x= 3,y=-1 5. x= \,y= 2'z=- 2

6. x=2,y=-1,z= 1 7. x= I,y=2,z=-I 8. x= 1,y=2,z=3.

• RANK OF A MATRIX

Definition: A positive integer r is said to be the rank ofa matrix A if it contains at least one
square submatrix oforder r x r, whose determinant is non-zero while any square submatrix
ofA of order (r + 1) x (r + I) or greater is singular i.e., having determinant zero. The rank
ofa matrix A is denoted by p (A).
It is obvious that the rank r of a matrix of order m x n may at most be equal to the smaller
of the numbers m and n, but it may be less.
I f the rank of a square matrix A of order n x n is rand r < n, then matrix A is said to be
singular, on the other hand if r = n, then the matrix is said to be non-singular.
Remarks
• If the rank of a matrix is zero, then matrix is a null matrix
• The rank of every non-zero matrix must be greater than or equal to 1.
• The rank of a unit matrix is equal to the order of the unit matrix.

III ECHELON FORM OF A MATRIX

Definition: A matrix A is said to be in Echelon form ifit satisfies following conditions:


(i) Every row of A has all its entries zero which occurs below the every row having a
non-zero entry.
(ii) The number of zeros before the first non-zero entry in the same row is less than the
number of zeros in the next row.
Matrices 173

Remark
• The rank ofa matrix is equal to the number of non-zero rows in Echelon form of the
given matrix.
For example:

Let
0 2 33 25]
A= 0 0
[0 0 0 0
This matrix A is in Echelon form and it has two non-zero rows since rank ofA is equal
to the number of non-zero rows. Hence rank of A = 2.
Theorem 1: The rank ofthe transpose ofa matrix is the same as that ofthe original matrix.
Proof: Let us suppose A is any matrix and A I is its transpose and let rank of A = r. This implies
that A contains at least one r-rowed square matrix whose determinant is non-zero, let it be B.
Obviously B' is a submatrix of A' but we know that det B' = det B and since det B *- 0 => det
B' *- O. Thus the rank of A' ?r. Now if A contains a (r + 1) -rowed square submatrix C, then det
C = 0 because rank of A = r. Obviously C' is a submatrix ofA' and det C' = det C = 0, it follows
that A does not contain (r + 1)-rowed square submatrix with non-zero determinant. Hence
I

rank of A'::; r and consequently we obtained rank of A' = rank of A .

• ELEMENTARY TRANSFORMATIONS OF A MATRIX

Definition: A transformation is said to be elementary transformation ~r it is one of the


following:
(1) Interchanging of any two rows (or columns).
(ii) Multiplying any row (or column) by any non-zero number.
(iii) Addition of any row to K times the other row, where K is any non-zero number.
Remarks
• lfthe elementary transformation (or E- transformation) is performed on rows, then it is
called row-transformation.
• If the E-transformation is performed on column, it is called column-transformation .

• ELEMENTARY MATRICES

Definition: A matrix which is obtained by a single E-transformation is called an elementary


matrix. For example
0
1 o , 0 0I :] cto
']['
[: 0 o 0 0

A~ [~ ~']
-10
8
-4 -2
174 Remedial Mathematics

Here first E- matrix is obtained from 13 by interchanging C 1 and C3 columns and the
second E-matrix is obtained by RI~R2 + 2R2.
Remarks
• All the elementary matrices ar~ non - singular.
• Each elementary matrix possesses its inverse.

III INVARIANCE OF RANK UNDERE-TRANSFORMATION


Casel. Elementary transformation (E- transformation) do not change the rank ofa matrix.
Case ll. Multiplication of the elements of a row by a non-zero number does not change the
rank.
Case ID. Addition of any row to the product of any number K and other row does not
change the rank.
Remarks
• The rank of a matrix does not change by a series of E-transformation.
• The rank ofa matrix does not change by a column-transformation.

III NORMAL FORM

Definition: If a matrix is reduced to the form (I; ~) . Then this form is called normalform
of the given matrix.

Theorem 3: Every matrix oforder m x n ofrank r can be reduced to the form


(
Ir
0 ~) bya
finite number ofE-transformations, where Ir is the unit matrix of order r x r.
Remark

• The form ( ~ ~) of A is also called first canonical form.


Corollary 1. The rank ofmatrix oforder m x n is r ifitcan be reduced to (~ ~) byafinite
number ofE-transformations.
Corollary 2. IfA is a matrix oforder m x n ofrank r, then there exist non-singular matrices
P and Q such that

PAQ = (~ ~)
III EQUIVALENCE OF MATRICES
Definition: Let A be a matrix oforder m x n. Ifa matrix B oforder m x n is obtainedfrom A
by afinite number ofE-transformations, then A is called equivalent to B. It is denoted by A
- B (Read as A is equivalent to B).
Matrices 175

Theorem 4: The relation "-" in the set ofall m x n matrices is an equivalence relation.
Proof:
(l) Reflexivity. IfA is a matrix of order m x n, then A is equivalent to A i.e., A-A.
(il) Symmetry. Let A and B be two matrices of order m x n and A-B. This implies if B is
obtained from A by a finite number of E-transformation, then A can also be obtained
from B by a finite E-transformations. Hence B - A.
(iii) Transitivity: LetA, B, C, be three matrices of order m x n and A - B, B -CO This implies
that of B is obtained from A by a finite number of E-transformation and C is obtained
from B by a finite number of E - transformations, then C can also be obtained from A
by a finite number of E-transformations. Hence A - C.
Hence the relation "-" is an equivalence relation.
Remarks
• An equivalence relation is a relation which is reflexive, symmetric and transitive.
• Two equivalent matrices have the same rank.
• Two matrices of same order and of same rank are always equivalent.

_ _ RANK OF PRODUCT OF MATRICES

Theorem 7: The rank ofa product oftwo matrices cannot exceed the rank ofeither matrix.
Proof: Let A and B be two matrices of order m x nand n x p respectively. Let r 1 and r2 be the
ranks of A and B respectively and let r be the rank ofAB. We shall prove that r :S;r l and r:S; r2 .
Since the rank of A is rl' then there exist a non-singular matrix P such that

PA = [~] ... (i)

where G is a matrix of order r1 x n ofrankrl and 0 is a zero matrix oforder(m-r l ) n. Now post
multiplying both sides of (i) by B, we get

PAB = [~] B. ...(ii)

Since we know that


rank of (PAB) = rank of (AB)
rank of(PAB) = r [:. rank of (A B) = r]

or rank of [~] B = r. ...(iii)

Since rank ofG is r l so it has only r l non-zero rows, therefore the matrix.

[~] B
Cannot have more than r 1 non-zero rows. Thus we have

rank of [~] B :s; r l


176 Remedial Mathematics

or r:::; r l [from(iii)]
or Rank of (A B) :::; RankofA. ... (iv)
Further since we have rank (AB) = rank (AB)' and (AB)' = B' A'.
.. rank of (A B) = Rank of (B'A'):::; rank of B' using (iv)
or rank of (A B) :::; rank of B' = rank of B
or rank of (A B) :::; rank of B
or r :::;r2 .

Remark
• The rank of matrix does change by pre (post) multiplication with a non-singular matrix.

~~~~~~~I SOLVED EXAMPLES ~I~~~~~~


Example 1: Determine the rank a/the/allowing matrices

Solution: (i) The square submatrices of the given matrix are

AI = [~ ~
3
:] A2 =
6 9 3
[~ ~ 6
:] A3 =
12
[~
:
3 9
:1]
12
A4 = [~
6
:
9
: ]
12
det AI = 1 (36-36) +2 (18-18) + 3 (12 - 12)= 0
detA 2 = 1 (48-48)+2(24-24)+4(12-12)=0
detA 3 = 1(72-72)+3(24-24)+4(18-18)=0
det A4 = 2(72-72)+3 (48-48)+4(36-36)=0.
Therefore, determinant of all square submatrices of the given matrix of order 3 x 3 are
. zero so the rank of the given matrix is less than 3. Now the square submatrices of the given
matrix of order 2 x 2 are

[~ ~l[~ l~l[: ~l[: l~l[~ l~l[~ ~l[~ ~]


[~ l~l[~ ~l[~ l~l[! l~l
Obviously, the determinant of all square submatrices of order 2 x 2 are zero. Thus the
rank ofthe given matrix is less than 2. Since the given matrix is non-zero matrix. Hence, the
rank of the given matrix is 1.
Matrices 177

(il) A = [~ ! ~l
456
det A = 1(24-25)+2 (20-18) + 3 (15 -16)
=-1+4-3=-2+2=0
Therefore the rank A #:-3.
Now the square submatrices of A of order 2 x 2 are

Al = G !J.A2= G ~] etc.

detA I = 4-6 =-2 #:-0.


Hence the rank A = 2.

Example 2: IfA = l~ ~ ~ ~1'


000 1
Find the rank ofA and A2.

o 000
Solution: Since the matrix A is in Echelonform and there are three non-zero rows. Therefore,
rank of A is equal to the number of non-zero rows. Hence, rank of A = 3.
NextfindA 2

l~ m~ ll~ ~J
1 0 1 0 0 1
0 0 1 0 0
A2 =
0 0 0 0 0 0
0 0 0 0 0 0
Obviously, A2 is an Echelonform and having two non-zero rows. Hence the rank of
A2=2.
Example 3: Use E-transformation to reduce the following matrix A to triangular form and
hence find the rank ofA.

A~U
1
3 2 2
36]
-1 -3 4
Solution: Since we have

A~[j
!l
I 3
3 2
-I -3

-[ ~-I -I -3
3
3 2 ;] byC,-> i [i
C,-
0 0 10
1 3
3 2 ~l by R3 -> R3 + R,o

This matrix is a triangular matrix (Echelon form) and it contains three non-zero rows.
Hence, the rank of A = 3.
178 Remedial Mathematics

Example 4: Reduce the ma"ix A

hence, find the rank ofA.


~~l -1 2
1 0
3 0
1 0

Solution: Since the given matrix is

A ~ l~ -~ ~ -~1
Performing C 2 ~ C 2 + C j , C 3 ~ C 3 -2C j , C4 ~ C 4 + 3C j

-l~ 1~1
0 0
5 -8
3 0
0

-l~ 1~1
0 0
5 -8
Performing R2 ~ ~ - 4R j
3 0
0

-l~ 1~1
0 0
0
Performing R2 B R4
3 0
5 -8

{ -~1
0 0
0
Performing C 4 ~ C4 -2C2
3 0
5 -8
Performing R3 ~ R3 - 3R2, R4 ~ R4 - 5R2

-r~ -~]
0 0
0
0 0
LO 0 -8

-l~ -~1
0 0
0
Performing C 3 B C4
0 -2
0 4
Matrices 179

-[~
0 0
0
Performing R3 ~ - .!.2 R3 0 1
0 4 lj
[~ lj
0 0
1 0

- 0 1
0 0

[~ ~]
0 0
. 1 0
Perfiormmg R4 ~- 8" R4 0 1
-
0 0

Hence the rank of A = 4.


Example 5: Find two non-singular matrix P and Q such that PAQ is in the normal form
where
1 1
4 = 1 -1
[
3 1
Solution: Since we have

i.e., ... (1)

Now applying E-transformation on the matrix A on the L.H.S. of(1) until A reduced to
the normal form. In this process we apply E-row transformation to pre-factor 13 ofR.H.S. of(1)
and E-column transformation to post-factor 13 ofR.H.S. of(l). Now performingR2 ~ R2 -RI'
R3 ~ R3 -3R J , we get

[ ~o ~2
-2
~2l [~1 ~ ~lA[~ 0 ~l
-2
=
-3 0 1 0 0 1
180 Remedial Mathematics

~J[! ;H~
0

[~ ~ll
0 -1
1
Performing R2 ~ - !2 R2 1 --
2
-2 0
0

~H~
0

[~ r] "[i ~1]
0 -1
1
Performing R3 ~ R3 + 2R2 1 1
2
0 0
-1

[~ ~] "[i ~H~
0

~1]
0 -1
Performing C3 ~ C3 - C2 1 1
2
0 0
-1

.. (I~ ~) =PAQ

P"[~ ~H~
0

~1]
-1
where 1
2
0
-2 -1
Hence rank of A = 2.
Example 6: With the help of E-transformation, find the rank ofthe following matrix
1 2
3 0

Solution: Let
li -2 -3
2 -i1
1 2
3 0

A" fi -2 -3
2 -i1
Matrices 181

1 1
o 1
Perfonning R3 ~ R2 + 3R2 ~
[
o 0
o 0
This is an Echelon form and having three non-zero rows. Hence, the rank of the given
matrix = 3.

1 1 1]
Example 7: Find the rank ofthe matrix A = a b c a, b, c being all real numbers.
[ 3
a b3 c3

IAI+ I]
Solution: Let

C,-+C,-C,
b c ,
C3~C3-Cl
a3 b3 c3
0 0
a b-a c-a , expanding vector R,
a 3 b3 _a 3 c3 _a3

= (b-a)(c-a) 1 2
b +ab+a 2,
1
C
2 1 21 C2
+ca+a
~ C2- C I

= ..
(b-a)(c-a) 1 I I 1
b 2 +ab+a2 , c 2 +ca-b 2 -ab
= (J-a)(c-a)[(c 2 +ca-b 2 -ab)-O]
= (b - a) (c- a) [(c 2 - b2 ) + a (c- b)]
= (b - a)(c-a) [(c- b)(c + b + a)].

IAI = (a-b)(b-c)(c-a)(a+b+c). ...(A)


Now following cases arise.
Case I. Let a = b = c, then

Therefore all minors of order 3 and 2 of A are zero. Also as no element of A is zero, so A
has non-zero minors of order 1.
Hence rank (A) = 1.
182 Remedial Mathematics

l~
Case II. Let a = b :#= c, then

A=
3
a
3
~ 1 °as C and C are identical.
3
= 1 2
a a c
Also A have a minor of order 2 viz.

I~ ~I =c-a:#=O.
Hence, rank (A) = 2.
Similarly, we can discuss the cases b = c :#= a and c = a :#= b.
Case III. Let a, b, c be all different such that a + b + c = 0.
By above discussion IAI = O.
Also, A has a minor of order 2

I~ ~I =b-a:#=O
Case IV, Let a, b, c be all different but a + b + C :#= 0. In this case from (case I), it is evident that
IAI i.e., Ahas a non-zero minor of order 3.
:#= 0,
Also A has no minor of order greater than 3.
Hence, r(A) = 3.

I EXERCISE 4.41
Determine the rank of the following matrices:
Matrices 183

10. [~ -~ -~ -!]
5 3 3 11

11.
[9
1
-3 ~ ~]

13. [I~ ~ :-~]


16 4 12 15
1

15. (a) ~1 2
_:
-2 2311]
[
I 2 0
16. Reduce the following matrix to its Echelon form and find its rank:

[~ ~ l~
I 3 4
(a) 3 9 12 (b) 401]
[
-1 -3 -4 I -1 2
17. Reduce the following matrix to normal form and find its rank:

(a) [;

1 -2 0
~ ! -1] (b)
I 24 3]
[3 6 107
2

18. Reduce the following rltatrix to normal form nI)O find its rank:

[~ ;~ ~ ~J
19. Find the ranks of A, B, A + B and AB, where

1 I -1] [-1 -2 -IJ'


A= 2
[3 -2
-3 4,
3
B= 6
5
12
10
6 .
5
20. If A and B are two equivalent matrices, then show that rank A = rank B.
184 Remedial Mathematics

21. Change the following matrix A into normal form and find its rank

WA ~ U_~ -! -~l (i,) A ~ [~ ~ ~l :

HINTS TO THE SELECTED PROBLEMS

1. A = [~ ~ ~l
012
detA = 1 (2-0)-2(4-0)+3(2-0)
=2-8+6=0
:. Rank A <3
Now the square submatrix of the given matrix are

A, = [~ ~ l A2 = [~ ~] etc.
and det A, = 1 - 4 = - 3 *0
Hence Rank A = 2.

Obviously det A = 0 => Rank A < 4


Also determinant values of all square sub matrices order 3 x 3 and 2 x 2 are also zero.
Thus Rank < 2. Since A is not a null matrix. Hence Rank (A) = 1

3. A = [~ ! ~l
456
Now detA = 1 (24-25)-2 (18-20)+3(15-16)
=-1+4-3
=0
Rank(A) < 3.

The square submatrices of A are Al = [~ !~], A2 =[! !] etc.

and det(A,) = 4-6=-2*0


Rank(A) = 2
Matrices 185

4. LetA =
1 25 -70 85]
0
[o 0 0-8
Since the matrix A is of Echelon form and contains three non-zero rows. Hence Rank
ofA is3.
2 -1

5. A=
o 3 3 4]
4 1
2 3 7 5
[
2 5 11 6

. 1
Performmg R2 ~ "3 R2

4]
1/3

-1I~

Thus A is converted into Echelon form, having 3 non-zero rows. Hence Rank of
A=3.

6. LetA = [-~ -~ -~ =~]


1 0 1 I
o 1 1-1
Then use the following steps:
(1) Performing R2 B R]
(2) Performing R2 ~ R2 + 2R], R3 ~ R3 - R J
(3) Performing R4 B R2
(4) Performing R3 ~ R3 + 2R2, R4 ~ R4 - 3R2, we get
186 Remedial Mathematics

[~ ; i =~]
Then A is converted to Echelon form having two non-zero rows.
Hence Rank of A = 2.

2 3 -1 -4-I]
1 -1 -2
7. A=
[6 3 0-7
3 1 3-2

Then using the following steps:


(1) Performing R2 ~ Rl
(2) PerformingR2 ~ R2 -2Rp R3 ~ R3 -3R p R4 ~ R4 -6R t
.
(3) Performmg R2 ~ '51 R2
(4) Performing R3 ~ R3 - 4R2, R4 ~ R4 - 9R2
(5) Finally performing R4 ~ R4 - R3
1 -1 -2

A-
o
o
1 3/5 -4]
7/5

[
0 33/5 22/~
o 0 0
Thus A is converted into Echelon form and having 3 Non-zero rows.

:~ ~l
Hence the Rank of A = 3.

abO
c d 1
Here Rl = R3 and R2 = R4
Let A = 0 => Rank of A <4
Also, all the square submatrices of order 3 x 3 having two rows or two columns
identical so that their determinant value will zero. Thus rank of A< 3. Further the
square submatrices of order 2 x 2 are

At = [~ ~lA2= [: !] etc.
and det At = c - 0 = C ;a!: 0
Hence Rank of A = 2.
Matrices 187

10. ~tusasswneA~ [~ -2 0 6] 202


-1 0 3
-2 1 2
Performing R3 ~ R t

-[~ ~p ~]
Then, using the following steps:
(1) PerformingR2 ~R2-4Rt,R3 ~ R3 -2Rt, R4 ~ R4 -R t
(2) Performing R4 ~ R2
(3) Performing C2 ~ C2 + C t , C4 ~ C4 - 3C t
(4) Performing R4 ~ R4 + 6R2
(5) Performing C3 ~ C3 + C2> C4 ~ C4 - C2
(6) PerformingR3 ~R4
. 1
(7) Performmg R3 ~ 6' R3
. 8
(8) Performmg C4 ~ C4 + 3' C3

(9) Finally perl'onning R, -+ H) R ,. we get A - [ ~


or

Thus A is reduced to normal form and hence the rank of A = 3.


19. Rank of A:
det A = 1(-9+ 8)-1 (6-12)-1 (-4 + 9) =-1 + 6-5 =0
=> Rank of (A) < 3
The square submatrices of A are

A =
t
[12 -31] [1-3 -I]4 etc.
,A 2 =

and detA t = -3 -2 =-5 *0 => Rank (A) =2.


Hence RankA = 2

-1 -2 -Ij
Now B = 6
[ 12 6
5105
188 Remedial Mathematics

Performing R2 ~ R2 + 6R], R3 ~ R3 + 5R]

_[~1 ~ ~1]
:. B is reduced to Echelon form, which having one non-zero row.
Hence, Rank of B = 1.

Further A+B =
[
1 1 -1] [-1
2 -3 4 + 6
-2
12 -1] [0
6 = 8
-19 -2]
10
3 -2 3 5 10 5 8 8 8
det (A + B) = -1 (80-64)-2 (64-72)
=-16+16=0
Rank of (A + B) < 3
Now the square submatrices of (A + B) are

(A + B)] = [~ ~Il [~ -2]


10
etc.

det(A +B)] = 0+8=8;t:0


~Rankof(A + B)=2.

Next, AB = [~ ~3 ~I][~1 ~~ ~ll = [~ ~ ~l


3 -2 3 5 10 5 0 0 0
:. AB is Null matrix, then Rank of (AB) = 0
20. Since A and B are equivalent. Then by definition, B is obtained by a finite chain
E-transformation applied on 'A' and vise-versa and further since we know that
E-transformation do not change the Rank. Hence
Rank A = Rank B
21. Since we have

Performing R2 ~ R2 - 2R], R3 ~ R3 + R]
1 2 -1
A ~ 0 0 5
[
o 0 5
Matrices 189

~ [~ ~ ~ -~l
. 0 000
1
Perfonning R2 ~ "5 R2
o
[~ -3/~l
0
o
- o 0

or A~ (I~ ~J
Thus A reduced to nonnal form and hence rank of A = 2.

ANSWERS
1. 2 2. 3. 2 4. 3
5. 3 6. 2 7. 3 8. 2
9. 4 10. 3 11. 2 12. 3
13. 2 14. 4 15. (a) 3 (b) 3 16. (a) 2 (b)3
17. (a) 3 (b)2 18. 3
19. Rank A = 2, Rank B = 1, Rank (A + B) = 2 RankAB= 0
21. (i) 2.

• LINEAR EQUATIONS
In this section we shall study of two type of linear equations:
(1) Homogeneous linear equations
(2) Non-Homogeneous linear equation.

1. Homogeneous Linear Equations


Let us consider a system of linear homogeneous equation as follows:
=O}
a\\x\ +al2x2 +···+alnxn

a2l~1:~~~'.~:::~'.:.~:~.~ . ... (1)

amlxj +an12 x 2 +···+amnxn-O


190 Remedial Mathematics

These are m equation in n unknowns. Any set ofnumbersx l ,x2 , ... xn that satisfies all the
equations (1) is called a solution of(1).
Trivial Solution: The solution xI = 0, x 2 = 0, ... xn = 0 of the equation (1) are called trivial
solution.
Non -trivial solution: Any other solution other than trivial, if exist, is called a non-trivial
solution of equations (1).
Let the coefficient matrix be

A=
l a~.I.
all

amI
al2
a22

am2
o
o
o

Xn nxl
o mxl
Then the equation (1) can also be written as
AX=O. ...(2)
This equation (2) is called a matrix equation.
Theorem 1: If Xj and X 2 are two non-trivial solutions of (2), then kjX j + k72 is also a
solution of (2), where k j and k2 are any arbitrary numbers.
Proof: Since the equation (2) is AX = 0
and AXI= 0, AX2 = 0 are given.
Now consider A (kIXI + kzX2) = kl (AXI) + k2 (AX2) = k (0) + k2 (0).
Hence, klXI + kzX2 is the solution of (2).

III NATURE OF THE SOLUTION OF EQUATION AX = 0

Since AX = 0 is a matrix equation of a system of m homogeneoll,,') linear equations in n


unknowns and A is a coefficient matrix of order m x n. Let the rank of A be r. Then obviously
r can not be greater than n. So, that either r is n or r is less than n. Therefore these are
following cases:
Case I: Ifr= n, then the equationAX=O will have no linearly independent solution. So in this
case only trivial solution will exist.
CaseII: Ifr < n, then there will be (n-r) linearly independent solution ofAX= 0 and thus in
this case we shall have infinite solutions.
Case III: Suppose the number of equations are less than number of unknowns i.e., m < nand
since r :s; m, then obviously r < n thus in this case a non - zero solution will exist. Therefore
the equation AX = 0 will have infinite solution.
Matrices 191

~~~~~~~I SOLVED EXAMPLES ~I~~~~~~


Exanple 1: Find the solution of the following system of linear homogeneous equation
2x\-x2 +x3 =0
3x\ +2x2 +x3 =0
x\-3x2 + 5x3 =0.
Solution: The given equation in mattix form AX = B can be written as

[: ~: m:} [~]
A ~ ~: ~]
[:

=> Performing R2 ~ 2R2 - 3R\, R3 ~ 2R3 - R\

[~ ~: ~,]
Now R3 ~ 7R3 + 5R2

0> [~ ~J[~] ~ m
:'

Hence rank of A =3, i.e., r= 3


So, trivial solution exists which.is given by
2X\-X2 +X 3 =0
7x2 -x3 = 0
58x3 = 0
2x\ =0
7x2 =0

X = - =0
o
58
3
x\ =0,x2=0,x3 =0.
Interchanging R\, R2 and R3, we get

~
1 0 0] =/3'
[o 0 -1
0 1 0

Hence, the rank of A is 3 and equal to the number of unknowns x\, x 2 and x 3 •
Hence,
192 Remedial Mathematics

Example 2: Find the solution o/the/ollowing system o/linear homogeneous equations:


x+y+z=O
2x-y-3z = 0
3x-5y+4z =0
x+ 17y+4z = O.
Solution: The coefficient matrix is given by

A=
[ ~ ~1 ~31
3 -5 4
1 17 4
First reduce A into Echelon form
Performing R2 ~ R2 - 2R 1, R3 ~ R3 - 3R 1, R4 ~ R4 - R1

1
-[i ~ d
Performing R2 ~ "3 R2, we get
1 1 1
5
0 1
3
0 -8 1
0 16 3
Performing R3 ~ R3 + 8R2, R4 ~ R4 - 16R4, we get
111
o 1 5
3
o 0 43
3
o 0 71
3
. 3
Performmg R 3 ~ 43 R 3, we get

1 1 1

o 1 5
3
001
71
o 0
3
Matrices 193

11
Performing R4 ~ R4 + "3 R3, we get

1 1 1
5
0 1
3
0 0 1
0 0 0
This is an Echelon form and having three non-zero rows so A has the rank 3. Since there
°
are 3 unknowns, hence a trivial solution exists here i.e., x = 0, y = 0, Z = is the only solution .

• NON-HOMOGENEOUS EQUATIONS
Let us consider a system of equations which are non-homogeneous as follows:

aUxl +a12 x 2 +···+alnxn =ht }

:,l;:;:::~~~;.~::~;:::':~?~,:
...(1)

There are m equations in n unknowns. Let

A = [::: :::
Cltn]
a2n Xl]
X2 [htb2 ]
X= B=

amI a m2
[
a mn IIIxn Xn nxl bm 1IIxi

Then the system of equations (1) can also be written as


AX=B. ...(2)
This equation is called matrix equation. If Xl' x 2 , .. .xn simultaneously satisfy the equation
(2), then (xI' x 2' ...x n) is called the solution of(2).
Consistency and inconsistency: When there will exist one or more than one solution of the
equation AX = B, then the equations are said to be consistent otherwise said to be inconsistent.
Augmented matrix: The matrix ofthe type

[AlB] = [ ; : ~: Cltn
a2n ht
b2 1
amI a m2 ~:I/n ~,:!
is called the Augmented matrix of the equations .

• CONDITIONS FOR CONSISTENCY


Theorem 1: The equation AX = B is consistent ifand only ifthe rank ofA and the rank ofthe
augmented matrix [AlBJ are same.
194 Remedial Mathematics

Proof: Since the equation is AX = B ...(1)


The matrix A can be written asA = [Cl' C2 ... Cn ]
Where Cl' C2 ",Cn are column vectors. Then the equation (1) can be written as

or xlC l + x 2C2 + '" + xnCn = B. - ... (2)


Suppose the rank of A is r, then A has r linearly independent columns. Let these columns
be C l , C 2 ""Cr and C l , C2""Cr are linearly independent and remaining (n - r) columns are in
linear combination ofC I, C2 ... Cr .
Necessary condition: Suppose the equations are consistent, there must exist kl k2 ... kr such
that
kl C l + k2C2 + ... + knCn = B. ...(3)
But Cr + l' Cr +2' ... , Cn is a linear combination ofC l , C2 ... Cr, then from (2) it is obvious
that B is also a linear combination of C l , C2, ",Cr and thus [AlB] has the rank r. Hence, the
rank of A is same as the rank of [AlB].
Sufficient condition: Suppose rank A = rank [AlB] = r. This implies that [AlB] has r linearly
independent columns. But Cl' C2 , ",Cr of [AlB] are already linearly independent.
Thus B can be expressed as
B = klC I + k2C2 + .. , + krCr. ...(4)
where k]l ~ ... kr are scalars.
Now, equation (4) becomes
B = klC I + k2C2 + ... + krCr + O.Cr + I + ... + O.Cn· ...(5)
Comparing (2) and (5), we get
xl =k l ,x2 =k2 , ... xr=kr,xr + l =0, ... xn=O
and these values of xI' x 2 ...xn are the solution of AX = B. Hence, the equation are consistent .

• CONDITIONS FOR ASYSTEM OF N-EQUATIONSIN


N-UNKNOWNS TO HAVE A UNIQUE SOLUTION

Theorem 1: if A be an n-rowed non-singular matrix, X be an n x 1 matrix B be an n x I


matrix, then the system of equation AX = B has a unique solution.
Proof: If A be an n-rowed non-singular,matrix, the ranks of both the matrices A and [AlB] is 17.
Therefore the system of equations AX = B is consistent.
Multiplying both sides ofAX= B by A-I, we have
A-I AX=A- l BorIX=A- 1 B
or X = A-I B is a solution of the equation AX = B.
To show that the solution in unique, let us suppose that Xl and X 2 be two solution of
AX=B.
Matrices 195

Then AXI = E, AX2 = B => AXI = AX2


A-I AX = A-I AX
I 2
=> IXI = IX2 => XI = X 1·
Hence, the solution is unique.
Remarks
• IfrankA"# rank of[AIB], then there is no solution.
• If r = n, then there will be a unique solution.
• If r < n, then (n - r) variables can be assigned arbitrary values. Thus there will be
infinite solution and (n - r + 1) solution will be linearly independent.
• If m < nand r ~ m < n, then equation will have infinite solutions.

I SOLVED EXAMPLES ~I~~~~~~


Example 1: Express the following system ofequations in matrix form
9x+ 7y+3z = 6; 5x+y+4z= I; 6x+ 8y+2z=4.
Solution: The given equation are
9x+7y+3x = 6
5x+y+4z = I
6x+ 8y+2z = 4.
:. The required matrix form of these equations is given by AX= B

when A" [:

Example 2: Solve by matrix method


~ ~l;X" m MdB" m
x + y + Z = 6, x - y + Z = 2, 2x + Y - z = l.
Solution: The given equation are
x+y+z=6
x-y+z=2
2x+y-z=l.

Let

and assume that the... exists a matrix X" [~l such that AX" B.

Then [~ ~1 iJ[l[H
196 Remedial Mathematics

Performing R2 ~ R2 - R\ and R3 ~ R3 - 2R\

we get [~~2
o
~] [~]
-1 -3 z
= [ :4]
-11
Performing R \ ~ R2 + R3

~ [~;~ ~2][~] ~ [~l:J


=> x-2z =-5,-2y=-4,-y-3z= 11
=> x-2z = -5,y=2, 2 + 3z= 11
=> x =2z-5,y=2,z=3
=> x = I,y=2,z=3.
Example 3: Solve the/allowing equation by matrix method:
x+y+z=9
2x+5y+7z=52
2x + y-z =0.
Solution: The given equation can be written as

i.e., AX=B.
:. Augmented matrix is

[AI~ ~
H
1 1
[: 5 7
1 -1 :
Performing R2 ~ R2 - 2R\ and R3 ~ R3 - 2R\, we get

[A~ [~ ~1 ~al[l [~:l


-

Performing R2 ~ ~ + 3R3

~ [~o ~ ~4][~] [-~o].


-1 -3 x
=
-18
The matrix equation is equivalent to the equations
x+y+z=9
-4z =-20
-y-3z =-18
which gives x = 1,y= 3,z= 5.
Matrices 197

Example 4: Solve the following equations by matrix method


x-2y + 3z =6
3x+y-4z =-7
5x-3y+2z = 5.
Solution: The given equation can be written as

i.e., AX=B
,'. Augmented matrix is

[A~l ~
-2 3 :
[: 1 -4
-3 2
;7]

This is an Echelon form and having two non-zero rows and rank A = rank [A I B] = 2.
Thus, the equation are consistent.

i.e., x-2y+3z = 6
7y-13z = -25.
Let z = c then
25 13 8 5
y =- - + -candx=- - +-c
7 7 7 7'
Hence, the solution is
8 5 25 13 h' b'
x = -;:; + ;:; c,y = -7 + "7 c, z = c, were c IS an ar Itrary constant.

Example 5: Solve the following equation by matrix method:


2x+3y+z=9
x+2y+3z = 6
3x+y+2z = 8.
198 Remedial Mathematics

Solution: The given equation can be written as

[~ ml[:]
3
2
1
i.e., AX=B

:. Augmented matrix is [AlB} ~ [~ 3 1 · 9]


2 3 : 6
1 2 : 8
Performing RI ~ RI - 2R2 and R3 ~ R3 - 3R 3, we get

~ [~ ;1;5 ~3]
o -5 -7 -10
Performing R3 -+ R3 - 5R I , we get

Which gives -y-5z =-3


- [~ ~1 ~ tl
x+2y+3z =6
1& = 5
th 1·· 35 29 5
Hence, e so utlon IS x = 18 ' Y = 18 ' z = 18 .
Example 6: Show that the equation
x + 2y-z = 3, 3x-y+2z= 1, 2x-2y+ 3z=2,x-y+ z=-1
are consistent and solve them.
Solution: The given equation can be written as

i.e., AX=B.
Therefore augmented matrix is

[A IB] = [11: ~~ t
-1 1
Matrices 199

~81
-4
-4

- [~o ~ t ~41
-3 2
-4
-4
Performing R3 --) R3 - 6R2 , R4 --) R4 - 3R2 we get

- [~o ~1 t ' ~l.


0 2 : 8
. 1 1
Performmg R3 --) 5" R3, R4 --) 2" R4

Performing R4 --) R I - R3

This is an Echelon form and having three non-zero rows. Thus rank A = rank of
[A IB] = 3. Therefore the equations are consistent

and

x-2y-z = 3,-y=-4,z=4.
Hence, the solution is x = - I, Y = 4, z = 4.
Example 7: If the system offollowing equations is consistent thenjind the solution
x + y + 4z =6
3x + 2y-2z =9
5x + Y + 2z =13
200 Remedial Mathematics

Solution: The equations can be written as

i.e .• AX=B
.'. Augmented matrix is
1 4
[Aim" [: 2 -2
1 2 : l·
13
Perfonning ~ -+ R2 -3R\. R3 -+ R3 -R]

~9l
If 1 4
- 0 -1 -14
[
4 0 -2
Perfonning R3 -+ R3 - 4R\. we get

- [~ ~1 ~14 6-9 1
o-4 -18 -17
Perfonning R3 -+ R3 - 4R2• we get

- [~ ~1 ~14 ~9l.
o 0 38 19
1
Perfonning R3 -+ 38 R3• we get

- [~ ~1 ~14 ~ ~9l.
o
0 1 : 112
PerfonningR\ -+ R\ -4R3' R2 -+ R2 + 14R3• we get

-
[1 0 : 4]
0
0 -1 0 : -2 .
o 0 1 : 112
Perfonning R\ -+ R\ + R2• we get

-
[1 0 0
0 -1 0 2-2 ]
o 0 1 112
Matrices 201

Performing R2 ~ - R2, we get

r1 o 0
2 ].
-l~
1 0
o 1 1/2
This is an Echelon form and having three non-zero rows and rank A = rank [A IB] = 3.
Thus the system of equations are consistent.

Hence, the solution is x = 2,y=2,z= 112.


Example 8: Show that the equations
x+y+z=-3
3x + y-2z =-2
2x + 4y + 7z = 7
are not consistent.
Solution: The given equations can be written as

~2][:H]
1
1
[: 4
i.e., AX=B
:. Augmented matrix is

[AI~~
1 1
[: 1 -2 : -2
, -3]
4 7 : 7
PerformingR2~R2-3Rl,R3 ~R3-2Rl' we get

[A IB]- [~ ~2 ~5 ::]

Performing R3 ~ R2 + R3

-3]
[~
1 1
-2 -5 7 .
- 0 0 20
This is an Echelon form and having three non-zero rows therefore the rank
[A I B] = 3 and we see that

A- [~ ~l:
202 Remedial Mathematics

Obviously rank A = 2.
Since, rank A :F- rank [A IB].
Therefore, the given equations are not consistent.-
Example 9: Investigate for what value of A, ",the simultaneous equations
x+y+z=6
x + 2y + 3z =10
x + 2y + A.z ="
have (i) No solution (ii) a unique solution (iii) an infinite solutions.
Solution: The given equations can be written as

i.e., AX=B.
Therefore, augmented matrix is

[AlB] =
1 1
1 : 6]
1 2 3 : 10 .
[ 1 2 A : 11
Performing R2 ~ R2 - R J , R3 ~ R3 - Rl' we get

[~
1 1 6

Performing R3 ~ R3 - R2
-
1 2
1 A-I
4
11- 6
1
[~
1 1 6

-
1 2
1 A-3
4
11- 10
1
If A :F- 3, the rank A = rank [A I B] = 3. Thus in this case a unique solution exists.
'*
In. = 3 and 1l:F- 10 then rank A rank [A IB] is 3. Thus rank A :F- rank [A IB]. Hence, in this
case equations are inconsistent.
In. = 3 and 11 = 10, then rank A = rank [A IB] = 2. Thus, in this case infinite solutions exist.

1. Use matrix method to solve the equations


2x-y + 3z= 9,x +y +z = 6,x-y+ z=2.
2. Use matrix method to solve the equations
x+2y+ z=2, 3x+ 5y+ 5z=4, 2x+4y+ 3z= 3.
3. Show that the equations are consistent and hence solve them
~
x-3y-8z+ 10=0, 3x+ y-4z =0, 2x+ 5y+ 6z- 13 = 0.
Matrices 203

4. 5x+3y+7z=4,3x+26y-2z=9, 7x+2y+ lOz=5.


5. 5x-6y+4z= 15, 7x+4y-3z= 19, 2x+y+ 6z=46.
6. 2x-y+3z= 8,-x+2y+z=4, 3x+y-4z=O.
7. Show that the following equations are not consistent.
2x-y+ z=4, 3x-y+ z= 6, 4x-y+ 2z= 7 -x+y-z= 9.
8. Prove that the following system of equations have a unique solution
5x+3y+ 14z=4,y+2z= l,x-y+2z=O.
9. Use matrix method to equation to solve 3x+y+2z=3, 2x-3y-z=-3,x+2y+z=4

HINTS TO THE SELECTED PROBLEMS

1. The given system of equation can be written in the form:


AX=B

=>
[:
-1
1
-1 :t] ~m
Consider the augmented matrix

[AI~~[:
-1 3
1 1 :] (by R\ --> R,)
-1 1

[~ ~]
1 1
We get -1 3
- -1 1
Now R2~R2-2RI,R3 ~R3-RI

~ [~
1 1
[A IH] -3 1
-2 0
-:]
-4

[~ ~] (bYR3~Ra-~~ )
1 1
-3 1
- -2 -2/3 -2
:. Rank [A IB] = The Number of Non-Zero rows in Echelon form = 3.
3. The given system of equation can be written in the form of
AX=B

such that [~ ~3 =:][~] [-~O] =


2 5 -6 z 13
Now consider the augmented matrix.

[AlB] ~ [~ !~ -1~]
13
204 Remedial Mathematics

Now R2 ~R2-3R2,R3 ~R3-2RI

[~ -10]
-3 -8
We have 10 20 30
- 11 22 33

- [~
-3 -8
1 2
1 2
-I:] (by ~ ~-..!..~,R3
10
~-..!..Rl)
11 '

Now R3 ~R3-R2

[~ -I~]
-3 -8
We get 1 2
- 0 0
The Rank of [A IB] = Number of Non-Zero rows = 2.
Here we observe that
Rank (A) = Rank [A IB]
Therefore, the given system of equation is consistent
Now Rank(A) = 2, which is less than the number of unknown
Therefore, the given system of equation having infinite number of solutions. Now
the given, system of equation reduces to

which gives x-3y-8z =-10


y+2z = 3
Letz = c, theny = 3 -2c andx= 2c-1.
Hence the solution of the given system of equation isx= 2c-l,y = 3 -2c, andz= c.
7. Consider the augmented matrix of the given system

[A IB] = [ :
-1
=:
1 -1
~ ;j
9

-n -~ ~] ~yR, ~R4)
=i
Now R2 ~R2 +3Rl'R3 ~R3 +4R 1,R4 + R4 +2R 1

We get [-~o =~ :::]


:
1 -1 : 22
Matrices 205

1 -1
1 -1
2 -2
3 -2
1 -1
1 -1
o 0
9]
22
-11
o 1 23
1 -1
1 -1
o 1
9]
22
23
o 0 -11
This is Echelon fonn.
Here, we observe that
Rank of [A I B] = 4 and Rank of A = 3
Here, the given system of equation is inconsistent.
8. The given system of equations can be written as
AX=B

[~ ~1 ~][~] m
Consider the augmented matrix

[AIBJ~[~ _:
14 : 4]
2 : 1
2 : 0

[~
14

J
3
We get 1 2
- -4 -12

[~
3 14

- 2 : 1
1 4] (by R3 ~R3 +4R 1)
0 -4 : 0
Which is in Echelon form
Rank of [A IB] = 3 = Rank of A.
~ System is inconsistent and have a unique solution such that
x+3y+14z =4
y+2z = I
-4z = 0
z=O
y = I, andz=O
Hence, the given system of equations have a unique solution given by
x = 1,y= 1 andz=O.
206 Remedial Mathematics

ANSWERS
1. x= 1,y=2,z=3 2. x=3,y=0,z=-1
7 3
3. Consistent:x=2c-l,y=3-2c,z=c 4. x= 11' y =11' z =0
5. x=3,y=4,z=6 6. x=2;y=2, z=2.
9. x= 1,y=2,z=-1

OBJECTIVE EVALUATION

MULTIPLE CHOICE QUESTIONS


1. The vertical lines in a matrix are called.
(a) Rows (b) columns (c) elements (d) None of these.
2. The Horizontal lines in a matrix are called
(a) Rows (b) Columns (c) Elements (d) None of these.
3. The single entity in a matrix is called.
(a). Rows (b) Column (c) element (d) None of these.
4. The elements of a matrix are defined for
(a) Only real no's (b) Complex no's
(c) Both real or complex (d) None of these.
5. The elements' aij' of a matrix A = [a,j]n x n for which i = j are called.
(a) Row elements (b) Column elements
(c) Diagonal elements (d) None of these.
6. A square matrix A = [aij] is a unit matrix if.
(a) aij= I when i = j and aij = 0 when i ±j
(b) aij = 1 when i *- j and aij = 0 when i =j
(c) aij*- 1 when i = j and aij = 0 when i*-j
(d) ai, *- 1 when i *- j and a ij = 0 when i *- j
7. If A = [a,j]m x n and B = [b'j]m x n then 'A + B' is a matrix of the type.
(a) n x m (b) m x n (c) m x m (d) n x n.

8. A = [3 2 -I], [I -2 7]
3 -3 1
B=
3 2 -I
then A + B=

(a) [~ ~3~] (b) [~ ~I~] (c) [~I ~ ~] (d) [-I0 60 7]4


9. A row matrix has only
(a) One element (b) One row with one or more columns
(e) One column with one or more rows (d) One row with one element.

10. If [ 5
k+1
k+2] = [k+3
-2 3
4]
-k
then k=

(a) 0 (b) 2 (c) -2 (d) 1.


Matricc!s 1207

=[~
2 0 3l
11.
IfP
5
8
:],nd Q =[: 7
J
0 ~ then 2P = 3Q is

4 -4
IS]
(a)
[:, 10
37 18
29 (b)
[:
-10
5 -:8]
(c)
[-4
-9
4
10 -J]
-1 (d) None of these.
-3 -5 18
12. If 'A' is a matrix of order 3 x 5, then each row of 'A' has.
(a) 3 elements (b) 5 elements (c) 8 elements (d) 2 elements
13. If'AB' is the product of matrix 'A' and matrix 'B'thenmatrix 'A'iscalled
(a) Pre-factor (b) Post-factor (c) Co-factor (d) None of these

A
14. If = [ ; ;j. B= [~ ~]
and A + B - C = 0, then C =

(a) [~ ;]

15. IfA=[~4 -22 ~]

MdB=[~ i]ili"
(a) AB. BA exist and are equal (b) AB. BA exist and are not equal
(c) AB. exist and BA does not exists (d) AB. does not exist and BA exists
16. Which is true for the product of matrix A and adjoint A when A = [aij]n x n
(a) Oifi=j (b) IAlifi;t) (c) IAlifi=j (d) None of these

17. If A = [~ :l then Acij. A

(a) [; ~] (b) [8
-y
-13]
a
(c)
8
[a
-y]
-13 (d) None of these.
208 Remedial Mathematics

19. IfA= 1
[' , 'l
2
2
-1
-3 thenA(A4IA)=
3

[~ ~3l
0 1
(a)
[T -11
0
-u
(b) 2
-1

[" ,~J
0
(c) ~ 11 (d) None of these.
0

20. If A = [; ;] then A (Ad) A) =

(a) [; !] (b) [-pU ~] (c) [~ ~] (d) None of these.

FILL IN THE BLANK


1. If AlB are square matrices of the same order such that _ _ _. then (ABt = AnBn .

2. The matrix A = [ ~ ~ !1 is a impotent matrix of index __


. _.
-2 -1 -3
3. The sum of two idempotent matrices A and B is idempotent if AB= BA ___.
4. If A and B bare symmentric matrices, then AB is symmetric ~ _ _ _.
S. If A and B are symmetric matrices of oreder n, then AB + BA is _ _ _ .
6. If A are singular matrix, then det (A-I) = - - -
7. If A is a skew symmetic matrix of odd order then 1A 1= ___
S. If A is an orthogonal matrix, then 1A 1= _ __

9. If A and B are two invertible matrices such that AB = C, then det (B) = ___
10. If A and B be two non-null square matrix such that AB is a null matrix, then A and B
both are =

TRUE I FALSE
1. A diagonal matrix is both an upper triangular and a lower triangular. (T/F)
2. If A, B are two matrices such that AB and A + B are both defined, then A, B are square matrices
of different order. (T/F)
3. If A, B are square matrices of the same order, then (ABt = An Bn. (T/F)

4. The matrix A = [ ~ ~ ~1 is on nilpotent matrix of index 4. (T/F)


-2 -1 -3
S. If A, Bare n -rowed square matrices, then AB = 0 implies that at least one of A and B is the null
matrix. (T/F)
6. Every matrix can be written as then sum ofa symmetric matrix a skew symmetric matrix. (T/F)
7. All positive integral powers ofa skew symmetric matrix are skew symmetric. (T/F)
Matrices 209

8. Positive odd integral powers of a skew symmetric matrix are symmetric. (T/F)
9. Positive even integral powers of a skew symmetric matrix are symmetric. (T/F)
10. If A and B are two symmetic matrices of order n, then ABA is a symmetric matrix. (T/F)
11. Every invertible matrix is orthogonal. (T/F)
12. If A and B are two invertible matrices, then (ABt I = A-I Ir I. (TIF)
13. If A is a non-singular matrix, then (ATrl = (A-Il. (TIF)
14. The inverse of a non-singular diagonal matrix is a diagonal matrix. (T/F)
15. The product of two diagonal matrices of the same order is a diagonal matrix. (TIF)

I ANSWERS
MULTIPLE CHOICE QUESTIONS
1. (b) 2. (a) 3. (c) 4. (c)
5. (c) 6. (a) 7. (b) 8. (b)
9. (b) 10. (b) 11. (c) 12. (b)
13. (a) 14. (a) 15. (b) 16. (c)
17. (c) 18. (a) 19. (a) 20. (d)

FILL IN THE BLANKS


1. AB=BA 2. 3 3. o(Null matrix) 4. AB=BA.
I
5. Symmetric 6. --- 7. 0 8. ±J
Det(A)
Det(e)
9. 10. Singular
Det(A)

TRUE OR FALSE
1. True 2. False 3. False 4. False
5. False 6. False 7. False 8 False
9. True 10. True 11. False 12. False
13. True 14. True 15. True

REFRESHER

Can we know? (Frequently Asked Questions)


1. Find the value of x such that

[lIXJ[~ ; ~l[}o IUpruB.Ph.. ma2002,~1


2. IfA=[~ ~]thenprovethatA2=6A+5I=[~ ~lWhereI= [~ ~].
(UPTU B. Pharma 2002]
210 Remedial Mathematics

3. If A = [~I _13 ~], B= [ ~ 4 3]


4 6 show that A2 B2 = A2
5 -5 5 -1 4 4
IUPTU B. Pharma 20051

4. Find the inverse ofthe matrix A " [i ~I ~Il


3 -I
IUPTUB. Pharma 2001]

5. Find the inverse of the matrix A " [:


~ !] IUPTU B. Pharma 2001]

6. Find the inve"e ofthematrixA "[; ~ :] IUPTU B. Pharma 2002]

7. If A" [~ ~ ~] usingA'-4A - 51" 0 and hence find A-I.

[UPTU B. Pharma 2003, 2008]

8. Show that the matrix A = [~1 ~1 ~ I] satisfies the equation A


3
- 6A' + 9A + 41" 0
1 -1
and hence find A-i. IUPTU B. Pharma 2006]

9. Find the inverse of the matrix [; ~ :] IUPTU 8. Pharma 20061

10. Solve for x andy by inverting the matrix in the following

[21 31] [yx] __ [47] [UPTU B. Pharma 2001]

11. If A = [~2 -
1
-1
~] find A-i. Hence solve the equations x + y + Z = 6,

x - y + z = 2 , 2x + Y - z = 1. [UPTU B. Pharma 2005]


13. Solve using matrix method, the following equations x + y + z =O,y + z = 1, z + x= 3.
[UPTU B. Pharma 2003, 06]

DOD
MEASURE OF CENTRAL TENDENCY

III INTRODUCTION
In previous chapter, we have studied about the classification and tabulation of data. But our
study about classification and tabulation is not enough to get all the desirable results since
when two or more series of same type are under observations, we can not classify and
tabulate them. So we need an arithmetical idea or characteristic about the distribution. These
characteristics are measure of central tendency, measure of dispersion, skewness and the
peakedness.
In this chapter, we will study the measure of central tendency or'average. According to
Dr. Bowley, "Statistics may rightly be called the science qf averages" and averages are
"statistical constants which enable us to comprehend in a single effort the significance of
the whole."
"Average is a point about which all the values of the variate cluster."

III KINDS OF STATISTICAL AVERAGES


The statistical averages can be divided into five parts:
1. Arithmetic mean (A.M.)
2. Median (Md)
3. Mode (Mo)
4. Geometric Mean (GM.)
5. Harmonic Mean (H.M.)

III ARITHMETICMEAN (A.M.)


"The arithmetic mean is the amount obtained by dividing the sum of values ofthe items in a
series by their number."
The arithmetic mean of some observations is the value which we can obtain by pividing
the sum of all the numbers by total number of terms.

x (A.M.) = Sum of all the observations


Total number oftenns
Let xI' x 2' x 3' .... xn are the observations. Then arithmetic mean is given by
212 Remedial Mathematics

n
LXi
Xl + x2 + x3 + ... + xn i=I
n n

n
If the variate xI' x 2' ... , xn occursjj, h, /3, ... ,In times, then the arithmetic mean is known
as due arithmetic average and,given by
x = fixl + hX2 + ... + fnxn
fi+f2+ .. ·+fn
:E'·x·
X = -'-;'-'
:EJ,
If wI' w2' .... wn be the weights assigned to the n values xI' x 2' ... ~ xn respectively, then
arithmetic mean is known as weighted mean and is defined by
Weighted mean = wlxl + w2 x 2 + ... + wnxn
11'1 + w2 w3 + ... + wn
:Ewx
:Ew

• METHODS OF CALCULATING ARITHMETIC MEANIN


INDIVIDUAL SERIES
We can calculate the arithmetic mean by following three methods:
1. Direct Method
2. Short-cut method
3. Step-deviation method
1. Direct Method: In this method, the mean is calculated using the following formula
n
LXi
XI + X2 +",+xn ;=1
=--
n n

STEP KNOWLEDGE

Step 1. Add all the observations to find :Exi -


Step 2. Divide this sum:Ex; by total number of observations, i.e., n.
2. Short-cut Method: In this method, we assumed a middle number as an assumed mean.
Here, we use the following formula.
:Ed
x =A+-
n
where, x = arithmetic mean; A = assumed mean; d = (x - A)
:Ed = sum of deviations
Measure of Central Tendency 213

STEP KNOWLEDGE
Step 1. Select the assumed mean i.e., A.
Step 2. Calculate the deviation from A, i.e., d=x-A.
Step 3. Find the sum of deviation as 'i.d.

Step 4. Using tpe formula x=


A + 'i.d, we get the required mean.
n
3. Step Deviation Method. Let us assume a number h called scale then
x
d' = -
h
'i.d' = 'i.x LX = hLd'
h '
'i,x hLd'
n n
x = hd'
AM. = hLd'
n

~~~~~~~I SOLVED EXAMPLES I


Example 1. Calculate the arithmetic mean of
129,117, 112,200, 172, 138, 183
Solution: Arithmetic mean ofthe above data can be given by:
129+ 117 + 112+200+ 172 + 138+ 183
x (AM) 7

= 1051 = 150.14
7
So, AM. = 150.14
Example 2. Hemoglobin percentage (Hb %) of a patients of a ward of hospital ~ "
obtained as 6 mg, 7 mg, 5 mg, 4 mg, 8 mg, 7 mg, 9 mg, 6 mg and 8 mg. Find out the arithmetl"
mean of the data.
'i,x
Solution:
N
6+ 7 +5+4+8+ 7 +9+6 + 8 = 60=6.66mg
9 9
Example 3: WBC's in Number of 10 malefrogs (Rana trigina) are 8.19, 9.21,10.40, 10.95,
12.14,12.52,13.41,13.92,14.78, and 15.74 laclmm 3. Findmean WBC'sNumber.

Solution: ..Here x= 'i,x


N
8.19 +9.21 + 10.40 + 10.95 + 12.14 + 12.52 + 13.41 + 13.92 + 14.78 + 15.74
10
= 121.26=12.13
10
214 Remedial Mathematics

Example 4: Find the arithmetic mean offirst n natural numbers.


Solution: Arithmetic mean is given by:
Sum of all the observations
X (A.M.)
Total number of terms
1 + 2 + 3 + 4 + ... + n ~n
n n
= ~ n(n + 1)
n 2
1
= - (n + 1)
2

So the arithmetic mean of first n natural numbers is given by ~ (n + 1) .


2
Example 5: Show that the arithmetic mean of the series 1,2,22,23 ,24 , ... , 2n is given by
(2 n + 1 _ 1) / n + 1.
· A'thm' M Sum of all the observations
S o Iu t IOn: n ebc ean= - - - - - - - - -
Total number of terms
1 + 2 + 22 + 23 + ... + 2 n
n+l
Sum of the G.P.
n +1
(2n+l_1) (2n+l_1)
(2-1)(n+l) n+l
(2n+l_l)
Arithmetic mean of the given G.P. = 1
n+
Example 6: A candidate obtain the following marks in an examination in a paper of 100
marks each
Subjects English Maths Physics Chemistry Biology
Marks (out of 100) 48 82 70 64 6)

It is agreed to give double weight to physics and mathematics as compared to other


subjects. What is the arithmetic mean? [UPTU B. Pharma 2005)
Solution: Since weights are given, we shall calculate weighted arithmetic mean in place of
simple arithmetic mean.
Weights are given accordance with the statement given in the example
Subjects Marks (X) Weight(W) WX
English 48 1 48
Maths 82 2 164
Physics 70 2 140
Chemistry 64 1 64
Biology 60 1 60
DV=7 LWX=476
Measure of Central Tendency 215

x = LWX = 476 =68


w LW 7
Example 7: The mean ofn numbers ofa series is x and the sum offirst (n - 1) numbers is A
Find the value of the last number.
Solution: Mean ofn numbers = x
Sum of first (n - 1) numbers =;'"
. hm . M Sum of all the observations
Ant etIc ean = - - - - - - - - - -
Total number of terms
Sum of all the observations = Mean x Total no. ofterm
= xxn
Sum of all the observations = nX"
Now, sum of first (n - 1) numbers = A
Then the last number = (Sum ofn terms) -(Sum of (n - 1) terms)
= nX"-A
The value of the last number = (nX" - A)
5. Calculation of arithmetic mean in a Discr~te frequency distribution
1. Direct Method: In case of discrete frequency distribution, we multiply the values of a
variable (x) by their respective frequencies (f). Then, we use the following formula
__ Lf x Lf x
x - --=--
Lf n
where, L.fx = The sum of the products of observations with their respective frequencies
"Lf = n = Total number of frequencies

STEP KNOWLEDGE
Step 1. Multiply the value of the variable x by the corresponding frequency fto find.fx.
Step 2. Calculate sum L.fx ofthe product obtained in step (1).
Step 3. Putting the values in the formula
- _ L.fx
x --
n
2. Short-cut Method: Firstly, we shall assume a mean and then take deviation of the variable
from this assumed mean. In this metfiod, we use the following formula: .
-x -- A +Lfd
-
n
where, A = assumed mean; d = x - A deviation; f = frequency

STEP KNOWLEDGE
Step 1. Select the assumed mean, i.e., A.
Step 2. Calculate the deviation from A, i.e., d=x -A.
Step 3. Calculatef d.
216 Remedial Mathematics

Step 4. Sum all the deviation to obtain I:.fd.

Step 5. Using the formula x=


A + I:.fd
n
3. Step-Deviation Method: In this method, we divide our deviation by the common factor h.
Therefore
Lfd'
x = A+--xh
n
where, h = Common factor of the deviation d !!. = x - A
h h

STEP KNOWLEDGE
Step 1. Choose the assumed mean A.
Step 2. Calculate d = x - A.
Step 3. Calculate d = d / h.
Step 4. Calculate fd'
Step 5. CalculateI:.fd'

Step 6. Using the formula x= A + Lfd' x h


n
6. Calculation of arithmetic mean in a continuous frequency distribution
In this case, we have to calculate the mid point ofthe various class intervals and denote it by
x . Then, proceed same as above.

~~~~~~~I SOLVED EXAMPLES I~~~~~~~


Example 1. Compute the mean of the following data by direct and short cut method.

Height (cm) 195 198 201 204 207 210 213 216 219
Children 4 5 7 11 10 6 4 2
Solution:

Height x I Ix d(x-A) Id
(em)
195 1 195 -12 -12
198 4 792 -9 -36
201 5 1005 -6 -30
204 7 1428 -3 -21
207 11 2277 0 0
210 10 2100 3 30
213 6 1278 6 36
216 4 864 9 36
219 2 438 12 24
Total 50 10377 27
Measure afCentral Tendency 217

Here, A =207
By direct method, we can calculate the A.M. by the formula

A.M. = 'Lfx = 10377 =207.54cm


N 50
By short- cut method, the formula is given by

A.M. = A + 'Ljd = 207 + 27 = 207 + 0.54 = 207.54


N 50
Example 2: Values of fecundity (Rate of reproduction) of 50 fishes of a species offish
(Macrognathus aculatus) were obtained and on the basis of that, a frequency table is
given below. Calculate the mean of fecundity by long method (Direct Method).
Class Interval 1-10 11-20 21-30 3140 41-50 51-60 61-70 71-80
Frequency 3 11 7 4 15 o 7 3
Solution:
Class Interval Mid"aluex Frequencyf ft
1-10 5.5 3 16.5
11-20 15-5 11 170.5
21-30 25.5 7 178.5
31-40 35.5 4 142.5
41-50 45.5 15 682.5
51-60 55.5 0 0
61-70 65.5 7 458.5
71-80 75.5 3 226.5
Lf=50 Lfx= 1815

x= 'Lfx = 1875 =37.5


'Lf 50
Example 3: Compute the mean of the following distribution by step deviation method
Class O-ll 11-22 22-33 3344 44-55 55-66
Frequency 9 17 28 26 15 8
Solution:
x - 38.5
Class Mid value x f (x - 38 5) d'= fd'
J1
0-11 5.5 9 -33 -3 -27
11-22 16.5 17 -22 -2 -34
22-33 27.5 28 -II -I -28
33-44 38.5 26 0 0 0
44-55 49.5 15 II I 15
55-66 60.5 8 22 2 16
Total "iJ= 103 Lfd' =- 58
218 Remedial Mathematics

Here, the assumed mean, A = 38.5 and h= 11.


A.M. by step deviation method is given by
M = A+h 'Ljd' =38.5+ llx(-58) =38.5-6.194=32.306
N 103
Hence, A.M. =32.306
Example 4: What is the arithmetic mean ofthefollowing data
Variate 0 1 2 3 ... n
Frequency nco nC nC nC ... nC
I 2 3 n

Solution: The arithmetic mean is given by = 'Lft


'Lj
'Lft = nCo.O + nCI xl + nC2 x 2 + nC3 x 3 + ... nCn X n
r.f nCo + nCI + nC2 + nC3 + ... + nCn
n(n-I) n(n-I) (n-2)
0+1'1.1+--.2+ x3 ... +1.n
2! 3!
(1+It

n[1 + (n -1) + (n - I)(n - 2) + ... + 1]


= 2!
n
2
n [n-1Co + (n-I)C +
1
... + (n-l)C _ ]
n 1
n
2
n. (1 + It -\ n . 2n -\ n
n n
2 2 2
So the arithmetic mean of the given data = !!.
2
Example 5. Compute the arithmetic mean of the following data by using step deviation
method
Age (below) 25 30 35 40 45 50 55 ro
No. of employees 8 23 51 81 103 113 117 120
Solution: Since the given data of the age is given as the age below x, so each column will
contain all the previous frequencies. So at first we will construct the frequency table.
d
Age Mid value (x) Frequency d= (x -A) d'=- Id'
h
20-25 22.5 8 -15 -3 -24
25-30 27.5 15 -10 -2 -30
30-35 32.5 28 -5 -I -28
35-40 37.5 30 0 0 0
40-45 42.5 22 5 1 22
45-50 47.5 10 10 2 20
50-55 52.5 4 15 3 12
55-60 57.5 3 20 4 12
Total '£1= 120 '£Id' =-16
Measure of Central Tendency 219

Here, assumed mean is taken asA = 37.5 and h = 5.


Now, by the above table, we can calculate 'Lf= 120 and 'f.fd' = -16.
Then the arithmetic mean by step deviation method is given by

A.M. = A + h 'f.fd'
N
= 37.5+ 5x (-16)
120

= 37.5 - 80 = 37.5 - 0.667


120
A.M. = 36.833 years
Thus, the required arithmetic mean is given by 36.833.
Example 6: A student passed the examination with the marks stated as follows
Mathematics = 59, Physics = 55, Chemistry = 63, English = 75, Statistics = 60
Compute the simple mean and also find out the weighted mean if weights are 3,3,1,2,1,
respectively.
Solution: We have
Sub Marks obtained (x) Weight (w) wx

Maths 59 3 177
Physics 55 3 165
Chemistry 63 1 63
English 75 2 150
Statistics 60 1 60
Total ~=312 LW= 10 LWX = 615
. r.x 312
Simple mean = - = -=62.4
n 5
Therefore, 62.4 marks are the simple mean for the given data
. 'f.wx 615
Weighted mean = - - = - = 61.5
LW 10
Hence, 61.5 are the weighted mean ofthe given data
Example 7. Find the weighted mean offirst n natural numbers whose weights are equal to
the squares of the corresponding number.
Solution: The table for the natural number and their respective weight is as follows
x w wx
1 12 13
2 22 23
3 32 33

n n2 n3
220 Remedial Mathematics

Now, we have to calculate the value of~u and ~w.x.


~w = 12 + 22 + 3 2 + ... + r? = ~r?
n(n + 1) (2n + 1)
6
~w = n(n + 1)(2n + 1)
6
~wx = 13 + 23 + 33 + ... + n3
=~n3

= [n(n + 1)
2
r 2
. ~wx n(n + 1) 6 n( n + 1) 3
Then, the welghted mean = - - =
~w
2 [ J x n(n + 1) (2n + 1) = (2n + 1) x -2
. 3 n(n + 1)
Welghted mean = 2" (2n + 1)

Remark
• Weight is the numerical value which shows the relative importance of the
corresponding variable.

III PROPERTIES OF ARITHMETIC MEAN


Therorem 1. If every variable is increased by a particular value a , then the arithmetic
mean is also increased by a.
Proof. Let Yl' Y2' Y3 ... ,Yn be n variables and the frequency for these variables is given by
ii, h, ... , In·
Then arithmetic mean
.:... ~fy
Y =-
~f
let us introduce a new variable z such that
z =y+a

and z = ~fz
~f

= ~f(y+a)
~f

_ ~fy a~f
--+--
~f ~f

z = y+a
Thus, it shows that if every variable is increased by a particular value a, then the
arithmetic mean is also increased by a .
Theorem 2. The algebraic sum ofthe deviations ofall the variate values from their arithmetic
mean is zero.
Measure of Central Tendency 221

Proof. Let zp z2' z3' ... , zn be the variables and the frequency for these variables is given by
Ii ,/z, ····,In·
Then the arithmetic mean will be
_ 'Lfz
Z =-
'Lf
or 'Lfz ='Lf·z
'Lfz = z'Lf ... (1)
Now the deviation from arithmetic mean is given by
d=z-z
then fd = f(z-z)
'Lfd = 'Lf(z - z)
= 'Lfz -'Lfi
= z'Lf -z'Lf [From(l)'Lfz= z'Lf]
'Lfd =0
So, we can say that the algebraic sum of the deviations of all the variate values from their
arithmetic mean is zero.
Theorem 3. The sum of the squares of the deviations of all the values taken about their
arithmetic mean is minimum.
Proof: Let zl' z2' z3' ... , zn be the variables andfi ,/z, .. ·,In be their respective frequencies.
Then the arithmetic mean z is given by
__ 'Lfz
'7' --
- 'Lf
or 'Lfz = z'Lf ... (1)
Now deviation from the arithmetic mean
d = z-z
then U ='Lj(z_A)2 (Sum of squares of deviation) ... (2)
The condition of maximum and minimum is given by
au =0
aA
a2u
and if --2 > 0, then Uwill be minimum.
aA
Now differentiating equation (2) w.r.t. A we get
au
aA = -2'Lf(z - A)
= -2'Lfz + 2A'Lf
= -2z'Lf + 2A'Lf

au = -2zN +2AN (Since 'Lf= N)


aA
222 Remedial Mathematics

au
But aA =0
-2ZN +2AN =0
-ZN+AN =0
A=z ... (3)
2
au
Now, - 2 =2N
aA
where, N>O

-->0
a2u
So,
aA 2
So, Uwill be minimum when A = z (which is mean).
Remark
• If M1, M2, ." Mk be the arithmetic mean of k distributions with respective frequencies
nl' n 2, ... , n k , then the mean M of the whole distribution with frequency N =
(n l + n2 + ... + nk ) is given by
k
M= lIN'L nr M r
r =1
Theorem 4: Arithmetic mean is not independent ofthe change of origin and scale.
Proof: Letx l , x2 ' ... , xn be the n variables andfi ,.12, ... ,In be their respective frequencies. Then
arithmetic mean is given by
M = 'Lfx
'Lf
Now, we will change the origin and scale. For this, let us assume a new variable ~, which
is given by
x-A
~=-­
n
- 'Lfd'
Then Mean M=-
'Lf
Put the values of d', we get
'Lf(x - A)
h
'Lf
=! 'Lf (X-A)
h 'Lf
=! 'Lfx _ A'Lf
h' 'Lf 'Lf

M = i(M-A)
Thus, we can say that the arithmetic mean is not independent of the change of origin
and scale.
Measure of Central Tendency 223

1. Compute the arithmetic mean of first n natural numbers whose weights are equal to

the corresponding number .!. (2n + 1) .


3
2. Compute the mean marks of a student from the following table:

Mark No.o/students
Above 0 80
Above 10 77
Above 20 72
Above 30 65
Above 40 55
Above 50 43
Above 60 28
Above 70 16
Above 80 10
Above 90 8
Above 100 0

3. The rainfall of a certain town in centimeters for the first six months of the year are
102, 103,95,98, 100, 105. Compute the average rainfall of the town.
4. Compute the arithmetic average in rupees from the data given below:
Salary 100 150 200 250 300 500
No. of Labours 30 20 15 10 4
5. Find the missing frequency from the following data, it is being given that 19.92 is
the average number of the given data:
Tablets 4-8 8-12 12-16 16-20 20-24 24-28 28-32 32-36 36-40
No. of 11 13 16 14 ? 9 17 6 4
persons cured
6. Calculate the mean marks of a student from the given data:

Marks No.o/students
Below 10 15
Below 20 35
Below 30 60
Below 40 84
Below 50 96
Below 60 127
Below 70 128
Below 80 250

7. Find the combined average daily wages for the workers of two factories:
No. of workers 250 200
Average 2.00 250
224 Remedial Mathematics

8. If the arithmetic average of data given below be 165 rupees, compute the missing term:
Monthly salary 100 150 200 300 500
No. of Labours 30 20 15 10 4
9. Compute the weighted arithmetic average rate of31 building trade workers from the
following table:
Kind of worker Daily wages (Rs) Frequency
Masons 15 4
Labourers 8 20
Carpenters 12 5
Painters 10 2

10. Compute the arithmetic average of the marks obtained by 9 students in a test:
75,43,52,65,48,35,40,70,40
11. Compute the missing frequer;tcy term from the following data whose arithmetic
average is given by 35.64:
Class 20-25 25-30 30-35 35-40 40-45 45-50
Frequency 18 44 102 57 19
12. Compute the arithmetic average of the following data:
0-5 5-8 8-10 10-12 12-15 15-17 17-20 20-25 25-30
2 5 7 5 6 4 4 9 6
13. The arithmetic average of a group of 40 items is 100 and that of another group of 50
items is 70. Find the mean of the combined group of size 90.
14. Compute the arithmetic mean for the following data:
Class 0-5 5-10 10-15 15-20 20-25
Frequency 4 16 2 15 2
15. Ifthe arithmetic average of the following frequency distribution is 7.85. Calculate
missing frequency term.
Salary 5 6 7 10 12 15
Labourers 10 13 8 5 4
16. The average salary of 500 workers in a factory running in two shifts of360 and 140
workers respectively is Rs. 70. The average salary of360 workers working in day
shift is Rs. 75. Find the average salary of 140 workers working in the night shift.
17. Find the mean of the following distribution:
Height (cm) 65 66 67 68 (f) 70 71 72 73
Plants 4 5 7 11 10 6 4 2
18. Find the mean of the following distribution:
Class 0-7 7-17 14-21 21-28 28-35 35-42 42-49
Frequency 19 25 36 72 51 43 28
19. IfP + q = 1, compute the mean ofthe following:
x 0 1 2 3
f qn
Measure oJ Central Tendency 225

ANSWERS
2. 51.75 5. 250 6. 50.4 7. 2.22
8. 250 9. 9.68 11. 160 12. 15.417
13. 83.33 15. 15.05 19. np

• COMBINED MEAN
If x,and x2 are the mean oftwo groups ofsizes n J and n2' then the combined mean x is
the mean of two groups, given by
__ nixi + n2 x 2
x -
n1 +n2
Proof: Letx"x2 , ... ,xn be the variates ofa group of size n, andYI'Y2' ""Yn be the variates of
a group of size n2 . Then

YI + Y2 + Y3 + ... + Y n -
::::>n2 x 2 =y, + Y2 + Y3 + ... + Yn
and x2 =
n, ...(1)

Let x be the mean of these two groups, then


x = (XI +x2 +x3 + ... +xn ) + (y,,,:r Y2 + Y3 + ... + Yn)
n, +~

x = nix, + n2 x 2 [Using (1) and (2)]


n, +n2

~~~~~~~I SOLVED EXAMPLES ~I~~~~~~


Example 1: The mean ofthe marks secured by 25 students ofsection A ofclass BCA is 47.
that of 35 students of section B is 51, and that of 30 students of section Cis 53. Find the
mean of marks secured by 90 students ofclass BCA.
Solution! Let n" n 2, n3 be the numbers of students respectively in section A, Band C and
Xi, x2 and x3 be the mean of marks secured by them.
n, =25,n2 =35,n3 =30
and Xi =47, x 2 =51 and x) =53

25 x 47 + 35 x 51 + 30 x 53
25 + 35 + 30

= 4550 = 50.56
90
226 Remedial Mathematics

Example 2: The school has two sections. The mean marks ofone section ofsize 40 is 60 and
mean marks ofother section ofsize 60 is 80. Find the combined mean ofthe students ofthe
school.
Solution: Here given that,
n l =40, n2 =60
xi =60, x2 = 80
.. Combined mean x = nlXi + n2 x2
nl +n2
40 x 60 + 60 x 80
40+60
x = 2400 + 4800 = 7200 =72
100 100
Example 3: The average score ofboys in an examination ofschool is 71 and that ofgirls is
73. The average score of school in that examination is 71.8. Find the ratio of number of
boys to the number of girls appeared in the examination.
Solution: Let there be n l boys and n2 girls in the school.
Here xI = 71, x2 = 73 and x = 71.8
. nlxl +n2 x2
.. Combmed mean x = --'--'------"=-=-
nl +n2
71.8 = nl x 71 + n2 x 73
nl +n2
71.8(n l +n2) =71nl +73~
71.8n l + 71.8n2 =71nl + 73n2
0.8n l = 1.2n2
nl 12 3
-=-=-
n2 8 2
Hence, n l : n2 =3: 2.
Example 4: Three teachers ofstatistics reported mean marks oftheir classes, consisting of
69, 64 and 71 students as 30, 26 and 18 respectively. Determine the mean marks of all the
three classes. [UPTU B. Pharma 2008]
Solution: Total marks ofIst class = 69 x 30
Total marks of 2nd class = 64 x 26
Total marks of3rd class =71 x 18
Total marks of all the three classes
Total number of students =
Mean marks of all the classes =

1. The mean wage of 150 workers of the first shift in a factory is Rs. 400. The mean
wage of75 workers of the second shift is Rs. 600. Find the combined mean wage of
the workers of the factory.
MeaslIre afCentral Tendency 227

2. There are 50 students in a class out of which 20 are girls. The average weight of20
girls is 45 Kg and that of30 boys is 52 Kg. Find the mean weight in Kg of the entire
class.
3. The average marks obtained by 30 students of group I is 60 and average marks of 40
students of II is 55 and that of 30 students of group III is 70. Find the combined
average of students of all three groups.
4. There are 100 students in a class. The mean height of the class is 150 cm. If the mean
height of60 boys is 170 cm. Find the mean height of the girls.
5. The mean weight of 150 students in a class is 60. The ~ean weight of boys is 70 Kg
and that of girls is 55 Kg. Find the number of boys and girls in the class.

ANSWERS I
1. 466.67 2. 49.2Kg 3. 61 4. 120 em
5. 50,100

• GEOMETRIC MEAN
LetxJ• xl' x J• .... xn be the n variates ofavariablex. then the geometric mean G ofn variables
is defined by
G -- (XI' ) lin
X 2 ' X 3 "'Xn
Iffi,fi,J3' ·.. ,fn be the frequency of these variables and
N =fi +fi+f3+···+fn
Then, G = (xf'x{:xf ···xf,n)'IN
1
log G = - [fi 10gxI + h logx2 + f3 10gx3 + ... + fn logxn]
N

10gG = ~ [~/; 10gX


1=1
i]

Thus, we can say that the logarithm of the geometric mean can be calculated by taking
weighted mean of the logorithm of the variables XI'

STEP KNOWLEDGE
Step 1. Find the logarithm ofthe variable x
Step 2. Obtain r,f log x

Step 3. Obtain ....:'i:f,---Io=g_x


r,f
. r,flogx
Step 4. Calculate the antIlog of -"---=--
r,f
228 Remedial Mathematics

~~~~~~~I SOLVED EXAMPLES ~I~~~~~~


Example 1: Calculate the geometric mean 3, 7,8,5,2.
Solution: The geometric mean of the terms 3, 7, 8, 5,2, is given by
G.M. = (3 x 7 x 8 x 5 x 2)1/5
=(1680)115
=4.416
Example 2: On 1" March a baby weighted 14 lbs. On ]-" May it weighted 20 lbs. What was
the approximate weight of the said baby ion 1" April.
Solution: G.M. =(xlx2x 3.. x)lIn
G.M. = (XIX/12
=(14 X 20Y.12
= (2 x 7 x 2 x 2 X 5)112
= (2J70)=2x8.36=16.72
The weight of the baby was 16.72lbs on 1st April.
Example 3: Calculate the geometric mean ofthe following data:
2574,0.005,0.8,0.0009,5,75,475,0.08
Solution: We know that the geometric mean of the data can be calculated by the formula
G.M. = ~ ~Iogxi. Then, we will solve it by this formula.
N
x logx
2574 3.4106
0.005 3.6990
0.8 1.9031
0.0009 4.9542
5 0.6990
75 1.8751
475 2.6767
0.08 2.9031
2.1208

I 1
10gG.M. = N ~logxi = gx2.1208
log G.M. =0.2651
Then, G.M = Anti log 0.2651
= 1.841
Example 4: Calculate the geometric mean ofthe given data
8, 15, 36, 40, 45, 70, 75, 85, 250, 500
Measure o/Central Tendency 229

Solution:
x logx
8 0.9031
15 1.1761
36 1.5563
40 1.6021
45 1.6532
70 1.8451
75 1.8751
85 1.9294
250 2.3979
500 2.6990
17.6373

Now, the geometric mean is given by


1
logG.M. = -l:logxj
N
= ~xI7.6373
lO
log G.M. = 1.7637
GM. = Antilog 1.7637
GM. =58.03
Example 5: The number ofBasophiles (a kind ofWBe) in blood of30 patients ofa hospital
and their frequency were recorded as [sources II, 14, 17, 19,22 andfrequencies 5, 6, 8, 7,
4]. Find out the G.M
Solution: Scores (x) Frequencies (j) logx j10p
11 5 1.0414 5.2070
14 6 1.1461 6.8766
17 8 1.2304 9.8432
19 7 1.2788 8.9516
22 4 1.3424 5.3696
30 36.2480

. . = Ant I·1og (1:fIOgx) = antI·1og (36.2480)


GM
1:f 30
= antilog (1.20826) = 16.15
Example 6: Calculate the weighted geometric mean of the data given below:
Articles Price Weight
A 125 40
B 150 25
C 100 5
D 122 20
E 75 10
230 Remedial Mathematics

Solution:
Articles Price Weight (w) log X wlogx
A 125 40 log 125 = 2.0969 40 x 2.0969 = 83.876
B 150 25 log 150=2.1761 25 x 2.1761 = 54.4025
C 100 5 log 100=2 5x2=10
D 122 20 log 122 = 2.0864 20 x 2.0864=41.728
E 75 10 log 75 = 1.8751 10 x 1.8751 = 18.751
100 208.7575
Weighted geometric mean can be calculated as
. ~wlogx
log (weighted G.M.) = _-'='_
~w

= _1_ x 208.7575 = 2.0875


100
Weighted G.M. = Anti log 2.0875
= 122.3
So, Weighted G.M. = 122.3

• PROPERTIES OF GEOMETRIC MEAN


Property 1: Ifwe put the value ofgeometric mean in place ofthe each value ofa series, then
the product of the value of the series will be unchanged
Proof: Let us assume a series with geometric mean G whose values are given by xl' X2' X3'
...xn. Then
G = (X I.xZ.x3 ••• x)lIn
Ifwe replace each value by G, then
G = (G. G.G. ... G)'ln. (ntirnes)
=(Cl')lln=G
Thus, it proves that if we put the value of geometric mean in place of every value of the
series, then the product of the value of the series will remain unchanged.
Property 2: If G 1 is the geometric mean ofthe series xI' XZ' X 3' ••. , xII' Gz is the geometric mean
of the series Y 1' YZ' Y3' ••• , Yn and G is geometric mean of the series obtained by the ratios of

corresponding observations. Then G will be equal to G/G z' i.e., G = ~.


G2
Proof: Here, G is the geometric mean of the series obtained by the ratios of corresponding
observations,
Measure of Central Tendency 231

G=§.
G2
Property 3: Let us consider n series with frequencies N I, N 2, N 3, ... , Nil' respectively and
geometric means G I , G 2, G 3, ... , Gn respectively. Then the combined geometric mean qfn
series with frequency NI + N2 + N3 + ... + N n is given by
N N N
G -- (GI IG2 2G3 3 .,.GNn n )IIN
Proof: Taking log of both sides, we get

Iog G -I
- og (GNIGN2GN3
I 2 3 ...GNn)I/N
n

1
= -[NllogGI +N210gG2 +N3 10g G3 + ... +NnlogGnl
N
n
NlogG = LN;logG;
;=1
Property 4: Let us consider n sets ofobservations whose geometric means are respectively
G I , G 2, ... ;Gn. Now, ifG is the geometric mean afthe product ofthese n sets, then the product
ofthe geometric means of these series will be equal to the value ofG.
G =G I.G2.G3...Gn
n
10gG = LlogG;
;=1
Property 5: Let us consider a series xI' x 2, ... , xp' xp+ I ... xn whose geometric mean is given
by G. In -Which G is greater than from the each value xI' x 2' ... , xp and less thanfrom each of
the values xp + I' xp + 2 ",xn then
an = (x l x 2 ... xp.xp+ I",xn)
GP.Gn-p = (x l .x2 .. .xp)(xp + I".xn)
G G G _ Xp + I XP + 2 Xn
xIx2 Xp -0'0- G
~~~~~~~I SOLVED EXAMPLES ~I~~~~~~
Example 1: Let us consider two series with m and n number of items and whose geometric
mean is given by G and g respectively. Then find out the geometric mean of combined
distribution. "
Solution: Let xI' x 2' ... , x"i. and Yl' Y2' ...Yn be the two series whose geometric means are
respectively given by G and g.
Then, G is given by
'n
G = (x l .x2,x3 .. .xm) 11m and g= (Y1'Y2'Y 3 ...yi
So the geometric mean of the combined distribution is given by
I

Gc = (XI,X2,X3,,,Xm·YI·Y2 .. ·Yn)m+n
232 Remedial Mathematics

mn
Gc = (G. g)m+n
m n
G = Gm+n . gm+n
c
Taking log of both sides, we get
m n
log Gc =log Gm+n . gm+n
1
= --[m log G + nlogg]
m+n
Gc =antilog _1_[mlogG+nlogg]
m+n
Example 2: The rate ofa certain item increases 5/4 times in first year, II/8 times in second
year and 5/6 times in third year. What is the annual average increment.

Solution: Let x be the rate of item. It is increased by S/4 times in first year means it is ~ x in
4
filrst year, -11. . t h'lrd year.
x m secon d year an d -S x m
8 6
Now, we have to calculate the geometric average of the rate, so

G= (~x x!..!.x ~x)1/3


4 8 6
Taking log of both sides, we get
1 S 11 S 3
log G = - log (- x - x - x x )
3 4 8 6
I[ S
348
11 + log- + logx
= - log- + log-
6
S 3]
1
= -[logS + 10g11 + logS + 310gx -log4 -log8 -log6]
3
1
= - [210gS + 10g11 + 310gx - 210g2 - 310g2 -log2 -log3]
3
1
= -[2 x 0·6990 + 1·0414 + 310gx -6x 0·3010 - 0·4771]
3
1
= - [1· 398 + 1· 0414 + 310gx -1· 806 - 0·4771
3
1
=-[310gx+0·lS63]
3
= 10gx[0,l] + O· OS21
log G-logx = 0·OS21
10g(Glx) = 0·OS21
(Glx) = Anti log 0.00S21 = 1.127
G = 1·127 x
Hence, the average annual increment will be 1·127 times of the price in first year.
Measure a/Central Tendency 233

1. Find the geometric mean of the following data:


50,100,1920,143740,204980,1206740,154910
2. Find the geometric mean of the series:
1,2,2 2,2 3,24 , ... .2n
3. The price ofcertain article rises 5% in first year, 8% in second year and 77% in third
year. What is the average change per year?
4. The geometric mean of 10 data are calculated as 16.2. It was later found that one of
the data was wrongly read as 12.9, in fact it was 21.9 Calculate the correct geometric
mean.
5. Calculate the geometric mean from the following frequency distribution
Marks obtained 11 12 13 14 15
Frequency 3 7 8 5 2
6. Find the geometric mean of2, 6, 18,54, 162.
7. Find the geometric mean of the following series
Class 0-10 10-20 20-30 30-40 40-50
Frequency 10 15 12 8 5
8. Find the geometric mean from the following table:
Marks obtained 5 7 9 11 13 15
No. of students 2 3 5 11 9
9. Calculate the average rate of increment in population which is increased by 20% in
first year, 25% in second year and 44% in third year.
10. Find out the geometric mean of the following distribution:
Marks obtained 0-10 10-20 20-30 30-40
No. of students 5 8 3 4

ANSWERS
1. 12700 2. 2n/2 3. 26% 4. 17.08
5. 12.79 6. 18 7. 19.\0 8. 11.86
9. 28.02% 10. 14.64

III HARMONIC MEAN


The harmonic mean of any series is given by the reciprocal of the arithmetic mean of the
reciprocals of the variables.
For different type afseries, it can be calculated by different method
(i) For individual series: Letx\, x 2 , x 3 .• .xn be the n variables, then the harmonic mean
of these variables [s' given by
234 Remedial Mathematics

n
H = I 1 1
-+-+ ... +-
xl X2 Xn

(ii) For discrete series: Letx l , x2,x3.• .xn be n variables andJ;,/i, .. 1n be the frequency
of them. Then the harmonic mean is given by

1
-
H
= -:L
I n (-/ ) ,where N = :L /
Nix

H= N~(;}
(iii) For grouped series: When the grouped series are given, we take the mid value of
each group and named them as Xl' X2' x3 ...xn and if the frequencies of these groups
are J; ,/i,fj.. In, then the harmonic mean can be calculated by

H= :LU/x)
N
~~~~~~~I SOLVED EXAMPLES I~~~~~~
Example 1: Find the harmonic mean o/the/ollowing data
12,8,6,24
Solution: Harmonic mean for individual series is given by
n
H = 1 1 I
-+-+ ... +-
Xl X2 xn

4 4
~+..!.+..!.+~ 0.0833 + 0.1250+ 0.1666+ 0.0416
12 8 6 24
H = _4_ =9.6038
0.4165
Example 2: Find the harmonic mean o/the/ollowing data: 4, 8, 16
Solution:
n
H= I 1 1
-+-+-
Xl X2 x3

3 3
.!.+..!.+~ 0.25+0.125+0.0625
4 8 16
= _3_ = 6.8571
0.4375
So the harmonic mean is given by 6.8571.
Measure o/Central Tendency 235

Example 3: Hemoglobin percentage offive persons were measured as 1,5,10,15, and 25.
Find out the Harmonic mean.
Solution:

=.!. (!+.!. + -.!...+-.!...+...!....)


HM 5 1 5 10 15 25
=.!. [150+30+ 15 + 10+6]
HM 5 150
1 211 211
=-x- = -
5 150 750
HM = 750 = 3.55
211
Example 4: Find the harmonic mean ofthefollowingfrequency distribution:
Class 0-10 10-20 20-30 3040 40-50
Frequency 4 5 11 6 4
Solution:
Class Mid value x Frequency J/x j7x
0-10 5 4 0.2 0.800
10-20 15 5 0.0666 0.333
20-30 25 11 0.04 0.04
30-40 35 6 0.0285 0.171
40-50 45 4 0.0222 0.088
30 1.832
Now, harmonic mean for grouped series is given by
N 30
H.M. = "( ) = - =1.63755
L..; f / x 1.832
Example 5: Find the harmonic mean ofthefollowingfrequency distribution:
Class 0-10 10-20 20-30 3040
Frequency 5 8 3 4
Solution: We have
Class Mid value x Frequency(f) J/x j7x
0-10 5 5 0.200 1
10-20 15 8 0.0666 0.5336
20-30 25 3 0.0400 0.12
30-40 35 4 0.0285 0.114
Total 20 1.7676
Now, the harmonic mean for grouped series is given by
N 20
H.M = "( ) = - - =11.3147
L..; f / x 1.7676
236 Remedial Mathematics

Example 6: Hb% and its frequencies in 10 members ofa family was studied andfollowing
results were obtained. Find the HM of the given series.
Hb% mg I 100ml Frequencies
12mg 3 -
l3mg 3
14mg 1
15mg 2
16mg 1
Solution:
Hb% mg/JOOml Frequencyf l/x jlx

12 3 0.083 0.25
13 3 0.076 0.23
, 14 I 0.071 0.071
15 2 0.066 0.133
""-
:~:
16 I 0.0625 0.0625
. 10 0.7465
.
- 'i,f= 10 'i,j7x = 0.7465

_ ~)flx) _ 0.7465
HM - --10- If
=0.07465

HM = =13.39
0.07465
Example 7: A man drives a car for three days by covering a distance of360 km per day. First
day he drives for a time of 10 hours and drive with the speed of36kmlh. On the second day,
he drives 15 hours at a speed of 24kmlh and on the third day, he drives for 12hours at a
speed of 30 kmlh. Calculate the average speed of the car.
Solution: It is given that he covers a constant distance of360 km per day.
His speed on the first day is given by = 36 kmlh
His speed on the second day is given by = 24 km/h
His speed on the third day is given by = 30 km/h
Since the distance is given to be constant so the average speed can be calculated by
taking harmonic mean of the speeds.
n 3
So, average speed = 1 I 1 1 1 1
-+-+- -+-+-
VI v2 v3 36 24 30
3
0.0277 +0.0416+0.0333
_3_=29.2397 kmIh
0.1026
Measure of Central Tendency 237

Remark
• Where the distance in each part of the journey is given to be constant, then average
speed will be calculated by harmonic mean. In the case when time being constant, the
average is given by arithmetic mean.
Example 8: In a certain factory a unit of work is completed by A in 4 minutes, by B in 6
minutes, by C in 8 minutes and by Din 12 minutes. What is their average rate of working?
At this rate, how many units will they complete in a 8 hour day?
Solution: The average rate ofwo]rking can be calculated by harmonic mean. So
n 4
H= 1 1 1 1 1 1 1 1
-+-+-+- -+-+-+-
Xl X2 X3 x4 4 6 8 12

4 _4_ = 6.41 minute


0.25 + 0.166 + 0.125 + 0.083 0.624
So, 6.41 minute per unit is the average rate of working
8 hour = 8 x 60 = 480 minutes

They will complete the units= _1_x 480 .


6.41
Example 9: A train travels first part of its journey of 100 kms at a speed of20 kmlh, second
part of 100 kms at a speed of 2 5 kmlh and the third part of the same distance at 30 kmlh,
Find its average speed
Solution: Since the train travels a distance of 100kms in each part. So, its average speed can
be calculated by the harmonic mean.
n 3 3
H= 1 1 1 1 1 1 0.05 + 0.04 + 0.033
-+-+- - + - + -
xl X2 x3 20 25 30
= _3_ = 24.390
0.123

II1II PROPERTIES OF HARMONIC MEAN


Theorem 1: Ifxl and x 2 are any two observations, then
A.H =G2
Where, A = Arithmetic mean
H = Harmonic mean
G = Geometric mean
Proof: We know that for two observations, arithmetic mean is given by
A = xl +x2
2
Harmonic mean is given by
2
H = 1 I
-+-
xl X2
238 Remedial Mathematics

2
Now, H = 1 1
-+-
xl X2

H = 2XIX2 ,A = xl +X2
Xl +x2 2
and geometric mean G is given by

G = ~XIX2
(Xl +X2) 2XIX2
Then, AH = x --,--~=----
2 (Xl + X2)
AH =x l x 2
Hence, AH = G2
Theorem 2: /fx l , x 2' x 3, ... ,xn be the n positive observations then A cG 2 H
The sign ofequality will hold if the values ofall observations under consideration are
same.
Proof: Let xI' x 2, x 3, ... ,xn be the n positive observations. For these observations
A = xI +x2 +x3 + ... xn
Arithmetic mean,
n
n
Hannonic mean, H = 1 1 1
-+-+ ... -
xI x2 xn
and Geometric mean, G = (x l ·x2·x3···xn)l/n
Now, we have to prove A 2: G 2: H
Firstly, we will prove that A 2: G

We know that 2(;-;; - ~)2 2: 0

=> (xl +X2 -2~XIX2) 2:0

... (1)

> (xl +X2) (X3 +X4)


2 - 2 2
2: ~{~[XIX2]X3X4} [by equation (1)]

(XI+ X2+ X3+ X4) 1/4


=> l 2: (x x r3 x 4) ... (2)
4
In the same way, we can show that
xI + x2 + x3 + x4 + Xs + x6 + x7 + Xg IIg
8 2: (xI x2x3x4xSX6X7Xg) ...(3)
Measure a/Central Tendency 239

By eqn. (I), (2) and (3), we see that A ~ G for n = 2, 4, 8, ... means it is true for
n=21 ,2 2 ,23, ... , 2m .
Now, we have to prove it for all values ofn.
For this, let us assume the n observations
x l ,x2,x3' ... , xk,xk + I ...x 2n (k < 2 n ,n = 2m)
Let
A = xI +x2 +",+xk 1 ... (4)
and G = (XIX2 ....\k)lIk
and all x k + 1 = xk+2=xk+3= .. .x2n=A
Now, we can write
XI + X2 + "'Xk + Xk+1 + ... x2m > ( m)I/2m
2m - XI X2 "'XkXk +1 ... x 2

From equation (4), it can be written as


(2m -k)A+KA (Gk A2m-~1I2m
~
2m
KA+2m A-KA (Gk A2n1 _k)1/2n1
~
m
2

2m.A
2m

... (5)
Now, we will prove that G ~ H

or

Now, put X;=-


Y;

1 1 1 )l/n n
Then ( - ' -... - >
, YI Y2 Y n - YI + Y2 +"'Yn
240 Remedial Mathematics

or {,
V'I . Y2....Y n
)lln ;<; --"-'---'c.=.._----"~
(YI+Y2+···+Yn)
n
Gy ;<;1
(where Gy and Ay are the Geometric mean and arithmetic mean with respect to y)
1 ~Gy
1
H ~Gy
H;<;G
G~H ... (6)
Thus, by equation (5) and (6), we get
A~G~H

Theorem 3: JfxI and x 2 be any two observations, A is the arithmetic mean and G is the
geometric mean of these two observations, then AH = G2.
Proof: We know that
A = xl +X2 G=(x X )112
2' I 2

2 2xlx2
H = 1 1
_+_ xl +X2
Xl x2

Now, A x H = (Xl + X2 ) ( 2XI X2 )


2 xI +X2

(xI +X2) 2xlx2


2 (xI +x2)
=x I x 2
=G2
Hence, AH = G2
Theorem 4: A variate takes values a, ar, a? , ... a~-I each with thefrequencies one. Show
that
AM =a(l-~)I n(l-r)
GM =ar<n-I)/2
HM =a. n(l-r)~-l /(l-rt
Proof: Given variables are a, ar, a?, ... , a~-l
~X =a+ar+a?+ ... +a~-l

a(l-r n )
(l-r)
n
Arithmetic Mean, A = Lx = a(l-r )
n (l-r)
Measure of Central Tendency 241

. mean, G-
Geometnc - [ a. ar. ar2 ... ar.JI-I]lIn
=[~ rl +2 +...+n-I]
=a[r'(n-I)/2r
=ar(n-I)/2

n
Hannonic Mean, H=1 1 1 1
-+-+-2 +"'+--n:J
aarar ar

anr n- I
r n- I + r n- 2 + r n- 3 + ...r +1
anr n- I an(l- r )r n- I
(l-rn) / (1-r) l- r n

EXERCISE 5.4
1. Find the harmonic mean ofthe following data: 5,10, 15,20,25,30,35
2. Find the harmonic mean ofthe following data:
0.00002853,0.0003425,0.004656,0.07834,0.676,9.45,78.3,800
3. Calculate theA. M., G.M. and H.M. of the observations and show that: A.M. > G.M.
>H.M.
37,32,36,35,43,39,41
4. Calculate the geometric mean of the following data

I M"ks obtained I
Frequency
5
3
6
4
7
8
8
7
9
2

5. Calculate the harmonic mean of the following frequency distribution

Class 40-50 504> 60-70 70-80 80-90 90-100


Frequency 19 25 36 72 51 43

6.Calculate the harmonic mean of the following frequency distribution

Class 04 4-8 8-12 12-16 16-20


Frequency 4 12 20 9 5

7.Compute the harmonic mean of the following frequency distribution

Class 40-50 504> 60-70 70-80 80-90 90-100


Frequency 12 10 15 17 8 3

8. A car runs at the rate of 15 km/h during the first 30 km, at 20 km/h during the second
30 km and at the rate of25 kmlh during the third 30 km. Find out the average speed
ofthe car.
242 Remedial Mathematics

9. A train starts from rest and travel a distance of 1 km in four parts each of 0.25 km
with average speed 12, 16, 24 and 48 kmlh. Explain the statement that the average
speed over the whole journey of 1 km is 19.2 kmlh and not 25 kmlh.
to. A variate takes values 1, r, ?, ... f'I- 1 each with frequency unity. Show that
~
n l
A= G = r n- 1/2 H = n(l-r)r -
n(l- r)' , 1- rn
From the above observations, also show that
AH = G2 and A> G> H.
11. Find out the average speed of a car running at the rate of20 km/h during the first 30
km; at 25 kmlh during the second 30 km and at 30 kmlh during the third 30 km.
t2. Calculate the average speed of a train running at the rate of20 km/h during the first
100 km, at 25 kmIh during the second 100 km and at 30 kmlh during the third 100km.

I ANSWERS I
1. 13.5030 2. 0.0002095 4. 6.84 5. 82.5669
6. 7.246 7. 32.049 8. 19.15 11. 24.32 km
12. 24.39 km/h

III MEDIAN
If wearrange the whole data in ascending or descending order, then the value of the
middle variable is known as median.
In case when the number of variables are odd, then the middle value is known as median.
If the number of variables are even, i.e., (2n), the value of the mean of nth, (n + l)th
variables will be median.

Computation of Median
1. Formula for individual series: When the data given are ungrouped, then firstly, we
arrange them in ascending or descending order.
Then, if number of data are odd number, then the value of the middle variable will be
median.
Ifnumber of data are even number (2n), then the value of the mean of the nth and
(n + I)'h variable will give the median.
2. Formula for discrete series: Let us assume that xl' X2 ....Xn are the n observation
whose frequencies are given by fi, ii, ..
In . To calculate the median of such series
first of all we calculate the cumulative frequency and then calculate the sum of the
frequency. Now, we calculate the median of series according to the N (sum of the
frequency) is odd or even.
\ 3. Formula for Continuous series: In these type of questions all the data are divided
into particular classes and their respective frequencies are given. Firstly, we calculate
the cumulative frequencies. Then, we calculate the sum of the frequencies (N).
According to N is even or odd, we fmd out the median. The class which contain this
median is known as median class.
Mea~ure ofCemral Tendency 243

Now, the median for this series can be calculated by the formula

IN-F
Median = 1+ 2 xi
f
I = lower limit of the median class
N = sum of all the frequencies
F = Sum of all the frequencies preceding the median class
f = Frequency of median class
i = Width of the median class

~~~~~~~I SOLVED EXAMPLES ~I~~~~~~


Example 1: The height of the 11 students in inches of a hockey team is given. Find the
median of these data .
65,67,69,6160,65,66, 70, 71,62, 72,
Solution: First of all, we arrange all the terms in ascending or descending order
60,61,62,65,65,66,6769,70,71,72
Number of terms are 11 which is odd.
· . .
Some d lanlsglVen bythe term = - (n+l)
-=-- =
11+1 6
2 2
Hence, the value of 6th term will be the median ofthese data.
Median = 66 inches.
Example 2: RBC's Number of8 patients is 35, 44,38,36,39,40,42 and 41laclmm 3. Find out
the median ofthis series.
Solution: First of all data is processed in ascending order i.e.,35, 36, 38, 40, 41 42, and 44
(Iac/mm3)
th ( )th
(
~ ) item+ ~+I item
Median = 2 2 (Since n = 8 is even)
2

= (%f item+(%+lf item


2
4th item + 5 th item
2
= 39+40 =39.5
2
Example 3: The daily wages in Rupees often labourers ofafactory are 4,6,9,12,11,8,5,
10, 11,8 Calculate the median ofthese wages.
Solution: Firstly we arrange the data in ascending order
4,5,6,8,8,9,10, II" 11, 12
244 Remedial Mathematics

Here, n = 10, i.e., number of data are even so the mean ofnl2th and (% + 1)th value will
be actual median.

n =5 ~+1 =5+1=6
2 ' 2
th
5 term is given by = 8
th
6 term is given by = 9
· Value of 5th term+value of 6th term
M elan
d = -------------
2
8+9 17
=-=-
2 2
Rs. 8.5
=
Example 4: Hb % ofan animal was recordep as 6, 7, 4, 5, 5, 3 and 4gmlJ 00 mi. Calculate the
median. .
Solution: First of all above data is arranged in an ascending order i.e.,
3,4,5,5, 6 and 7.
Total Number of scores is 7 (an odd number)

. (n+ l)th Item


Median = -2-
.

7+
= ( -2-
l)th Item
.

= (i
Median = 5gm1100ml.
r item = 4 th item = 5

Example 5: Find the median in the following frequency distribution:

5 7 9 11 13 15
3 12 28 10 9 6

Solution:
(x) (f) Cumulative frequency
3 7 7
5 3 10
7 12 22
9 28 50
11 10 60
13 9 69
15 6 75
N=75

Here. N =75, which is odd.


Measure a/Central Tendency 245

So, the median = Value of the (75 + I) th term.


2
= Value ofthe 38 th term.
In the table, we see that the cumulative frequency 50 contain the 38th term. So the value
of x for this column will be the median
Median = 9
Example 6: Compute the median ofthefollowingfrequency distribution:
Size 8 10 12 14 16 18 20
Frequency 3 7 12 28 10 9 6

Solution: (x) (f) Cumulative frequency


8 3 3
10 7 10
12 12 22
14 28 50
16 10 60
18 9 69
20 6 75
N=75
Here, N= 75, which is odd.

So, median is given by the value of (N + 1) th term.


2
Median = Value ofthe (75 + 1) th term = Value of the 38 th term.
2
The 38 th term will fall in the cumulative frequency 50. So the median for this distribution
is given by the value of x for this frequency.
Median = 14.
Example 7: Percentage of body water of J5 Fishes and their frequencies given as follows.
Find median of the given data.
Water'% 60 62 64 70 72 74 76 78 82 84 86
Frequency 1 2 1 2 1 3
Solution: Cumulative frequency table
Water % Frequency(/) Cumulativefrequency
60 I 1
62 1 2
64 1 3
70 2 5
72 1 6
74 1 7
76 2 9
78 1 10
82 1 II
84 3 14
86 I 15
246 Remedial Mathematics

Median will fall in ( ~)th item

= C;Jh item=7.5 th item

So median = 76.
Example 8: Calculate the median in the/ollowing/requency distribution:
x 20-30 3040 40-50 50-60 60-70
/ 8 '2fj 30 20 16
Solution: (x) (j) Cumulative frequency
20-30 8 8
30-40 26 34
40-50 30 64
50-60 20 84
60-70 16 100
N= 100

Here, N = 100. So the median number is given by N = 50. The median number 50 falls in
2
the class 40-50. So, 40-50 will be the median class.
Lower limit of median class 1= 40
Sum of all the frequencies, N = 100
Sum of all the frequencies preceding the median class, F = 34.
Frequency of median class/= 30.
Width of the median class i = 10.

~N-F
Then, median is given by = I + 2 / xi

(~x 100-34)
= 40+ 2 xl0
30
(50-34)
= 40+ 30 xl0 =40+5.3333=45.3333
Example 9: From the/allowing table, calculate the median a/the cost a/living index:
(UPTU B. Pharma 2005]
Costoflivingindex(Rs) 140-150 150-160 160-170 170-180 180-190 190-200
No. of weeks 5 10 20 9 6 2
Measure of Central Tendency 247

Solution: Calculation of median

Cost o/living index No.'o/weeks Cf


140-150 5 5
150-160 10 15
160-170 20 35
170-180 9 44
180-190 6 50
190-200 2 52
N=52

Median number = ( ~) th item = ( 5; ) th item = 26th item.


which lies in the class 160-170. Thus, 160-170 is the median class in which
I) = 160,/=20, F= 15, i= 10.
Using the formula

Median = f. +-
I f 2
(N
i --F )
= 160 +.!Q.(26 -15) = 160+.!..!.
20 2
= 160+5.5= 165.5.
Example 10: Find the medium ofthe following data:
Daily wages (in Rs) 22 24.50 28 31.50 34 36.50
No. of Workers 10 23 32 28 12 5 [UPTUB.PHarma20081
Solution: We have
Daily wages x Frequency Cumulative frequency
22 10 10
24.50 23 33
28 32 65
31.50 28 93
34 12 105
36.50 5 110
HereN= 110.
110
then - =55.
2
which lies between 33 and 65.
Hence medium = 28.

~~~~~~~I EXERCISE 5.5 ~I~~~~~~~


1. Compute the median ofthe data: 9,10,15,7, 11,9,8,11,7,9,10
2. The marks obtained by the ten students of class 8th is as follows:
75,80,96,92,89,94,100,82,63.105
Find the median
248 Remedial Mathematics

3. In a factory the daily wages of labourers are given by the following frequency
distribution.
Find the median.
Wages (Rs) 6 8 10 12 14
No. of labourers 6 3 4 5 2
4. Compute the median for the following frequency distribution
Age 5-7 8-10 11-13 14-16 17-19
No. of students 7 12 19 10 2
5. Compute the median for the following frequency distribution
Variable 45-50 SO-55 55-60 60-65 65-70 70-75 75-80 80-85 85-90 90-95
Frequency 2 3 5 7 9 11 7 2 3
6. Calculate the median for the following frequency distribution
Variable 0-5 5-10 10-15 15-20 20-25
Frequency 4 16 2 15 2
7. Calculate the median for the following frequency distribution
Variable 0-10 10-20 20-30 30-40 40-50 50-60 60-70 70-80
Frequency 2 18 30 45 35 20 6 3
8. Compute the median ofthe following frequency distribution:
Age 15-19 20-24 25-29 30-34 35-39 40-44
Frequency 4 20 38 24 10 4
9. Compute the median ofthe following frequency distribution:
Class 0-4 4-6 6-8 8-12 12-18 18-20
Frequency 4 6 8 12 7 2

ANSWERS
1. 9 2. 90.5. 3. Rs. 10 4. 11.447
5. 7.583 6. 36.559 7. 34.45 8. Median = 28.49
9. Median = 8.5.

_MODE
The variable whose frequency is maximum, is known as the mode of the distribution. In
other words, we can say that the value which occurs most frequently in a distribution is
known as the mode of the distribution.

Computation of the mode:


1. For individual series: For individual series, we can find the mode by inspection
only. If number of variables are very large, then we arrange the data into discrete
series and then we check the frequency for each variable to know the mode of the
series.
Iffrequency for different variables are same in the frequency table, then we use the
method of grouping to calculate the mode of the distribution.
Measure a/Central Tendency 249

2. For discrete series: Firstly, we arrange all the data in the frequency table. If the
maximum frequency has the unique value, then it will be the mode of the series and
if the maximum frequency occurs more than once, then mode can be calculated by
grouping of data.
3. For continuous series: The class with maximum frequency is known as the modal
class and we can obtain the mode of this series by calculating the formula

Mode = 1+ f - f-l xi
2f - f-1- It
where, I = Lower limit of the modal class,
f= Frequency of the modal class,
1-1 = Frequency preceding the modal class,
fi = Frequency succeeding the modal class,
i = Class width.
If1-1 andfj are both (or one) is greater than fthen we use the following formula

Mode = 1+ It xi
fi + f-1

~~~~~~~I SOLVED EXAMPLES ~I~~~~~~


Example 1: Find the mode ofthe data given below
0, 1,6,7,2,3,7,6,6,2,6,0,5,6, °
Solution: In the given data, we see that the frequency of 6 is 5, which is maximum and no
other frequency is equal to this frequency. So, 6 is the mode of the given data.
Example 2: Find the mode ofthe data given below
25, 15.23, 4~ 2~ 2~23, 25, 20
Solution: In the given data, the frequency for 25 is given by 3 which is maximum, since no other
data has the same frequency or more than 3. So the mode ofthe given data is given by 25.
Example 3: Water percentage offtfteenfishes ofa species offish wave recorded as 60, 64. 62.
76. 70, 84, 82, 72, 76, 84, 78. 84 and 86. Find the mode of this series.
Solution: First of all, data is arranged in ascending order. Not even single observation is
spared. It comes as 60, 62, 64, 70, 72, 76, 76, 78, 82, 84, 84,84 and 86.
By simple observation one can s,ay that 84 is the mode, because 84 is repeated maximum
times (three times) in the above series.
Example 4: Find the mode of the following frequency distribution
Mid value 15 20 25 30 35 40 45 50 55
Frequency 2 22 19 14 3 4 6 1 1
Solution: Here, the mid value for each classes is given. So, firstly, we have to convert the
given data into grouped frequency. So,
250 Remedial Mathematics

Mid value Class frequency


15 12.5-17.5 2
20 17.5-22.5 22
25 22.5-27.5 19
30 27.5-32.5 14
35 32.5-37.5 3
40 37.5-42.5 4
45 42.5-47.5 6
50 47.5-52.5 1
55 52.5-57.5 1

Here, by inspection, we see that the maximum frequency is given by = 22. So the class
17.5-22.5 will be the modal class.
For the modal class
f=22, 1= 17.5,1=22'£1 =2,fi = 19, i= 5

Then, mode M = 1+ f - f-1 xi


a 2f - f -1 +fi
22-2
= 17.5+ x5
2x22-2-19
20
= 17.5+---x5
44-21
20
17.5+-x5
=
23
= 17.5+4.3478
Hence, Mode Mo =21.8478
Example 5: The expenses of lOOfamilies are given below. Ifthe mode ofthis distribution is
48, thenjind the missingfrequency.

Solution: Expenses (Rs.) Frequency


0-20 14
20-40 x
40-60 27
60-80 Y
80-100 15
N=56+x+y

It is given that N = 100


So, 56+x+y = 100
x + y =44 ...(1)
Now, it is given that mode of this distribution = 48 which falls in the class 40-60. So for
this
1=40,1=27'£1 =x,fi =y, i=20
Measure of Central Tendency 251

Then mode,

M = 1+ 1-1-1 xi
o 21 - 1-1 - fi
48 = 40+ 27-x x20
54-x- y

8 = 27-x x20
54-x- y
8 (54-x-y) = 20 (27-x)
2 (54-x-y) = 5 (27-x)
108-2x-2y = 135-5x
3x-2y =27 ... (2)
Solving equation (1) and (2)
5x = 115
x = .!..!2. =23
5
Put this value in equation (1)
23 +y =44,y=21
Hence, the missing frequency is given by = 23, 21.
Note: Empirical Formula
The empirical relationship between mean, mode and median is given by
Mode = 3 Median - 2Mean
Example 6: Calculate the mode of the following data
Wages (Rs.) Below 100 100-200 200-300 300-400 400-500 500 and above
No. of weeks 8 12 25 15 10 5
[UPTUB. Pharma - 2005J
Solution: As the frequencies are regular and highest frequency 25 belongs to the class 200-
300. Thus.200-300 is the modal class in which 1=200,J= 25,L 1 = 12,J= 15,;= 100.
Using the formula

Mode = 1+ 1-1-1 xi
21 - 1-1 - fi
= 200+ 25-12 xl00
50-12-15

= 200 + 1300 = 200 -r 56;52


23
=256.52
Example 7: Compute the mode ofthefollowing distribution
Class 10-20 20-30 30-40 40-50 50-60 60-70 70-80
Frequency 22 35 40 62 50 45 40
Solution: Here, the frequencies are regular and highest frequency 62 belongs to the class 40-
50. Thus 40-50 is ~emodalclass in which 1=40,J=62,1-1 =40,fi =50, i= 10.
252 Remedial Mathematics

Using the formula


Mode = 1+ f - f-1 xi
2f - f-1 - fi
62-40
= 40+ xlO
2x62-40-50
= 40 + 220 =40 + 6.47
34
=46.47
Example 8: Calculate the mean, median and mode of the following data of 120 articles
Weight in gms 0-10 10-20 20-30 30-40 40-50 50-60
No. of articles 14 17 22 26 23 18
[MEERUT B.Se. BIOTECH-2005]
Solution: Table for Calculation of Mean

Weight in 9 ms No ofartic.:lesf Mid value x d=x-35 d' = !l.- Id'


10
0-10 14 5 -30 -3 -42
10-20 17 15 -20 -2 -34
20-30 22 25 -10 -1 -22
30-40 26 35 -0 0 0
40-50 23 45 10 1 23
50-60 18 55 20 2 36
N= 120 100 'Lfd' =-39

" fd' (-39)


X = A+-L..--xi = 35+--xlO
N 120
= 35 -3.25 = 31.75
Table for Calculation of Median
Wt ingms Number ofarticles Cumulative frequency
0-10 14 14
10-20 17 31
20-30 22 53
30-40 26 79
40-50 23 102

r
50-60 18 120

Median No. = C~O item = 60 th item.

It belongs to 30-40 class. Thus, using the formula

Median = 1+ ~ (~ - F )
= 30+~(60-53) = 30+~x 7 = 30+ 70 = 32.69
26 26 26
Measure o/Central Tendency 253

Mode
Since highest frequency is 26 which belongs to 30-40 class. Thus, using the formula

Mode = 1+ I - I-I X h
21 - I-I - Ii
26- 22
= 30+ xlO
2x 26 -22 -23

= 30+~ =35,71.
52-45
Example 9: The rise in prices ofacertain commodity was 5% in 1954,8% in 1955 and 77%
in 1956. It is said that the average price rise between 1954 and 1956 was 26% and 30%.
JustifY this statement and show how you would explain it before a layman.
Solution: Assuming the price in year 1954 is 180.
105 + 108 + 177
Arithmetic mean = 3 = 130

i.e., an average change 000%. However, if we use the average change of30% per annum of
result would not tally with the actual change in prices.
If we use the geometric mean of 105, 108 and 177, the results would reflect the real
change in prices.
G.M. ofthe prices would be

% olpri~e Price at the end log X


olyearX
5 105 2.0212
8 108 2.0334
77 177 2.2480
LlogX= 6.3026

6.3026
G. M. = antilog - 3 - = 126.2

Average price rise during the period was 26.2% per annum.
Thus, if we use G. M. The % increase would be 105 for 1954 and 113.4 (105 + 8% of 105)
for 1955 and200.7(113.4+77% of113.4)for 1956.
But if we use A.M., it would be 130 (for 1954),169 for 1955 and219.7 for 1956. Thus A.M.
does not give a correct value, while G.M. tally the price.

~~~~~~~I EXERCISE 5.61~~~~~~~


1. Find the mode ofthe following data: 4,5,8,6,9,8,8,6,5,11,9,8
2. Find the mode of the following data: 0, 1,6,7,2,3,7,6,6,2,6,0,5,6, °
3. Find the mode of the following data:
2 3 4 5 6 7 8 9 10 II 12 13
3 8 10 12 16 14 10 8 17 5 4 )
254 Remedial Mathematics

4. Find the mode of the following data:

Monthly rent 2040 40-60 60-8080-100 100-120120-140 140-160 160-180180-200


No. 0/students 6 9 11 14 20 15 10 8 7
5. Find the mode ofthe following frequency distribution:
Height 120-124 125-129 130-134 135-139 140-144 145-149 150-154
Frequency 2 5 8 15 20 10 5
6. Find the mode of the following frequency distribution:
Marks 10-25 25-40 40-55 55-70 70-85 85-100
Frequency 6 20 44 26 3
7. Find the mode for the following frequency distribution:
Age 20-25 25-30 30-35 35-40 40-45 45-50 50-55 55.(j()
No.o/persons 50 70 80 180 150 120 70 50·
8. Compute the mode for the following frequency distribution:

Age below (in years) 5 10 15 20 25 30 35 40


No.o/persons 2 4 14 27 48 64 72 75

9. Find the missing frequency for given frequency distribution when the mode is
given by 47.5:

Class 0-10 10-20 20-30 30-40 40-50 50-60 60-70 70-80


Frequency 7 8 10 40 35 10 7

10. Find the missing frequency for the given frequency distribution:

Classes 0-10 10-20 20-30 30-40 40-50


Frequency 14 27 15

The mode and the median for the distribution is given by 24 and 25.
The empirical relationship between Mean, Mode and Median
Mode = 3 Median - 2 Mean
11. In a moderately asymmetrical distribution, the mode and mean are 32.1 and 35.4.
Calculate the median.
ANSWERS

1. 1.8 2. 6 3. 6 4. 110.9
5. 191.25 6. 58.75 7. 42 years 8. 23.1
9. 25 10. 25,24 11. 34.3

DOD
TRIGONOMETRY

• INTRODUCTION
The word trigonometry is derived from two Greek word "trigon" and "metron ", means
"triangle" and "to measure" respectively. Therefore, trigonometry means to measure a triangle,
i.e.,
"Trigonometry is that branch of Mathematics which deals with angles, whether of a
triangle or any other figure ",

• ANGLES AND QUADRANTS


Consider the Fig. 6.1, the angle is obtained by rotating a given
ray about its end points. The original ray is called the initial
side and the ray into which the initial sides rotates is called 0 ~------- X
Initial side
the terminal side ofthe angle.
Fig. 6.1
Remarks
1. The measure of an angle is the amount of rotation required to get the terminal side
from initial side.
2. Ifthe revolving line revolves in anticlockwise direction, then add the angle is positive
and if revolving line revolves in clockwise direction, then it is called negative angle.
This may be clear in the following Fig. 6.2.
p
o~---'r-------x

+ ve angle
o "----'I..------x p

Fig. 6.2

• MEASUREMENT OF ANGLES
Generally, we measure the angles in degrees or in radian, which are defmed as follows:
(a) Sexagesimal System or English System (Degree Measure):
We can divide the right angle into 90 equal parts, and each small part is known as degree.
256 Remedial Mathematics

Thus a right angle is equal to 90 degrees. Similarly we can say that, the circumference of a
circle can be divided into 360 equal parts. One degree is denoted by 1°.
Again we can divide a degree into sixty equal parts. Each small part is known as a minute
and is denoted by 1'.
i.e., 1° = 60' (sixty minutes).
A minute can also be divided into sixty equal parts and each small part is known as
second and is denoted by I" c
i.e., I' = 60" (sixty seconds).
(b) Radians (Circular System): B
Let us take a circle of radius a then 'a radian' is an angle subtended
at the center of a circle by an arc equal in length to the radius of the
circle. One radian angle is denoted by as 1C in the fig 6.3, LBOC= 1c. Fig. 6.3
Remark
• Relation between degree and radian: n radian = 180 degree

Grade Measure:
Q
If we divide the right angle into 100 equal parts, then each small part is

L:J
said to be grade. Ifwe subdivide each grade into 100 equal part, then each
part is known as minute and when we divide the minute into 100 parts
then each part is known as seconds.
o P
So, I right angle = 10<%
Fig. 6.4
Ig = 100' and l' = 100"
0
n radian = 180 = 20<%.

III VARIOUS TYPES OF ANGELES


(a) An angle 8 is to be acute angle if it remains in quadrant number 1
i.e., 0° ::; 8 < 90°.
(b) An angle 8 is said to be right angle if8 = 90°
(c) An angle is said to be obtuse angle ifit remains in quadrant
number 2
i.e.,
(d) An angle 8 is said to be straight angle, if8 = 180°.
(e) An angle 8 is said to be reflexive angle if it remains in
quadrant number 3 and 4 i.e., 180° < 8 < 360°.
1t (A Constant Number):

The ratio between the circumference and the diameter of a circle is known as constant
numbern.
circumference
So, n=
diameter
Trigonometry 257

Let us take a circle of radius a, then


circumference
1t=
2a
So circumference = 21ta.

Also 1t is an irrational number. We use 2: for 1t, which is its appropriate value.

Relation between an Arc and an Angle:


arc = radius x angle in radians A t---~.....l...-:---1 B
From figure 6.6, we have

Area ofthe Sector: Fig. 6.6


Let OPQ be the sector having an angle e c at the center. This sector form the circle of radius
1
- if e .
c
a, then area of the sector OPQ =
2

~~~~~~I SOLVED EXAMPLES ~I~~~~~~


Example 1: Find the degree measure ofthe given radian measure,'

(b) (~J (c) _3 c

Solution: (a) We know that


180
1 radian = - degree
1t

So,
(b) We know that
C;J = e 12
1t
x 180r =75°
1t

180
1 radian = - degree
1t

(~J (~x 1!0)


2
Therefore, [ taking the value of 1t as 27 ]

9°32'43.6" = 9°32'44".
(c) We know that
180
1 radian = - degree
1t

Therefore, _3 c = [(-3)~I!OJ
[ (-3)x 180 x7
n J [ taking the value of 1t as 2: ]

= -171°49'6".
I 258 Remedial Mathematics

Example 2: Find the radian measure ofthe give degree measure:


(a) 210° (b) - 22°30' (c) 135°
Solution: (a) We know that

1degree = ~ radian.
180

Therefore,
(b) We know that
210° = (210X 1:0J = e: r
Therefore,

(c) We know that


1t .
1 degree = 180 radIan.

Therefore, 135° = (135X I;OJ = e:J


Example 3: Find the degree ofthe angle sub tended at the center ofa circle ofdiameter 50
cm by an arc oflength 11 cm.
Solution: Given that the diameter ofthe circle = 50 em
i.e., radius ofthe circle = 25 em
and are length s = 11 em
We know that

e= (;J
Therefore,
0

e= (,!,!,x I80 )O [-: I radian =C !O ) ]

25 1t

=> e= (.!.!.x 180


25 22
X7) [Takingn:= 22J
7

=> e = C~6} = (25~)O = (25+~}


Trigonometry 259

Example 4: Find the area ofa sector which has a central angle of60° and a radius of 3 em.
Solution: We know that the area of a circle which subtends an angle of 21t radians at the
center = 1t?-.
1t .
Again 60° = "3 radian.
Therefore, area of a sector which has a central angle of 60° (i.e., ~ radian)

1tr2 1t
= --x-
21t 3
= ~x32 [since r=3cm]
6
22 I
= -x-x9
7 6
= 4.714 square cm (app.).
Example 5: Find the length ofan arc ofa circle ofradius 5 cm subtending a central angle 15°
Solution: Let s be the length of the arc subtending an angle 8 at the Center of a Circle of
radius r. Then 8 = sir.

Here r = 5 cm and 8 = 150 = 15 x~


160
= (~)e
12

8=':: ~ s=8xr
r
= (~X5)
12 em
51t
=-cm
12
Example 6: What is the ratio oftwo circles at the centres of which two arcs ofsame length
subtend angles of 60° and 75°.
arc
Solution: We know that, radius = angle·
Here, it is given that the length of arc of two circles is same.
Let the length of arc be s.
1t 1t
Now given 8 1 = 60° = 60 x - = - radian
180 3
8 =75°=75x~=51tradian
2 180 12

Also, r, s s
= -andr2 = -
8, 82

So,
_Ij 82
= -=--=-
51t1l25
~ 8, 1t/3 4
Hence r 1 : r 2 = 5: 4.
260 Remedial Mathematics

Example 7: A horse is tied to a pole by a rope. If the horse moves along a circular path
always keeping the rope tight and describes 88 metres when it has traced out 72° at the
centre. Find the length of the rope.
Solution: In the fig. 6.7 the position of pole is indicated by O. Let P be the initial condition
of the horse and OP is the rope by which the horse is tied by the rope and the pole is in tight
condition. The horse moves along the path PQ
Q
and given PQ = 88 m.
Also

and
OP = OQ = r (say)

LQOP = 72° = (72 x~)


180
radian
o
£j p
Fig. 6.7
( 21t ) radian.
5
s
Now we know that r =

88
therefore, => r= --x7x5
2x22
=> r = 70 metres.
Example 8: The minute hand ofa clock is 10 cm long. How far does the tip ofthe hand move
in 20 minutes?
Solution: We know that the minute hand of a clock describe a circle in 60 minute and given
that the length of the circle is 10 cm.
So minute hand describe a circle of radius 10 cm.
Therefore, the distance covered by the minute hand in 60 minute = 21tr = 21t x 10 cm.
So the distance covered by the minute hand in 20 minute
22
-x21txI0
60
1 22
-x20x- (taking 1t = 2217)
3 7
20.95cm
21 cm(appr.)
Example 9: Find the number of degrees in the angle subtended at the center ofa circle oj
radius 10ft. by an arc oflength 20 ft. [UPTU B. Pharma 2005)
Solution: Using the relation s = r8, we get
8 C = angle subtended at the center
20 .
10 = 2 radIans

2XC!0)"

(2X 1;~ X7)0


114°32'43".
Trigonometry 261

Example 10: An electric fan makes 100 revolution per minute. Find the angular speed in
radian per second. Find also the linear speed in feet per second of the tips of the blades.
The tips being 10 inches from the centre.
Solution: (i) Given that the number of revolution per minute = 1000.
1000 100 50
So number of revolution per second = 60 = 6 = 3
Now, the angle described by the blade in one revolution = 2n radian
So, angular speed (00) = 2n (number of revolution per second)
50) lOOn .
= 2n ( 3 =-3- radian/sec.
(ii) Also given that, radius of the blade = 10 inches
10. 5
= - = - feet [Since 1 feet = 12 inches]
12. 6
So, linear speed of the tip ofthe blade = roo
[Since linear speed = radius x angular speed]
5 lOOn 5 100 22
-X--=-X-X-.
6 3 6 3 7
Hence, linear speed = 87.3 feet/sec.
Example 11: !fa, D and C be the number of degrees and radians in an angle. Prove that
G D 2C
(a) - = - = - (b) G-D = 20C.
100 90 n n
Solution: Let the measure of the given angle be x right angle.
Then, 1 right angle = 10<% ~ x right angle s = (1 OOx)g ... (1)
1 right angle = 90° x right angles = (90x)0

r
... (2)

2 right angle s = rf ~ x right angles = (~x ... (3)

(a) Now from equation (1) ,(2),(3), we have

l00x = G, 90x = D, n; = c,
G D 2C G D 2C
Hence, -=-=x ~ -=-=-
100 90 n 100 90 n
2c
(b) Again G = 100x, x = ~ G-D= 100x-90x
n
2C 20C
lOx = l O x - = - -
n n
20C
G-D =
n
Example 12: The angles ofa triangles are in A.P The number ofgrades in the least is to the
number ofradians in the greatest as 40 : n. Find the angles in degrees.
262 Remedial Mathematics

Solution: Let the angles be (a - d)0, a O, (a + d)0.


As we know that the sum of the angle of a triangles is 180°.
So, a - d + a + a + d = 180 ~ 3a = 180°
~ a=60°.
Therefore, the angles are (60 - d)0, 60°, (60 + d)0.
Clearly, the least angle is (60 - d)0 and the greater is (60 - d)0.

Now (60-d)0 = [(60-d)X 200]g


180

(60-d)° = [600~lOdr .

And (60+d)° = [ (60+ d)~]C


180
Now according to the given condition, we have
(600 -lOd) 180 40
x ----:---"7" 120+2d=600-lOd
9 1t( 60 + d) 1t
~ d=40.
Hence, the angles ofthe given triangle are 20°, 60° and 100°.
Example 13: The circumference ofa circle is divided into 5 parts inA.P' and ifthe greatest
part is 6 times the least, find in radian the magnitudes ofthe angles that the parts subtend
at the ~enter of the circle.
Solution: Let r be the radius ofthe circle. Ifthe five parts of the circumference ofthe cir~le are
inA.P.
Then we can take five parts as x - 2y, x, - y, x. x + y, x + 2y.
Also the sum of five part = circumference ofthe circles
i.e. x-2y+x-y+x+x+y+x+2y =2nr
5x =2nr
21tr
x= - .
5
According to the given condition, we have
greatest part = 6 x least part
(x + 2y) = 6 (x-2y)
x + 2y = 6x- 12y
5x = 14y
14y
x =
5
5x
or y= 14 .
Trigonometry 263

Putting the value of x in equation (2) from (I)


1tr
y =-
7
. . . . 41tr 91tr 21tr 191tr 241tr
:. FIve parts mA.P. of pen meter of the CIrcle are 35 ' 35 ' -5- , 35 ' 35
arc
Again we know that 9 (angle at the center) = -d-'- .
ra lUS
So angles in radians subtended at the center of the circle by the five parts which are
inA.P.
41t 91t 21t 191t 241t
- - --
35 ' 35 ' 35 ' 35 ' 35 .

1. Find the degree measure of the given radian measure

(a) C; J (b)
2. Find the radian measure of the given degree measure
(~J
(a) 240° (b) 15° (c) 5°37'30".
3. Find the radius of a circle in which a central angle of 45° intercepts an arc of 187 cm.
4. The large hand of a big clock is 70 cm long. How many cms does its extremity move
in 6 minute time.
5. Find the angle between the minute hand and the hour hand of a clock when the time
is 7.20.
22
[Hint. Angle making by hour hand at 7.20 i.e., 3 = 9 1
Angle making by minute hand in 20 minute = 92
Angle between two hands = 9 I - 92]
6. A point on a turbine wheel of radius 3 metres moves with a linear speed of 15 metres
per second. Find the rate at which the wheel turns (i.e., the angular speed) in a
radian per second.
7. The perimeter of a certain sector of a circle is equal to the length of the are of a
semicircle having the same radius; express the angle of the sector in degrees,
minutes and seconds.

ANSWERS
1. (a) 105°, (b) 14°19'5". (c)-l 14°32'44"

2. (a) (~1t J ,(b) C~J ,(c) (3~J '


3.238cm. 4. 44cm. 5. 100°

6. 5 rad/sec. 7. CI2
10)".
264 Remedial Mathematics

• TRIGONOMETRIC RATIOS OR FUNCTIONS


In a right angled triangle ABC, if LeAB = e
=> BC = perpendicular (y)
and AC = Base (x)
AB = Hypotenuse (r)
We define the following trigonometric ratios which are also known as trigonometric
functions: c
Perpendicular y
(i) sine e = = - , and is written as sin e.
Hypotenuse r
y
Base x
(li) cosine e = = - , and is written as cos e.
Hypotenuse r
A "'--'-"0_ _ _ _ _--'-' B
Perpendicular y . . x
(iii) tangent e = = -, and IS written as tan e.
Base x
Hypotenuse r
(iv) coseant e = d' I = -, and is written as cosec e.
Perpen ICU ar y
Hypotenuse r . .
(v) secant 8 = = - , and IS written as sec 8.
Base x
Base x .. 8
(vi) cotangent 8 = . = - , and IS wntten as cot .
perpendicular y
From the above definitions, it follows some definitions:
sin 8 1 1
tan e = --e' cosec 8 = -:-8 ' sec 8 = --e '
cos Sin' cos
1 cose
and cot e = or
tan 8 sine
Remarks
o Sine and cosine functions are called primary functions whereas tangent, cotangent,
secant and cosecant functions are called secondary trigonometric functions.
o When the terminal ray coincides with x-axis, cosec e and cot e are not defined.
o When the terminal ray coincides with y-axis, sec e and tan e are not defined.
o The domain of the sine function is the set of real number, whereas its range is the set
of real numbers from -I to 1.
o The domain of the cosine function is the set of all real numbers and range is the set of
real numbers from -I to I.
o The domain of the tangent function is the set of all real numbers except odd multiples
of nl2 and its range is the set of all real numbers.
o The student should not commit the mistake of regarding sine as sin X8; sin e means
the sine of angle e, it is absolutely wrong to perform such operations as:
sin (A + B) = sin A + sin B
sin 2x + sin x = sin (2x + x).
o Power notation for trigonometric function (sin ei is written as sin2 e and is read as sin
square e, (sin e)3 is written as sin3e and is read as sin cube e. )
Trigonometry 265

• TRIGONOMETRIC IDENTITIES AND EQUATIONS


An expression involving trigonometric function which is true for all those values of 8 for
which the functions are defined is called a trigonometric identity. Otherwise, it is a trigonometric
equation.
I 1
For Example (i) sec 8 = --9 ' cosec e = -:--e etc. are true for all values of8 except those
cos Sin .
for which cos e = 0, sin e = o. So these are called trigonometric identities.
For Example (ii) sin 8 = cos 8 is an expression which does not hold for all values of8. So it
is an equation and not identity.

• FUNDAMENTAL TIUGONOMETRIC IDENTITIES


For any angle e, we have
(i) sin2 8 + cos 2 8 =1 (ii) 1 + tan 2 8 = sec 2 8
sin8
(iii) 1 + cot2 e = cosec2 e (iv) tan 8 = - -
cose
cos9
(iv) cot 8 = --:--9
Sin

Proof: Let a ray starting from OX; trace out any angle 8 in any ofthe four quadrants and let
it take the final position OP. From P draw PM perpendicular to x-axis.
Now, in right angled triangle 1'10Mp, we have
y
OP2 = OM2 +PM2 p

PM 2 +OM 2 x' x
OP2
PM)2 PM2
1+ ( - - =1+--
OM OM 2 y'

OM 2 +PM 2 = Op2 = (OP)2 Fig.6.S


OM 2 OM 2 OM

(iii) 1 + cot2 e = 1+ (~~f


OM 2 PM 2 + OM 2 Op2 Op2
1+ - -
= = ------,,-- = - -2 = -- = cosec 2 8.
PM 2 PM 2 PM PM 2
sine PM OP PM
(iv) - - = - - x - - = - - = tan 8.
cos 8 OP OM OM
(v) cos9 = OM x OP = OM = cote
sin9 OP PM PM
. 1
(VI) cot e = --8 => tan 8 cot 8 = \.
tan
266 Remedial Matheniatics

Trigonometric Ratios of Standard Angles:

The values of Trigonometric Ratio of trigonometric angles of .:: (30°),':: (45°), .:: (60°), .::
6 4 3 2
(90°) given in the following table 1:
Table 1
8 0 d5 1tftl. 1t/3 1t1l 1t 31t1l 2n:
1 1 fj
sin 0 - - 1 0 -1 0
2 J2 2
fj 1 1
cos 1 - - 0 -1 0 1
2 J2 2
1
tan 0 1 fj 00 0 00 0
fj
1
cot 00 fj 1 0 00 0 00
fj
2
sec 1
fj
J2 2 00 -1 00 1

2
cosec 00 2 J2 fj 1 00 -1 00

SOLVED EXAMPLES

I + COS 8
Example 1: Show that --- = cosec 8 + cot 8.
I-cos8
Solution: Here, we have
1 + cos 8
L.H.S. =
1- cos8

1 + cos 8 1 + cos 8
/---x---
I-cos 1 + cos8

1 +cose 1 cos 8
---=-+--
sine sine sine
= cosec 8 + cot 8 = R.H.S.
tan8-cot8
Example 2: Show that . 8 8 = sec2 {}- cosec2 {}
Sin cos
Trigonometry 267

tanS - cotS
Solution: L.H.S. =
sinS cos S

sinS cosS sin 2 S - cos 2 S


-----
cosS sinS sinS cosS
sinS. cosS sinS cosS
sin 2 S - cos 2 S sin 2 S cos 2 S
2
2
sin S cos S 2 2
sin S cos S sin S cos 2 S
2

I 1
--
2
- - 2- = sec2 9 - cosec2 S = R.H.S.
cos S sin S
1- sin A
Example 3: Show tNat (sec A - tan Ai = .. [RGPV B. Pharma 20021
I'+sinA
Solution: Consider L.H.S. = (sec A - tan A)2

1 sin A)2 (I-sin A)2


= ( cosA - cosA = cosA

(1- sin A)2 = (1- sin A)2 = (1- sin A)2


2
cos A (l-sin 2 A) (l-sinA)(l +sinA)
= I-sinA =R.H.S.
1+ sin A
tanS + seeS -1 1 + sinS eosS
Example 4: Sh ow t hat = =- - - [RCPV B. Pharma 20041
tan-seeS + 1 easS I-sinS
[UPTU B. Pharma 20041
tanS + seeS -1
Solution: Consider S S·
tan - sec + 1
Using 1 + tan2 S = sec2 S, i.e., sec 2 S - tan 2 S = 1, we get
tanS + seeS -1 tan S + sec S - ( sec 2 S - tan 2 S)
tan S - sec S + 1 tan S - sec S + 1
(tan 8 + sec8) -(sec8 - tan 8)(sec8 + tan 8)
tan 8 - sec 8 + 1
(seeS + tanS) (1- seeS + tanS) S S
-'-------'-"'-------....:.. = sec + tan
1- sec S + tanS
1 sinS 1 + sinS
--+--=---
cosS cosS cosS
(1 + sinS) (I-sinS) cos 2 S
~--...:.. x =- - - - -
cos S (1 - sin 8) cos8(1-sin S)
cosS
1- sin. S R.H.S.
268 Remedial Mathematics

Example 5: Show that 2 (sin 6() + cos 6 fJ) -3 (sin 4 ()+cos 4 ()) + 1 = 0 [UPTUB. Pharma2005\
Solution: Consider L.H.S. = 2 (sin6 e + cos6 e) -3 (sin4 e + cos4e) + 1
[.,' a' + b3 = (a+ b)3-3ab (c?+ b2 )
= 2[(sin2 e)3 + (cos 2e)3] - 3 [(sin2e)2 + (cos 2 e) + 1
2 2
= 2 [(sin2e + cos e)3 - 3 sin2 e cos e (sin2e + cos 2e)
- 3 [(sin2e + cos 2 e)2 - 2 sin2 cos2e] + 1
= 2 [(1)3 - 3 sin2 e cos 2 e '1] -3 [(1)2 - 2 sin2 cos2e] + 1
= 2 [1 - 3 sin 2e cos 2e] - 3 [1 - 2 sin 2e cos 2e] + 1
= 2 - 6 sin2e cos2 e - 3 + 6 sin2 e cos 2 e + 1
= O=R.H.S.
sine sinS
Example 6: Show that = 2 +-----
cot e + cosec S cot e - cosec e
sine sine
Solution: Consider L.H.S. =
cot S + cosec e - - + -1-
cose
sine sine
sine 2
sin e I-cos 2 e
cose + 1 1 + cose 1 + cose
sinS
(1- cosS) (1 + cosS)
-'-----'----- = (1 - cos e). ...(1)
1 + cosS
sinS
Now, R.H.S. = 2 + - - - - -
cot S - cos ece
2
2+ sine = 2 + sin S = 2 + sin S
cosS cos e - 1 eos e - 1
sine sinS sine
1- eos 2 e (1- eose)(1 + cos e)
2+ = 2 - -'-----'--'----
cos e - 1 1 - eos e
2 -(1 + eos S)
2 -1- cos S
= I-cos e ... (2)
From (1) and (2), we conclude that
sine sine
2+-----
cot S + eos eeS cot e - cos eee
111 1
Example 7' Show that - -- = -- - ----
. seeS + tane case case sece - tane
1 1
Solution: +---- --+--
seeS + tan e seeS - tan S cose cose
Trigonometry 269

I I
If --------+--------
see8 + tan 8 see8 - tan 8
--+--
eos8 cos8
sec8 - tan 8 + sec8 + tan 8 2
i.e.,if (sec+tan8)(sec8-tan8) cos8

2sec8 2
i.e., if
cos8

i.e., if 2 see 8, whieh is always true.


I
Example 8: Prove that the expression 2 (sin 6 {}+ cos6 B) - 3 (sin4 {}+ cos 4 B) is independent
ofB.
Solution: We have
sin68 + cos6 8 = (sin2 8 + cos2 8) (cos4 8 + sin4 8 - sin2 8 cos2 )
I. ( cos 48 + sin 4 8 + 2 sin2 8 . cos 2 8 - 2 sin2 8 cos 2 8 - sin2 8 cos2 8)
(eos 2 8 + sin 2 8)2 - 3 sin 2 8 cos 2 8.
Now using this result in L.H.S. we get
2(1 - 3 sin2 8 cos 2 8) - (sin4 8 + cos 4 8)
(1- 3 sin 2 8 cos 2 8)-3 [sin 48 + cos4 8 + 2 sin2 8 cos 2 8 -2 sin2 8 cos 2 8]
= 2 - 6 sin2 8 cos 2 - 3 [(sin2 8 + cos 2 8)2 - sin2 8 cos 2 8]
= 2 - 6 sin2 8 cos2 8 - 3 + 6 sin2 8 cos2 8
= -I

which is independent of8.

(I+sin8-cos8)21-cos8
Example 9: Prove that 2 I + cos 8 . [Meerut B.Se Biotech 2006)
(I + sin 8 + cos 8)
I + sin 2 8 + cos 2 8 + 2 sin 8 - 2 cos 8 - 2 sin S cos S
Solution: L.H.S. =
1+ sin 2 8 + cos 2 8 + 2 sin8 + 2 cos S + 2sin8 cosS
I + I + 2 sin S - 2 cos S (1 + sin S)
I + 1 + 2 sin S + 2 cos 8 (I + sin 8)
2 (1 + sin8) - 2 cosS (1 + sinS)
2 (1 + sinS) + 2 cos8 (1 + sinS)
2 (1 + sin S) (1 - cos 8)
2 (1 + sinS) (I + cosS)
1- cos8
=R.H.S.
1 + cos S
Example 10: If tan {} + sin {} = m and tan {}- sin {} = n, show that (m 2 - n2) = 16 mn.
Solution: Here the given equation are
tan 8 + sin 8 = m ... (1)
and tan 8 - sin 8 = n ...(2)
270 Remedial Mathematics

Adding (1) and (2), we get


2tan8 = m+n.
m+n 2
tan 8 = => cot8= - - .
2 m+n
Subtracting (2) from (1), we get
2 sin 8 = m - n => sin 8 = m - n .
2
2
cosec e =
m-n
Putting the values of cot e, cosec 8 in equation
cosec2 8 - cot2 = 1,
4 4
we get
(m_n)2 (m+n)2
or 4(m+nyl-4(m-n)2 = [(m-n)(m+n)f
Therefore, 4(m 2 + n2 + 2mn)-4 (m 2 + n2-2mn) = [(m-n)(m + n)f
=> 4m 2 + 4n 2 + 8mn - 4m 2- 4n2 + 8mn = (m 2- n2i
=> 16mn=(m2-n 2i
or (m 2 _n 2)2 = 16 mn.
sec ()
Example 11: Show that ( J + cot () + tan ()) (sin ()- cos()) = ---=-
cosec 2()

Solution: Consider L.H.S. = (1 + cot e + tan 8) (sin e - cos e)


COS e sin e ) .
(
1 + - . - + - - (SIn e - cos 8)
sme cose
2
sinecose+cos +sin28) (. e
SIn - cos
e)
( sin e cos e
[.: if _b 3 = (a-b)(~ + b2 + ab)
(sin8 - cos8) (sin2 8 + cos 2 + sin 8 cos 8)
sin 8 cos8
3 3
sin e - cos 8
... (1)
sin e cose
secS cosec8
R.H.S. = - - - ---
cosec 2 sec 20
1
2 2
cose sin e
-----=-----
sin cos e
cose sine
2 2
sin e cos e
sin 3 e - cos3 8
... (2)
sine cose
Trigonometry 271

From (1) and (2), we get


sec e cosec e
(sin e- cos e) (1 + cot e + tan e) =
cosec e 2
sin 2 e
1 1
Example 12: Ifsec e = x + -4 ,show that sec e + tan e = 2x or - .
x 2x
Solution: We have
I 4x 2 + I
sec e = x+ - => sece= - - - ...(1)
4x 4x
Now, tan2 e = sec2 e - I

(4X;x+lr -1= (4x2 +1~~2-16X2 = (4~::21)2


4x 2 -I
tane = ± - - - ... (2)
4x
Adding (1) and (2), we get
4x 2 + I 4x 2 -1
sece+tane = - - - + - - -
4x - 4x
2
4x2+1 4x2-1) (4x2+1 4X -1)
( ---+ or ----
4x 4x 4x 4x

- - - - - - or - - - - - -
4x 4x
8x 2 2
-or-
4x 4x·
1
Hence, sec e + tan e = 2x or - .
2x

1. Show that
1- sine 1- cose
(a) / - - - = sec e -tan e (b) / - - - = cosec e - cot e.
I + sine 1+ cose
(c) 2 4 2
2 sec e - sec e - 2 cosec e + cosec e = cot4 e - tan4 e.
4

(d) sec6 e = tan6 e + 3 tan2 e sec2 e + 1. (e) sec4 e - sec2 e = tan4 e + tan2 e.
2
(f) tan2 e _ sin2 e = tan2 e . sin2 e. (g) I + cos e = tan e .
1- cose (sece -Ii
sece - tane
(h) e = I - 2 sec e tan e + 2 tan 2 e.
sece + tan
1- cose sine
(i)
sine 1 + cose·
0) tan2e + co2 e + 2 = sec2 e . cosec2 e IUPTU B. Pharma 2001)
272 Remedial Mathematics

2. Eliminate 8 from the following equations:


(a) x = a cos 3 8. y = b sin 3 8
(b) x=asec 3 8,y=btan 3 8
(c) sec 8 + tan 8 = m, sec 8 - tan 8 = 7t
(d) acot8+ bcosec8=x2 ,bcot8+dcosec8=/
cos8 sin8
3. Prove that 1 8 +1 8 = sin 8 + cos 8 [RGPV B. Pharnia 20011
- tan - cot
4. Prove the following:
(a) sins 8 - cos s 8 = (sin2 - cos 2 8) (1 - 2 sin2 8 cos 2 8).

(b) sinS + 1 tanS S = sec 8 cosec 8 + cot 8.


1- cosS + cos
sin8 1 + cos8
(c) + = 2 cosec 8.
1 + cos8 sin8

(d) cosec8 + cot8 - -si-n-8 = -si-n-8 - cosec8 - cot 8


21
5. (a) If cos 8 = 29 and 8 lies in the fourth quadrant, find sin 8 and tan 8.

(b) If cos 8 cosec 8 = - 1 and 8 lies in the fourth quadrant, find cos 8 and cosec 8.
6. Prove that
(a) sin 8 cot 8 + sin 8 cosec 8 = 1 + cos 8.
(b) sec 8 (1 - sin 8)(sec 8 + tan 8) = 1.
tan 8 cot 8
(c) 8+ 8 = 1 + sec 8 cosec 8.
I-cot I-tan
(d) (1 + cot 8 - cosec 8)(1 + tan 8 + sec 8) = 2.
(e) (cosec 8 - sin 8) (sec 8 - cos 8) (tan 8 + cot 8) = 1.
. 1
(t) (sec 8 - cos 8) (cosec 8 - SIll 8) = 8 8
tan + cot

(g) ~sec2 + cosec 28 = tan 8 + cot 8 = sec 8 cosec 8.


(h) (sin 8 + cosec 8i + (cos 8 + sec 8i = tan2 8 + co~ 8 = 7
(i) cosecS + cosecS = 2 sec2 8.
cosecS - 1 cosecS + I
sin A - sin B cos A - cos B
G) + =0
cos A + cos B sin A + sin B
1 + sin 8
--- = sec 8 + tan 8.
(k) sec8 - tan 8 cos8

sin A
7. If - . - =
cosA
m and - - = n, show that tan A =
m~-n2
± - -2-'
SIll B cos B n m - 1
Trigonometry 273

9. (a) If sin 8 = ~~, show that sec 8 + tan 8 = %, if 8 lies between 0 and ~,
(b) What will be the value of the expression when 8 lies between ~, and 1t.
2

HINTS TO THE SELECTED PROBLEMS

(i) (e). sec4 8 - sec 2 8 = sec2 8 (sec 28 - 1)


= (l + tan2 8) [tan2 8]
= tan2 8 + tan4 8
= tan 4 8 + tan2 8 .
sec 2 S -tanS (sec 8 -tan8) x (sec8 - tan 8)
(h) - - -
secS+tanS (sec 8 + tan 8) (sec8 - tan 8)

(sec8 - tan 8)2


sec 2 e - tan 2 8
sec 2e + tan 2e - 2 seeS tanS
1
1 + tan 2 8 + tan2 8 - 2 sec tan 8
1 - 2 sec 8 tan 8 + 2 tan2e.
2. (a) x = a cos3 8 ,Y = sin3 8.
x
- =cos 3 8
a '

i
113 113
cos 8 = ( ; ) sin8= ( )
2 2
cos e + sin 8 = I

(~f/3 + (1;)2/3 = 1.
(c) sec 8 + tan 8 = m ... (1)
sec8-tan8=n ... (2)
(l) x (2) (sec e + tan 8) (sec e - tan e) = m x n
= sec2 e - tan2 8 = mn
~mn=1

4. (a) sin8 8 - cos 8 8 = (sin4 8 + cos4 8) (sin4 e - cos 4 e)


= [(sin2 e + cos 2 e) - 2 sin 2 8 cos 2 8)] [(sin 2 8 + cos 2 8) (sin2 8 - cos 2 8)]
= (1- 2 sin2 8 cos 2 e) (sin2 e - cos 2 e)
= (sin2 e - cos2 e) (1 - 2 sin2 8 cos2 8).
1 1 1 1
(d) cosece + cot 8 - -si-n-e = -si-n-e - cosec8 - cot 8

2
or - ----+-----
cosec e + cot e cosec e - cot e sin e
274 Remedial Mathematics

S. (a) If cos 9 = ~! and q lies in fourth quadrant.


Then in fourth quadrant sin and tan both are negative
20
sin 9 = - 29
20
tan 9 = _ 20
21
tan 9 cot 9 21
6. (c) +I 9 = I + seq e cosec 9. Fig. 6.9
1- cot 9 - tan
sin9 cos9
sin 2 9 cos 2 9
L.H.S. = coS9 +
I_COS 9
sin.9
9
I-~
------------+------------
cos9(sin9 - cos a) sin a(cos 9 - sin9)
sin a cosa
sin 3 9 - cos9 3 (sin9 - cos9) (sin2 9 + cos 2 + sina cos 9)
sin 9 cos a (sin a - cos a) sin a cos a (sin a - cos a)
1+ sin9 cosa
sin 9 cos9

(g) ~sec2 9 + cos ec 29 = tan 9 + cot 9 = sec a cosec 9

I I
L.H.S.: ~sec2 a + cos ec 2a = ----+----
2 2
cos 9 sin 9

sin 2 a + cos 2
sin 2 9 cos 2 9

- - - - = sec 9 cosec 9 = R.H.S.


sin a cos9
Middle Term: tan a + cot 9
sina cos9 sin 2 9 + cos 2 9 I .
= --+-- = = . 9 9 =sec9cosec9=R.H.S
cos 9 sin 9 sin 9 cos 9 Sin cos
1 I+sin9
(k) = = sec a + tan a
sec9 - tan a cos9

L.H.S. = . 9
Sin
seca---
cos9
cos 9 cos 9(1 + sin 9)
1 - sin 9 (1 - sin 9)(1 + sin 9)
cos a (1 + sin 9)
(1 - sin 2 a)
Trigonometry 275

cosa (1 + sin a)
=>
cos 2 a
1+ sina
=> => Middle Tenn.
cosa
1 sina
--+--
cosa cosa
sec a + tan a => R.H.S

ANSWERS

X)2/3 (y)213_ (b) ( ~ )


2/3 (
f
)2/3
2. (a) (-a . + -b -1, - = 1,

(c) mn = 1, (d)x4 _y4 = b2 -c?


20 20
7. (a) - 29' -21'

III SIGNS OF TRIGONOMETRIC FT)NCTIONS


(a) When the angle is-x radians (x> 0):
Take a circle with center 0 at the origin and radius equal to 1 unit.
Take L. AOP = x radian
a = cos x, b = sinx. Y
=>
Let L.AOQ = -x
[clockwise rotation from OX]
Co-ordinates of point Q are (a, - b).
X' ---t----:=*"-'r--+-- X
x-eo-ordinates = cos ( -x) A

=> a = cos (-x)


=> cos(-x) = a= cosx [By()]
:. y-coordinate = sin (-x)
Y'
=>-b=sin(-x)
Fig. 6.10
=> sine-x) =- b =-sinx
sin ( - x) - sin x
Now tan (-x) = = - - =-tanx.
cos( - x) cosx
Hence, sin (-x)=-sinx, cos (-x) = cos x, tan (-x)=-tanx.
(b) Trigonometric Ratio for 0 <x < 1t/2 (First quadrant) :
The terminal side OP lies in quadrant I and as such both a and b are positive i.e., > 0
cos x = x-eo-ordinate of P = a > 0
sin x = y-co-ordinate of P = b > 0
276 Remedial Mathematics

Y
sinx 1
tan x = --=->0
cosx a
Taking the reciprocals, we get
1
sec x = ->0 X' --t----::-f<--''---+-A- X
a
1
cosec x = ->0
b
a Y'
cot x = ->0
b Fig. 6.11

Hence, for 0 <x < 2: (first quadrant) all six trigonometric ratios are positive i.e., > 0
2
For, 0<x<1t/2
sin x > 0, cosx> 0, tan x > 0, cosec x > 0, secx> 0, cot x > 0
Also for every P (a, b) on the circle of radius 1 unit, we have
- 1 ~ a ~ 1 and - 1 ~ b ~ I
- 1 ~ cos x ~ 1 and - 1 ~ sin x ~ 1.
Remark
• For any angle xc,
-1 ~ sinx< 1
and -1 ~ cosx ~ I.

(c) Trigonometric Ratios for 2: <x < 31t (second quadrant):


2 2

For '21t <x < 1t i. e., 90° <x < 180°, the terminal side CQ of LA OQ lies in quadrant if, when
co-ordinates of Q are (-a, b)
Y
~ cos x = -a<0
sinx = b>O·
b
tan x = - < 0
-a
1 x' A
X
sec x = - <0
a
I
cosec x = - >0
b
Y'
-a
cot x = - < 0 Fig. 6.12
b
1t
Hence, for - < x < 1t (quadrant II)
2
Trigonometry 277

sin x and cosec x are positive and remaining four ratios are negative.

, 2 <x<1t
For'::'
sinx> O,cosx<O,tanx<O.
cosec x > 0, secx < 0, cot x < O.

(d) Trigonometric Ratios for1t<x< 31t (Third quadrant) :


2

For 1t < X < 31t , the terminal side OR of the angle lies in quadrant III where the co-ordinates
2
of point R are ( - a - b)
Y
=> cos x = -a < 0
sin x = -b < 0
sin x
tanx =
cosx X'
-b b
- =->0
-a a
1
cosec x -<0
-b Y'

1 Fig. 6.13
sec = - < 0
-a
a
cotx = ->0
b
=> Only tan x and cot x are positive and the remaining four ratios are negative in-sign.
31t
For1t<x< --
2'
sin x < 0; cos x < 0; tan x > 0
cosec x < 0; sec x < 0; cot x> O.

(e) Trigonometric Ratios for 31t <x < 2 1t (Fourth quadrant) :


2

For 31t < x < 21t, the terminal side OS of angle x lies in quadrant IV and as such the
2
Y
co-ordinates of point S are (a, - b)
cos x = a>O
sinx = -b <0
-b
tanx = - < 0 X'---+----~~~--~A--.X
a
a
cot x = - <0
-b
B
1 Y'
sec x - >0
a Fig. 6.14
278 Remedial Mathematics

I
cosec x = -b <0.
=> Only cos x and sec x are positive and the remaining four trigonometric ratios ar~
negative.
31t
For - < x<21t
2
sin x < 0; cos x > 0; tan x < 0
cosec x < 0; sec x > 0; cot x <0.
The above results may l1e summarized as
y

Quadrant II Quadrant I
1t 1t
-<x<1t O<x<-
2 2
Sine and Cosecant are + ve All T-ratios ar + ve
All other are -ve
X' .x
Quadrant III Quadrant IV
3x 31t
1t<x<- -<x<21t
2 2
Tangent and Cotangent are +ve Cosine and secant are +ve
All other are -ve All other are -ve

y'
We have already shown that one complete revolution followed by a turn of x radians
brings the terminal line in the same position OP as in the case with only a turn of x radians.
=> For both angle x and angle 21t + x, the co-ordinates of point P are (a, b)
cos x = a; cos (21t + x) = a
sin x = b; sin (21t + x) = b.
This may be followed up by any number of revolutions to give cos (41t + x) = a.
sin(41t+x)=b... Y
Hence, cos (2n1t +x) = cos x = a,n e Z
sin (2n1t + x) = sin x = b, n e Z
sec (2n1t + x) = sec x, ne Z
cosec (2n1t + x) = cosec x, n e Z. X' _-I--"T"f~=-"...1---JI--_ X
A
However in case of tan x and cot x
tan(n1t+x) = tan x, neZ
cot(n1t+x) = cot x, neZ
Y'
Fig. 6.15
Trigonometry 279

as shown below:
cos x = a
sin x = b
b
tanx =
a
a
cotx =
b

sec x =
a
1
cosec x = b
cos (1t + x) = - a ¢ cos x
sin(1t+x) = -b ¢sinx
1
sec (1t +x) = - ¢ cosec x
-a
I
cosec (1t + x) = -b ¢ cosec x

-b b
tan x = - = - =tanx
-a a
-a a
cot x = - =- = cot x
-b b
:. the values of tan x and cot x repeated a rotation of an angle 1t.
Hence tan (n1t + x) = tan x, n E Z
cot (n7t + x) = cot x, n E Z
Remarks
• In trigonometric ratios of sine, cosine, secant, cosecant we may add or subtract a
multiple of21t to the angle without changing the value of the T-ratios .
• In trigonometric ratios of tangent and cotangent, we may add or subtract a multiple of
1t and this will not change the value of the T-ratios.
For Example:

(i) sin (765°) = sin (765° -2 x 360°) = sin 45° = .1 [from tables]

-6 = cos (231t
.. cos (231t)
(11) -6 + 2 (21t) ) = cos (1t)
"6 = cos 30° = 2' J3
(iii) tan (420°) = tan [420°-2(180°)] =tan600= J3.
280 Remedial Mathematics

Some Important Results:


1. cos (2mt + 8) = cos 8 and sin (2mt + 8) = sin 8, where n is any integer.
2. cos (- 8) = cos 8 and sin (- 8) = - sin 8,for all values of8.
Behaviour of cos e and sin e as e Varies From 0 to 21t :
y
II
x../- x..L- As x=cos8
y../- yt y=cos8
cos 8..L- cos 8../-
sin 8 . L- sin 8 t
(-1,0) (1,0) t -+ lndicates for increment
x' 0 x ,j.. -+ Indicates for decrement
cos 8 t cos 8 t
III sin 8 . L- IV
xt xt
y..L- yt

y'
3. cos 8 = x and cos (-8) = x cos (-8) = cos 8 for all real numbers x ;

(a) cos(~ - x) = sinx (b) cos(i + x)= - sin x

(c) sin ( ~- x ) = cos x (d) sin (i + x ) = cos x.

4. For all real number x


(a) cos(1t-x)=-cosx (b) cos(1t+x)=-cosx
(c) sin (1t-x) =sinx (d) sin(1t+x)=-sinx.
5. For all real number x,
(a) cos (x + 21t) = cos x (b) sin (x + 21t) = sin x.
6. For all real number x and y,
(a) sin (x + y) = sin x cos y + cos x sin y
(b) sin (x - y) = sin x cos y - cos x sin y.
7. For all real number x and y,
(a) cos (x + y) = cos x cos y - sin x sin y
(b) cos (x-y) = cos x cosy + sin x siny.
8. For all real x,
(a)sin2x =2sinxcosx
(b) cos 2x = cos2 X - sin2 x = 2 cos 2 x-I = 1 - 2 sin2 x.
9. For all real x
(a) sin 3x = 3 sin x - 4 sin3 x (b) cos 3x = 4 cos 3 X - 3 cos x.
Trigonometry 281

10. For all real number x andy


(a) 2 cos x cosy = cos (x + y) + cos (x-y)
(b) 2 sin x cosy = sin (x + y) + sin (x-y)
(c) 2 sin x siny = cos (x-y) -cos (x + y)
(d) 2 cos x siny = sin (x + y)- sin (x-y).
Now, subtracting (4) and (3) we get
2 cos x sin y= sin (x + y) - sin (x - y) this proves the result (d).
11. Forallx,y E R

(a)sinx+siny=2sin (x; Y)cos( x; y)

(b) sinx-siny = 2 cos (x; y) sin ( x; y)

(c) cos x + cos Y = 2 cos ( x ; y) cos ( x ; y)

. (x+y) . (x- y
(d) cos x - cos y = 2 sin ( x ; y) sin ( y ; x) =- 2 sIn -2- Sin -2- ) .
12. Prove for all real number x andy.
(a) sin (x + y) sin (x - y) = sin 2 x - sin2 y [UPTU B. Pharma 2001]
2
(b) cos (x + y) cos (x-y) = coi x-sin y
Proof. (a) We have
sin (x + y) sin (x -y) = [sin x cos y + cos x siny] [sin x cos y - cos x siny]
= sin2 x cos 2 Y - cos2 x sin2 y
= sin2 x (I - sin2 y) - (I - sin2 x) sin2 y
sin2 x - sin2 y.
(b) We have
cos (x + y) cos (x - y) = [cos x cos y - sin x siny] [cos x cos y + sin x sin y]
cos 2 x cos2 Y - sin2 x sin2 y
cos 2 x (1- sin2 y) - (I - cos2 x) sin2 y
= cos 2 x - sin2 y.
13. For all real number x prove that tan (-x) = - tan x.
sin (-x) -sinx
Proof. We have tan (-x) = ( ) = - - =-"-tanx.
cos -x cosx '
14. For all x E R, prove that

(a) tan (~-x)""cotx (b)tan(~+x)=-cotx


(c) tan (1t -x) =- tan x (d) tan (1t + x) = tanx.

1t) sin(~2 - x) cosx


Proof. We have tan (- - x = ( ) "" - . - = cot x.
2 1t smx
cos --x
2
282 Remedial Mathematics

sin(~+ x)
(b) We have tan (% + X) 2
1t )
cosx
= -sinx
=-cotx.
cos ( - + x
2
sin (1t - x) sin x
(c) We have tan (1t-x) = = - - =-tanx.
cos(1t - x) -cos x
sin(1t + x) -sinx
(d) We have tan(1t+x) = = - - =tanx.
cos (1t + x) -cosx
Table 2 gives the sine, cosine tangent of some angles less than 90°
Table 2
9 0 15° 18° 22.so 36° 67.5°
sin 0
.J6-h .Js-.Ji ~2-h ~IO -2.Js ~h +1
4 4 4 4 J2h

.J6+h ~IO+ 2.Js ~h+ I /5 +1 ~2-h


cos I
4 4 ~2h
- 4
- 2

~25 -10/5
tan 0 2-J3
5
h-I ~5-2.Js h+1

~~~~~~~I SOLVED EXAMPLES ~I~~~~~~

Example 1: Given that sin (A + B) = Jj and cos (A - B) = Jj ,find A and B, where A and
2 2
B are positive acute angles.
Solution: Here, the given equations are

sin (A + B) =
J3 ... (1)
2

and cos (A -B) =


J3 ... (2)
2
From (1), we have
A+B=60° ...(3)
Also, from (2), we have
A -B = 30° ...(4)
Adding (3) and (4), we get
2A = 90° => A =45°.
Put this value in (4), finally, we getB= 15°.

Example 2: Show that cos 9 + sin(-9) _ tan(90 + 9) = 3.


sin(90 + 9) sin(J80 + 9) cot 9
Trigonometry 283

Solution: The L.H.S. of the given equation is


cose + sine-8) tan(90 + 9) cos9 -sin -cot9
=--+-----
sin(90 + e) sip(l80 + 9) cot 9 cos 9 -sin 9 cot 9
= 1 + 1 +=3 =R.H.S.
1
Example 3: Show that cos 24° + cos 55° + cos 125° + cos 204° + cos 300° = -.
2
Solution: L.H.S. = cos 24° + cos 55° + cos 125° + cos 204° + cos 300°
= cos24°+cos 55° +cos (180°-55°)+ cos (180 0+24°)+cos(3600-600)
= cos 24° + cos 55° - cos 55° - cos 24° + cos 60°

= cos 60° = .!. = R.H.S.


2
Example 4: Calculate
(I) cos 15° [UPTU B. Pharma 2004)
(ii) cos 75°
Solution: (i) We know that
cos 15° = cos(45°-300)
cos 45° cos 30° + sin 45° sin 30°
1 Jj 1 1 J2+1
-x-+-x-=--
J2 2 J2 2 2J2'
(ii) We have cos 75° = cos(45°+300)
cos 45° cos 30° - sin 45° sin 30°
1 Jj 1 1 Jj-l
J2 x T- J2 x "2= 2J2 .
Example 5: Find the value o/cot75°. [UPTU B. Pharma 2002)
Solution: cot75° = cot(45°+300)
cot 45° cot 30° - 1
cot 45° + cot 30°
l.Jj -1 Jj-l
I+Jj = Jj+l'
. 57t 47t r.. 7t
Example 6: Show that Sin 18 - cos 9" = ,,3 Sin 9 [UPTU B. Pharma 2003)
Solution: We have
57t 47t . 57t
sin
18 - cos 9 = Sin 18 -Sin. (7t2"-9
47t)

. 57t . 7t (57t +.2:] (57t


= Sin - - Sin - = 2 cos 18 18 sin 18 18
-~]
18 18 2 2
7t . 7t
=2 cos - SIn-
6 9
= (2X ~) sin %= Jj sin %.
284 Remedial Mathematics

Example 7: . Find the vlaue ofsec(-15000) x sin 390° [UPTU B. Pharma2008]


Solution: sec (-1500°) x sin 390° = sec(-4 x 360-60) x sin(360 + 30)
= sec(-60°) x sin 30°
= sec 60° x sin 30°
1
=2x-=1.
2
7t 97t 37t 57t
Example 8: Prove that 2 cos - cos _ + cos -3 + cos - = 0 [UPTU B. Pharma 2007]
13 13 1 13
7t 97t 37t 57t
Solution: We have 2 cos - cos - + cos - + cos-
13 13 13 13

7t 97t) ('It 97t) 37t 57t


cos ( 13 + 13 + 13 -13 + cos 13 + cos 13

107t 87t 37t 57t


cos - + cos - + cos - + cos-
13 13 13 13 .

cos (7t _ 37t) + cos 87t + cos 37t + cos (7t _ 87t)
13 13 13 13
37t 87t 37t 87t
= ~os - + cos - + cos - - cos -
13 13 13 13
=O.
Example 9: If tan A +tan B = a and cot A + cot B = b, prove that
1 1
cot (A + B) = a b .

Solution: We have cot (A + B) =


tan(A + B)
I-tanAtanB tan A tanB
tanA+tanB tanA+tanB tanA+tanB

tan A + tanB
tanA+tanB tan AtanB

tanA+tanB tan A tan B


----+----
tan A tan B tan A tan B

tan A + tanB 1 1
--+--
tanB tanA
1
tan A tanB cot A + cotB
1
a b'
Trigonometry 285

Example 10: Obtain the Values of


(i) sin 15° [UPTU B. Pharma 2004)
(ii) tan 15° [UPTUB. Pharma 2006]
Solution: (i) We have sin 15° = sin (45° - 30°)
sin 45° cos30° - cos 45° sin 30°
1.J3 1 1 _ .J3-1
J2T- J2'2 - 2J2 .
(ii) We have tan 15° = tan(45°-300)
tan 45° - tan 30°
1 + tan 45° tan 30°
1 __
1
= .J3=.J3- 1
1+_1 .J3+1·
.J3
cos 8° + sin 8°
Example 11: Prove that 80 . 80 = cot 37° [UPTU B. Pharma 2002]
cos -Sin
Solution: We have
cos 8° + sin 8° 1 + tan 8 0

[Diving Nr and Dr by cos BO]


cos 8° - sin 8° 1- tan 8°
tan 45° + tan 8°
1- tan 45°tan8°
tan (45° + 8°)
= tan 53°
= tan(90-37)
= cot 37°
4 5
Example 12: Ifsin A = 5 and cos B = 13' Find the values ofsin (A - B) and cos (A + B)
where OLA L7fl2 and 0 LB L7fl2. [UPTUB.Pharma2004[

Solution: We have sin A =


4
- and 0 < A < - .
1t

5 2
cos A = )1- sin 2
A

~1-(~r =)1- ~~ =~ =~
5 1t
Again cos B = - andO<B<-

r
13 2

sin B = )1 - cos 2 B = ~1 - C53


/44 =g.
169 13
286 Remedial Mathematics

sin (A - B) = sin A cos B - cos A sin B

( ~x~)-(~xg)
5 13 5 13
20 36 -16
---=-
65 65 65
Also cos (A + B) = cos A cos B - sin A sin B.

( ~x~)
5 13
_(~x5 12)
13
15 48 -33
---=-
65 65 65
Example 13: Prove the following identity:
sin6 0 + cos6 0 = J - 3 sin 2 0 cos 2 0 [UPTU B. Pharma 2007)
Solution: We have LHS = sin 8 + cos = (sin 8)3 + (cos2 8)3
6 6 2

= (sin2 8 + cos 2 8) [(sin4 8 + cos4 8 - sin2 8 cos 28]


= sin4 8 + cos4 8 - sin2 8 cos 2 8
= [(sin2 e + cos 2 8i -
2 sin2 8 cos 2 8] - sin2 8 cos 2 8
= 1 - 3 sin2 e cos 28 = RHS
Example 14: IfcosO+sinO= J2 cos O,showthatcos O-sinO= J2 sinO
[UPTU B. Pharma 05, RGPV B. Pharma 06

Solution: We have cos 8 + sin 8 = J2 cos 8


sin 8 = (J2 -1) cos 8
~ (J2 + 1) sin 8 = (J2 + 1) (J2 - 1) cos 8

~ (J2 + 1) sin 8 = (2 - 1) cos 8

~ J2 sin 8 + sin e = cos e


~ cos 8 - sin e = J2 sin 8.
Example 15: Show that cos 15°-sin 15°= ~.
Solution: L.H.S. = cos 15° - sin 15°

J2 (.1 cos 15° - .1 sin 150)

J2 (cos 45° cos 15°-sin45°sin 15°)


J2 cos(45° + 15°)
J2 cos 60° = J2 x.!. = ~ = R.H.S.
2 ,,2
Trigonometry 287

Example 16: Show t"at tan 13A .- tan 9A - tan 4A = tan 13A . tan 4A tan 9A.
[UPTU B. Pharma 20021
Solution: Since, we know that
13A = 9A +4A.
Therefore, tan 13A = tan (9A + 4A)
tan 13A tan 9A + tan 4A
=> 1 I - tan 9 A . tan 4A
=> tan 13A -tan 13A . tan 9A . tan 4A = tan 9A + tan 4A
=> tan 13A -tan 9A -tan 4A = tan13A . tan 9A . tan 4A.

Example 17: Show that sin 2 2: + sin 2 37t + sin 2 57t + sin 2 77t = 2.
8 8 8 8

· 2 7t . 2 37t . 2 57t . 2 7rt


Solution: Consider, L.H.S. = sm -+sm -+sm -+sm -
8 8 8 8
7t . 2 3rt . 2(
· 2 - + sm - + sm rt - -
3rt ) + sm
. 2 ( rt - -)
rt '
sm 8 8 8 8

· 2 rt . 2 3rt . 2 3rt . 2 rt
sm -+sm -+sm -+sm -
8 8 8 8

= i
2 (sin 2 + sin 2 3; )

= 2[sin2i+sin2(~-i)]
2
= 2 [sin2i+cos i] =2(1)=2.

2 7t 2 37t 2 57t 2 7rt


Example 18: Show that cos -8 + cos - + cos - + cos - = 2.
8 8 8 ~
rt 2 37t 2 57t 2 77t
Solution: Consider L.H.S. = cos 2 - + cos - + cos - + cos -
8 8 8 8

cos 2 i 2
+ cos 3; + cos 2 ( 7t - 3;) + cos 2 ( 7t - i)
rt 2 3rt 2 3rt 2 77t
cos 2 - + cos - + cos - + cos -
8 8 8 8
2
2 ( cos i + cos 2 3; )

= '8 + cos 2(7t'2 - '8rt)]


2 [ cos 2rt

= 2 [ cos 2 '87t + sm
. 27t]
'8 = 2.
288 Remedial Mathematics

Example 19: Prove that sin (A + B) sin (A - B) = sin2A - sin2 B = cos2B - coiA
[RGPV B. Pharma 2004]
Solution: sin (A + B) sin (A - B) = (sinA cosB + cosA sinE) (sinA cosB - cosA sinB)
sin2 A cos2 B - cos2 A sin2 B
sin2 A (1- sin2B)- (1- sin2A) sin2B
sin2A - sin2A sin2B - sin 2B + sin2A sin2B
sin2A - sin2B.
(1 - cos2) - (1 - cos 2B) = cos 2 B - cos2A.
Example 20: Solve sin 75° + cos 75° + sin 15° + cos 15° [RGPV B. Pharma 2001]
Solution: sin 75° + cos75° + sin 15° + cosl5°
= sin (30 +45°) + cos(30 +45°)+ sin(45° -30°) + cos(45° -30°)
= (sin 30 cos 45° + cos 30° sin 45) + (cos 30 cos 45° - sin 30 sin 45°)
+ (sin 45° cos 30-sin 30 cos 45°) (cos 30 cos 45° -sin 30° sin 45°)
1 1 Jj 1 Jj 1 1 1 1 Jj 1 1 1 Jj 1 1
= -x-+-x-+-x---x-+-x---x-+-x-+-x-
2.J2 2.J2 2 2 2.J2 2 2 2.J2.J2 2.J22
1 Jj Jj 1 Jj 1 Jj 1
= --+--+-----+-----+--+--
2.J2 2.J2 2.J2 2.J2 2.J2 2.J2 2.J2 2.J2
_2Jj
- .J2'

Example 21: Prove that sin} 05° + cos 105° = cos 45° [RGPV B. Pharma 2003]
Solution: L.H.S = sin 105° + cos 105°
sin(60° +45°)+ cos(60° + 45°)
sin 60 cos 45° + cos 60° sin 45° + cos 60° cos 45° - sin 60° sin 45°
Jj 1 1 1 1 1 Jj 1
-x-+-x-+-x---x-
2 .J2 2 .J2 2.J2 2 .J2
2 1
2.J2 = .J2 = cos 45° = L.H.S.

Example 22: Show that tan 50° = tan 40° + 2 tan 10°.
Solution: Since, 50° = 40° + 10°.
Therefore, tan 50° = tan (40° + 10°)
!lID 50° = tan 40° + tan 10°
1 1 - tan 40° tan 10°
tan 50° - tan 50° tan 40° tan 10° = tan 40° + tan 10°
tan 50° - tan (90° - 40°) tan 40° tan 10° = tan 40° + tan 10°
1
tan 50° - - - - tan 40° tan 10° = tan 40° + tan 10°
tan 40°
tan 50° - tan 10° = tan 40° + tan 10°
tan 50° = tan 40° + 2 tan 10°.
Trigonometry 289

tanS - tancjJ 13
Example 23: Using tan (S-cjJ) = 1 S cjJ ,evaluate tan ~
+ tan tan 12

Solution: Consider
131t
tan -
12
tan(1t+~)
12
= tan ~ = tan 15°
12
1__
1
tan 45° - tan 30° J3
tan (45° - 30°) = = ---
I + tan 45° tan 30° I I
+'J3
J3-1 J3-1·J3-1
--=--x--
J3+1 J3+1 J3-1
3 + 1- 2J3 = 4 - 2J3 = 2 _ J3
3 -1 2 .
sin (A - B) sin(B - C) sin(C - A)
Example 24: Show that + + = O.
sinA sin B sinB sinC sinC sinA
Solution: Consider
sin (A - B) sin(B - C) sin(C - A)
L.H.S. =. . B + sm
. B sm
. C+ . C .
sm A sm sm sm A
sin A cos B - cos A sin B sin B cos C - cos B sin C sin C cos A - cos C sin A)
= + +---------
sin A sin B sin B sin C sin C sin A
sin A cos B cos A'sin B sin B cos C cos B sin C~sinC cos A cos C sin A
= - + - .f' -----
sin A sin B sin A sin B sin B sin C sin B sinC sin C sin A sin C sin A
= cot B-cotA + cot C -cotB + cot A -cot C= 0 = R.H.S.
Example 25: IfA + B = Jr!4, prove that (1 + tan A) . (1 + tan B) = 2.
[UPTU B. Pharma 20051
Solution: Given A +B = 1ti4
tan (A +B) = tan (1t/4) = 1
tan A + tan B
l-tanAtanB
tanA+tanB = l-tanAtanB
=> tan A +tanB+tanA tanB =
Adding (1) to both sides
1 +tanA+tanB+tanAtanB = 2
(1 +tanA)(1 + tan B) = 2
cos 8A cos 5A - cos 12A cos 9A
Example 26: Prove that . 8A
Sin cos
5A
+ cos 12A'. Sin
9'A = tan 4A
[Meerut B. Sc. Biotech 20031
290 Remedial Mathematics

Solution: Multiplying Numerator and Denominator by 2, we get


2cos SA cos 5A - 2cos 12A cos 9A
L.H.S. =
2sin S A cos 5A + 2cos 12A. sin 9A
cos (SA + 5A) + cos (SA - 5A) -[cos (12A + 9A) + cos (12 A - 9A)]
[sin(SA + 5A) +sin (SA - 5A)] + [sin (12A +9A) -sin (12A -9A)]
(cos 13A + cos 3 A) - (cos 21 A + cos 3 A)
(sin 13A + sin 3A) - (sin 21A - sin 3A)
cos 13A - cos 21A
sin 13A + sin 21A

. (13A + 21A) .sm


2 sm . (21A -13A)
2 2
. (13A + 21A) . cos (21A -13A)
2 sm
2 2
sin 17Asin 4A
. = tan 4A = R.H.S.
sm 17A cos 4A

~~~~~~~I EXERCISE 6.31~~~~~~~


1. Given angle C of a triangle ABC to be obtuse, find all angles when

.
sm (A + B) = 2J3 and c'os (A - B) =
_1
J2
2. Show that
cos (90° + 8) sec (-8) tan (1S0° - 8)
(i) =-1
sec (360° - 8) sin (1S00 + 8) cot (90° - 9) .
sin (IS0° - 8) cos(360° - 8) cot (90° - 8)
(ii) sin 8.
tan (90° + 8) . tan(lSO° + 8) sin (- 8)
(iii) sin 75°-sin 15°=cos 105°+cos 15°.
(iv) sin 105° + cos 105° = cos 45°.
(v) sin (45° -A) cos (45° - B) -cos (45° -A) = sin (A + B).
(vi) sin (n + l)A sin (n+ 2)A + cos (n+ l)A cos (n +2)A = cosA.

(vii) cos (45° + 8) cos (10° + 8) + sin (40° + 8) sin (10° + 8) = J3.
2

(viii) cos 8 - sin 8 = J2 cos (8 + ~).


3. Show that cos ISO + sin ISO = cot 270
cos ISO -sin ISO
Trigonometry 291

4. Show that

(i) .tan( ~ + 8 )tan (~- 8) = 1. (ii) tan 38 tan 28 tan 8=tan 38-tan 28-tan 8

(iii) cos 28 cos 2$ + sin2 (8 - $) - sin2 (8 + $) = cos (28 + 2$).


S. If A,B, C,D are the angles of a cyclic quadrilateral in order, show that
cos A + cos B + cos C + cos D = O.
15 12
6. Ifsin a = 17 and cos p = 13 ,find the values of sin (a + P), cos (a - P) and tan
(a + P).

7. (i) Find all positive values of x less than 2n which satisfy the equation cos2 x = ...!...
4
(ii) Find all positive values of x less than 2n which satisfy the equation 3 tan2 x = I.
I3n tan8+tan$
8. Evaluate tan - using tan (8 + $) = 8,f,
12 I -tan tan 'I'
9. Show that

(i) cos 70° cos 10° + sin 70° sin 10° = ...!...
2
(ii) cos 130° cos 40° + sin 130° sin 40° = O.

10. If cos A = ...!.., cos B = ~, (A and B, being positive acute angles), show that A - B
7 14
=60°.
11. If8 + $ = 45°, show that (cot 8 - I) (cot $ - I) = 2.
n
12. If8 - $ = - , show that
4
(i) (1 + tan 8) (1 - tan8) = 2
(ii) (1 +tan 8)(1 + tan 8) = 2 tan 8.
13. Show that
. cos (360° - 8) cosec (180° + 8) cot (90° - 8)
= cosec 8.
(1) sec (90° + 8) cos (- 8)
(ii) cos (270° - 8) sec (- 8) tan (180° - 8) + sec (360° + 8) sin (180° + 8) cot (90° - 8) = O.

(iii) sin 420° cos 390° + cos (-660°) sin ( -390°) = ...!...
2
sin 135° - cos 120 0 r;::;-
(iv) sin 1350 + cos 1200 = 3 + v2.
14. Show that
cos 15° - sin 15° I
(i) cot ( %+ 8 ) cot ( %- 8 ) = I (ii)
cos 15° + sin 15° = .fj
...) cos 8 cos ,f, 28- 8 . 28+8
(111 'I' = cos - - - sm - -
2 2
(iv) tan 2 8 - tan 8 = tan 8 sec 2 8.
292 Remedial Mathematics

nsin A cos A
15. If tan B = 2' show that (A - B) = (1 - n) tan A.
1- n sin A

I ANSWERS I

1. A = 52"!'o B= 7..!.o C=120 0 • 6. 220 171 220.


2' 2' 221'221'221
. 1t 21t 41t 51t .. 1t 51t 71t II1t r;:;-
7. (1) 3'3'3'3' (n) 6'6"'6'6' 8.2-",3.

• TRIGONOMETRICAL RATIO OF COMPOUND ANGLES


(TO BE USED DIRECTLY
1. (a) sin (A + B) = sin A cos B + cos A sin B
(b) sin (A - B) = sin A cos B - cos A sin B
(c) cos (A + B) = cos A cos B - sin A sin B
(d) cos (A - B) = cos A cos B + sin A sin B
tan A+tan B
(e) tan (A + B) = 1 _ tan A tan B

tanA-tanB
(f) tan (A - B) = 1 A B
+ tan tan
cot A cot B -1
(g) cot (A + B) = A
cot + cot B
cotAcotB+l
(h) cot (A - B) = cot A _ cot B .

2. (a) sin (A + B) sin (A - B) = sin2 A - sin2 B = cos2 B - cos 2 A


(b) cos (A + B) cos (A - B) = cos2 A - sin2 B = cos 2 B - sin2 A.
tan A + tan B + tan C - tan A tan B tan C
3. tan (A + B + C) = .
1 - (tan A tan B + tan B tan C + tan C tan A

/4. (a) sin 2 A = 2 sin A cos A = 2 tan A


1 + tan 2 A
. 2
(b) cos 2A = cos 2 A - sin2 A = 2 cos 2 A-I = 1 _ 2 sin2 A = l - tan A
1 + tan 2 A
2 tan A
(c) tan 2A = 2'
I-tan A
5. (a) sin 3A = 3 sin A - 4 sin3A
(b) cos 3A =.4 cos3 A - 3 cos A
3
(c) tan 3A = 3 tan A - tan A
1-3tan3 A
Trigonometry 293

6. (a) 2 sin A cos B = sin (A + B) + sin (A - B)


(b) 2 cos A sin B = sin (A + B) - sin (A - B)
(c) 2cosAcosB=cos(A +B)+cos(A-B)
(d) 2 sin A sinB=cos(A-B)-,cos(A + B).
. .), . D . C+D C-D
a smc+sm
7. () =2sm - - c o s - -
2 2
. C -cos D = 2 cos -
(b) sm C+D.
-smC-D
--
2 2
. C+D C-D
(c) cosC+cosD=2sm--cos--
2 2
. C +D . D - C
(d) cos C - cos D = 2 sm - - s m - - .
2 2
8. (a) sin (A + B + C) = sin A cos B cos C + cos A sin B cos C
+ cos A cos B sin C - sin A sin B sin B sin C
(b) cos (A + B + C) = cos A cos B cos C - cos A sin B sin C
- sin A cos B sin C - sin A sin B cos C
tan A + tan B + tanC - tan A tan B tanC
(c) tan (A + B + C) = -l---tan-A-ta-n-B---t-an-B-t-an-C---t-an-C-t-an-A

_ TRIGONOMETRICAL RATIO FORHALF ANGLES (Obtained

by Replacing A by A in the above Formulae)


2

' A=2 .
l . sm A A
sm-cos-.
2 2
A
2. cos A = cos2 -A - sm
. 2 -A = 2 cos22- - 1 = 1 - 2' A.
sm2 -
2 2 2
. A l-COSA
3. sm"2 =± 2

A =± l+COSA
4. cos-
2 2
A
2tan-
2
5. tan A = 2 A'
I-tan -
.2
A
2tan-
6. sin A = 2 A' 7. cos A = A
1 + tan 2 - 1 + tan 2 -
2 2
294 Remedial Mathematics

~~~~~~~I SOLVED EXAMPLES ~I~~~~~~


Example 1: Show that cos 2A + cos 2 B-2 cos A cos B cos (A + B) = sin2 (A + B)
Solution: cos 2 A + cos 2 B-2 cos A cos B cos (A + B)
Consider L.H.S. = cos2 A + cos 2 B - [cos (A + B) + cos (A - B)] cos (A + B)
= cos 2 A -cos 2 B - [cos2 (A + B) + cos (A + B) cos (A - B)]
cos2 A + cos2 B - cos2 (A + B) - cos2 A + sin 2 B
(cos 2 B + sin 2 B) - cos2 (A + B)
1 - cos 2 (A + B)
sin2 (A + B) = R.H.S.
Example 2: Show that
(i) sinA+sinB _tan(A+B)
[UPTU B. Pharma 2001J
cos A +cos B 2
sinA+sin3A 2A
OJ) = tan . [RGPVB. Pharma 2001 J
cos A +cos3A
sin A + sinB
Solution: (i) Consider L.H.S. =
cos A + cos B
. A+B A-B . A+B
2sm--cos-- sm--
2 2 2
A+B A-B A+B
2 cos - - cos-- cos--
2 A 2
A+B
tan - - = R.H.S.
2
sin A + sin 3 A sin 3 A + sin A
(ii) Consider R.H.S. =
cos A + cos 3 A cos 3 A + cos A
. 3A +A 3A- A
2 sm - - cos - - - s1·n 2 A
____~2~____~2__ =----
3A+A 3A-A cos2A
2 cos - - cos - - -
2 2
= tan2A =R.H.S.
Example 3: Show that

0) (cos A + cos Bi + (sin A + sin Bl = 4 cos2 A - B.


2

.(ii) (cos A - c~s Bl + (sin A - sin Bl = 4 sin2 A - B.


2
Solution: (i) Consider L.H.S. = (cos A + cos B i + (sin A + sin Bi

A+B A_B)2 ( . A+B A_B)2


( 2 cos -2-cOS - 2 - + 2 sm -2-cos-2-
Trigonometry 295

A+B 2 A-B .2A+B 2 A-B


4 cos 2- - cos - - x sm - - cos - -
2 2 2 2

A +-
4 cos 2 - A +-
B [ cos 2 - B + sm A +-
. 2- B]
2 2 2

2A+B 2A-B
= 4 cos - - [1] = 4 cos - - = R.H.S.
2 2
(ii) Consider L.H.S. = (cos A - cos B)2 + (sin A - sin Bi

. A+B A - B)2 ( A + B . A _ B)2


( - 2 sm - 2 - cos - 2 - + 2 cos-2- sm - 2 -

.2 A+B .2 A-B 2 A+B .2 A-B


4sm - - s m --+cos - - s m - -
2 2 2 2
. 2 A- B[
4 sm A+B
- - cos 2 --+sm A +-
. 2- B]
222

4 sm A+
. 2- -B. 1 = 4 sm
. 2-
A --B = RH S
...
2 2

Example 4: Show that sin A + (sinA + 2;rr) + sin ( A + 431t ) = 0

Solution: Consider L.H.S. = sin A + sin ( A + 2; ) + sin ( A + ~1t)

sin + [sin ( A + ~1t) + sin ( A + 231t )]

sin A + [ 2 sin
A + 41t + A + 21t
3 2 3 cos
A + 41t - A + 21t
3 2 3
1
sin A + [ 2 sin (A + B) cos ~] = sin A + [2 (- sin A) x ~]
sinA-sinA =O=R.H.S.

Example 5: Show that tan (~+ 8) -(tan ~ - 8) = 2 tan 28.

Solution: Consider L.H.S. = tan (% + 8) -(tan % - 8)


= sin(% + 8)
cos(% + 8) cos(%- 8)
296 Remedial Mathemntics

2 sin 28 2 sin 28 = 2 sin 28 = =


' 28 2 tan 28 R.H.S.
O+ cos 28 cos
cos 2: + cos 28
2
1
Example 6: Show that sin]O° sin 30 0 sin 50 0 sin 70 0 = 16' (UPTU B. Pharma 2003)

Solution: Consider L.H.S. = sin 100 sin 300 sin 500 sin 700

..!.. sin 10 0 sin 50 0 sin 70 0


2

..!.. sin 10 0 [ 2 sin 50 0 sin 70 0 ]


4

..!.. sin 10 0 [cos (70 0 - 500 ) - cos (70 0 + 50 0 )]


4

± sin 10 0 [~os 20 0
- cos 1200 ]

±sin 10 0
[cos 20 +
0
~J [ .: cos 1200
1
= - - ]
2
1 1
"8 sin 10 0
[2 cos 20 0 + 1] = "8 [2 cos 20 0
sin 100 + sin 100 ]

.!.8 [sin (20 0


+ 100 ) - sin (20 0 - 10 0 ) + sin 10 0 ]

i [sin30 0 -sin 10 0 +sin 10 0 ]

1 1 1 1
"8 sin30 0
= "8 x "2=16 =R.H.S.
sin 88 cos 8 - cos 38 sin 68
E xample:
7 Sh ow t h at = tan 28.
cos 28 cos 8 - sin 38 sin 48
Solution: Consider
sin 88 cos 8 - cos 38 sin 68
L.H.S. =
cos 28 cos 8 - sin 38 sin 48
2 sin 88 cos 8 - 2 sin 68 cos 38
2 cos 28 cos 8 - 2 sin 48 sin 38
[sin (88 + 8) + sin (88 - 8) - [sin (68 + 38) + sin (68 - 38)]
[cos (28 + 8) + cos (28 - 8)] - [cos (48) - 38) - cos (48 + 38)]
Trigonometry 297

sin 99 + sin 79 - sin 99 - 39


cos 39 + cos 39 - cos 9 + cos 79
79 + 39 . 79 - 39
2 cos SIn---
sin 79 - sin 39 2 2
cos 79 + cos 39 79 + 39 79 - 39
2 cos cos---
2 2
2 cos 59 sin 29 sin 29
= -29 = tan 29 = R.H.S.
2 cos 59 cos 29 cos
l+m
Example 8: Ifcos (9 + 2a) = m cos 9, show that cot 9 = - - tan (9 + a).
I-m
Solution: Given that cos(9 + 2a) = m cos 9
cos (9 + 2a) m
cos 9
Using componendo and dividendo, we get
cos (9 + 2a) +cos9 l+m
cos9 - cos (9 + 2a) I-m
9+2a +9 9 +2a-9
2 cos cos 1+ m
2 2 - -
=> 9 + 9 + 2a 9+2a-9 I-m
2 sin sin
2 2
cos (9 + a) cos~ 1+ m
=> --
sin (9 + a) sin a I-m
cot a 1+ m
tan (9 + a) I-m
l+m
Hence cos a = 1 _ m tan (9 + a).

Example 9: Show that cos 79 + cos 59 + cos 39 + cos 9 = 4 cos 9 cos 29 cos 49.
Solution: Consider
L.H.S. = cos 79 + cos S9 + cos 39 + cos 9 = (cos 79 + cos 9) + (cos 59 + cos 39)
79 + 9 79 - 9 2 59 + 39 59 - 39
2 cos - - - cos - - - + cos cos - - -
2 2 2 2
2 cos 49 cos 39 + 2 cos 49 cos 9 = 2 cos 49 (cos 39 + cos 9)
39+9
2 cos 49 ( 2 cos - 39-9)
2 - cos - 2- = 2 cos 49 (2 cos 29 cos 9)

4 cos 9 cos 29 cos 49 = R.H.S.

Example 10: Prove that cos 20° cos 40° cos 80° = "81 [UPTU B. Pharma 2005]
298 Remedial Mathematics

Solution: We have
L.H.S. = (cos 40° cos 20°) cos 80°
1 I
- [2 cos 40° cos 20°] cos 80° = - (cos 60° + cos 20°) cos 80°
2 2
~ [~ + cos 20 0
] cos 80° = ~ cos 80° + ~ (2 cos 80 0 cos 20°)
2 2 4 4
1 1
- cos 80 0 + - (cos 100 0 + cos 60 0 )
4 4
1 1 1
- cos 80 0 - - cos 80° + - cos 60°
444
1 1 1
"4 . "2 = "8 = R.H.S.
1t 1t
Example 11: Calculate sin 100 and cos "5 [UPTU B. Pharma 20031

Solution: We have
31t
cos- . (1t 31t)
21t
SID 2-10 = SIDW
.
IO
31t . 21t
Now, cos- SID-
IO 10
1t 1t 1t 1t
=> 4 cos 3 - - 3 cos - 2 sin - cos -
10 10 10 10
[.: cos 3 A = 4 cos3 A - 3 cos A and sin 2A = 2 sin A cos A].

(cos 1~) [ 4 cos 2 .2:. - 3 - 2sin.2:.] = 0


10 10
1t
2- 1t
4 cos 10 - 3 - 2 sin 10 0

4 (I - sin 2 .2:.)
10
- 3 - 2 sin .2:.
10
0

.21t .1t
4 SID - + 2 SID - - 1 0
10 10
1t -2±J4+16
sin-
10 8

[solving the quadratic equation in sin I~]

-1±J5
4
. 0 1t 1t h ti . 1t. ..
sIDce < 10 < 2 ,t ere ore SID 10 IS posItive.
Trigonometry 299

Hence
, 1t
sm - sin 18°=
-1+15 15-1
= --,
10 4 4
1t , 2 1t
. Now cos -
5
1-2 sm 10'
1-2, ( 15 -
4
1 .r = I _ 5 +1 ; 215 8 - 6 + 215
8
2(15+1) 15+1
---
8 4

cos-
1t 15 + I
Hence cos36°= - - ,
5 4

Example 12: Prove that 1 cos.8 8 = tan


+ sIn
(450 - ~).
2
[UPTUB, Pharma 2003]

cos8' sin (90 - 8)


Solution: We have
I + sin 8 I + cos (90 - 8)

2 sin (T)cOS (T) :;:


sin( 450-%)
cos (450 - %)

J5 -1
Example 13: Prove that sin2 72° - sin2 60° = - 8 - . [UPTU B. Pharma 2001]

Solution: We have sin 2 72° - sin2 60°


, 2
sm
8)
(1t8+2' '2(1t
-sm 8-2'8)

·1
sm - tsm
' 8 = - 1 sm
'8 ,
4 J2
, '8 = -
1t
sm - sm I,
4 J2
·300 Remedial Mathematics

9 p
Example 14: !ftan - = - . find the value of(P sin B+ q cos ()) [UPTUB.Pharma 2007)
2 q

. . 2tan8/2 I-tan 2 8/2


Solution: We have p sm 8 + q cos 8= p 2 +q 2
l+tan 8/2 l+tan 8/2

3
Example 15: Prove that sin 20° sin 40° sin 60° sin 80° = 16· [ RGPV B. Pharma 2002

Solution: L.H.S. = sin 20° sin 40° sin 60° sin 80°

sin 20° . sin 40° x Ji x sin 80° = Ji x sin ~Oo (sin 40° sin 80°)
2 2

Ji sin 20° (2 sin 40° sin 80°)


4

Ji sin 20 [cos (40 - 80°) - cos (40 + 80)]


4

-
Ji.sm 20° cos 40° + -Ji.sm 20°
4 8

gJi. Ji.
(2 sm 20° cos 40°) + g sm 20°

= 8Ji. . Ji .
[sm 60° + sm (-20)] + g sm 20°

Jj Ji Ji. Ji. 3
= - x - - - sm200+ - sm200= - =R.H.S.
8 2 8 8 16
sinA - sin 5A + sin 9A - sin 13A
Example 16: Pralle that A 5A cot 4 A
cos - cos + cos9A - cos 13A =

[RGPV B. Pharma 20021


sin A - sin 5A + sin 9A - sin 13A
Solution: L.H.S.
cosA - cos 5A + cos9A - cos 13A
. A-5A A+5A 2· 9A-13A 9A+13A
2 sm--- cos - - - + sm cos - - -
2 2 2 2
. A+5A . 5A-A 2. 13A+9A . 9A-13A
2 sm---sm---+ sm sm---
2 2 2 2
Trigollometry 3&1

2 sin (-2A) cos 3A + 2 sin A (-2A) cos IIA


2 sin3A sin2A + 2 sinIIA sin(-2A)
-2 sin 2A cos 3A - 2 sin 2A cos IIA
2 sin 3A sin 2A - 2 sin A sinA sin 2 A
- 2 sin 2A( cos 3A + cos IIA)
- 2sin 2A (- sin 3A + sin IIA)
cos3A + cosIIA
sinIIA - sin3A
2 cos7A x cos (-4A) cos4A
. 4A 7A = -·-4- = cot 4 A = R.H.S.
2 sm x cos sm A
cos38 + 2 cos58 + cos78 cos59
E xamp Ie 17 : Prove th at - -- [RGPV B. Pharma 2002)
cos 8 + 2 cos38 + cos58 sin39
cos38 + 2 cos58 + cos78
Solution: L.H.S. =
cos8 + 2cos38 + cos58
(cos38 + cos 78 )+ 2 cos 58
(cos 8 + cos 58) + 2cos38
2 cos 58 cos (-28) + 2 cos 58
2 cos 38 cos (-28) + 2 cos 38
2 cos 58 (cos 28 + 1)
2 cos 38 (cos 28 + 1)
cos 58
-38 =R.H.S.
cos
Example 18: Prove that 4 sin A x sin (60 0 + A) x sin (60 0 - A) = sin 3 A.
[RGPV B. Pharma 2001)
Solution: L.H.S. = 4 sin A sin (60 0 + A) sin (60 0 -A)
4 sin A (sin2 60 - sin2 A)

= 4 sin A ( ~- 2
sin A )

= 3 sin A -4 sin3 A = sin 3 A = R.H.S.


Example 19: Prove that cos 4x = 1 - 8 sin 2 x. cos2 x. [RGPV B. Pharma 2003)
Solution: L.H.S. = cos 4 x
= cos 2 (2x) = 1 - 2 sin2 2x.
= 1 - 2 [2 sin x cos x f
2
= I - 8 sin2 x cos x = R.H.S.

Example 20: !f0 ~x ~2 ",find sin ~ , cos ~


4
when tan x = -3 , x lies in second quadrant
2 2,
[RGPV B. Pharma 20001)
302 Remedial Mathematics

Solution: As tan x = 3-4 the value of cos x is negative in second quadrant


3
cos x =
5

Now
. x
sm -
2
t~osx ~ ~l:% 4
5

2
J5
3

and
x
cos -
2
l+~osx ~ ~l~% ~ Js.
Example 21: Prove that sec a + tan a = tan (~+ ~). IUPTU B. Pharma 2003, 2006]

1 sin a 1 + sin a
Solution: L.H.S. = - - + - - = - - -
cos a cos a cos a

( cOs2~+sin2
2
a)+2sin~.cos~
2 2
2 a . 2 a
cos --sm -
2 2

(cos 2"a.
+ sm a )( cos 2"
a- sm. 2"a)
a . a
cos-+sm-
2 2 [Divide Nr and Dr by cos a/2]
a . a
cos--sm -
2 2
a. a]
cos "2+:m"2
[ cos -
2
a. a]
cos "2-:m"2
[ cos -
2
1 + tan ~ tan ~ + tan ~
_ _",,-2 = _ _4"---_--'2"-
[.,' tan 1t/4 = 1]
I - tan ~ I - tan ~ tan ~
2 4 2

tan (%+%) =R.H.S.


Trigonometry 303

Example 12: Prove that tan (~_ A) = J - sinA . [UPTUB. Pharma 2003, 20061
4 2 J + sinA
2
iccos AI2 +sin 2 A12) - 2 sin AI2 .cosAl2
SolUtion: RH.S. = \/ 2 2
V(cos AI2 + sin A12) + 2 sin AI2 cosAI2

(cos A 12 - sin A12)2 cos AI2 - sin A 12


(cos A 12 + sin A I 2)2 cos A I 2 + sin A I 2
i-tanAl2 tan1t/4-tanAI2
[Dividing Nr arid St by cos All}
1+tanAl2 tanAl4-tanAl2
= tan(1t/4-AI2)=L.H.S.

tan 3 A cot 3 A
Example 23: J + tan 2 A + J + cot 3 A = sec A. cosec A - sin 2A. [Meerut B. Sc. Biotech 2006)

3
8m. A 1 cos 3 A 1
Solution: L.H.S. = ~.--+--.
2 3
'Cos A sec A sin A cos ec 2A
sin 3 A cos 3 A sin 4 A + cos 4 A
- - + - - =- - - - - -
cos A sin A sin A cos A
sin4 A + cos 4 A + 2 sin 2 A cos 2 A - 2 sec 2 A cos 2 A
sin A cosA
(cos 2 A +sin 2 A)2 2sin2 Acos 2 A
sin A cosA sin A cosA

. A' - 2 sin A cos A


sm cosA
= sec A 'COsec A - sin 2A = R.H. S

Example 24: Ifsj.1f lif <!!!. !... find the value oftan A (0 SA !S: 1t/4) [UPTU B. Pharma 1006)
5
4
Solution: We have sin2A '"
5
2 tan A 4
1 + tan A 2 = 5
=> 4 + 4 tan2A = 10 tan A
=> 2
2 tan A - 5 tan A + 2 = 0
=> (tanA-2)(2tanA-l) = 0
=> tan A = 2 or 112.
But 0 SA S 1t/4, so rejecting value 2. Hence, tan A = 112
3Q4 Remedial Mathematics

1. Show that
(i) tan (45° + 8) + tan (45° -8) = 2 sec 28
(ii) sin 2 A + sin 2 (A - B) - 2 sin A cos B sin (A - B) = sin 2 B
sin 5A - sin 3A
(iii) = tan A
cos 3A + cos 5A
cos 4A + cos 3A + cos 2A
(iv) . 4 . 3A . 2A = cot 3A.
sm A+sm +sm
2. Show that
(i) (sin 3A + sinA) sin A + (cos 3A - cosA) cos A = 0
OO) 2 cos-cos-+cos-+cos-
1t 91t 31t 51t =
0
C11
13 13 13 13
(iii) (cos A - cos B)2 + (sin A - sin Bi = 4 sin2 A - B
2
. A 3A 9A . . 5A
(IV) cos 2 A . cos - - cos - = sm 5 A sm - .
2 2 2
3. Show that
(i) cos 20° + cos 100° + cos 140° = 0
(n) cos 20° cos 40° cos 60° cos 80° = 1/16

(in) cos 10° cos 50° cos 60° cos 70° = Jj


16
(iv) cos 52° + cos 68° + cos 172° = O.
sin A + sin 3A + sin 5A + sin 7 A 4A
4. Show that = tan .
cos A + cos 3A + cos 5A + cos 7 A
5. Show that
2 cos 2A + 1
(i) 2 cos 2A -1 = tan (60° + A) tan (60° - A)

(ii) cos A + cos (A + 221t) + cos( A _ 2;) =0.

6. Show that cot 48 (sin 58 + sin 38) + cot 8 (sin 58 - sin 38).
sin 8 + sin 8 + sin 48 + sin 58
7. Show that = tan 38.
cos8 + cos 28 + cos48 + cos 58
8. Show that
4 cos A cos B cos C
= cos (A + B + C) + cos (B + C - A), + cos (C + A - B) + cos (A + B -C).
9. If S9S (A +~B) sin ( C - D) = cos (A - B) sin (C + D), than show that tan A tan B tan
C+tanD=O
Trigonometry 305

10. Ifsinx + siny = a and cos x + cosy= 6


Show that

. x+y a
(I) tan - - = -
2 b

ADDITIONAL SOLVED EXAMPLES

(Based on '2A' and '3A' Formulae)

Example 1: Show that sin J8° =


J5 -I
-- [UPTU B. Pharma 2001)
4
Solution: Let 8 = 18°
~ 58 = 90°
~ 28+38 = 90°
~ 28 = 90°-38
~ sin 28 = sin (90° - 38) = cos 38
~ 2 sin 8 cos 8 = 4 cos 3 8 - 3 cos 8
~ 2 sin 8 = 4 cos2 8 - 3
:::> 2 sin 8 = 4 (I -sin2 8)-3
~ 4 sin2 8 + 2 sin 8 - I = 0
. -2±)4+I6 -2±2J5 -:±J5
sm8 = =---
8 8 4
Since, 18° lies in the first quadrant, therefore sin 18° is positive.
-I-J5
Rejecting the value < 0, we get
4

sin 18° =
J5 -1
4
Remark

)10+25 J5 +1 ~IO-2J5
• Similarly we canfind cos 18° = , cos 36° = - - ,sin 36° = .
4 4 4
Example 2: Show that
J + sin 28 - cos 28
------ = tan 8.
J + sin 28 + cos 29
306 Remedial Mathematics

(1- cos 29) + sin 29


Solution: Consider L.H.S. = ( )
1+ cos 29 + sin 29
2 sin 2 9 + 2 sin 9 cos 9
2 cos 2 9 + 2 sin 9 cos 9
2 sin 9 (sin 9 + cos 9) sin 9 .
-----:,..----:...= - - = tan 9 = R.H.S.
2 cos 9 ( cos 9 + sin 9) cos 9
Example 3: Show that
sin2A
(I) - - - - = tan A
1 + cos2A
(ii) cosec 2A + cot 2A = cot A

(iii) sin 3A + sin 2A - sin A = 4 sin A cos A cos 3A.


2 2
sin 2A 2 sin A cos A cos A A R S
Solution: (i) L.H.S. = 1 + cos2A = 2 cos 2 A = sin A =tan = .H ..
1 cos 2A
(ii) Consider L.H.S. = cosec 2 A + cot 2 A = - - + - - -
SID 2A sin 2A
1 + cos 2A 2 cos 2 A cos A
= = - - = cot A = R.H.S.
sin 2A 2 sin A cosA sin A
(iii) Consider L.H.S. = sin 3A + sin 2A - sin A
= ( sm . 2A) -sm
. 3A +sm . A = 2' SA A . A A
sm-cos--2sm-cos-
2 2 2 2

= 2 cos A [. 5A . A]
smT-sm2
2

2cos A2 ¥+ 1sin2
[2 cos 2 ¥-1]
A
2 cos-
2
[2 sm-sm
.2
3A . A] = 4smAcos-cos-
. A 3A =RHS
2 2 ...

Example 4: lftan 9 = !!....,jind the value ofa cos 29 + b sin 29.


a
Solution: Consider a cos 29 + b sin 29
2
1-- 2-

C+tan e) (l+tan e)- 1+ ~


2

1+ :: 1
=a 1 - tan 9 + b 2 tan 9 _ a ---...rl. + b _ _
a_
2 2 b ] [ b
[
Trigonometry 307

Example 5: Find the value a/tan 2r30 ~


Solution: We know that
A 1- cos A
tan-
2 1 + cosA
Put A =45°, we get

tan 22.!.0
2
1 - cos 45° _ H _ .fi - 1
1 + cos 45° - 1 + _1_ - -./2 + 1 .
.fi
Example 6: Show that
4 tan 8 (1 - tan 28)
tan 4 B = ---'------'-
J - 6 tan2 8 + tan4 8
Solution: L.H.S. = tan 48° = tan C2 x 28)
2x 2 tan 8
2 tan 28 1- tan 2 8
1- tan 2 28
1-( 2tan8)2
1- tan 2 8
(1- tan 2 8)2 - 4 tan 2 8

(1- tan 2 8t
4tan8 (1- tan 2 8)
2 4 =R.H.S.
1 - 6 tan 8 + tan 8

1. Show that
tan 58 + tan 38 sin 8 + sin 28
Ci) 58 8 = 4 cos 28 cos 48 (ii) = tan 8.
tan - tan 3 1 + cos 8 + cos 28
2. Show that
Ci) sin 4A = 4 sin A cos 3 A - 4 cos A sin 3 A
Cii) cos 4A = 1 - 8 sin 2 A cos 2 A
CiiI) cos 5A = 16 cos5 A - 20 cos3A + 5 cos A
(iv) cos 6A = 32 cos 6 A -48 cos4 A + 18 cos2 A-I.
3. Show that

Ci) sin2 72° _ sin2 600 = J5 - 1 ..) . 7t . 137t I


CII Sln-+Sln-=--
8 10 10 2
...)
C111
. 7t . 27t . 37t . 47t
Sln-Sln-Sln-Sln-=-
5
(iv) 2 cos 8 = ~2 + -./2 + 2 cos 48.
5 5 5 5 16
308 Remedial Mathematics

5. Show thattan 15° + cot 15° = 4.


6. Prove that
cos 8 - sin8
(i) sec 28 - tan 28 = 8 . 8 (ii) cosec 8 - 2 sin 8 = 2 cot 28 cos 8.
cos + sm
7. Iftan2 8 = 2 tan2 ~ + 1, then show that cos 28 + sin 2 ~ = \
1 3 1
8. If x + - = 2 cos 8, show that x + 3" = 2 cos 38.
x x

9. Ifcos8=
cos ~ - e 8
,j..' show that tan -=±
Jfg+
e ~
--.tan-.
1 - e cos,!, 2 1- e 2
sin 213
10. If2 tan a. = 3 tan 13, show that tan (a. -13) = 5 2A
-cos I-'
11. Show that
(i) cosec 2A + cos 2A = cot A

(ii) sec 2A + tan 2A =


cos A + sin A
.
(7t
= tan - + A
J
cosA -smA 4

(iii) I-sinA =tan(.!:.-


1 + sin A 4 2
A).
12. If cos 8 = - 1( + -a1) ,show that 2 cos 48 =
2
a
a
1
a 4 + 4"" .

I ANSWERS I
4. ~4-~+Ji )4+~+Ji .-(Ji+l)+~4+2.J2.
2 2 2 2

• CONDITIONAL IDENTITIES
Type 1. Identities which involve sines and cosines.
Type 2. Identities which involve squares of sines and cosines.
Type 3. Identities which involve tangents and cotangents.

Based on Type 1:
IfA + B + C = 7t,prove that
Example 1: sin 2A + sin 2B + sin 2C = 4 sin A sin B sin C. (Meerut B. Sc. Biotech 2005\
Solution: We have,
+ + +
sin 2A sin 2B sin 2C = 2 sin (A B) cos (A - B) sin 2C +
2 sin C cos (A - B) + sin 2C [given A + B + C:= 7t
C~7t-(A + 'B)
sin C = sin [7t - (A +'-8)]]
Trigonometry 309

2 sin C cos (A - B) + 2 sin C cos C


2 sin C [cos (A - B) + cos C]
2 sin C [cos (A - B) - cos (A + B)
[cos C = cos [1t - (A + B)] = - cos (A + B)] .
2 sin C [2 sinA sin B]
4 sin A sin B sin C.
Example 2: sin (B + C - A) + sin (C + A - B) + sin (A + B - C) = 4 sin A sin B sin C.
Solution: We have
sin (B + C - A) + sin (C + A - B) + sin (A + B - C)
= sin (1t - 2A) + sin (1t - 2B) + sin (1t - 2C)
[since B + C = 1t - A, C + A = 1t - B, A + B = 1t - C]
= sin 2A + sin 2B + sin 2 C
= 4 sin A sin B sin C [from Ex. no. 1]
Example 3: cos 2A + cos 2B - cos 2C = 1 - 4 cos A cos B cos C.
Solution: We have,
cos 2A + cos 2B - cos 2C = 2 cos (A + B) cos (A - B) - cos 2C
= 2 cos (1t - C) cos (A - B) - 2 cos 2 C + 1
= - 2 cos C cos (A - B) - 2 cos 2 C + 1
1 - 2 cos C [cos (A - B) + cos C]
1 - 2 cos C [cos (A - B) + cos {1t - (A + B)} ]
1-2cosC[cos(A-B)-cos(A + B)]
1 - 2 cos C [2 cos A cos B] = 1 - 4 cos A cos B cos C.
. A+ sm
E xamp Ie 4: sm . C = 4 sm-sm-cos-.
. B -sm '. A ,B C
2 2 2
Solution: We have
. A +sm
sm . C
. B -sm =
2' A+B A-B - 2'
sm--cos-- C C
sm-cos-
2 2 2 2

= 2 sm (1t
. - - C) A-B 2'
- cos--- C C
sm-cos-
2 2, 2 2 2
C A-B . C C
= 2 cos-cos---2sm-cos-
2 2 2 ·2,

=
C[ A-B
2 cos2' cos- .
-sm2'
2
CJ.
= C [ cos-
2 cos2' A- B-sm. {1t'2--2-
A+ B}]
2
= 2 cos C [cos! - B _ cos A + BJ =2 cos C [2 sin A sin BJ
2 2 2 2 2 2
. A . B C
= 4 sm-sm-cCls-.
2 2 2
310 Remedial Mathematics

Example 5: cos A + cos B-cos C = 4 cos.icos B sin C -I.


222
Solution: We have
A B
eosA+eosB-cosC = 2 eosA+Beos - _(1_2Sin 2C )
2 2 2

2 eos{~ _ C}eos A - B -1 + 2 sin2 C


2 2 2 2
. C A-B 2' C
= - 2 sm-eos--+ sm 2 - - 1
2 2 2

. {1t
. C [{ sm
= 2 sm
2 2"- (- 2
+- A-2-B1
A B)}} -eos- -1

A +--eos-
. C [ cos-
= 2 sm B . A -- B] -I
2 2 2

= 2 sin C [2eos cos


222
A B]_I
A B. C I
= 4 eos-eos-sm-- .
2 2 2
Example 6: sin (B + 2C) + sin (C + 2A) + sin (A + 2B) = 4 sin B - C cos C - A sin A - B.
2 2 2
Solution: We have, sin (B + 2C) + sin (C + 2A) + sin (A + 2B)
sin (B + 2C) = sin [(A + B + C) - (A - C)]
sin [1t - (A - C)] = sin (A - C).
Similarly. sin(C+2A) = sin(B-A)
sin (A + 2B) = sin (C - B).
Therefore, L.H.S. can be written as
sin (A -C) + sin (B-A) + sin (C-B).

'~ = 2 sini (B - C) eosi (2A --B - C)-2 sini (B- C) eosi (B-C)

= 2 sin.!. (B - C) [eos.!.(2A - B - C) - eos.!.(B - C)]


2 2 2

= 2 sin.!. (B - C) [2 sin .!.(.A( - C)sin .!.(B - A)]


222
. B-C. C-A . A-B
=4sm--sm--sm--
222

Example 7: IfA + B + C =
1t
- , prove that
2
cos 2A + cos2 B + cos2 C = 2 + 2 sin A sin B sin C. [UPTU B. Pharma 2004, 2006)
Trigonometry 311

Solution: We know that


cos 2A = 2 cos2 A-I
cos 2B = 2 cos2 B-1

cos
2 A
=
(1 + cos 2A) cos
2 B
=
(1 + cos 2B) .
2' 2

L.H.S. = .!. (1 + cos 2A) + .!. (1 + cos 2 B) + cos2 C


2 2
1
= 1 + - (cos 2A + cos 2B) + cos2 C)

e
2

= 1+ ~ [2 cos (2A ; 2B ).COS


A
; 2B)] + cos 2 C

= 1 + cos (A + B). cos (A - B) + cos2 C

= 1 + cos (~ - C ) cos (A - B) + cos2 C as A + B + C = ~


= 1 + sin C . cos (A - B) + (1 - sin2 C) as cos ( ~ - C ) = sin C

= 2 + sin C [cos (A - B) - sin C]

= 2+sinC [COS(A-B)-Sin( %-(A+B»)]

= 2 + sin C [cos (A - B) - cos (A + B)], as sin ( %- e) = cos e


. C [2 sm
= 2 +sm . (A - B + A + B) sm
. (A + B - A - B )-]
2 . 2
=2 +2 sin A sinB sinC=R.H.S.

cosA cosB cosC 2


Example 8: + + = .
sinB sinC sinC sinA sinA sinB
Solution: Taking
L.H.S. = sin A cos A + sin B cos B + sin C cos C
sin A sin B sin C
2 sin A cos A + 2 sin B cos B + 2 sin C CQS C
2sin A sinB sinC
sin 2A + sin 2B + sin 2C
2 sin A sin B sinC
4 sin A sin B sin C
2 sin A sinB sinC =2=R.H.S.
Example 9: cos 4A + cos 4B + cos 4C = - 1 + 4 cos 2A cos 2B cos 2C.
312 R-emedial Mathematics

Solution: cos 4A + cos 4B + cos 4C = 2 cos (2A + 2B) cos (2A - 2B) + 2 cos2 2C - 1
2 a:s (;at - 2C) cos (2A - 2B) + 2 cos2 2C - 1
2 cos 2C cos (2A - 2B) + 2 cos 2 2C - 1
2 cos2 C [cos (2A -2B) + cos {27t -(2A + 2B)}] - 1
2 cos 2 C [cos (2A - 2B) + cos (2A + 2B)] - 1
= 2 cos 2 C [2 cos 2A cos 2B - 1
= - 1 + 4 cos 2A cos 2B cos 2C.
Based on Type 2

ExamplelO:Showthatsin2i +sin2B +sin2C =I-2sinisinB sinC,


2 2 2 2 2 2
Given that A+B+C = 7t
Solution: We have

sin2 A + sin2 B + sin2 C 1 - cos2A '2 -B + sm


- + sm '2 -C
222 222
A +-Bc o sA--- B] +sm
1- [ cos- ,2 [7t A + B]
2'--2-
2 2
:, (cos 2 A - sin2 B = cos(A + B) cos (A -B»

= A+B)
1 - cos ( - 2 - cos (A-B)
- 2 - - cos 2(A+B)
-2-

= A2
1- cos ( - + B)
- [ cos ( -
A2- B)
- - cos ( -
A2+ B)]
-

= 1-cos(-A2+ B)- [2 sm'2


. Asm, '2B]
= 1 - cos
_2
[~- C]
2
[2 sin Asin B]
2 2
,A,B,C
I - 2 sm-sm-sm-,
=
222
Example ll: IfA + B + C = p, Prove that
A 2B 2 C 22,A,B,C
cos 2 - + cos - + cos - = + sm - sm - sm -
2 2 2 2 2 2
Solution: We have,
2A 2B 2C . 2A 2A 2C
cos -+cos -+cos - = I-sm -+cos -+cos -
2 2 2 2 2 2
.2b ,2 A 2 C
= 1+sm --sm -+cos -
2 2 2
= 1 + [cos 2 A + B cos B - A] + cos2 C
2 2 2
Trigonometry 313

.C[cos (A-B)
2 +sm .C]
-2- -sm
2 2
2+sm A- B) .
. C [ cos ( -2- -sm {1t
"2- (- 2
+- A B)}]
2
2+sin ~ [COS( A;B)-COs( A;B)]
A-B) . (A B)]
+
. C [ cos ( -2- -sm -2-
2 +sm
2
. C [2 sm-sm-
2 +sm- . A . B] = 2 + 2 sm-sm-sm-.
. A . B . C
2 2 2 2 2 2
Based on Type 3
Example 12: If A + B + C = 1t,prove that tan A + tan B + tan C = tan A tan B tan C
[UPTU B. Pharma 2004, 071
Solution: Give that
A+B+C =1t => A+B=1t-C
tan (A + B) = tan I1-C
tanA+tanB
- - - - - = - tan C
I-tanA tanB
=> tan A + tan B =-tan C+ tan A tan B tan C
=> tan A + tan B + tan C= tan A tan B tan C.
Example 13: IfA + B + C = 1t, show that
cot B + cot C cot C + cot A cot A + cot B
+ + =]
tan B + tan C tan C + tan A tan A + tan B .
Solution: We have
cotB+cotC cotC +cotA cot A +cotB
-----+ +-----
tanB+tanC tanC+tanA tanA+tanB
] 1 1 1
--+-- --+-- --+--
~B ~C+~C ~A+~A ~B
tanB+tanC tanC+tanA tanA+tanB
1 1 1
-----+ +-----
tan B tan C tan C tan A tan A tan B
= cot B cot C + cot A cot C + cot A cot B. ...(1)
314 Remedial Mathematics

Again, A +B+C =n ~ A +B = n - C ~ cot (A + B) = cot (n - C)


cot A cosB-l
----- = - cot C
cot A cotB
~ cot A cot B-1 = - cot C cot A - cot B cot C
~ cot A cot B + cot B cot C cot C + cot A = 1.
cot A + cot B cot B + cot C cot C + cot A
H ence, + +-----
tan A + tan B tan B + tan C tan C + tan A
= cot A cot B cot C + cot C cot A = 1.
Example l4:.if A + B + C =n,
tanA tanB tanC tanB tanC tanA
--+--+--+--+--+--
tanB lanC lanA lanA lanB lanC
=~A~B~C+~B~C~A+~C~C~B

Solution: We have,
tanA + tanc)+(tanB + tanA)+(tanC + tan B)
( tanB tanB tanC tanC tanA tan A
sin A sinC sinB sin A sinC sinB
--+-- --+-- --+--
~A ~C+~B ~A+~C ~B
sinB sinC sin A
cosB cosC cos A
sin (A + C) cos B sin (A + B) cosC sin (B + C) cos A
-~-~--+ +-~-~--
cosAcosCsinB cosAcosBsinC cosBcosCsinA
sin (n - B) cos B sin (n - C) cosC sin (n - A) cos A
-~-~--+ +-~-~--
cos A cosC sinB cosA cosB sinC cosB cos C sin A
sin B cos B sin C cos C sin A cos A
------+ +------
cos A cosC sin B cos A cos B sin C cos B cos C sin A
= cos B sec A sec C + cos C sec A sec B + cos A sec B sec C.

If A + B + C = n, prove that

·A + SID
l • SID · BSID· C=4 cos-cos-cos-
ABC
222
2. cos 2A + cos 2B + cos 2C = - 1 - 4 cos A cos B cos C.

3. cos A + cos B + cos C = 1 + 4 sin A sin B sin C .


2 2 2
sin2A +sin2B+sinC . A . B . C
4. - - - - - - - - = 8 SID-SID-SID-.
sin A + sin B + sin C 2 2 2
Trigonometry 315

. 3A + SIn
5. SIn . 3C = 4 cos-cos-cos-.
. 3B + SIn 3A 3B 3C
222
ABC 1t-A 1t-B 1t-C
6. cos- + cos- + cos- =4 cos--cos--cos--.
22222 2
. "2
7. Sin A + SIn
. "2
B + Sin C = 1 + 4 SIn
. "2 . (1t-A) . (1t-B)
-2- Slll . (1t-C)
-2- SIn -2- .

.2 A ·2 B .2 C 1 2 A B. C
8. Sin -+SIn --SIn - = - cOS-COS-SIn-.
2 2 2 2 2 2
2A 2B 2C A B. C
9. cos -+cos --cos - =2cos-cos-sm-.
2 2 2 2 2 2
2A .2B .2C . A . B . C
10. cos --SIn --Sill -. =2 Sill-Slll-Sill-.
2 2 2 2 2 2
11. sin2 A + sin2 B - sin2 C = 2 sin A sin B cos C.
12. cot B cot C + cot C cot A + cot A sin cot B =1.
13. tan 2A + tan 2B + tan 2C = tan 2A tan 2B tan 2C.
A B B C C A
14. tan-tan-+tan-tan-+tan-tan-= 1.
2 2 2 2 2 2

OBJECTIVE EVALUATION

MULTIPLE CHOICE QUESTIONS


Choose the most appropriate one:
1. In Correct statement is
(a) sin8=-1I5 (b) cos 8 = 1
(c) sec 8 = 112 (d) tan e = 20.
2. The value of sin2 20° + sin2 70° is
(a) 1 (b) 0
(c) -1 (d) 112

3. If sin e = -( 112) and cos 8 = (.J3 /2») , then e lies in the quad rant
001 ~ll
(c) III (d) N
4. If tan e + cot e = 2, then sin e is
(a) 1 (b) ± J2
(c) J2 (d) ±1I J2
5. Values ofe which satisfy the equation sin e + cos e = 2 sin e cos e, are

( a) 0 21t 31t (b) ~ ~


, 3 ' 2 3' 2

(c) 21t 41t 21t (d) None of these


3' 2'
316 Remedial Mathematics

cos 54° tan 20°


6 . Th evaueo
I f --+--
tan 36° cot 70°
(a) 3 (b) 1
(c) 0 (d) 2
7. The value of sin 4620° is
(a) 112 (b) J3/2
(c) -J3/2 (d) -112
1 1
8. If tan A = - and tan B = -, then (A + B) equal to.
2 3
(a) rrJ6 (b) 0
(c) rrJ4 (d) 7t

9. If sin A = ~ , sin B = ~ then sin (A + B) is


5 13
(a) ± 35 or ± 45 (b) ±35 or± 63
65 65 65 65
(c) ± 63 or± 33 45 33
(d) ±-or±-.
65 65 65 65
10. Which of the following in not true?
(a) sin2 9 = 1 - cos2 9 (b) 1 + tan 2 9 = sec2 9
2 2
(c) cot 9 - cosec 9 = 1 (d) sin 2 9 + cos2 9 = 1
11. If sin 3A = cos (A - 26°) where 3A is an acute angle, then value of A is
(a) A = 29° (b) A = 39°
(c) A = 64° (d) None of these
12. If sin (9 + 36°) = cos 9, (9 + 36°) is an acute angle. Then 9 is equal to
(a) 25° (b) 54°
(c) 27° (d) 29°

5sin9-4cos9
13. If 5 tan 9 - 4 = 0 , then the value of is
5sin9 + 4cps9

(a) 5 5
(b) -
3 6

(c) 0 (d) -
6
14. Value of tan 5° tan 25° tan 30° tan 65° tan 85° is

(a) J3 (b) _1
J3
(c) (d) O.
15. If 8 tan x = 15 then sin x - cos x is equal to
8 17
(a) (b)
17 7
(c) 7
1 (d) -
17 17
Trigonometry 317

16. If tan 8 = -3 , then the value of I-cos8


is
4 1 + cos8
5
(a) - (b) 2
9 5
(c) I (d) I
5 9
2tan30
17. --""'"2- is equal to
IHan 30
(a) sin 60° (b) cos 60°
(c) tan 60° (d) sin 30°
18. If 8 tan x = 15 then sin 2 x - cos2 x equal to
8
(a) (b) 2..
17 17
(c) 30 (d) None of these.
17
- 12 th l+sin8.
19. 1ftan 8 - - en - - - IS
5' l-sin8
5 (b) 25
(a) 12
13
13
(c) (d) 25.
25

20. x =cotA+cosAandy=cotA-cosA,then (;:;J +(x;2f is

(a) 0 (b) - I
(c) I' (d) None of these.

PILL IN THE BLANKS

1. /f2 sin ~ = 1 , then x is equal to


2 -----
2. /f8 is a positive acute angle such that
sec 8 = cosec 60° , then 2 cos28 - I is
-----
3. (1 + tan 0 + sec 8) (1 + cot 8 - cosec 8) is equal to _ _ __

4. In a ~ ABC if L B = 90° and sin A = ~ , tan A is equal to _____


5. /f x = a cos 8 and y = b sin 8, b2x 2 + a21 is _____.
6. /f6 + A = 3. Then cosec A equal to _____.

TRUE I FALSE STATEMENT


2tan8
1. /f8 = 30°, then tan 29 = 2. (TIF)
I-tan 8
2. A,B,C are interior angles of ~ ABC ,then cos B + C = sin.:i.. (TJF)
2 2
318 Remedial Mathematics

3. An equation involving trigonometric ratios of an angle is called a trigonometric identity if it


is true for all values of the angle. (T/F)
4. I - sec2 e = tan 2 e. (T/F)

5. If A.B, C are interior of A ABC, then tan ( C ; A) = tan f. (T/F)


6. The measure of an angle is the amount of rotation from the initial side to the terminal side.
(T/F)
7. tan (A + B) = tan A + tan B. (T/F)
8. sec 5 A = cosec (A - 36°), where 5A is an acute angle then A is 21°. (T/F)

9. cosec .JI- cos 2 e = 1. (T/F)

ANSWERS

MULTIPLE CHOICE QUESTIONS


1. (c) 2. (a) 3. (d) 4. (d)
5. (c) 6. (d) 7. (c) 8. (c)
9. (c) 10. (c) 11. (d) 12 (c)
13. (c) 14. (b) 15. (d) 16. (d)
17. (a) 18. (d) 19. (d) 20. (a).

FILL IN THE BLANK


1. 60° 2. 1/2 3. 2 4. 3/J7
2 2
5. a b 6. JIO.
TRUE/FALSE
1. False 2. True 3. True 4. False
5. True 6. True 7. False 8. True
9. True

REFRESHER

Can We Do? (Frequently Asked Questions)


1. Find the length of an arc ofa circle of 5 em subtending a central angle 15°
[UPTU B. Pharma 2007)
2. Find in degrees the angle subtended at the centre of a circle of radius 10ft by an are
oflength 20 ft. [UPTU B. Pharma 2005]
3. Prove the following:
(l) t::de + co~e + 2 = sec 2e + cosec 2e [UPTU B. Pharma 2001)
(ii) sin 6e + cos 6e = 1 - 3 sin2e cos2e . [UPTU B. Pharma 2007)
...) tane+sece-l 1+sine
( III [UPTU B. Pharma 2004, RGPV B. Pharma 2005]
tan e - sec e + 1 cose
Trigonometry 319

4. Prove that the expression, 2 (sin 68 + cos 6) - 3 (sin48 + cos48) is independent ofthe
angle 8. [UPTU B. Pharma 20051

5. If cos 8 + sin 8 = .J2 cos 8, show that cos 8 - sin 8 = .J2 sin 8
[RGPV B. Pharma 20031
6. Prove that sin (A + B) sin (A - B) = sin2A - sin2B [UPTU B. Pharma 20031
[RGPV B. Pharma 20011
7. Obtain the values of
(I) sin 15° IUPTU B. Pharma 2004)
(il) C~! 75 [UPTU B. Pharma 2002)
(iii' 1 1° rUPTU B. Pharma 20061
cos8°+sin8°
8 . rwvt:th at = cot 37° [UPTU B. Pharma 2002J
cos8°-sin8°
9. Prove that tan 138 - tan 98 - tan4 8 = tan 138 tan 98 tan 48. [UPTU B. Pharma 2005)

10. If sin A = i and cos B = ~


,find the values of sin (A - B) and cos (A + B)
5 13
where <A < n/2 ,and 0 < B<n/2. [UPTU B. Pharma 2001 J
m i n
11. If tan A = - - and tan B= - - ,showthatA + B = -
m +1 2m+1 4
[UPTU B. Pharma 2007)
12. Prove that
. 5n
SID- -
18
4n
cos- =
9
"jr:;3.sm-.n9 [UPTUB. Pharma 2003)

13 . Prove that sinA+sinB = tan (A+B)


-- [UPTU B. Pharma 20011
cosA+cosB 2
n 9n 3n 5n
14. Provethat2 cos-cos-+cos-+cos- =0 rUPTU B. Pharma 20011
13 13 13 13

15. Prove that sin 10° sin 30° sin 50° sin 70° = ..!.- [UPTU B. Pharma 2003)
16

16. Prove that cos 20° sin 40° cos 80° = .!. rUPTU B. Pharma 2005J
8

17. Prove that sin 20° sin 40° sin 60° sin 80° = ~ IUPTU B. Pharma 2004,08)
16
18. Find the values of IRGPV B. Pharma 2002)

(i) cos 22.!. ° IU.P. T.U. B.Pharma 2001)


2

(ii) tan 22.!. ° IUPTU B. Pharma 2002J


2

19. Calculate sin.2:. and cos":: IU.P.T.U. B.Pharma 2003)


10 5
320 Remedial Mathematics

20. Prove that

(i) cosS = tan (45S


--2) [UPTU B. Pharma 2001[
I +sinS

(ii) I-sinS = tan(~-~) [UPTU B. Pharma 2003,06[


l+sinS 4 2

21. Prove that sin2 72° _ sin 2 600 = J5 -1 [UPTU B. Pharma 20011
8

· 2(1t
22 · P rove th at sm - + -S) - '>In
. 2(1t
- - -S) = - I sm
. S [UPTU B. Pharma20031
8 2 2 2 J2
23. Ifsin 2A = ~ , find the values of tan A. ( 0 ~ A ~ ~)
24. Iftan ~= P , find the value of (p sin S + q cos S) [UPTU B. Pharma 2007)
2 q
25. IF A + B + C= 1t, prove
tan A + tan B + tan C = tan A tan B tan C. [UPTU B. Pharma 2004, 07)
26. IF A + B + C = 1t/2 prove that
cos2 A + cos 2 B + cos 2 C = 2 + 2 sin A sin B sin C [UPTU B. Pharma 2004,06)
27. Prove that
cosS sin S
---+ = sin S + cos S. [URGPV B. Pharma 2001)
1- tan S 1- cot S
I-cos 2 A 2
28. Prove that 2 = tan A [RGPV B. Pharma 2002)
I-sin A
I+cosS+sinS l+sinS
29 · P roveth at = --- [RGPV B. Pharma 2004]
I +cosS -sinS cosS

30. Find sec S and tan S if sin S = g is second quadrant [RGPV B. Pharma 20041
13

31. If 0 ~ x ~ 21t find cos x and tan x ,when sin x = -.!.. ,x lies in fourth quadrant
2
[RGPV B. Pharma 2003)
32. Solve sin 75° + cos 75° + sin 15° + cos 15°. [RGPV B. Pharma 200 I)
I,

33. Prove that sin 105° + cos 105° = cos 45°. [RGPV B. Pharma 20031

34. Prove that


sin A + sin 3A
= tan 2A. [~k;pv B. Pharma 2001[
cosA+cos3A
sinA-sin5A+sin9A-sinI3A
35· Prove that = cot 4A [RGPV B. Pharma 2002]
cos A - cos5A - cos9A + cos13A
cos3S+2cos5S+cos7S cos5S
36· P roveth at =-- [RGPV B. Pharma 2002)
cosS + 2cos3S + cos5S cos39
37. Prove that sin A x sin (60° -A) sin (60 + A) = sin 3A [RGPV B. Pharma 2001)
,
Trigonometry 321

38. Prove that cos 4x = 1 - 8 sin2x. cos 2x. [RGPV B. Pharma 2005]

223
_i,
39. IF o::;;x::;; 2'1t ,fmd sin::' and cos::' , when tan x = x lies in lInd quadrant.

(RGPV B. Pharma 2001]


1t 21t 1
40. Prove that cos
S - cos S ="2 (UPTU B. Pharma 2008]

41. Find the value ofsec(-15000) x sin 390° [UPTU B. Pharma 2008]

000
LOGARITHMS

• INTRODUCTION
Sometimes, to simplify the numerical expression involving multiplication, division or rational
powers, we use the logarithms. It is very useful for such type of lengthy and typical
calculations.
Definition: Let there be a number a > 0 and a "* 1. A number x is called the logarithm
of another variable y > 0 to the base a if d = y. [Meerut B. Sc. Biotech 2005, 20061
.. d =y <=> x = logo y. ...(1)
For Ex;ample:
(A) (I) 24 = 16 <=> log2 16 = 4
(i/) 102 = 100 <=> loglO 100 = 2
(iii) 8° = 1 <=> logg 1 = 0
1
(iv) (64)1/6 = 2 <=> log64 2 = -.
6
log2 128 = A real number x such that 2 x = 128 => x = 7
1
log4 2 = A real number x such that 4x = 2 => x = - [.: 4 1/2 = 2]
2 .
Remarks
• Logarithm of a number satisfying the condition (1) is unique. For, if a, ~ are two
distinct loganthms of the number y to a base a, then by definition, we have
aU = y and a~ = y, when aU = aP. ... (2)
But by properties of powers with positive base different from 1, we conclude from
(2) that a = ~. This, if the number y has a logarithm to base a, this logarithm is
unique. We denote it by the definition
x = logo y if d = y.
• 'log' is the abbreviation of "logarithm".
• The logarithm of a number to a given positive real number ("* 1) as base is th!;l index
or the power to which the base must be raised in order to make it equal to the given
number.
Logarithms 323

• PROPERTIES OF LOGARITHMS
Here, we assume a > 0, a :1:- 1 ,m > 0 ,n > 0
1. if = Y then x = loga Y'
Here, L.H.S. is called exponential form, whereas R.H.S. is corresponding logarithmic
form.
2. a l = a, b l = b etc., therefore, logo a = 10gb b = 1.
3. aO = 1, bO = 1 :::) log0 1 ='0 logb 1 = 0 •
1
4.
5. Base Change Formula
10gb a = loge a . 10gb c
loge a
or 10gb a = - -
logc b
6. The log of the product of two numbers is equal to the sum of their logs.

Remark
• IfXI' x2, ... , xn are positive rational numbers then
log (XI' x 2' ... , xn) = logo xI + logo x 2 + ... + logo x n·
7. The log of the ratio of two numbers is equal to the difference of their logs.
8. logo mn = n logo m.
p
9. logoq rf = -logo n.
q
10. a logan = n.
11. If a > 1, then 0 < ex < ~ :::) logo ex < logo ~.
12. If 0 < a < 1, then 0 < ex < ~ :::) logo ex > logo ~
13. If a > 1, ex > 1, then logo ex > O.
14. If 0 < a < 1, 0 < ex < 1, then logo ex > O.
15. If 0 < a < 1, ex> 1, then logo ex < O.
16. If a> 1,0 < ex < 1, then log dJ. < O.
17. J fa> 1, ex > I and ex < a, then 0 < log aU < 1.
18. If a> 1, ex> 1 and ex > a, then logoex> 1.
19. If 0 < a < 1, 0 < ex < 1 and ex > a, then 0 < logoex <
20. If 0 < a < 1, 0 < ex < 1 and ex < a, then loga ex > 1.

• SYSTEM OF LOGARITHMS
(a) Common Logarithm. In this system we take the base 10. This is also known as Bring's
system.
For Example. loglo 10 = I, loglo 100 = loglO IO~ = 2, loglo 1000 = 3.

Remark
• If no base is mentioned, the base is always taken as 10.
324 Remedial Mathematics

(b) Natural Logarithm. In this system, we take the base e, where e" is an irrational number
lying between 2 and 3 and is given by
1 1 1
e =1 + -+-+-+ .....
l! 2! 3!

III STANDARD FORM OF DECIMAL


To calculate the logarithm of any positive number in decimal form, we always express the
given positive number in decimal form ao the product of an integral power of 10 and a
number between 1 and 10
i.e., c = m x 10k
where k an integer and 1 ~ m ~ 10.
For Example:
(i) 1234.56 can be written as 1.23456 x 1000 = 1.23456 x 103
(ii) 0.0023 = (0.0023 x 1000) x 10- 3 = 23 x 10-3

• CHARACTERISTIC AND MANTISSA


The integral part of a logarithm is called the characteristic and the decimal part is
called the mantissa. Logarithm to the base 10 are called common logarithms. The
characteristics of common logarithms can be written by inspection, using the following
rule.
"The characteristic of the logarithm (base 10) of a number greater than 1 is less by
one than the number of digits in the integral part, and is positive. The characteristic of
the logarithm of a positive decimal fraction less than 1, is greater by unity than the number
of consecutive zeroes immediately after the decimal point and is negative."
On the other hand, to find the mantissa, we used the table of logarithms of numbers.
The position of the decimal point in a number is immaterial for finding the mantissa. To
find the mantissa of a number, we consider first four digits from the left most side of the
number. If the number in the decimal form is less than one and has four or more consecutive
zeroes to the right of the decimal point, then mantissa is calculated with the help of
number formed by digits, starting with the first non-zero digits.
Significant Digit. The digit which are used to find the mantissa of a given number
are known as significant di[jits.

~~~~~~I SOLVED EXAMPLES I~~~~~~~


Example 1: Express each of the following in exponential form
(i) log2 64 = 6 (ij) /oglo 0.01 = - 2.
Solution: (i) log264 = 6 ~ 26 = 64.
(i/) (oglO 0.01 = - 2 ~ 10- 2 = 0.01.
Example 2: Find the values of each of the following form :
(i) log9 81 (ii) log 12.4-
Logarithms 325

Solution: (1) Let lo~ 81 = x.


Then x = log9 81 =:> <JX" = 81 =:> <JX" = 92
=:> x = 2.

(ii) Let log h 4 = x.


Then, logh4 = x=:> ( hy = 4
=:> [(2)1/2y = 4 = 22
=:> 2x/2 = 22

=:> ~ = 2 =:> x = 4.
2
Example 3: Rewrite the following equations in the logarithm form :

(i) 42/ 3 = 8 (ii) 50 = J (iii) (2 h) -2/3 = 1


2

Solution: (I) 4 3/2


= 8 can be written as log4 8 =i.2
(ii) 50 = I, can be written as logs I = o.
(iii) (2 h ) -2/3 = ~,
can be written as log h ~ =
2
2 2v2 2 3
Example 4: Rewrite the following equations in the exponential form :
1
(i) log2 32 = 5 (ii) log3 - = - 5
243
2 1
(iii) log ~ 5 = - (iv) logioo (0.1) =
5,,5 3 2
Solution:
Logarithmic Form Exponential Form
(i) log2 32 = 5 32·= 2 5
I
(il) log3C:3)= - 5 - =r5
243
2
(iii) log5JS (5) = "3 5 = (5$)213

I
(iv) loglOO (0.1) =-2 0.1 = 100- 1/2

Example 5: If logto x = a, find the value of lOa - I in terms of x.


Solution: Here, we have
loglo x = a =:> x = 100.
Now, lOa - I = 100 x 10-1

100 x
10 10
Example 6: If log5 x = a and loGY2 y = a. Find 10020 - I in terms of x and y.
326 Remedial Mathematics

Solution: Here, we have


logs x =a and log2 y = a,
Therefore, x = 50 and y = 20
Now, 20
100 - 1 = (52 x 22i o - 1
= (5 2)20 - 1 x 240 - 2
= 540 - 2 x 240 - 2

540 240
=-x-
52 22

(50
=--x--
t (2 0 )4
52 22
= x4 x y4 = x4y4
52 22 100
Example 7: Evaluate each of the following:
(I) log 5 + log 2
(il) log 500 - log 5
(iii) 4 log 5 + 2 log 4
(iv) log 6 + 2 log 5 + log 4 - log 3 - log 2
1
(v) -log 36 + log 5 - log 30
2
(VI) log 5 + 2 log 0.5 + 3 log 2.
Solution: (i) log 5 + log 2 = log (5 x 2) = log 10 = 1.
[By using log (mn) = log m + log n ]

(ii) log 500 - log 5 = log( 5~0) = log 100 = 2.

(iiI) 4 log 5 + 2 log 4 = log 54 + log 4 2


= log 625 + log 16 = log (625 x 16)
= log 10000 = 4. [': log 10000 = log 104 = 4]
(iv) log 6 + 2 log 5 + log 4 - log 3 - log 2 = log 6 + log 52 + log 4 - log 3
-log 2
= log 6 + log 25 + log 4 - (log 3 + log 2)
= log (6 x 25 x 4) - log (3 x 2)

= IOg(6X25
3x2
X 4)
= log'100 = 2.

(v) .!.log 36 + log 5 - log 30 = log (36)112 + log 5 - log 30


2
= log 6 + log 5 - log 30
= log (6 x 5) - log 30
= log 30 - log 30 = O.
Logarithms 327

(vi) log 5 + 2 log 0.5 + 3 log 2 = log 5 + log (0.5)2 + log 23


= log 5 + log (0.25) + log 8
= log (5 x 0.25 x 8)
= log 10 = I.
Example 8: If log (m + n) = log m + log n .

Show that m = _n_ [RGPV B. Phrama 20041


n-1
Solution: Gives log (m + n) = log m + log n
=> log (m + n) = log mn
m+n=mn
n =mn - m
n = men - 1)
n
=> m =--
n-l
Example 9: If log (mn) = log m - log n Show that n = J [RGPV B. Phrama 2005)
Solution: Gives log (mn) = log m - log n
=> log m + log n = log m -- log n
=> 2log n = 0
=> log n = 0
=> log n = log I
=> n = I
Example 10: Show that
16 25 81
(i) 7 log- + 5 log- + 3 log- = log 2
15 24 80
70 22 7
(ii) log- + log- - log- = 3 log 2 - 2 log 3.
33 /35 18

Solution: (i) 7 log~ + 5 log 25 +3 log!!.


15 24 80
= 7 (log 16 - log 15) + 5 (log 25 - log 24) + 3 (log 81 -log 80)
= 7 [log 24 - log (3 x 5)] + 5 [log 52 - log (2 3 x 3)] + 3 [log 34 - log (2 4 x 5)]
3
= 7 [log24 - (log 3 + log 5 )] + 5 [log 52 - (log 2 + log 3)]
+ 3 [log 34 - (log 24 + log 5)]
= 7 [4 log 2 - log 3 - log 5] + 5 (2log 5 - 3 log 2 - log 3)
+ 3 (4 log 3 - 4 log 2 - log 5)
= 28 log 2 - 7 log 3 - 7 log 5 + 10 log 5 - 15 log 2 - 5 log 3 + 12 log 3
- 12 log 2 - 3 log 5
= 28 log 2 - 15 log 2 - 12 log 2 - 7 log 3 - 5 log 3 + 12 log 3 - 7 log 5
+ 10 log 5 - 3 log 5
= log 2.
328 Remedial Mathematics

70 22 7 = log (70
(ii) log-+log--log- - x22
-) -Iog-
7
33 l35 18 33 135 18

= log[~x~J log(70xE....x~)
~
=
33 l35 18
18

= IOg(%) = log 8 -log 9

= log 2 3 - log 32 = 3 log 2 - 2 log 3.


Example 11: Find the values of x in each of the following:

(i) log 144 = log x (ii) log 125 = x


log 12 log 25
(iii) logx 4 + logx 16 + logx 64 = 12.
Solution: (i) Here. we have
log 144
- - - = log x
log 12
log 122
- - - =Iogx
log 12
2log 12
= log x
log 12
log x = 2
i.e., logtO x = 2
x = 102 = 100.
(ii) Here, we have
log 125
- - - =x
log 25
log 53
log 52 = x

3 log 5 = x
i.e.,
2log5
3
- =x
2
3
i.e., x =-.
2
(iii) logx 4 + logx 16 + logx 64 = 12
::::::> lo~ 22 + logx24 + logx 26 = 12
Logarithms 329

=:> 12 logx 2 = 12
=:> 10gx 2 = 1
i.e., xl =2

x =2.

Example 12: If log x = log y = log z , then show that x b + C- a yC + a - b~ + b - C = I.


b-e e-a a-b
Solution: Let us suppose
logx = logy = logz = k
b-e e-a a-b .
which gives
log x = k (b - c), log y = k (e - a), log z = k (a - b)
=:> (b + e - a) log x + (e + a - b) log y + (a + b - c) log z
= k (b + e - a) (b - c) + k (e + a -- b) (e - a) + (a + b - c) k (a - b)
= k {(b 2 - e 2) - a(b - c)} + k {(e 2 - ~) - b(e - a) + k{(~ - b 2) - e(a - b)}
= k {b
2
- e2 + e2 - ~ + ~ - b 2 } - k{a (b - c) + b (e - a) + e (a - b)}
= k . 0 - k . 0 = O.
Therefore,log x b + c - a + logyc + a - b + logza + b - c = 0
=:> log (x b + c - a. y + a - b.~ + b - C) = 0 = log 1.
Hence, x b + c - a. y + a-b. ~ + b - c = 1.

log a 10gb loge b


Example 13: If - -= -- = - - = prove that ~ . b . eC = 1.
b-e e-a a-b
. log a 10gb loge
Solution: Let - - = - - = - - = k
b-e e-a a-b
Then log a= k (b - c)
log b = k (e - a)
and log e = k (a - b).
Adding all those, after multiplication by a, b, e, respectively, we get
a log a + b log b + clog e = ak(b - c) + bk(e - a) + ek(a - b) = O.
Therefore, log ~bbec = 0 = log 1
~bbec = 1.
Example 14: Solve the equation clx = bX - C ~ + 5, a, b, e, > '0 but * 1.
[Meerut B. Sc. Biotech. 20051
Solution: Take logarithm to both sides, we get
log? = log bX - C + log ~ + 5

=:> 2x log a = (x - c) log b + (x + 5) log e


=:> x[2 log a - log b - log e] = 5 log e - clog b
330 Remedial Mathematics

109(fJ
x

Example 15: Solve the system of equations


109(:: r
(i) 5(logy x + logx y) = 26
(ii) xy = 64. IMeerut B. Sc. Biotech 2005)
1
Solution: We know that logx y = - - , therefore
logy x

(i) ~5(lOgyx+_l_)=26.
logy x

Putting logy x =p, it gives 5 (p +~ J= 26


or 5p2 - 26p + 5 = 0
5p2 - 25p - P + 5 = 0
~ 5p (p - 5) - 1 (p - 5) = 0
~ (p - 5)(5p - 1) =0
~ P =5,1/5.
When p = 5, when
logy x = 5 ~ x = i.
(ii) ~ x . y = y5 . Y = 64 or i
= 64 ~ y3 = ± 8
1=+ 8 = 23 or y3 = - 8 = (- 2)3
y =2 or y = -2.
64 64 64
Then (ii) ~ x = - = - =32 orx= -=-32.
y 2 -2
But y and x both are used as base in equation (i), so x, y cannot be negative
~ one solution is x = 32, y = 2.
1
When p = 5' then logy x = 51 ~ x = yl/5 or x 5 = y

:. (ii) ~ x, y = x 5 = 64 or x 6 = 64 or x 3 8 "*
~ x =2 or x = - 2 [reject negative value]
64 64
Again (ii) ~ x =-= - = 32.
y 2
This gives another solution.
Thus, we get two solutions of the gives system of equations as follows:
x = 32, y = 2 or x = 2, y = 32.
Logarithms 331

~~~~~~~I EXERCISE 7.1 ~I~~~~~~~


1. Write the following into logarithmic form
(i) 2 8 = 256 (ii) 10 3 = 1000 (iii) 7 3 = 243
(iv) 4- 4 = _1_ (v) 4 3/ 2 = 8.
256
2. Write the following into exponential form
(i) log5 25 = 2 (ii) loglo 0.001 = - 3.
3. Find the value of b which satisfies

(i) logJ8 b = 3~ (ii) loge 210gb 625 = loglO 16 loge 10

1
(iii) log J3 x = 4 (iv) log4 x = 15 (v) logl25 X = -.
6
4. Find
(i) log6 16, if logl2 27 = a
(ii) log25 24, if log6 15 = a and logl2 18 = [3
(iii) log30 8, if lof30 3 = a and log30 5 = b.
5. If logl2 18 = a and log24 54 = [3, show that a[3 + 5 (a -(3) = I.
6. Without using the table, show that

(i) _1_+_1_ > 2 (ii) log2 17 logl/5 2 log(l/5) > 2


cos2 1t log4 1t
(iii) 110gb a + loga b I ~ 2, where a and b are positive integer not unity.
7. Compute, without using tables
(i) log3 4 log4 5 log5 6 log6 7 log7 8 logg 9
(ii) log3 2 log4 3 log5 4 .... logl5 14 logl6 15.
8. Show that log2 3 is an irrational number.
9. Prove the following:
(i) log a n = I + log b
logab n a
10. Show that:
50
(i) log 2 + 2 log 5 - log 3 - 2 log 7 = log 147

(ii) IOgC~) + logG!)- lOge!) = 0



(iii) -log 25 - 2 log 3 + log 18 = 1
2
(iv) .!.. log2 54 + log2 10 - log2 625 = 1
3
(v) loglo 10 + loglo 100 + loglo 1000 + loglo 10000 = 10

(vi) .!..Iog 9 + 2 log 6 + .!..Iog 81 -log 12 = 3 log 3.


2 4
332 Remedial Mathematics

11. Evaluate each of the following


(i) 210g3 5 - 510g32
(ii) (81)lIlog5 3 + 2i og 936 + 3(41og9 7)
(iii) log 15 + 2 log 0.5 + 3 log 2 - log 3 - log 5
(iv) log 21 + log 4 + 2 log 5 - log 3 - log 7.
12. Show that
(i) a loga I + 2 log a 2 + 310g a 3 + ... + n loga n = 22 . 33 . 44 ... nn
(ii) a loga I + 2 log a 2 + 2 log a 3 + ... + 210ga n = (n!i
(iii) loglo tan 1° 10glO tan 2° loglo tan 3° ... loglo tan 50° = 0
(iv) loglo tan 1° + 10glO tan2° + ... +loglO tan 89° = o.
s
I ANSWERS I
I. (i) log2 256 = 8 (ii) log", 1000 = 3 (iii) log7243 = 3
(iv) log4 (1/256) = - 4 (v) log4 8 = 3/2.
2. (i) 52 = 25 (ii) 10-3 = 0.001.
2
3. (i) 32 (ii) (J2t
4(3 -a) 5-~
4. (i) (ii) (iii) 3 (1 - a - b).
3+a 2a~ + 2a - 4~ + 2

1
7. (i) 2 (ii)
4
11. (i) 0 (ii) 890 (iii) log2
(iv) 2

• METHOD TO DETERMINE THE CHARACfERISTIC AND


MANTISSA
The characteristic is determined by using the following rule :
(i) The characteristic of the logarithm of any number greater than I is one less than
the number of digits to the left of the decimal point in the given number.
(ii) The characteristic of the logarithm of any number less than 1 is negative and
numerically one more than the number of zeroes to the right of the decimal point.
For Example: See the following table:
S.NO Given number Characteristic Explanation
1. 63 One less then the number of digits

2.
389.6
3986
6.36

0.4
0.04
n
-1 )
-2
to the left of the decimal point.

One more than the number of zeroes


0.004 x 10- 1 -3 to the right immediately after the
decimal point.
Logarithms 333

Method to Determine the Mantissa


The mantissa is determined by using the following rule :
(i) The mantissa is the same for the same significant figures in the same order and
does not depend on the position of the decimal point.
(ii) The mantissa is always taken as positive.
For Example:
Given number Characteristic Mantissa Logarithm
5978 3 0.7766 3.7766
597.8 2 0.7766 2.7766
0.005978 -3 0.7766 - 3 + 0.7766
The given number contain the same significant figures, namely 5, 9, 7, 8 in the same
order and so the mantissa of their logarithm is the same, they differ only in the characteristic.
In log 0.005978, which is equal to - 3 + 0.07766, the characteristic - 3 is negative and
the mantissa 0.7666 is positive. To indicate that the negative sign applies to the characteristic
only, the '-' sign is put above the characteristic .Thus
logO.005978 = - 3 + 0.766 6 = 3.7766.
It is read as "bar 3 point 7766"

Remark
• To find the mantissa of the logarithm of a number which contain less or more than
four digits, make it afour digit number by having zeroes on its right or by condensing
it by the rule of approximation which is given below.'

- or more t h
(I) ,,1 an i
- .IS takes as I ; andless th
an i
- .IS negIected"
.
2 2 2
(ii) "5 or more than 5 is taken as 10; and less than 5 is neglected"
For Example:
(i) 6.76236 = 6.7624, upto four decimal places.
(ii) 6.7634 = 6.763, upto four decimal places.

Mantissa of the Logarithm of a Given Number


To find the mantissa ofiogarithm of a given number we use the standard table ofiogari'thms.
The table of logarithms consist of 90 rows and 20 columns. Every row begins with a two
digit number 10, 11, .... , 98 99 and every column is headed by a one digit number 0, 1.
2, ... , 9. On the right of the table, a big column is divided into 9 sub columns headed by
the digits 1, 2, 3, ... , 9, known as column of mean differences. To find the mantissa of
a number, consider first four digits from the left most side of the number. If the number
is the decimal point, then mantissa is calculated with the help of the number formed by
digits beginning with the first non-zero digits.

Remark
• Tofind the logofa given number x, use theformula logx = characteristic + mantissa.
334 Remedial Mathematics

~~~~~~~I SOLVED EXAMPLES I~~~~~~~


Example 1: Find log 756.8.
Solution: By neglecting the decimal point, we obtain 7568, which is a four digit number.
See the number 75 in the extreme left hand column in the logarithm table. In the horizontal
line of75 and under 6 (next digit in the number) we found the number 8785. In the same
horizontal line and under 8 (4th digit of the given number), the number found is 8. Adding
8 to the already obtained number 8785, we get 8793. Therefore, the mantissa is 0.8793
=> log 756.8 = 2.8793 [.,' characteristic is 2]
Similarly log 75.68 = 1.8793 [characteristic is 1]
log 0.07568 = 2.8793 [characteristic - 2]
Example 2: Find the logarithm of the following number
(i) 5395 (ii) 0.002359
(iii) 25795 (iv) 0.005.
Solution: (i) The given number 5395 is a four digit number. See the number 53 in the extreme
left column is the logarithm table. In the horizontal line of 53 and under 9 (next digit in the
given number) we found the number 73.16. In the same horizontal line and under 5 (4 th digit
of the number ), the number found is 4. Add this number 4 to 7316 to get 7320, which is
the required mantissa of log 5395. Also the characteristic of 5395 is 3.
Therefore, log 5395 = characteristic + mantissa
= 3.7320.

(ii) Firstly, find the four digit number, by getting the first four digits beginning with
the first non-zero digit on the right of the decimal point which is 2359. The
mantissa of the given number can be determined by the procedure discussed
in (i) and given by
mantissa of2359 = 3711 + 17 = 3728.
Also, the characteristic of 0.00359 is - 3.
Hence, log 0.002359 = 3.3728.
(iii) Clearly, the characteristic of the logarithm of25795 is 4. To find the mantissa of
the given number 25795, consider the four digit number 2579 and apply the same
process. we get
mantissa of2579 is 4114.
Therefore, the logarithm of the given number 2595 is 4.4114.
(iv) The characteristic of 0.005 is - 3. To find the mantissa, consider the number 50.
See in the row 50, under the column headed by 0 and get the number 6990.
Therefore, log 0.005 = - 3 + 0.6990 = 3.6990.
Example 3: Find log 11.648.
Solution: The characteristic of 11.648 is 1. Now leaving the decimal point, the given
number consists of five digits. Condensing it to a four digital number, by the rule of
approximation, we get the number 1165. Now follow the same procedure, as above, the
mantissa of 1165 is 0.0664.
Hence, log(11.647) = log (1 1.65)= 1.0664.
Logarithms 335

• ANTILOGARITHM
If log x = n, then x is called the antilogarithm of n and is written as
x = antilog (n).
For Example:
(i) log 10 = 1 <=> antilog (1) = 10.

(ii) log 0.0681 = 2.8331 <=> antilog (2.8331) = 0.0681.


Method of Finding the Antilogarithm
To find the antilog of a given number, we make use of the table of antilogarithms. The
method is almost the same as that for finding the logarithm of a number. The table is
divided into three similar sets of columns and the four digits are to be taken in the same
manner starting with digits immediately to the right of the decimal point, not excluding zero.
The following points must be kept in mind for convenience :
(i) If the given number is negative, first make it positive by adding one to the
decimal part and by subracting one from the integral part.
(ii) Apply the method, same as that used for finding the logarithm of a number.
(iii) If the characteristic of the given number is positive and is equal to n, then insert
the decimal point after (n + 1) dIgits in the obtained number.
If n > 4, then write zeroes on the right side to get (n + 1) digits.
(iv) If the characteristic of a given number is negative and is equal to - n or n then
on the right side of the decimal point, write (n - 1) consecutive zeroes and then
write the obtained number

~~~~~~~I SOLVED EXAMPLES ~I~~~~~~


Example 1: Find antilog 2.3456.
Solution: The mantissa is positive and is equal to 0.3456. Now look into the line starting
with 0.34. In the horizontal line of 0.34 and under 5 (the next digit of the mantissa), the
number obtained is 2213. In the same horizontal line and under 6 (the fourth digit of the
given number) in the mean difference columns, the number found 3. Adding 3 to 2213,
we get 2216. Now, since the characteristic is 2, the required number must have 3 digit in
the integral part.
Therefore, antilog 2.3456 = 221.6
Similarly, we can find
antilog 0.3456 = 2.216, antilog 1.3456 = 0.2216
antilog 2.3456 = 0.02216, antilog 5.3456 = 0.00002216.
Example 2. Find antilog 3.0675
Solution: In the horizontal line of 0.06 and under 7 we get 1167. In the same horizontal
line and under 5 in the mean difference column, we get I. Adding 1 to 1167 we get 1168.
Since the characteristic is 3, the required number must have 2 zeroes immediately to the
right of the decimal part.
Therefore, antilog (3.0675) = 0.001168.
336 Remedial Mathematics

Example 3: Find the antilog, each of the following


(i) - 1.2084 (ii) - 0.62.
Solution: (i) The given number is - 1.2084.
Here, we observe that the mantissa of the given number is negative. First we make
it positive by adding 1 in following manner
- 1.2084 = - 1 - 0.2084
= - 1 - 1 + 1 - 0.2084
=-2+0.7916
= 2.7916.
Now, using the antilogarithm table, we find that the number corresponding to the
mantissa 0.7961 is 6189.
Since, the characteristic is 2 i.e., - 2, put one zero just after the decimal point to get
the antilogarithm of the given number.
Therefore, antilog (- l.2084) = antilog (2.7916.)
= 0.06189.
(li) Consider the given number - 0.62
= - 1 + 1 - 0.62
= -1 + 028
= 1.028.
Now, using the antilogarithm table, the number corresponding to 0.23 is 1905 also,
the characteristic is T i.e., - l.
Therefore, antilog (- 0.62) = antilog (1.28)
= 0.l905.
Example 4: Find the values of the following
(1) 2.76x4 (ii) 6.45 x 98l.4
(iii) 6.42 + 3 (iv) 0.0064 x 1.507.
Solution: (i) 2.76x4 = (-2 + 0.76) x 4
= - 8 + 3.04
= - 8 + 3 + 0.04
= - 5 + 0.04
= 5.04.
(li) Let x = 6.45 x 98l.4
::::;> log x = log 6.45 + log 981.4
= 0.8096+2.9919=3.8015.
Therefore x = antilog (3.8015)
= 6331.
1
(iii) 6.42+3 = -(-6+0.42)
3
-6 0.42 -
= - + - = - 2 + 0.l4 = 2.l4.
3 3
Lo~arithms 337

(iv) Let x = 0.0064 x 1.507, then


log x = log (0.0064 x 1.507)
= 3.8062 + 0.1781

= 3.9843.

=> x = antilog (3.9843.)


0.09645.
=
Example 5: Find log {(27i x (0.81lls + (90/14), where log 3 = 0.4771213.
Solution: The required value is
27 4 81 5
log {(27)3 x (0.81)4/5 + (90)5/4} = 3 log-+-log---log 90
10 5 100 4
, 4 5
= 3 (log3 3 - I) + -(log3 4 - 2) --(log3 2 + I)
5 4

= (9+ I; -%}Og3-(3+~+%)
97 17
= -log3-5-
10 20
= 4.6280766 - 5.85
2.7780766.=
Example 6: Using logarithmic table, evaluate the following

41.32x20.18
(i) (iO
12.69
112
· (.) L 41.32x20.18
So Iutton: I etx = ( )
12.69
Then, we have

I I (41.32X20.18)1/2 = ..!..lo (41.32X20.18)


og x = og 12.69 2 g 12.69

1
= -(log 41.32 + log 20.18 -log 12.69)
2

= ..!.. (1.6162+ 1.3049-1.1035)


2
1 1
= "2(2.9211-1.1035)= "2(1.8176)
= 0.9088.
Therefore, x = antilo (0.9088)
= 8.106
338 Remedial Mathematics

(ii) Let

Then, we have

Iog x = -1 Iog [ (45.4 )2 )


3 (3.2)2 x (5.6)2

= 3"1 [2 log (45.4) - 2 log (3.2) - 3 log (5.6)]

1
= -(2 x 1.6571-2 x 0.5051-3 x 0.7482)
3

= .!.(3.3142-1.0102-2.2446)
3

= .!. (0.0594) = 0.0198.


4
Therefore, x = antilog (0.0198.)
= 1.047.

~~~~~~~I EXERCISE 7.21~~~~~~~


1. (i) 25.795 (ii) 1270 (iii) 0.005
(iv) 431.5 (v) 0.0074 (vi) 0.002598
(vii) 0.3582.
2. Find the antilogarithm of each of the following :
(i) 1.4114 (ii) 3.1038 (iii) 3.6990
(iv) 2.6350 (v) 3.8692 (vi) 3.4146
(vii) 1.5541.
3. If log x = 1.4914 and log y = 2.4669, find the value of each of the following:
x x2
(i) xy (ii) (iii)
. y l
3
x x2
(iv) i Y
(v)

4. Using logarithmic table, find the value of each of the following :


76.03 x 9.08 (73.56)3 x (0.0371)2 (25.36)2 x 0.4569
(i) (ii) (iii)
101.2x63.17 68.21 847.5
8113 x 73 / 4
(iv) (v) (0.OOOOI427)ln
(7.29)1/3 x3.26
(6.45)3 x (0.0034)113 x981.4
(vi) .
(9.37)2 x (8.93)114 x(0.0617)
Logarithms 339

• APPLICATION OF LOGARITHM IN PHARMACEUTICAL


PROBLEMS

(1) Half life Period


The half life period T of C 14 gives the value of disintegration Constant k from the equation

k = - - . The d··
0.693 . eauatlOn
IsmtegratlOn . .IS k - Iog -a- IS.
2.303
=- .
T I t a-x
l4
Exa mple 1: The amount of C isotope in a piece of wood found to be one sixth of its
amount present in a fresh piece of wood. Calculate the age of wood:
Solution: Half life of C I4 = 5577 years.
Half life period => T = 0.693
k
k = 0.693 0.693
T 5577
·· . equation,
Th e dIsmtegratlOn . name Iy k = - - Iog--.
2.303 a
t a-x
0.693 2.303 I 1
X
5577 = - t - og 1/6
2.303 x 5577 x log 6
t = ---------="---
0.693
2.303 x 5577 x 0.7782
0.693
t = 20170 year.

(2) Calculation of Boiling Point or Freezing point


If the freezing point or the boiling point of a liquid at one pressure is known, it is possible
to calculate it at another pressure by the use of the Chaperone classes equation
p, ~Hv [ I I]
loglo~ = 4.576 7i - T2 .
Example 2: Calculate the vapour pressure of water 90.0°C if its value at JOO.O°C is
76.0 cm. The mean heat of vaporization of water is the temperature range 90° and JOO°
C is 542 calories gm.
Solution: l1Hv = 542 x 10 caliper mole
P2 =? PI = 76.0 cm
T2 =90 + 273 = 363° Abs TI = 100 + 273 = 373° Abs.
According to Chaperone clauses equation
P2
log1o- ~Hv
= - - [T2
--- -11 ] ,we have
l't 4.576 11 T2
10 P2 =542 X I8[363-373]
glO PI 4.576 363 x 373
340 Remedial Mathematics

542 x 18[363 -373]


Hence, loglo P2 - loglo 76 = 4.576 363 x 373
P2 = 52.88 cm or 528.8 mm.

(3) Calculation of Equilibrium Constants and Concentrations


If the concentration of reactants and products are known at equilibrium is a reaction, the
equilibrium constant can be calculated.
Example 3: An equilibrium system for the reaction between hydrogen and iodine to
given hydrogen iodine at 670 k in a 5 litre flask contains 0.4 mole of hydrogen, 0.4 mole
of iodine and 2.4 moles of hydrogen iodine calculate equilibrium constant.
Solution: For the reaction
H 2(g) + 12 (g) ~ 2HI(g).

k = [Hlf
[H 2 ][1 2 ]
molar concentration of various species at equilibrium

[H] = 0.4 [I] = 0.4 = 0.08 mol L- I


2 5' 2 5'
[HI] = 2.4 = 0.48
5
k =(0.48i
(0.08)(0.08)
log k = log (0.48)2 - log (0.08)2 = 2 log 0.48 - 2 log 0.08
=2 log 4.8 x 10- 1_ 2 log 8.0 x 10- 2
=2 (-I + .6812) - 2 (- 2 + .9031)
= 1.5562
k=36

(4) P-H Equation


PH =k + log [salt] .
a [acid]
Example 4: Some of 0.2 m acetic acid are mixed with 50 ml of 0.2 m sodium acetate
solution. What will be the PH the mixture ka = 1.85 x ur 5.

[Acid] = .Q3..
100
[Salt] = .Q3..
100
We have pH =pka - log [Sa~t] = - log 1.85 x 10- 5 + log 0.2/100
[Acid] 0.21100
pH = - log 1.85 x 10- 5
pH = 4.7325.
Logarithms 341

Example 5: Calculate the PH. Value


(i) 0.001 MHC/ (ii) 0.001 M NaOH
Solution: (i) Since HC is a strong Acid, H30+ ion concentration is equal to that of the
acid
[HP1 = [HCl] = 0.001 m - I x 10- 3 m
pH = - log [H30+]
or pH = _ log [I x 10- 3] = - (-3) log 10 = 3.
(ii) Since NaOH is a strong base, it completely ionizes.
:. hydroxyl ion concentration is equal to that of the base
[OW I] = [NaOH] = 0.01 m = I x 10- 2
kw = [HP+] [OH-]
k IxlO- 14
[H 01 = - - = = I x 10- 12
3 [OW] Ix 10-2
PH = - log [H30+] = - log (I x 10- 12) = 12.

I EXERCISE 7.31
1. Calculate the PH of 0.5 molar solution of sulphuric acid
2. Gives that the half life period of Radium is 1580 years. Calculate the disintegration
constant and average life
3. The equilibrium Constant for the reaction
N2 (g) + 3H2 (g) = 2 NH3 (g).
at 715 k is 6.0 x 10- 2 . Ifin a particular reaction there are 0.25 mol L- 1 ofH2 and
0.06 mol L-I of NH3 present, calculate the concentration of N2 at equilibrium.
4. At what temperature will water boil under a pressure of787 mm ? The latent heat
of vaporization is 536 cal per gram.
5. The pH of a soft drink is 4.4 . Calculate [HP+] and [OH-].

HINTS TO THE SELECTED PROBLEMS

1. H2 S04 ~ 2H+ + SO~­


In a 0.5 M H2So4 solution
[H1
= 2 x 0.5 mol L- 1
pH = - log [W] = - I x log I = O.
·s:: • dT 0.693
2. Ha If Ihe perlO 112 = - -
k
k = 0.693
TI/2

0.693
1580
342 Remedial Mathematics

Average life A= ..!. = __1_-:-


k 4.3 x 10-4 .

4. 10glO P2 = ~Hv [~-...!..]


PI 4.576 1\ T2

10 10 787 = 536 x l8[T2-373]


g 762 4.576 373T2

536 x 18[T2 -373]


10glO 787 - log 762 = 4.576 3 73T2
T2 = 374°k.
s. pH = - log [HP+]
- log [H30+] = 4.4
log [H30+] = - 4.4
lew lxlO- 14
[OHl = - - = ----:- 5
[H3 0 +] 3.98x10-

I ANSWERS
1. 0 2. 4.3 x 10-4 years-I, 2325 years. 3. 3.84
4. 3740 k or 101 0 C. 5. [Hp+] = 3.98 x lO-l mol L-I , [OH-] = 2.5 x 10-10 mol L-I

OBJECTIVE EVALUATION

MULTIPLE CHOICE QUESTIONS


Choose the most appropriate one:
1. If t.f = bY = cf and logha = log} . then which one of the following will hold True?
(a) y = x - z (b) y = x + z
(c) ;; = yz (d) Y. = xz .

2. The domain of the function ~Iogo.sx is


(a) (0.5, 00) (b) (I, 00)
(c) (0,1) (d) (0.5,1).

3. If loga 6 = m. and loga 3 = n. then loga (%}s equal to

(a) 1 - loga2 (b) 1 + m + n


(c) 1- m - n (d) 1- m + n.

4. If the logarithm of a number to the base·-.[g· is 6, then the number is


(a) 512 (b) 343
(c) 216 (d) 36.
Logarithms 343

5. The equation log3 (3x - 8) = 2 - x has the solution


(a) x = 4 (b) x = 3
(c) x = 2 (d) x = 1
6. Evaluate log tan 1° + log tan 2° + ... + log tan 89°
(a) 0 (b) 1
(c)1/2 (d) .fi
· I'f I 14 I 28 I 405
7. SImp I y og 15 + og 27 = og 196
(a) I (b) 1092
(c) 1/2 (d) .fi
8. Solve for x if logl6x + (log I6x)2 + (Iogl6x)3 + ... + to OCJ = 1/3.
(a) 1/2 (b) 1/2
(c) 2 (d) 3/5
9. If loglO (x 3; ) = 3a + 2b and 2 loglo (x 2y 3) = 2a + 3b, then the value to x at a = 3 in
W 100 ~ I~
(c) loglO2 (d) loglo3.
10. If log9o 2 = a, then loglo 25 is equal to
(a) (I + a) (b) 1 - a
(c) 2 (1 - a) (d) 2 (l + a).

FILL IN THE BLANKS


1. If log .J8 x = 3 i- The value of x in _ _ _ _'
1. If log 2 = 0.3010 and log 3 = 0.4771 the value log 25 _ _ __
3. log927 - log279 = _ _ __

4. log7 logs (~ x + 5 + J;) = a then x ____ '


5. The value of log 3.fi (5832) is equal to _ _ __
6. The value of 641og8 5 is _ _ _ _ '
7. If logIQ5 + loglo (5x + 1) = loglo (x + 5) + 1 then x is equal to _ _ _ _'
8. If log 27 = 1.431 then value of log 9 is _ _ __
I
9. 2 loglo 5 + logIQ8 - '2log10 4 = _ _ __

10. 1 + 1 + I ] .IS egua I to _ _ __


[ (log a be + I) (Iogh ea + I) (log,. ab + I)

TRUE/FALSE

~ 1. The equation ~ log (~ + 2x) - log J; + 2 = 0 has the solution x = I. (T/F)

1. IfIOg(a;b) = ~ [Ioga+logb]a=b. (TIP)


344 Remedial Mathematics

bq
3. IF log,r- = p and log"x = q. the log(alb) x = q _ p . (T/F)
log 2 log 2 log 2
4. a Y' a Z'a a = Z. (T/F)
5. The Number log27 is a rational Number. (T/F)
6. The value of 3210g 3 is equal to 49. (T/F)
7. The value of loglo 50000 - log lO 5 is equal to 10. (T/F)

I ANSWERS
MULTIPLE CHOICE QUESTIONS
1. (d) 2. (c) 3. (d) 4. (a)
5. (c) 6. (c) 7. (b) 8. (c)
9. (b) 10. (c).

FILL IN THE BLANK


1. 32 2. 1.398 3. 516 4. 4
5. 6 6. 25 7. 3 8. 0.954
9. 2 10. I

TRU'E/FALSE
1. True 2. True 3. True 4. True
5. False 6. False 7. False

REFRESHER ~I~~~~~~~
Do you know? (After reading this Chapter you must be able to learn the following
concepts)
Let there be a Number a > 0 and a * 1. Number x is called the logarithm of another variable
Y > 0 to the base a if cI = Y .
cI'= Y ¢:> loga Y = x.
• logaa = logbb = 1.
• loga 1 = 0 ,10gb 1 = O.
• logba. logca = 1.
• logba = logca logb c .
• loga(m x n) = logam + logan.

• IOga(:) = loga m - logan.


• logamn = n logam.
• The integral part of a logarithm is called the characters and the decimal part is
called mantissa.
Logarithms 345

Can we do? (Frequently Asked Questions)


n
1. If log (m + n) = log m + log n, show that m = - - [RGPV B. Pharma 2004]
n -1
2. If log (mn) = log m - log n, show that n = 1 [RGPV B. Pharma 2005]
3. Solve the equation ;a = b - c·cf + 5, a, b, c> 0 but not equal to 1
X

[Meerut B. Sc. (Biotech) 2005]


4. Solve the following equation
xy=64 [Meerut B.Sc. (Biotech 2005]

DDD
SYSTEMS OF COORDINATES

III INTRODUCTION
Coordinate Geometry is the branch of mathematics in which two numbers are used to repre-
sent the position of a point with respect to two mutually perpendicular number lines called
coordinate axes.
The french mathematician and philosopher Rene Descaotes first published his book
La Geometric in 1637 in which he used algebra in the study of geometry. This he did by
representing points in the plane by ordered pairs of real number called cartesi an coordinates
and representing lines and curves by algebraic equations.

Coordinate Axes
The adjoining figure 8.1 shows two number lines XoX and YoY intersecting each other at
their zeros.

1
0
X' X
-3 -2 -1 2 3
-1

-2

-3

Y'

Fig. 8.1
XoX' and YoY' are called coordinate axes out ofwhichXoX is called the x - axis,
Yo Y is called y-axis and their point of intersection is called the origin.
System a/Coordinates. 347

Remark
• Number lines XoX and YaY are sometimes also called rectangular axes as they are
perpendicular to each other.
Convention of Signs: The distance measured along OX and Of are taken as positive and
those along OX' and Of' are taken as negative as shown in figure 8.1.

Coordinate of a point in a plane


Let P be a point in a plane. Let the distance of P from the y-axis = a units.
And, the distance of P from the x-axis = b units. Then we say that the coordinates of
P are (a, b) a is called the x - coordinates or abscissa of P and b is called the y - coordinate
or ordinate. of P.
Y

• (a. b)
I
I
I
I
I
I
b :
I
I
I
I
I

X'
',M
---=+-------'---'-'."--- X
0t---_ a -

Y'
Fig. 8.2

Remarks
• (x, y) and (y, x) do not represent the same point unless x = y.
e.g. (5,4) and (4,5) represent two different points.
• In stating the co-ordinates of a point the abscissa proceeds the ordinate. The two are
separated by a comma and enclosed in a bracket. Thus a point, whose abscissa is x
and whose ordinate is y designated by the notation (x, y) i.e., (abscissa, ordinate)
• Since at origin the value of x-coordinates is 0 and Y
the value of y-coordinate is also 0, Therefore, the
coordinates of origin = (0,0). II I
(-. +) (+. +)
• Since for every point on x axis, its distance from x -
axis is 0 i.e., ordinate is O.
x' -----:::+----. X
Therefore the Coordinate of a point on x-axis are 0
taken as (x, 0). (~,I~) (1~_)
• In the same way, for every point on y-axis its
distance from y-axis is 0 i.e., abscissa is 0, therefore,
the coordinate ofa point on y-axis are taken as (O,y). Y'
Fig. 8.3
348 Remedial Mathematics

Quadrants y

LetX' OX and YOY be the Coordinate Axes.


II I
These axes decide the plane of the paper into four (-,+) (+, +)
regions, called Quadrants. The RegionsXOY, YOx' ,X'Or'
and y' OY are respectively known as first, second, third
X'--------~-------- X
and fourth quadrants. 0
1I1 IV
Using the convention of signs, we have the signs (-,-) (+, -)
of the coordinates in various quadrant given below.
Region quadrant nature ofx and y signs ofcoordinates
Y'
XoY I x>O,y>O (+, +) Fig. 8.3
YoX II x<O,y>O (-, +)
XoY III x<O,y<O (-,-)
YoX IV x>O,y<O ( +,-).

~~~~~~~I SOLVED EXAMPLES ~I~~~~~~


Example 1: In which quadrant will the point lie if .
(i) the ordinate is 3 and the abscissa is - 4 ?
(ii) the abscissa is - 5 and the ordinate is - 3 ?
(iii) the ordinate is 4 and the abscissa is 5 ?
(iv) the ordinate is 4 and the abscissa is - 8 ?
Solution: (i) Second quadrant
(ii) Third quadrant
(m) First quadrant
(iv) Fourth quadrant.
Example 2: In which quadrant do the given point lie.
(i) (4, - 2) (ii) (-3, 7) (iii) (-I, - 2) (iv) (3, 6).
Solution: (i) fourth quadrant
(it) second quadrant
(iiI) third quadrant
(iv) first quadrant.
Example 3: On which axes do the given points lie ?
(i) (7, 0) (ii) (0, - 3) (iii) (0, 6) (iv) (-5, 0)
Solution: (i) In (7,0), we have the ordinate = °
:. (7,0) lies on the x-axis.
(ii) In (0, - 3) we have the abscissa =°
(0, - 3) lies on the y-axis.
(m) In (0, 6) we have the abscissa = °
:. (0, 6) lies on they-axis.
(iv) In (- 5,0) we have the ordinate = °
:. (- 5,0) lies on the x- axis.
System of Coordinates 349

Example 4: Plot the points (- 3, 0), (2, 3), (- 4, 3) and (3, - 5) in a rectangular Coordinate
system.
Solution:
Y

4
C(-4, 3) 8(2, 3)
1"------------- 3·----.,
2

A (-3, 0)
X' X
-4 -3 -2 -1 0 2 :3 4
-1

-2

-3

-4

-5 ---------- .. 0 (3, -5)

Y'
Fig. 8.5

• DISfANCE BETWEEN TWO POINTS


Theorem: The distance between two points A (x 1 ' y 1) and B (x2' y) is given by the formula

AB = ~(X2-X))+(Y2-y))2
Proof: Let A (x) ,y)) and B (x2 , Y2) be the given points
Letx'OXand YOY be the coordinate Axes.
Y

I
I
I
I
I
I
(x1. Yl) :
A I
I -----------: N
I I
I I
I I
I I
I I
X'----------~~---L--------~----~X
o L M

Y'
Fig. 8.6
350 Remedial Mathematics

Draw A L ..L Ox, BM..L OX and AN ..L BM. Then


OL = xl' OM=x2 ,AC= Yl' andBM=Y2
AN = LM=OM-OL=(x2 -x l ).
BN = BM-NM=BM-AC=(Y2- Y I) [.,' NM=AC].
Now, Ll ANB is a right - angled, so by Pythagoras theorem, we have
AB2 = AN2 + BN2
= (x2 _x l )2 + (Y2 - Yl)2

AB = ~(X2 _XI)2 +(Y2 _ YI)2.


Remark
• The distance of a point P(x, y) from the sign (0, 0) is given by

OP = ~(x_O)2 +(y_O)2

= ~x2+i

STEP KNOWLEDGE
In order to prove that a figure is a.
Step 1. Square, prove that four sides are equal and the diagonals are also equal.
Step 2. Rhombus, prove that the four sides are equal.
Step 3. rectangle, prove that opposite sides are equal and the diagonals are also equal.
Step 4. a parallelogram, prove that the opposite sides are equal.

~~~~~~~I SOLVED EXAMPLES ~I~~~~~~


Example 1: Find the distance between the points (3, 4) and (6, -3)
Solution: The given points areA (3,4) andB (6, -3).
Here XI = 3, YI =4andx2 =6'Y2=-3.

AB = ~(X2-XI)2+(Y2-YI)2
= J(6-3)2 +(_3_4)2

= ~9+49 = J58.
Therefore AB = J58
Example 2: Show that the points (- 3,- 3) , (3,3) and C (- 3 J3 ,3 J3 ) are the vertices of
an equilateral triangle.
Solution: A (-3, - 3), B (3,3) and C (-3 J3 ,3 J3) are the three points.

AB = ~(3+3)2 +(3+3)2 = ~36+36 = JTi = 6.J2.


System of Coordinates 351

AC = J(-3Jj +3)2 +(3Jj +3)2

= ~27+9-18Jj +27+9+18Jj = JTi = 612.


BC = J(-3Jj _3)2 +(3Jj _3)2

= JTi = 612.
Then AB = BC=AC.
Therefore, triangle ABC is an equilateral triangle.
Example 3: Find the value of a if the distance between the points (3,0) and (4.1) is Fa
[UPTU B. Pharrna 20031
I 2 2 I
Solution: Here\j(4-3) +(I-a) =.JlO
~ 1 +(l-ai = 10~(1-a)2=9
~ l-a=±3
~ a =1 ±3=-2,4.
Example 4: Show that the points A(2, -2). B(8. 4), C(5. 7) and D(-I. 1) are the vertices of
a rectangle. [UPTU B. Pharrna 2007J

Solution: Here. AB = ~(8_2)2 +(4+2f =..j62;62 = JTi.


BC = ~(5_8)2 +(7_4)2 = ~(_3)2 +3 2 = Jl8
CD = ~(_1_5)2+(I_7)2 = )(_6)2+(_6)2 = JTi
DA =~(2+1)2 +(_2_1)2 = ~32 +(_3)2
Jl8 =

AC = ~(5_2)2 +(7 +2)2 = ~ = J90

BD = ~(-1- 8)2 + (I - 4)2 = ~(-9 f + (_3)2 = J90


Thus, the opposite sides of the quadrilateral ABCD are equal and its diagonals are also
equal. Hence, it is a rectangle
Example 5: Find the point on the y axis which is equidistant from the points (3. 4) and
(6.7).
Solution: Given points are A(3.4) and B(6,7). Required point P is· on the y-axis. Its
abscissa = O.
Suppose its ordinate = y.
Then Coordinate ofP are (O,y).
Now PA \"'" PB
~ )(0-3f +(y_4)2 = )r-(0-_-6-f-+-(-y-_-7)-2
~ 9+1-8y+16 = 36+1-14y+49
~ -8y+ 14y = 36+49-9-16
~ 6y = 60
~ y = 10.
Therefore, the required point is (0, 10).
352 Remedial Mathematics

Example 4: If the point (x ,y) is equidistantfrom the points (a + b, b - a) ana (a- b, a + b)


prove that bx = ay.
Solution: Let P (x,y) ,Q (a + b, a- b) and R (a- b, a +b) be the given points.
Then PQ = PR (given)
~ ~[x - (a + b)f + [y - (b - a)f = ~,.....[x---(a---b-)]-::-2-+-[y---(-b+-a-)-f

~ [x-(a+b)f +Lv-(b-a)f = [x-(a-b)f +[y-(b+a)]2


~ ~-2x(a + b)+(a+ by2+y-2y(b-a)+(b-a)2
=~+ (a-bi-2x(a-b)+y-2y(a+ b)+(a+ bi
~ -2x(a + b)-2y(b-a) = -2x(a-b)-2y(a+b)
~ ax + bx + by - ay = ax - bx + ay +by
~ 2bx = 2ay
~ bx = ay
Example 5: Find the value a/x, if the distance between the points (x, -1) and (3,2) is 5.
Solution: Let P(x, -1) and Q(3, 2) be the given points. Then
PQ = 5 [Given].

~(x_3)2 +(-I-2i = 5
(x-3i+9 = 52
(~-6x + 18) = 25
x 2 -6x-7 = 0
(x-7)(x+ 1) = 0
x =7 or x=-l.

• COLLINEAR POINTS
Three points A, B, C are said to be collinear if they lie on the same straight line.

Test for collinearity of three points

STEP KNOWLEDGE
In order to show that three given points A, B, C are collinear. We find distances AB, BC and
AC. If the sum of any two of these distance is equal to the third distance then the given
points are collinear.

~~~~~~~I SOLVED EXAMPLES I~~~~~~


Examples 1: Using distance formula, showthatthe points (-3, 2), (1, -2) and (9, -10) are
collinear .
Solution:-A (- 3,2), B (1, -2) and C(9, -10) are the given points.

AB = ~(1+3)2 +(_2_2)2 = .J16+16 = 4J2 units.


System of Coordinates 353

BC = ~(9-I)2 +(_10+2)2 = J64+64 = sJ2 units.

AC = ~(9+3)2 +(-10_2)2 = ,.1144+144 = 12J2 units.

Now AB + BC = 4J2 + sJ2 = 12J2 = AC.


Therefore, the three points are collinear.
Example 2: Show that the points (1, 1) , (- 2, 7) and (3 ,-3) are collinear.
Solution: LetA(l,I), B(-2, 7) and C(3, - 3) be the given points.
Then we have

AB = ~(-2 _1)2 + (7 _1)2 = J9+36 = 315 .


BC = ~(3+2)2+(_3_7)2 = ,.125+100 = 515.

and AC = ~(3_1)2 +(-3-li = ,.14+16 = 215


Clearly BC = AB+AC.
Hence, A, Band C are collinear

1. Find the distance between the points


(i)A(7,13),B(l0,9) (ii) P(-4,7)andQ(l,-5).
2. Find the distance of the point P(6 , -6) from the origin.
3. Find the value or values of k for which the distance between the point A (k, -5)
and B(2 ,7) is 13 units.
4. Prove that the points A(- 3,0), B (1, - 3) and C(4, 1) are the vertices of an isosceles
right angled triangle. Find the area ofthis triangle.

S. Prove that the points A (a, a),B(-a -a), C(- ~,~ ) are the vertices ofan equilateral
triangle. Calculate the area of this rectangle.
6. Provethatthe pointsA(l,-3), B(13, 9), C(lO, 12),D(-2, 0) taken in order are the angular
points ofa rectangle. Find the area of the rectangle.
7. Prove thatthe pointsA(I, 1), B(-2, 7) and C(3, -3) are collinear.
8. If P(2, -1), Q(3, 4), R(-2, 3) andS(-3 ,-I) be four points ina plane. Show that PQRSis
a rhombus but not a square. Find the area of the rhombus.

HINTS TO THE SELECTED PROBLEMS

S . LetA (a, a), B(-a, a) and C(-Jia, Jia)


AB = 2 J2 a units, BC = 2 J2 a units,
AC =2J2 a units, AB=BC=AC=2 J2 a
354 Remedial Mathematics

area of ~ABC = -J3. J3 r;;


(slde)2 = - x (2,,2 a)2
4 4
= (2 J3 a2) sq.units.
6. AB = 12 J2 units
BC = 3 J2 units, DC = 12 J2 units
AD = 3 J2 units
AB = DC&BC=AD,AC=3J34 units
BD = 3 J34 units,AC=BD.
Hence ABCD is a Rectangle.

ANSWERS

1. (i) 5 units (ii) 13 units 2. 6J2 units 3. k = - 3 or k = 7


2
4. 12. 5 sq units 5. (2.fja ) sq units. 8. 24 sq units.

III SECTION FORMULA


Theorem : The coordinates of the point P(x , y} which divides the line segment joining
A(x J, YJ} and B (x2' Y) internally in the ratio m .' n are given by
mx;+nx, mY2+ny,
x = ,y=
m+n m+n
Proof: LetX' OXand YOY' be the coordinate axes.
LetA(xpYI) and B(x2,y 2) be the end points of the given line segmentAB.
Let P(x, y) be the point which divides AB in the ratio m : n
Y

I
I
I
I
I
I
(x, y) , :
p ----------~ S
I I
I I
I I
I I
I I
I I
(x1. Y1) : :
A I I
I -----------1 R :
I I I
I I I
I I I
I I I
I I I
x'-----r----~------~~------~-----x
o L N M

Y'
Fig. 8.7
System of Coordinates 355

AP m
Then
PB n
Draw AL 1. OX; BM 1. OX; PN 1. Ox.
AR 1. PN;PS1.BM.
Now AR = LN=ON-OL=(x-x l )
PS = NM= OM-ON=(x2 -x)
PR = PN--RN=PN-AL = (y-y\)
BS = BM-SM=BM-PN=Y2-y
Clearly, MRP and APSB are similar and therefore, their sides are proportional.
AP AR PR
-=-=-
PB PS BS
m = x- XI = Y - YI
n x2 -x Y2 - Y
m = x - XI and!:!!.. = Y - YI
n X2 -x n Y2-Y
mx2 -mx = nx- nx\ and mY2-my = ny-nYI
(m +n)x = mx2 +nx\ and(m + n)y=nY2+ny\
mX2 +nxI mY2 +nYI
x = y=
m+n m+n
Hence, the coordinate of P are (m X2 + n x\ , mY2 + ny\ ).
m+n m+n

• MID POINT FORMULA


The coordinates of the midpoint M on a line segment AB with end points A(xl' Y\) and

B(x Y ) are (X\ +x2 Y\ + Y2)


2' 2 2' 2

Remarks
• The coordinate of the point P, which divides the line - segment joining A(xl,y\) and
B(x I ,Y\) and B(x2 'Y2) internally, in the ratio k: 1 are given by

(
kx2 +x\ ,ky2 + Y\).
k+l k+l
• The coordinate of the point which divides the line segment joining the points
(x \ ,Y I) and (x 2 'Y2) externally in the ratio m : n are given by
mx2 - nx\ _ mY2 - ny\
x= ,~- .
m-n m-n
356 Remedial Mathematics

~~~~~~~I SOLVED EXAMPLES ~I~~~~~~


Example 1: Find the coordinates of the point which divides the line segment joining the
points (6, 3) and (- 4 , 5) in the ratio 3:2 (i) internally and (ii) externally.
Solution: Let P (x ,y) be the required point. 3 2
AI I I B
. (i) for Internal division, we have (6,3) P (x, Y) (-4. 5)

x= 3x-4-4+2x6 d 3x5x5+2x3
3+2 ' an y= 3+2
21
x=O,andy = - .
5
So the coordinates of Pare (0, ~1)
(n) for external division, we have
3x-4-4-2x6 3x4-2x6
x= and=----
3-2 3-2
x =-24 andy=9.
- - - - - - - - . 3 ••- - - - - - -
- - - -•• 2 ••- - - -

A B (-4, 5) p
(6,3) (x, Y)
Fig. 8.8
So the coordinates of point (- 24, 9).
Example 2: In what ratio does the point (-2, 3) divide the line segment joining the points
(- 3 , 5) and (4, - 9) .
Solution: Let the required ratio be k: 1
k
A p B
(-3,5» (4, -9)

Compairing x-coordinate
kx4+1x(-3) =-2
k+l
4k-3 =-2 ::::>4k-3=-2k-2
k+l
::::> 6k = 1 ::::> k = 1/6.
Compairing y-coordinate
kx(-9)+(l)x5 = 3
k+l
-9k+5
::::> = 3 ::::>-9k+5=3k+3
k+l
::::> 12k = 2 ::::> k = 1/6.
Hence, the required ratio is 1 : 6.
System of Coordinates 357

Example 3: In what ratio does the y-axis divide the line segment joining the point (-4, 5)
and (3, -: 7) ?
Solution:
k=/~ I
I
A P (0. y) B
(--4. 5) (3.-7)
Fig. 8.9
The line segment joining the points A(- 4, 5) and (3, - 7) is divided by the y axis at the
point P(O,y) in the ratio k: 1.
Compairing x-coordinate, we have
kx3+lx-4
---- = 0=>3k-4=0=>3k=4=>k=4/3.
k+1
Therefore the required ratio is 4 : 3.
Example 4: Find the ratio in which the line 3x + y - 9 = 0 divides the line segment joining
A (1, 3) and B (2, 7).
Solution:

A B
(1. 3) (2.7)
Line
3x+y-9=0
Fig. 8.10
The equation ofthe given line is
3x+y-9 = O. ...(1)
meets the line segment joiningA(I, 3) and B(2 , 7) at the point P (x , y) and divides the segment
internally in the ratio k: I.
. 2k+1 7k+3
By sectIOn formula: x = --, y = - -
k+1 k+1
. '
I.e., CoordmateofPare= (2k+1
- - ,7k+3)
-- .
k+1 k+1
The point P lies on the line whose equation is given by (I)

Therefore, (2k+I)+(7k+3) =0.


k+1 k+1
=> 6k+3 +7k+3-9k-9 = 0=>4k-3=0
=> k = 3/4
Hence, the required ratio is 3 : 4
Example 5: Find the coordinates o/point which divides the line joining point (1, 2) and
(-3 ,4) in the ratio 2 : 3 internally. [RGPV B. Pharma 20051

".
358 Remedial Mathematics

Solution: LetA (1,2) andB(- 3.4) be the given points.


Point P divides A and B in the ratio 2 : 3
PA :PB = 2:3
.
:. Coordmates of Pare (2X-3+3XI ,2X4+3X2)
----
2+3 2+3

53, 8; 6) i.e., ( -; , I;).


i.e., ( -6 +

Example 6: IfA (- J, - 3), B(1 ,-J) and C(5, J) are the vertices ofa triangle ABC, find the
length of median through A. rUPTU B. Pharma 200S1
Solution: Let D be the middle point of BC. A (-1, 3)
Then AD is the median through A and D is
'
th epomt (I
+ 5 -1 + 1),I,e., (3 ,0)
-2-'-2-

AD = ~{3 _(_1)}2 +{O-3}2 B o C


(1, -1) (5,1)
=~42+(_3)2 =5. Fig. 8.11

Example 7: A quadrilateral has the vertices at the points (- 4 , 2) , (2, 6) ,(8, 5) and
(9, - 7). Show that the mid points of the sides of this quadrilateral are the vertices of a
parallelogram. [UPTU B. Pharma 2004]
Solution: LetABCD be the given quadrilateral with vertices A (-4,2), B(2, 6), C(8, 5) and
D (9, - 7). Let E, F, 0 and H be the mid points of the sides AB, BC, CD and DA respectively.
2 6
Then the coordinates of E are ( -4 + , 2; ) i.e., (- 1,4).
2
,.
The coordmates of Fare (2+8 6+5).I.e., ( 5'2
-2-'-2- 11) .

The coordinates of 0 are (8;9, 5;7}.e.,C; ,-I}


and the coordinates of Hare (-4 +9, 2;7}.e,,(%, ~}
2

Now, the coordinates of the mid point of EO are ( -1: I; , Il·. .c: ,n
4;

and the coordinate of the mid point of FH are (s: t,'; ; %} .•..c: ,n
Thus we see that the diagonals EO and FH of the quadrilateral EFGH bisect each other.
Hence EFGH is a parallelogram
Example 8: Three consecutive vertices ofa parallelogram are A(1 , 2), B (1, 0) and C(4, 0)
find the fourth vertex D.
System of Coordinates 359

Solution: Let the coordinates of the vertex


D be (x ,y) Diagonals AC and BD of the parallelogram ABCD bisect each other at M.
i.e., M is mid point of AC as well as of BD. o (x, y) C (4, 0)
~-----..,.

Midpointof AC=C~4,2;0)=(%,I) ... (1)

Mid point of BD = (x; I, y;O) . .(2)


(1) and (2) are same coordinates of the point M.
A(1.2) B (1,0)
Then x + 1 = ~ and l = I Fig. 8.12
2 2 2
x = 4 andy = 2 Therefore, the coordinates of Dare (4,2).

I EXERCISE 8.21
1. Find the coordinates of the point which divides the line segment joining the points
A(4, - 3) and B(9, 7) in the ratio 3 : 2
2. Find the coordinates of the mid point of the line segment joining the points A(- 5,4)
andB(7,-8)
3. Find the ratio in which the point P(m, 6) divides thejoiningA(-4, 3) and B(2, 8). Also,
find the value of m.
4. In what ratio does point P(2, - 5) divide the line segment joining A (-3,5) and (4, -9).
S. In what ratio is the line segment joining the point A (6, 3) and B (- 2, - 5) divide by the
x-axis.
Also, find the coordinates of the point of intersection of AB and the x - axis.
6. Find the ratio in which they-axis divides the line segment joining the points A(-4, 10)
andB(7,-I).
Also, find the coordinates of their point of intersection.
7. The coordinates of one end point of diameter AB of a circle are A(4. - I) and the
coordinates of the center of the circle are C(I. - 3). Find the coordinates of B
8. The three vertices ofa parallelogramABCD, taken in order are A(l, -2), B(3, 6), and
C(5, 10) . Find the coordinates of the fourth vertex D.
9. Find the lengths of the medians of a d ABC whose vertices are A(7. - 3) B(5, 3) , and
C(3,-I)
10. Let D(3, - 2), E(- 3, 1) and C(4, - 3) be the mid points of the sides BC, CA and AB
respectfully of d ABC. Then, find the coordinates of the vertices A, Band C.

I ANSWERS I
-2
1. (7,3). (3, 5) 2. (1.-2) 3. m=- 4. 5:2
5
5. (3,0) 6. 4:7,(0,6) 7. (2. -5) 8. (3.2)
9. 5,5. JI6 10. //(-2.0). B(10. -6), C(-4. 2)
360 Remedial Mathematics

• AREA OF A TRIANGLE
Theorem: The Area of a MBC with vertices A(x/,y/), B(x 2, Y} and C(Xj ,y) is given by

area (MBC) = 11[X/(Y2 - Y3)+ x2(Yj - Y/)+ Xj(Yl - Y2)]/

Proof: LetA(x),y), B(x2 'Y2) and C(x3' x 3) be the vertices of the given MBC.
Draw AL, BM and CN perpendiculars to the x-axis.
Then ML = (x) -X2) , LN= (x 3 -x) and MN = (x 3 -x2)
Y

sf-----+-----------"'!C
~2,y~ : ~~y~
I

x'------~----~----~----------~-------x
M L N

Y'
/ig.8.13
Area oft). ABC
= area (trap. BMLA + area (trap. ALNC) - area (trap. BMNC)

= [~(AL+BM)XML ] + [~(AL+CN)XLN ]- [~(BM+CN)XMN J.


I I I
= 2 (y) + Y2)(X\ -X2) + 2 (y) + Y3)(X3 -XI) -2 (Y2 + Y3)(x 3 -x2)
= 2I [XI (y\ +Y2- Y \-Y3)+ X2 (Y2 + Y3 -Y\ -Y2) + X3 (y\ +Y3 -Y2- Y 3)]
I
= 2 [X\(Y2-Y3)+X2 (Y3-Y\)+X 3 (Y\-Y2)]
Since, the area is never negative, we have
1
area (MBC) = -Ix\ (Y2 - Y3)+X2(Y3 - yl)+X3(Y\ - Y2)1·
2
Remarks
• The area of a triangle is always taken as positive real quantity. Some times the result
from the area formula given negative value in that case we reject the negative sign .
• The three points (xI'Yt), (x 2'Y2) and (x3 'Y3) are collinear i.e., in a line ifx t (Y2 - Y3) +
x 2 (Y3 - Yt) + x3 (y\ - Y2) = 0 because in this situation the area of the triangle reduces
to zero.
System of Coordinates 361

~~~~~~~I SOLVED EXAMPLES ~~~~~~~ I


Example 1: Find the area ofthe triangle whose vertices are (4,3), (5,4) and (1 1,2).
Solution: A (4, 3), B (5,4) and C (11, 2) are the three vertices of the given triangle.
xI =4,x2 =5,x3 =11
YI = 3'Y2=4'Y3=2

Area of t1ABC = 21 [XI (Y2 - Y3) + x 2 (Y3 - Y2) + x3 (Yl- Y2)


1
= - [4(4-2)+5(2-3)+11(3-4)]
2
1
= 2[8-5-11]=-4

Rejecting negative sign, we have the area of the given triangle equal to 4 square units.
Example 2: Find the value of k so that the point A (- 2 , 3) , B (3, - I) and C (5, k) be
collinear.
Solution: We have Xl =-2,x=3,x3 =5
YI =3'Y2=-I'Y3=k.
1
Area of t1ABC = 2 [Xl (Y2 - Y) + X2 (Y3 -Yl) + X3 (Yl- Y2)]

= 21 [-2(-I-k)+3(k-3)+5(3+1)]
1 1
= - [-2+2k+3k-9+20= - [5k+9]
2 2
Now, the three points are collinear if the area of flABC = 0
1
i.e., -(5k+9) =0
2
5k+9 = 0
k =-9/5.
Example 3: Find the area of the quadrilateral ABCD whose vertices are respectively
A (J,J), B (7, - 3), C (12,2) and D (7,21).
Solution: Area of quadrilateral ABCD = IArea of (flABC)1 + IArea of flACDI
1 1
Now Areaoft1ABC = 211x(-3-2)+7(2-1)+12x(l+3)1 = 21-5+7+481
= 25 sq units
1
Areaoft1ACD = 211x(2-21)+12(21-1)+7(l-2)1

= .!.1-19+240-71 = 107 sq. units.


2
:. AreaofquadrilateralABCD=25 + 107 = 132 sq. units.
362 Remedial Mathematics

Example 4: For what value o/k the points (k, 2 - 2k) (-k + 1 ,2k) and (- 4 - k, 6 - k) are
collinear.
Solution: Let the three points be A(x l , YI) !II (k, 2 - 2 k), B(x2 'Y2) !E ( - k + I. 2 k) and
C(x3'Y3). (-4 -k, 6- 2 k).
If the given points are collinear, then
xI (v2 - Y3) + x2 (v3 - YI) +x3 (vI - Y2) = 0
=> k(2k-6+2k)+(-k+ 1)(6-2k-2+2k)+(-4-k)(2-2k-2k) = O.
=> kC4k-6)-4Ck-I)+C4+k)(4k-2) = 0
=> 4~-6k-4k+4+4~+ 14k-S = 0
=> S~+4k-4 = 0
=> 2~ + k-l = 0
=> (2k-l)(k+ I) = 0
=> k = 1/2ork=-1.
Hence, the given points are collinear for k = 112 or k = - I.
Example 5: If the vertices 0/ a triangle have integral coordinates prove that the triangle
cannot be equilateral.
Solution: LetA (xl'YI)' B (x2 'Y2) and C(x3 'Y3) be the vertices of triangle ABC, then the area
of ~ ABC is given by

~ = '21 [xI (v2-Y3)+x2 CY3-YI)+x3 (vI-Y2)]


= A rational number
IfpossibJe let the triangle ABC be an equilateral triangle, then its area is given by

~= Jj Cside)2 = Jj CAB)2
4 4

=
Jj
""4 x
.. num ber
a positive
= an irrational number
This is a contradiction to the fact that the area is a rational number. Hence. the triangle
cannot be equilateral.
Example 6: Prove that the/ollowing points are collinear,' (-3, 0) ,(0, -9) and (-2, -3)
[RGPV B. Pharma 20041
Solution: The given points will be collinear if the area of the triangle formed by these points
is zero. Now area of the triangle
1
= 2'[x 1 (v2-Y3)+x2 (v3-YI)+x 3 (vI-Y2)]

= .!.[-3C-9+3)+O(3-0)+C-2)(O-C-a)]
2
= .!. [IS - IS] = 0
2.._ ~_-
System of Coordinates 363

Example 7: The co-ordinates a/vertices Band C a/triangle are (/, - 2) , (2, 4) lies on the
line 2x + Y- 2 = O. The area a/the triangle is 8 units. Thenfind the vertices coordinates
0/A. [RGPV B. Pharma 2001 I
Solution: Given points are B( 1,2), C(2, 3)
and A(x ,y) line on the line 2x +y - 2 = 0
The coordinate of A are (x, 2 - 2x)
1
Area of A ABC = '2 [xI (Y2 - Y3) + x2(Y3 - YI) + x3 (YI - Y2)
1
:1:8 = '2[x(-2-3)+ 1(2-(2-2x)+2(2-2x+2)]

:1:16 =[-5x+I+2x+8-4x]
:1:16 =-7x+9
16-9 = -7x (taking positive sign)
7 = -7x.
x = - 1 put x = - 1 iny = 2 - 2x and gety = 2 + 2 = 4 .
.'. coordinate of A are (- 1, 4) . Taking negative sign - 16 = - 7x + 9

=> -16-9 = -7x=>x = 2:


put x = 25/9iny=2-2x
25 18-50 32
we get y = 2-2 x - = - - = -

:. coordinate of A are e: '3:) 9

or (-1,4)
9 9

Example 8: Find the area o/the triangle whose vertices are (0, 5) ,(2, 3) and (4, 5)
IUPTU B. Pharma20021
Solution: Here xI =O'YI = 5,x2 =2 'Y2 = 3,x3=4'Y3 = 5.
:. area of the triangle

= ~[(XtY2+X2Y3+X3 Yt)-(YtX2+Y2x3+Y3Xt)]·
= 1. [(0 x 3 + 2 x 5 + 4 x 5) - (- 5 x 2 + 3 x 4 + 5 x 0)]
2
= 1.[(0+ 10+20)-(10+ 12 +0)]
2

= 1. [30-22] = 1. x 8 =4 sq. units


2 2
Example 9: Prove that the points (a,b + c) , (b, c + a), (c, a + b) are collinear.
[UPTU B. Pharma 2001, 071
364 Remedial Mathematics

Solution: Here x\ = a, y\ = b + e, Y2 = e + a, x3 = e, Y3 = a + b, x 2 = b
Now area of the triangle formed by the given points:
1
2" [(xI Y2 + x 2Y 3 + x3 Y\) - (y\ x 2 + Y2 x3 + YJ xI)]

1
= -[{a(e + a) + b (a + b)+e (b + c)} - {(b + c) b+ (e+a) e +(a + b)a}]
2

= .!..[ae + ~+ab + b 2 +be + e2-b2-be+e2-ae-~-ab] =0.


2
Hence, the given points are collinear.
Example 10: Find the area ofa triangle formed by the lines:
Y = 2x, y = x and y = 3x + 4. [UPTU B. Pharma 20011
Solution: Let the equations of the sides AB, BC and CA of ~ ABC be y - x = 0, y - 2x = 0 and
y - 3x - 4 = 0 respectively.
Solving these equations in pairs, the coordinates of A, Band Care (-2, -2), (0, 0) and
(-4, -8) respectively.
1
Area of ~ABC = 2"[x\ (Y2- Y 3) + x 2 (Y3 -y\)+x3 (Y\-Y2)]'

= 2"1 [(-2)(0+8)+0.(-8+2)+(-4) (-2-0)]


1
= -[-16+8]
2

= .!.. [- 8] = 4 sq. units, neglecting the negative sign.


2
Example 11: Four points A (6, 3), B(- 3, 5), C(4, - 2) and D(x, 3x) are given is such a way
I'lDBC 1
I'l ABC = 2"' find x. [UPTU B. Pharma 20061

.!..[x(5 + 2) -3(-2 - 3x) +4 (3x- 5)]


Area of ~DBC 2
Solution:
Area of I'lABC 1
2[6(5 + 2) -3(-2 -3) +4(3 - 5)]

1 7x+6+9x+12x-20
2 42+15-8
28x-14
2 49
4x-2
----
2 7

7 = 4x-2
2
11
x =-
8
System of Coordinates 365

~~~~~~~I EXERCISE 8.3~1~~~~~~~


1. Find the area otthe triangle whose vertices are A (2, 7) B (3, - 1) and C(-5, 6)
2. Find the value of k for which the area formed by the triangle with vertices A (k, 2k),
B (- 2,6) C (3, 1) is 5 square units.
3. Showthatthe points A (- 1,1), B (5, 7) and C(8, 10) are collinear.
4. For what value of K are the points A (1, 5) , B (k, 1) and C (4, 11) collinear.
5. If the vertices of a triangle are A (1, k) , B (4, -3) and C (- 9, 7) and its area is 15
sq.units, these find the value of K.

ANSWERS
1. 28. 5 sq. units. 2. K=2, K=2/3
21
4. K=-1 5. K=-3 or K=-
13

III LOCUS AND EQUATION TO ALOCUS


Locus: The curve described by a point which moves under given condition or conditions is
called its locus.
For example:
(i) Suppose C is a point in the plane of the paper and P is a
variable point in the plane of the paper such that its
distance from C is always equal to resay). Obviously all
the positions of the moving point P lie on the
circumference of a circle whose radius is r. The
circumference of this circle is therefore the Locus of the
point 0 when it moves under the condition that its
distance from point C is always equal to constant r.
Fig. 8.14
Equation of the locus of a point
The equation of the locus of a point is the relation which is satisfied by the coordinates of
every point on the locus of the point.

STEP KNOWLEDGE
Step 1: Assume the coordinates of the point say (n, k) whose locus is to be found.
Step 2: Write the given condition in mathematical form involving n, k
Step 3: Eliminate the variables, if any
Step 4: Replace h by x and k by y in the result obtained in step 3. The equation so obtained
is the locus of the point which moves under some stated conditions.
366 Remedial Mathematics

~~~~~~I SOLVED EXAMPLES ~I~~~~~~


Example 1: Find the locus of a points P such that the sum of the squares of abscissa and
ordinate is equal to the product of abscissa and ordinate.
Solution: Let P(h, k) be any point on the locus.
:. h is abscissa and k is ordinate of P.
By the given geometrical condition, we get
rt-+JC2= hk
Hence, locus of(h, k) is x2 + I = xy
Example 2: Find the equation to the locus ofapointequidistantfrom the points A (1, 3) and
B (-2,1).
Solution: Let P (h, k) be any point on the locus .Then
PA = PB (given)
=> PA 2 = PB2
=> (h-I)2+(k-3f= (h+2f+(k-If
=> 6h+4k = 5
Hence, locus (h, k) is 6x + 4y - 5.
Example 3: Find the equation of the locus of a point which moves so that the sum of its
distances from (3, 0) and (-3, 0) is less than 9.
Solution: Let P(h, k) be the moving point such that the sum of its distance from (3,0) and
(- 3,0) is less than 9.
Then PA+PB< 9.

~(h-3)2+(k-0)2 +~(h+3)2 +(k--;O)2 <9


=> ~(h_3)2+k2 < 9-~(h+3)2 +k2

=> (h- 3) + JC2 < {9 - ~(h-3)2 +k2 r


=> (h-3f+JC2 < SI +(h+3)2+JC2-1S~(h+3i+k2
=> -12h-Sl < -IS ~(h+3i +k2
=> 4h +27> 6 ~(h+3)2 +k2
=> (4h+27f > 36 [(h+3)2+JC2]
=> 2
16h +216h+729 > 36(h2 + 6h+9+JC2)
=> 20h2 + 36JC2 < 405
Hence, locus of(h, k) is 20x2 + 361 < 405.
Example 4: A point moves so that the sum of its distances from (ae, 0) and (- ae, 0) is 2a,
prove that the equation to its locus is
2
x
2+2
i = 1 whereb 2 =~ (J -~).
a b
System of Coordillates 367

Solution: Let P (h, k) be the moving such that the sum of its distance from A(ae, 0) and
B(-ae. 0) is 2a.
Th~ M+n=~

=> ~(h-ae)2 +(k-O)2 +~(h+ae)2 +(k -0)2 = 2a

=> ~(h-ae)2 +k2 = 2a- ~(h+ae)2 +k2


=> (h- ae) + ~ = 4if + (h + ac)2 + ~-4a ~(h+ae)2 +k2
(squaring both side)

-4aeh-4if = -4a~(h+ae)2 +k2


(eh + a) = ~(h+ae)2 +k2
(eh + a)2 = (h + aei + ~
e 2h2 + if + 2aeh = h 2 + ife2 + 2aeh + ~
h2 (1- e2) + ~ = if (l-e2)
h2 k2
-+
2 2 = 1
a a (1-i)
Hence, locus of(h, k) is
h2 k2
-+
2 2 = 1
a a (1-i)
x2 y2
or -+- = 1 whereb 2 =a2 (I-e2)
~ b2 '
Example 5: A rod of length I slides with its ends on two perpendicular lines find the locus
of its mid - point.
Solution: Let the two perpendicular lines be the coordinate Axes. Let AB be a rod of length
I. Let the coordinate ofA and B be (a, 0) and (0, b) respectively. As the rod slides, the values
of a and b change. So a and b are two variables.
y

8(0, b)

--~O+-------------~A----~X

(a. 0)

Fig. 8.15
368 Remedial Mathematics

Let P(h, k) be the mid point of the rod AB in one of the infinite position it attains. Then
a+O O+b a b
h and k = - -
= -- ~ h= - and k = - ... (1)
2 2 2 2
From Ll OAB , we have AB2 = OA 2 + OB2
=if+ b2 =P
(2h)2+ (2ki =P (from (1))
~ 4h2+4~ =p
Hence, the locus of (h, k) is 4x2 + 4; = P.
Example 6: If 0 is the origin and Q is a variable point on ~ = 4y. Find the locus ofthe mid
pointofOQ.
Solution: Let the coordinates of Q be (a, b) and let P(h, k) be the mid point of OQ. Then
a+o a o+b b
h = - - = - andk= - - = - ~a=2handb=2k. ... (1)
2 2 2 2
Here a and b are two variables which are to be eliminated. Since (a, b) lies on
x 2 = 4y.
Therefore, a 2 = 4b ~ (2h)2 = 4 (2k)
~ h 2 = 2k [using (1)]
~ h 2 = 2k.
Hence, the locus of(h,k) is x 2 = 2y.
Example 7: A point moves so that its distance from (3, 0) is twice the distance from
(-3, 0). Find the equation of the locus. [UPTU B. Pharma 20051
Solution: Let A represent the point (3,0) , B the point (-3,0). Further, point ( -3,0) and
P(h, k) be the moving point.
According to the question:
PA = 2PB
or (PAP = 4 (PBi
[(h - 3)2 + (k- 0)2] = 4[(h + 3i + (k- of]
~ h 2 + 9 - 6 h + ~ = 4h2 + 36 + 24h + 4~
2
~ 3h + 3~+30h+29 = 0
Hence, the required locus is 3x2 + 3; + 30x + 27 = O.
Example 8: Find the locus of a point such that the line segments having end points (2,0) ,
and (- 2, 0) subtend a right angle at that point. [UPTU B. Pharma 2006]
Solution: Let A(2, 0) and B(- 2, 0) be the given points and P(h, k) be the variable point.
According to the question
L.APB = 90°
P(h, k)
:. i.e., MPB is a right angle.
AB2 = PA 2 + PB2
[2 -(-2)f + [0-0] = [(2 -hi + (0 _k)2] + [(-2 -hi + (0 _k)2]
16 = (2-hi+~+(-2-hi+~
16 = 4 + h 2 - 4h + 2~ + 4 + h2 + 4h
16 = 2h2+2~+8
A(2.0)
h2+~ = 4.
Hence, the required locus is ~ + ; = 4. Fig. 8.16
System of Coordinates 369

Example 9: Find the equation to the locus of a point which moves so that the sum of its
distance from (3, 0) and (- 3,0) is less then 9. IUPTU B. Pharma 2003, 041
Solution: LetA(3, 0) andB(- 3,0) be the two given points and (h, k) be the coordinates of the
moving point P whose locus is to be found. According to the question
PA+PB <9

or ~(h-3)2 +k2 +~(h+3)2 +k2 < 9

~(h_3)2 +k2 < {9-~(h+3)2 +k 2 }


Squaring both sides, we get

(h-3)2+~ = <81-18 ~(h+3)2+k2 +(h+3)2+k

-12h-81 <-18~(h+3)2+k2
or 4h+27 > 6 ~(h+3)2 +k2
Again squaring both the sides, weqet
16~+729+216h > 36[h2+9+6h+~]
20~ + 36~ < 405.
The required locus of the point (h, k) is 20x2 + 36y < 405.

1. Find the equation to the locus of a point equidistant from the points A(1, 3) and
B(-2, I).
2. A = (2, -2), B is a point on the locusy= 4x. Find the equation ofthe locus ofapoint
which divides AB internally in the ratio I: 2 .
3. A = (-3, 0) and B(3, 0) are two given points. Find the equation of the locus of point
P such that PA + PB = 10.
4. If B is (4, - 3), qo, 2) and point A lies on the locus y = 1 + x. Then find the equation
oflocus of centroid oftriangle ABC.
5. Find the equation of the circle having segmentAB as a diameter where A (-1,4) and
B(3,-2)
6. Find the equation of the locus of a point such that the sum of the squares of its
distance from (5, -3) and (2, -2) is 20.
7. A = (0, 5) and B (0. -5). Find the equation of the locus of point P such that
PA-PB=4.
8. A (1,2) andB (4, -5) are two vertices of dABC. Find the locus of the third vertex C
if the centroid lie on the locus 2x + 3y = II.
370 Remedial Mathematics

ANSWERS

x2 y2
1. 6x+4y=5 2.91-12x+24y+32=0 3. -+-=1
25 16
4. 9x2_24x-3y+ 16=0 5. ~+1-2x- 2y-11=0

6. ~+21-14x+8y+ 19=0
_x 2
7. - + - = 1
i 8. 2x+ 3y = 32.
21 4

OBJECTIVE EVALUATION

MULTIPLE CHOICE QUESTIONS


Choose the most appropriate one:
1. Value of k for which (8, I), (k, -4) and (2, -5) are collinear is
(a) k=2 (b) k=-3
(c) k = 3 (d) k = -2.
2. The ratio is which y-axis divides the join of(5,-6) and (- I, - 4) is
(a) 1:3 (b) 1:5
( c) 5: I (d) 3: 1.
3. AB is diameter of a circle whose center is (2, - 3). If coordinates of B are (I, 4) then
co-ordinates of A are.
(a) (3,-10) (b) (-10,3)
(c) (5,2) (d) (- 3, 10)
4. A point on y-axis equidistant from (6,5) and (- 4, 3) is
(a) (9,0) (b) (0,3)
(c) (0,9) (d) (0, - 9).
5. The area of triangle PQR where coordinates of P. Q and R are respectively (4, 5),
(I, - 6) and (-4, -5) is
(a) 38 sq. units (b) 19 sq.units
19 .
(c) - sq.umts (d) None of these
2
6. The value of P for which (- 5, I), (I, P) and (4, - 2) are col1inear is
(a) p=-I (b) p=-2
(c) p = \ (d) None ot these.
7. The points (1,7). (4.2), (-\, \) and (- 4,4) are vertices ofa
(a) square (b) P. Rhombus.
(c) rectangle (d) parallelogram.
8. Area of MBC, where A(2, 3), B(- 2, I) and C(3, - 2) is
(a) 10 Sq. units (b) 22 Sq.units
(c) \\ Sq.units (d) 24 Sq. units.
9. IfA(5, 3), B(1I, - 5) and P(12 ,y) are vertices ofa right angled triangle, right angled atPtheny is
(a) -2,4 (b) -3,4
(c) 2,-4 (d) 2,4.
System of Coordinates 371

10. In the figure 8.17 if AOBA is equilateral y


then coordinates of vertex B are
(a) (a. a) B
(b) (a.2a)
(c) (a. lia)

(d) ( -a,-lia).
A
11. Value ofp for which (- 5, I), (I, p) and X' 0
X
(4, -2) are collinear (2a.0)

(a) p=1 y'


(b) p=-I Fig. 8.17
(c) p=2
(d) None of these.
12. If the point (2, 3) lies on the locus whose equation is y = 2x + c, find C.
(a) I (b) - I
(c) 0 (d) 2.
13. If A = (a. I) is a point on the locus whose equation is 4x + 3y = 5, find a.
(a) II2 (b) 3/2
(c) 213 (d) 4/5.
14. Find the equation ofthe locus of points P, such that distance of P from the (-1,2) is equal to 3.
(a) (x_I)2+(y +2i=7 (b) (x+I)-(y~2)2=5
(c) (x+I)2+(y_ 2)2=9 (d) (x-I)2+(y+2)2=9.
15. Find the equation of the locus of points P, such that twice the ordinate ofP execeeds three times
its abscissa by 5.
(a) 3x-2y + 5 (b) 2x-3y+5=0
(c) 3x+2y-5=0 (d) None of these

TRUE I FALSE
1. The distance of point (5, 3) from origin is 5 units. (T/F)
2. Ifthe vertices of a triangle have integral co-ordinates then the triangle cannot be equilateral.
(T/F)
3. The ratio in which (4, 5) divides the join of(2, 3) and (7,8) is 2: 3. (T/F)
4. The abscissa and ordinate ofa given point are the distances of the point fromy- axis respectively.
(T/F)
5. The distance between the points (cose ,sine) and (- cose ,- sin e) is J2. (T/F)
6. If the points (Ie, 2k), (3k, 3k) and (3, I) are collinear, then k = II3. (T/F)
7. If three points (0,0), (3, Ii) and (3, A.) form an equilateral triangle this A. is equal to (T/F)

FILL IN THE BLANKS


1. Coordinates of any point lying on x-axis are _ __
2. The distance betweenA(2, 4) and B(-2, 3) is,_ __

3. If(a,O),(O,b)and(1, l)arecollinear,then .!..+.!.. isequalto _ __


a b
372 Remedial Mathematics

4. IfA(-I,p) lies onBC, where B= (-5,3) and C=(O, 3). Thenp is equal to _ __
5. Ifthe points (t, 2t), (- 2,6) and (3, I) are collinear, then t is equal to _ __
6. If centroid of the triangle formed by (7, x) , (y, - 6) and (9 , 10) is at (6,3) then (x, y) is equal to

7. Distance of (I, 2) from the mid point of the line segment whose end points are (6,8) and (2,4)
is _ _ _,
8, Ifareaoftriangleformed by (x, 2.x), (-2,6) and (3,1) is 5 sq.units, then x is equal to _ __

9. ~ (3, 2), B (- 2, 1) are two vertices of I:;. ABC ,whose centered G has the coordinate (~,i).
Then the co-ordinates of the third vertex Care _ __
10. Area oftriangle whose vertices are (1,3), (- 2,4) and (0,6) is,_ __

I ANSWERS
MULTIPLE CHOICE QUESTIONS
1. (c) 2. (c) 3. (a) 4. (c)
5. (b) 6. (a) 7. (b) 8. (c)
9. (c) 10. (c) 11. (b) 12. (a)
13. (c) 14. (a) 15. (a)

TRUE/FALSE
1. False 2. True 3. True 4. True
5. True 6. False 7. True

FILL IN THE BLANK


1. (x, 0) 2. Jl7 3. 0 4. 2
4
5. - 6. (5,2) 7. 5 units 8. 2/3
3
9. (4, -4) 10. 4 sq.units.

I REFRESHER I

Do you know? After reading this chapter you should be able to learn the following
concepts:
• The distance between two points A(xl' Y\) and B(x2 , Y2) is given by
I 2 '
AB= ,,(x2 -xl) +(Y2 - Y\)
2

• Three points are said to be collinear if they lie on the same straight line
• The coordinate of the point P(x,y) which divides the line segmentjoiningA(x\ ,y\) and
B(x2, Y2) internally in the ratio m : n are given by
mx2 + 17Xl mY2 + ny\
x= ,y=
m+n m+n
System of Coordinates 373

• The coordinate of the mid point M ofa line segmentAB with end points A(xj,Yj) and
B(x y) are (XI +x2 YI + Y2).
2' 2 2' 2
• The Area ofa ~ ABC with vertices A (xl'YI)' B(x2'Y2) and C(x3'Y3) is given by

area(~ABC)= 1~[XI(Y2 - Y3)+x2(Y3 - YI)+X3(YI - Y2)]1


• Three points A, Band C are collinear, ifthe area of the MBC formed by these three
vertices is zero
• The equation to the locus of a points is the relation which is satisfied by the coordinates
of every point on the locus of the point.
Can we do? (Frequently Asked Questions)
1. Find the value of a if the distance between the points (3, a) and (4, 1) is .JIO.
[UPTU B. Pharma 2003j
2. Show that the pointsA(2, -2), B(8, 4), C(5, 7), D(-I, 1) are the vertices ofa rectangle ..
[UPTU B. Pharma 2007}
3. If A(-1,3), B(1, -1) and C(5, 1) are the vertices of a triangle ABC. Find the length of the
median through A. [UPTU B. Pharma 2005}
4. The coordinates of the mid points of the sides ofa triangle are (1, 2), (0, -1) and (2, -I).
Find the coordinates of its vertices. [UPTU B. Pharma 2007)
5. A quadrilateral has the vertices at the points (--4, 2), (2, 6), (8,5) and (9, -7). Show that
the mid points of the sides ofthis quadrilateral are the vertices of a parallelogram.
[UPTUB. Pharma 2004)
6. Find the area ofthe triangle whose vertices are (0, 5), (2, 3) and (4, 5).
[UPTU B. Pharma 2007)
7. Prove that the points (a, b + c), (b, c + a), (c, a + b) are collinear.
[UPTU B. Pharma 200], 07}
8. Find the area of a triangle formed by the lines.
Y = 2x,y= x andy= 3x+ 4. IUPTU B. Pharma 20011
9. Four points A(6, 3), B(-3, 5), C(4, 2) and D(x, 3x) are given in such a way that

~ DBC =..!.. findx. IUPTU B. Pharma 20061


~ABC 2'
10. A point moves so that its distance from (3, 0) is twice the distance from (-3, 0). Find the
equation of the locus. IUPTU B. Pharma 20051
II. Find the locus of a point such that the line segments having end points (2, 0) and
(-2,0) subtend a right angle at the point. (UPTU B. Pharma 20061
12. Find the equation to the locus of a point which moves so that the sum of its distances
from (3, 0) and (-3, 0) is less than 9. IUPTU B. Pharma 2003, 041
13. Find the equation of the locus of a point which moves so that its distance from (a, 0) is
equal to its distance form the y-axis. IUPTU B. Pharma 20071
374 Remedial Mathematics

14. Find the coordinates of point which divides the line joining points (1, 2) and (-3, 4) in
the ratio 2 : 3 internally. [RGPVB. Pharma 2005]
IS. Prove that the following points are collinear (-3,0), (9 -9), an<;i{-2, -3).
[RGPV B. Pharma 2004]
16. The coordinates of vertices Band C ofa triangle are (1, -2), (2, 4) and A lies on the line
2x+y-2 = O. The area of the triangle in 8 units. Then find the vertex coordinates ofA.
[RGPVB. Pharma 2001)

DOD
THE STRAIGHT LINES

• INTRODUCTION
A straight line is the locus of all those points which are collinear with two given points. Since,
we know that one and only one line can be drawn from any two given points. So straight line
is a curve such that every point on the line segment joining any two points on it lies on it.
Remarks
• Every first degree in x, y represents a straight line.
• The x-axis and all lines parallel to it are called horizontal lines.
• They-axis and all lines parallel to it are called vertical lines

SLOPE OR GRADIENT OF A LINE


.gonometrical tangent ofthe angle that a line makes with the positive direction ofthe
x - axis is anticlockwise sense is called the slope or gradient of the line.
y y

x' -------o~----~~----x x' ------~o~------~----x

y' y'
Fig. 9.1
The slope of a line is generally denoted by m .Thus m = tan 8.

Remarks
• Slope of any line paraillel to x-axis is zero.
• Slope of any line par:allel to y-axis i.e., perpendicular to x-axis is infinite. Thus not
defined.
376 Remedial Mathematics

• Slope of the general equation of line ax + by + c = is °


a cofficient of x
m=--,=- .
b coefficintof y
• Slope of a line equally inclined with axes is 1 or -1 as it makes 45° or 135° angle with
x-axis.
• The angle of inclination of a line with the positive direction of x-axis is anticlockwise
sense always lies between 0° and 180°.
• When m < 0, tanS < ° i.e., S> 90°. Therefore the angle of inclination is obtuse.
• When m = 0, tanS =°
i.e., S = 0°. Therefore the line is parallel to x-axis.
• When m > 0, tanS> 0, i.e., S < 90° . Therefore the angle of inclination is acute.

• SLOPE OF A LINE THROUGH TWO POINTS


Let L be the line through two fixed points A(xp YI) and B(x2, Y2) respectively. Let us draw
perpendiculars from A on x and y axis respectively so as to meet at M(x p 0) and P(O, y I)' Also
through B perpendiculars on x and y axes meet at N 2(x 2, 0) and Q(O, Y2)'
y

---------------------------
I B (x2. Y2)
I
I
I
I
: Y2-Y1
I
I
I
e______________ 1I
I
X2- X1

----~~~~------~--------------~---x
M
(x1.0)

Fig. 9.2
Now MN=(x 2-x l )=AC and PQ= (Y2-YI) =BC.
Let the line L makes an angle Q with the positive direction of x-axis. Then
tan S = BC => tan S = Y2 - YI
AC x2 -XI

m = tan S = Y2 - y,
X2 -XI
Which is the slope ofthe line joining the points A(x l , YI) and B(x2'Y2)
Remark
• LetA(x, y) and 0 = (0-, 6) = B
0
Then slope ofline AB = Y - = E. = slope of OA.
x-O x
The Straight Lines 377

Condition of Parallelism of Lines


If two lines of slopes m I and m2 are paraJlel, then angle e made by them with positive direction of
x-axis are equal.
So m 1 =m 2 .

Condition for perpendicularity of two lines:


Ifml and m 2 are the slopes of two mutually perpendicular lines
1
then m 1 =- - or m 1m 2 =-1
m2

~~~~~~I SOLVED EXAMPLES ~I~~~~~~


Example 1: Find the slope ofa line which makes an angle of30° with the positive direction
ofx-axis.
Solution: Let m be the slope of the line
1
Then e
m = tan = tan 30° = .J3'
Example 2: Find the slope of the line joining A (3, 4) and B (6, 8)
Y2 - YI 8-4 4
Solution: Slope ofline AB = - - - = - - = -.
x2 -xI 6-3 3
Example 3: Determine x so that the line passing through (3, 4) and (x, 5) makes J35° angle
with the positive direction of x-axis.
Solution: Since the line passing through (3, 4) and (x, 5) makes an angle of 135° with x-axis.
Therefore its slope is tan 135° = - 1.
But slope of the line is also equal to
5-4
x-3
5-4
-1 = - - =>-x+3 = 1 =>x=2.
x-3
Example 4: Show that the points A (2, 3), B (4, 5) and C(3, 2) are the vertices of a right
angled triangle.
Solution: The given points areA(2, 4), B(4, 5) and C(3, 2)
4-2 2
Slope ofline AB=--=-=2.
5-4 7
'
Slopeofl me BC __ 3 - 4 = -=! = _1
2-5 -3 3
3-2 -1
and slope ofline AC = - - = - .
• 2-4 2
:. (SlopeoflineAB) x (Slope oflineAC) =2 x (~l) =-1.
=> line AB is perpendicular to line A C.
:. ABC is a triangle right angled at A.
378 Remedial Mathematics

Example 5: Find k, if the points (- 1, 3), (8, k) and (2, 1) are collinear.
Solution: The points (- 1,3), (8, k)s and (2, 1) are collinear then slope of AB= slope ofAC.
k-3 k-3
Slope of AB = - - = - -
8-(-1) 9
1-3 -2
and Slope of AC = - - -
2-(-1) 3
k-3 -2
-9- = -3 ~3(k-3)=-18

3k-9=-I8
k =-3
Example 6: Find angle made by the lines x cos 30 0 + Y sin 30 0 + sin 120 0 = 0
with the positive direction of x - axis. [UPTU B. Pharma 2007)
Solution: The equation of the given line is
x cos 30 0 + y sin 30 0 + sin 120 0 = 0 ...(1)
-cos300x -sin 1200x
y=
sin30° sin 30°
o .J3 12
y=-cot30 x - - -
112
~ Y = tan 120 0 x - .J3 ,
which is the slope intercept form. Hence, the angle made by the
given line with the positive direction of x - axis is 120 0 •
Example 7: Reduce 4x -r 3y - 9 = 0 to the Normal form andfind the distance (perpendicular
distance p) from origin. [UPTU B. Pharma 2004)
Solution: We have 4x + 3y - 9 = 0 or 4x + 3y = 9
Dividing both sides by ~(4)2 +(3)2 = 5, we get

i x +~ y = ~, which is the normal form


5 5 5
~ ________-::-c

A ~--------...... B
Fig. 9.3

Hence, the length of perpendicular from the origin to the line is p = ~.


5
Example 8: Prove that the points (- 1, 0), (3, 1), (2, 2) and (- 2, 1) are the vertices of a
parallelogram.
~he Straight Lines 379

Solution: LetA(- 1,0), B(3, 1), C(2, 2) and D(- 2,1) be the vertices ofthe parallelogram, ABCD
taken in order. The mid points of diagonals AC and BD are

AC= (-1+2 0+2) =


2 '2
(.!.2' 1)
D C

and BD = (3-2 ~) = (.!. 1).


2 ' 2 2'
Since, the mid points of AC and BD are same.
Also, slope of AB x slope of AD #= - 1
and slope of AC x slope of BD #= - 1 A
Hence, ABCD is a parallelogram. Fig. 9.4

1. Find the slope of a line whose inclination to the positive direction of x - axis in
antic10ckwise sense is
(i) 60° (ii) 0°
(ii) 150° (iv) 120°.
2. Find the slope of the line passing through (2,3) and (1,4)
3. Show that the points (1, 3), (2, 5) and (4,9) are collinear.
4. LetA(6, 4) and B(2, 12) be two given points. Find the slope ofa line perpendicular to
AB.
5. Determine x so that 2 is the slope of the line through (2, 5) and (x ,3)
6. Without using Pythagoras theorem, show that the points (1, 2), (4, 5) and (6, 3)
represent the vertices of a right angle triangle.
7. Show that the points P(- 4, -5), Q(- 2,2), R(5, 4) and S(3, - 3) are the vertices ofa
rhombus.
8. Show thatthe following points represent a rectangle (0, 0), (0, 5), (6 ,5), (6, 0).
9. Show that the following points represent a square (3, 2), (0,5), (-3, 2), (0 -I).
10. Prove that the lines.
(i) x + 3y + 4 = 0 and 2x + 6y-7 = 0 are parallel.
(li) 2x + 3y + 3 = 0 and 3x - 2y + 5 = 0 are perpendicular.

HINTS TO THE SELECTED PROBLEMS

6. LetA(I, 2), B(4, 5) and C(6, 3) be the vertices of the given triangle.
5-2 3
SlopeofAB= 4-1 ="3 = 1 =m, (say).

3-5 -2
Slope of BC= 6-4 =""2 =-1 = m2 (say).

3-2 1
SlopeofAC= 6-1 ="5 =m 2 (say).
380 Remedial Mathematics

1x-I = - 1 => AB 1.. Be.


mIx m 2 =
Hence, MBC is right angled.
7
7. SlopeofPQ=-,
2
7
Slope of RS = -
2
2 2
Slope of QR = -, Slope ofPS = -
7 7
PQ II RS and QR II PS.
So PQRS is a parallelogram.
4+5 -3-2
Also (slope of PR) x (slope ofSQ) = - - x - - -
5+4 3-(-2)
9 -5
= - x - =-1
9 5
:. The diagonals PR and QS are perpendicular
.'. The Parallelogram PQRS is rhombus.

ANSWERS
i 1
I
I
1. (i) J3 (ii) 0 (iii)
-J3 (iv) -J3
2. 7 4. 5. x= 1
2

• EQUATION OF LINES IS STANDARD FORM


Slope or tangent form: Find the equation of a line whose y-intercept 'c' and slope 'm' are
given. y

Fig.9.S
Letthe given line meet y-axis inA and letP(x, 4) be any point on it. As they-intercept of
the line is e.
:. coordinates of A are (0, C).
The Straight Lines 381

Draw PB 1.. to x-axis and A C 1.. PB.

Then tan e = -PC


AC
PB-BC
DB
= ---
BP-OA
tan e = - - -
OB
y-c
=> m=--
x
=> y=mx+c
Which is called the slope intercept form of the equation ofa straight line.

Remarks
• If c becomes zero, the equationy = mx + c reduces to y = mx which is the equation of
a line through the origin.
• If m = 0, c:;t: 0, then equation y = mx + c reduces to y = c which is the equation of a line
parallel to x - axis at a distance c from it.
• If m °
= 0, c = 0, then the equation becomes y = which represents the x - axis.
Point Slope Form
To find the equation ofa line passing through the given point (x l' y 1) and having slope m:
y

(x, y) p

k....L.;;.----iR

M x

Fig. 9.6
Let the given point (xt,Yt) be represented by Q. Let P(x,y) be any point on the line.
Draw PM and QL perpendiculars to x - axis from points P and Q and QR 1.. MP. Then
PR =MP-MR =MP-QL=y-Yt
and QR =LM=OM-OL=x-x t

Then tane = PR = y- Yt
QR X-Xt

m= Y-Yt
x-xI
Y-Yt =m(x,-x t )·
Which is the equation of the line in the point slope form.
382 Remedial Mathematics

1\vo point form:


To find the equation of the straight line passing through two given points:
y

--~~~~-------L------~----~-- __ x

Fig. 9.7
Let the two given points be Q(x l' Y 1) and R(x2, Y2)· Let P (x,y) be any point on the line.
Draw RL, QM and PN perpendiculars to x - axis from points R, Q and P respectively. Let
RS ..L QM anQ QT ..L PN.
Then RS = LM= ON-OL=x 1 -x2
QS =MQ-MS=MQ-RL =YI-Y2
QT=MN=ON-OM=x-x l
PT=NP-NT=NP-MQ
=Y-YI

In MQS, tan9 = QS = YI - Y2 ...(1)


RS xl-x2

In flQTP, tan9 = PT = Y - YI ... (2)


QT x-xI
from (1) and (2), we get
YI- Y2 = Y- Yl
XI -X2 x-xI
X-XI = Y-YI
X-X2 YI-Y2
x-xI = Y-YI
X2 -XI Y2 -YI

Y _ YI = Y2 - YI (x-xI).
x2 -XI
Which is the required equation of line in two point form.
The Straight Lines 383

Intercept form:
To find the equation of the line which cuts off intercepts a and b on x-axis and y-axis
respectively.
y
Let the line meet x-axis at point A and y-axis is at
point B. As the respective intercepts are a and b. So
OA = a and OB = b.
Coordinates of A and B are (a, 0) and (0, b)
respectively.
Using two point form, the equation ofline is b

x-a y-o
=--
o-a b-o
o a x
-x +1 = ~
a b Fig.9.S

~+~ = 1.
a b
Which is the equation of the line in the intercept form.
Normal or Perpendicular form:
To find the equation ofa line in terms ofthe perpendicular segment p, from the origin to the
lines and the angle a which the perpendicular segment makes with the x-axis.
Let I be the given line meeting x-axis and y-axis at the points A and B respectively. Let OC
..LlandLAOC= aOC=p. Y

OA
Now = sec a
OC
OA
=seca
p
OA =pseca
OB
Again = cosec a ----~~--------~-----.x
OC
DB
= cosec a Fig. 9.9
P
OB =p cosec a
Using the intercept form of the equation of the line, the equation of the given line is
x y
- + - =1.
OA OB
__x_ + __y'--__ = 1.
p seca p coseca
or xcosa+ysina=p.
which is the required equation of the line
384 Remedial Mathematics

Parametric form:
To find the equation of a straight line in the parametric form:
x - x] = Y - y] = r, where r is the parameter.
cosS sine
Let the given line passes through the point A(xl' y]) and be inclined at an angle e with
the positive direction of x-axis.
y

P (x, Y)

~~----------~N

--~L-~~--------~--------------~--_x
B c
Fig. 9.10
Let P(x, y) be any point and AP = r.
Draw AB and PC perpendiculars to x-axis from A and P respectively and AN..l Pc.
Now AN=BC=OC -OB=x-x]
PN = PC - CN = PC - AB = Y - y]
Also AP =r.
In right angle triangle t.ANP.
cose = AN = x-x].
AP r
i.e., x-x] =r. ...(1)
cose
and sine = PN = Y - y]
AP r
y- y] =r. ...(2)
sine
From (1) and (2), we get x-x] =
cose
y:-
sme
Y] =r.

Which is the equation of the line in the parametric form.


Remarks
• Parametric form of equation of a line is also known as symmetrical form of equation
• from x-x] = Y- Y] =r we have
cosS sine '
x =x] + rcose ,Y = y] + rsine, thus the coordinates of any point at a distance r from
(xl'y])r are (x] +rcose,y] +rsine)
The Straight Lines 385

~~~~~~I SOLVED EXAMPLES I


Example 1: Find the equation ofa line which cuts offan intercept - 2 on the axis ofy cmd
makes an angle of 45° with the positive direction ofx-axis.
Solution: Here, c=.,..2, andm =tan45° =L.
Substituting these values iny= mx + c, we gety =x-2 which is required equation of the line.
Example 2: Find the equation ofa line through (4,3) with slope 2.
Solution: Equation of line passing through (xI'Y\) and with slope m is
y-y\ =m(x-x\)
:. The required equation of the line is
y-3 =2(x-4)
::::> 2x-y-5 =0.
Example 3: Find the equation ofa line which passes through the point (- 2, 3) and makes
an angle of 30° with the positive direction ofx-axis
Solution: Here 0 = 30,°
1
::::> m = tanO = tan 30° = .J3'

The point on the line is (- 2,3)


Using point slope form, the equation ofline is
1
y-3 = .J3 (x+2)

::::> .J3y - 3.J3 =x+2


::::> x-.J3y +(3.J3 +2) =0.
Example 4: Find the ratio in which the line segment joining the points (2, 3) and (4, 5) is
divided by the line joining the points (6, 8) and (- 3, 2).
Solution: The equation of the line joining the points (6, 8) and (- 3, - 2) is
y-8 x-6
- - = - - (Two point form).
-2-8 -3-6
y-8 = x-6
::::>
-10 -9
or 9y-72 = IOx-60
IOx-9y+ 12 =0 ...(1)
Let this line divide the join of (2, 3) and (4, 5) at the point P in the ratio of k: I.
.
Then the coordmates of Pare (4k+2
- - , 5k+3)
--.
k+l k+1
N ow, the point P on the line (I)
4k+2 5k+3
Therefore, 10 x - - - 9 x - - + 12 =0
k+l k+1
40k+20-45k-27+12k+ 12 =0
386 Remedial Mathematics

7k =-5
k =-517.
Since, the value of k is negative, the line is divided externally.
Hence, the required ratio is 5 : 7 externally.
Example 5: Find the equation ofthe line which passes through the point (3, 4) and the sum
its
of intercept on the axes is 14. IUPTU B. Pharma 2008]
Solution: Let the intercept made by the line on x-axis be a.
Then intercept on y-axis = 14 - a.
:. Equation of the line is given by

~+-y- =1 ...(1)
a 14-a
As the point (3, 4) lies on it, we have

~+_4_ =1
a 14-a
3 (14-a)+4a = 14a-c?
=> 42-3a+4a = 14a-c?
=> c?-13a+42 =0
=> (a-7)(a-6) =0
a =7,6.
Putting these values of a in (1), we get equation of the lines

~ +.[ = 1 or x + y = 7
7 7
and ~+.[ = 1 or4x+ 3y= 24
6 8
Example 6: A line is such that its segment between the axes is bisected at the point (xl' YI)
Prove that the equation of line is

~+L =1
2xI 2Yl
Solution: Let [be the given line which meetsx-axis atA andy-axis atB. Then segmentAB
is bisected at the point P (xl' Y\).
y

Letthe equation of the line be ~ +.[ = 1 ... (1)


a b
:. the coordinates of A and B are (a, 0) and (0, b)
respectively. As P is the mid point ofAB. Its coordinate,

therefore are given by (~,%).


We are given that coordinates of Pare (xI'Y\) --~~--------~~-----x
o
a b \
- =x and - =y Fig. 9.11
2 I 2 I
The Straight Lines 387

a b
.. =x and - =y
2 I 2 I
or a =2x1 andb=2YI
:. substituting the value of a and b in (I), equation of the line I becomes

~+L =1.
2xI 2YI
Example 7: Find the equation of the straight line passing through the points (- 3, 4) and
(1, - 3). [RGPVB.Pharma2004]
Solution: Equation Ollhe straight line passing through (xl'YI) and (x2'Y2) is given by

Y-YI = Y2-YI(x_XI) ...(1)


x2 -XI
xI =-3'YI =4
x 2 =I'Y2=-3
Putting these values in equation (1) we have
-3-4
Y -4--(x+3)
=
1+3
4(y -4) =-7x-21
4y -16 =-7x-21
=> 7x +4y+5 =0.
Example 8. Find the equation of a line which passes through (2, 3) and whose m is 3.
[UPTU B. Pharma 2005]
Solution: Equation of line passing through one point is
(y -YI) =m(x-x]) ...(1)
m =3,x l =2'YI=3
Putting the values in (1) , the equation of required line is given by
Y -3 =3 (x-2)
=> Y -3 =3x-6
=> 3x -y-3 =0.
Example 9. Find the equation ofa line passing through the point (3, - 2) and perpendicular
to the line X - 3y + 5 = 0 [UPTU B. Pharma 2006)
Solution: Slope of the given line x- 3y+ 5 = 0 is
1
ml = 3"
As the line is perpendicular to line passing through (3,-2).
m] xm 2 =-1.
1
=> -Xm =-1
3 2
=> m2 =-3.
and required equation is (y + 2) = -3(x - 3)
=> y+2 =-3x+9
=> ~+y-7=~
388 Remedial Mathematics

Example 10. Find the equation of perpendicular bisector of the line segment joining the
points A(2, 3) and B(6, - 5) .
-5-3 -8
Solution: Slope = - - = - =-2.
6-2 4

:. Slope of a line perpendicular to the line AB = ..!..


2

e
The coordinates of the middle point M of AB are

;6, 3 +~-5)) i.e., (4, -1).

Hence, the equation of the perpendicular bisector of AB, i.e., the equation of the line
passing through m and perpendicular to AB is
1
y + 1 = -(x-4)
2
=> x-2y =6.
Example 11. Find the equation of the straight line which passes through 0,2) and is
perpendicular to the line 4x - 3y = 8 [UPTU B. Pharma 2001)
Solution: The equation of any straight line perpendicular to the line 4x - 3y - 8 = 0 is
3x+4y+A. = 0 ...(1)
If the line (1) passes through the point (1,2) then
3+8+1.=0
1.=-11.
Putting A. = -11 in (1) the required equation of the line is
3x+4y-Il =0
Example 12: Find the equation of the straight line passing through the point
(a cos 3 e, a sin 3 B) and perpendicular to the line x sec e + y cosec e = a cos2 e.
IUPTU B. Pharma 20071
Solution: The slope of the given line x sec S + Y cosec S = a is
-secS. -sin S
---I.e.,--
cosecS cosS

:. the slope of a line perpendicular to the given line = c~s S


smS
Now, the equation of the straight line which passes through the point (a cos 3S ,a sin 3S)
. cosS .
an d w hose s1ope IS - - IS
sinS
. 3 cosS 3
Y - a sm S = - . - (x - a cos S)
smS
=> x COS S - y sin e = a (cos4 S - sin4 S)
or x cos S - y sin S = a (cos2 S + sin2 S) (cos 2 S - sin2 S)
Hence, x cos S - y sin S = a cos2 S.
Example 13: Find the equation of the straight line which makes equal intercepts on the
axes and passes through the point (3, -5). IUPTU B. Pharma 20021
The Straight Lines 389

Solution: Let the equation of the straight line be

-=-+E. =1 ... (1)


a b
The line (1) makes equal intercepts on the axes, i.e., a = b.

-=- + E.
= 1 or x + y = a
a a
If this line passes through the point (3, - 5) , then
3- 5 = a or a = - 2
Hence, the required equation is x + Y = - 2
or x+y+2 =0.
Example 14: Find the equation ofthe straight line, the portion ofwhich intercepted between
the axes is divided by the point (- 2, 6) in the ratio 3 : 2 [UPTU B. Pharma 2007]
Solution: Let the equation of the straight line be

-=-+E. =1
a b
The line (1) meet x-axis atthe point A (a, 0) andy-axis at the pointB(O,b). Then the point
(- 2, 6) divides the line AB is the ratio 3 : 2
By section formula, we have
_ = (2a+3XO 2XO+3Xb)
( 2, 6) - 2 +3 ' 2 +3

=> _2 = 29 and 6 = 3b
2 5
or a=-5b=10.
Putting the value of a and b jn (1) , the required equation of the line is
x y
- + - =1
-5 10
or y-2x = 10.

Example 15: A straight line, drawn through the point A(2, 1) makes an angle'::' with
4
positive x - axis and intersects another line x + 2y + 1 = 0 at point B. Find the length AB.
[UPTU B. Pharma 2003]
Solution: The equation of any line passing through the given point A(2, 1) and making an

angle .::. with x-axis is


4
~ = ....c..!... = r (say) ...(1)
cos 45° sin 45° \\
Where Il represents the distance of any point B on this line from the given point A(2,1)
The coordinates (x, y) of any point B on the line (1) are

(2+rcos45°, 1 +rsin45°) i.e., (2+r.~'1+r.~}


390 Remedial Mathematics

Ifthe point B lies on the line x + 2y + 1 = 0, then

(2+r.~)+2(I+r.~)+1 =0

(5+r·1) =Oorr= -%J2.


Hence, the length AB = - - -
5J2
3
Example 16: Find the equation of the line passing through the points (4. 3) and (7. 8).
IUPTU B. Pharma 2001 )
Solution: The two points are (xI' YI) = (4,3) and (x2,y 2) = (7,8).

Using y - YI = Y2 - YI (x - x I)
x2 -xI
8-3
y-3 = -(x-4)
7-4
5
y-3 = -(x-4)
3
5x-3y-11 =0.
Example 17: Find the slope and the equation of the straight line joining the points (2 -5)
and (4. 1). rUPTUB.Pharma2002]
Solution: The slope of the line joining the points (2, - 5) and (4, 1) is
= 1-(-5) = ~ =3.
4-2 2
Now, the equation of the straight line joining the points (2, - 5) and (4, 1) and whose
slope is 3 is
y -(-5) =3(x-2)
y+ 5 =3x-6
=> 3x-y =11.
Example 18: Find the equation of the straight line which divides the line joining the point
(5. - 2) and (- 5. 8) in the ratio 3 : 4 and is also perpendicular to it. rUPTU B. Pharm 2006]
Solution: The equation of the line joining the points (2, 3) and (- 5,8) is
8-3 5
y-3 = -5-2 (x-2)ory-3 = -7 (x-1)

- 7y + 21 = 5x - 10
5x+7y =31 ...(1)

The slope ofline (1) is -5 and so the slope of the line perpendicular to it will be 7...
~~ 7 5
The coordinates (h, k) of the point dividing line (1) in the ratio 3 : 4. are given by
h = 3x(-5)+4x2 andk= 3x8+4x3
3+4 3+4
36
i.e.• h = 1 andk = - .
7
The Straight Lines 391

Hence, the equation of the line passing through (h, k) and having slope 2. is
5
7
y-k=S(x-h)

36 7
y- - = - (x-(-I))or49x-35y+229=O.
7 5
Example 19: Find the equatiorr ofa line at a distance of3 units from the origin such that the
perpendicular from the origin to the line makes an angle tan- i (~) with the positive
direction ofx-axis. lUPTU B. Pharma 2006]

Solution: We have p = 3 and a = tan-1 i


4 c
3
tana=-
4
3
cos a = -4 an d. 3
sm a = -.
3 4 ex
Hence, the equation of the line in normal form is BL----4---...J....~ A
x cos a + y sin a = p Fig. 9.12
4 3
or xx-+yx-=3
5 5
~ 4x+3y=15
Example 20: Find the equation ofthe line which has length ofperpendicular segment from
the origin to the line is 4 and the inclination of perpendicular segment with the positive
direction ofx-axis is 30°.
Solution: The normal form ofthe equation of a line is
x cos a + y sin a = p
Now, here, p = 4 and a = 30°
:. Equation of line is
xcos300+ysin30° =4
.fj I
XX -+yx - =4
2 2
~ .fj x+y =8.
Hence, .fj x +Y - 8 = 0 is the required equation of the line.
Example 21: Through the point P(3, -5), a line is drawn inclined at 45° with the positive
direction ofx - axis. It meets the line x + y - 6 = a at the point Q. Find the length PQ.
Solution: The equation of the line through (3, -5) inclined at an angle of 45° (by using
parametric form) is
x-3 = y+5 =r.
cos 45° sin 45°
392 Remedial Mathematics

x-3 = y+5 =r.


-1- ) .

J2 J2
~

Now, the point Q ( h h-


+ 3, 5) lies on the line x + y - 6 = O.

~+3 ~-5-6 =0
J2 'J2
~ ~ =8
J2
~ r = 8.fi =4J2
2
Hence, required length of PQ = 4 J2 .
1EXERCISE 9.21
1. The x- intercept ofa line is double to itsy- intercept. Ifit passes through (2,3), find
its equation.
2. A line makes equal intercept on the coordinate axes and passes through (1,3) find its
equation.
3. Find the equation of the line passing through the I?oints (2,3) and (-1, -4).
4. Iflength and inclination of the perpendicular from the origin on the line is 4 and 135°
respectively. Find the equation of the line.
S. If A(O, 2), B(4, 1), C(1, 3) are the vertices ofa ~ABC, find the equation of 0) side AB
(ii) median CF and (iii) attitude on side BC.
6. Find the equation of the line which passes through the point (-3, 8) and the sum of
its intercept on the axes is 7.
7. Find the equation of the line through (2, 3) so that the segment of the line intercepted
between the axes is bisected at this point.
8. The length of the perpendicular from the origin to a line is 6 and the line makes an
angle of 30° with the positive direction ofy - axis_ Find the equation of the line.
9. Find the equation of the line through the point (2, 3) and making an angle of 45° with
the x - axis. Also determine the length of intercept on it between A and the line
x+y+ 1 =0_
10. Ifp be the length of the perpendicular drawn from the origin to the line bx + ay = ab
1 1 I
show that -+-=
2 2 2-
a b p
The Straight Lines 393

HINTS TO THE SELECTED PROBLEMS

5. Given A = (0, 2), B(4, I) and C(I, 3)


(i) Equation of AB is
y-2 x-o x
2-1 = 0-4 =>y-2= -4 =>-4(y-2)=x.
=>-4y+ 8 =x=>x+4y= 8.
:. x + 4y = 8 is the eqn. of side AB.

(ii) Median CF: F = the mid of AB = (0; 1


4 , 2; )

F= (2,%) and C= (1, 3)


U sing two point form y - YI = x - xI
YI- Y2 xl-x2
y-3/2 x-2 2y-3 x-2
=> =--=>--=--
3/2-3 2-1 3-6 1
=> 2y-3 =-3 (x-2)=>2y-3 =-3x+6
=> 3x + 2y - 9 = 0 is the required equation of median CF.
(iii) Allude AD 1.. Be. y
3-1 2
SlopeofBC= - - = --. A(2,3)
1-4 3
3
Slope of AD = - (': AD 1.. BC)
2 --~~--~------x
Equation of AD iSY-YI = m (X-XI)
3
y-2 ="2(x-O)
3
y-2 =-x
2
Fig. 9.13
2y-4 =3x=>3x-2y+4=0.
The equation of a line through A and making an angle of 45° with the x - axis is
x-2 y-3 x-2 y-3
cos45° = sin45° => -1- = -1-
Ji Ji
=> x-2 =y-3 =>x-y+/l =0.
Suppose the line meets the line x + y + 1 = 0 at P such that AP = r.
Then the coordinates of P are given by
x-2 y-3
- - =--=r.
cos45° sin 45°
r r
x =2+ -Ji,y=3+ Ji
394 Remedial Mathematics

Thus the coordinate of P are

Since P lies on x + y + I = 0

2 +~
J2
+3 +~
J2
+I = 0 => J2r = - 6

r =-3J2
length of AP = Irl = 3J2
Thus the length of intercept = 3J2
I ANSWERS
1. x + 2y+4=0 2. x+y +4=0 3. 7x-3y+2=0
4. x-y-4J2 =0 5. (i)x+ 4y= 8 (ii) 3x+2y-9= 0 (iii) 3x-2y+4 =0.
6. 4x+3y= 12 7. 3x+2y-12=0 8. .J3x+y-12=0
9. x-y+ I = 0, 3J2
III TRANSFORMATION OF GENERAL EQUATION IN DIFFERENT
STANDARD FORMS
The general equation of a straight line is ax +by + c = 0 which can be transformed to various
standard forms as discussed below:
(1) Transformation of ax + by + c = 0 in the slope intercept form (y = mx + c):
We have ax +by +c =0 => by=-ax-c => y =(-~)x +(-~)
This 0 is of the formy =mx +c where m =-!!.. and c =-:..
, b b
Thus for the straight line ax + by + c = 0

m =slope =--ab =----


coeff of x
coeff .of y
. c constant term
and y- mtercept = --=-
b coeff .of y

"Remark

• To determine the slope of a line by the formula m = coeff of x we must transfer all
coeff. of y
terms in the equation on one side. Transformation of Ax + By + C = 0 intercept form

l;+i=l}
The Straight Lines 395

We have Ax + By + C =O:::::>Ax+By=-C
Ax By
- + - =1
-c -c
x y
=1.
(-~)+(-~)
This is of the form ~ + 2::. = 1. Thus for the straight line Ax + By + C = O.
a b
. -C -const. term
Intercept on x-axIs = - = ----
A coeff. of x.
. -C - costant. term
Intercept on y-axis = _ = -----
B coeff .of y
(2) Transformation of Ax + By + C = 0 in normal form (x cos a + y sin a = p):
We have Ax + By + C =0 ... (1)
Let x cos a + y sin a - p = 0 ...(2)
be the Normal form of Ax + By + C = O. Then (1) and (2) represent the same straight line.
ABC
cosa sina - p

cosa = - Ap and sin a = - Bp ...(3)


C C
A2p2 B2p2
cos2 a+ sin2 a = -- + -2-
C2 C
2
1 = L(A2+B2)
C2
c
P =± - ; = = =
~A2 +B2
But, p denotes the length ofthe perpendicular from the origin to the line and is always
positive.

Putting the value of p in (3) we get


-A . -B
cosa = -.j?A=2=+=B=2= , sma = ~ A2 + B2
So, the equation (2) takes the form
-A
-,====x
~A2+B2
-A B C
-,=o=:===~x - Y- x
~ A2 + B2 ~ A2 + B2 ~ A2 + B2
This is the required normal form of the lineAx +By + C = o.
396 Remedial Mathematics

~~~~~~I SOLVED EXAMPLES ~I~~~~~~


Example 1. Reduce 3x - 4y + 5 = 0 to slope/orm andfind its intercept on y-axis.
Solution: The given equation 3x - 4y + 5 = 0 can be written as
4y = 3x+5
3 5
=> Y = '4x+ '4'
. 5
Intercept on y-axIs = '4'
Example 2. Reduce the lines 3x - 4y + 4 = 0 and 4x - 3y + 12 = 0 to the normal/orm and
hence determine which line is nearer to the origin.
Solution: We have 3x - 4y + 4 = 0
=> -3x+4y =4

This is the normal form of 3x - 4y + 4 = 0 and the length of the perpendicular from the
origin to it is given by.
4
PI 5
Now 4x-3y+ 12 =0
-4x+3y =12
-4x 3y 12
=> +-r=======
~(-4)+32 J(-4)2 +(3)2
4 3 12
=> --x+-y - -
5 5 5
This is the normal form of 4x - 3y + 12 = 0 and the length of the perpendicular from origin
.. . b 12
to It IS given y P2 = S·
Clearly P2 > PI therefore, line 3x - 4y + 4 = 0 is nearer to the origin.
Example 3: Reduce 3x + 5y + 4 = 0 to the intercept/orm andfind the y-intercept.
Solution: 3x + 5y + 4 = 0
3x + 5y =-4
3x 5y -4
or -+- =-
-4 -4 -4
x y
(-4/3) + (-4/5) = 1
Which is the required intercept form:
. . 4
Hence y-mtercept IS - - .
5
The Straight Lines 397

• POINT OF INTERSECTION OF TWO LINES


Let the two lines be
Alx+Bly+C I =0 ... (1)
A 2x +B21 + C2 = 0 ... (2)
Let (Xl' Yl) be the point of intersection of these two lines:
Then Alx l + BIYI + C I =0 ... (3)
and AzXl +B211 + C2 = 0 ...(4)
From (3) and (4), we have
YI

B C -B C
I 2 2 I
X = --'--"'-----"'-'-
I AIB2 -A2Bt
C1A2 -C2 A1
Y =
I AIB2 -A2BI
Hence, the coordinates of the point of intersection of the two lines (1) and (2) are
B1C 2 -B2C1 , C IA2 -C2A1 ).
( AIB2 - A2Bl AIB2 - A2Bl

Remarks
• To find the coordinates of the point of intersection of two non parallel lines, we solve
the given equations simultaneously and the values of x and Y so obtained determine
the coordinates of the point of intersection.
• The coordinates of the point of intersection determined above do not exist if
A 1B2 -A 2 B I =0.

i.e., if ~=!i*~
A2 B2 C2

• If ~ = .!!.L = ~, then the lines are coincident.


A2 B2 C2
• If there is only one point which satisfied both equation the system of equations is
called consistent.

In that case ~ * .!!.L * ~.


A2 B2 C2

• CONDITION OF CONCURRENCY OF THREE GIVEN LINES


Let the equation of the three lines be
a1x +bly + c I =0 ... (1)
a 2x + b21 + c2 = 0 ... (2)
a3x + b)Y + c3 = 0 ...(3)
398 Remedial Mathematics

For given lines to be concurrent, no two ofthese lines can he parallel or coincident i.e.,

E1.
a2 a3* * ...(4)
~ ~ ~
and the point of intersection of any two lines must lie on the third line.
Now, the point of intersection of (1) and (2) can be obtained as below:
x y 1
~c2 -b2c I cla2 -alc2 al~ -a2~
~c2 -~cI
x = -'-=---=-..!..
alb2 -a2~
y = cla2 -alc2
alb2 -a2q

Now, the point (~C2 - b2cI ,cla2 - alc2 ) lies on (3) because the lines are concurrent
alb2 - a2bl alb2 - a2 bl

a3 ( a~C2 -b2CI) + b
lb2 - a2~
3 (Cl a 2 -alc2 ) +c =0 .
a)b2 - a2q 3

a3(b lc 2 - b2 c l ) + b 3(c la 2 - a lc 2) + c 3(a l b 2- a 2 b l ) =0.


a l (b 2c 3 - b3c2) + b l (c 2a3 - a 2c3) + c l (a2b3 - a3b2) =0. ... (5)
Thus for the given three lines to be concurrent, the condition (4) and (5) must hold.

~~~~~~~I SOLVED EXAMPLES I


Example 1: Find the coordinates ofthe point ofintersection ofthe lines 2x - y + 3 = 0 and
x + 2y-4 = O.
Solution: Solving simultaneously the equation 2x - y + 3 = 0 and
x + 2y-4 = 0, we obtain
_x_ =-L=_I_
4-6 3+8 4+1
x y1 -2 11
=> -= - -=>x = -
= y=-.
-2 11
5 5' 5
Hence, (-2/5,1115) is the required point ofintersection.
Example 2. Show that lines x - y - 6 = 0, 4x - 3y - 20 = 0 and 6x + 5y + 8 = 0 are
concurrent. Also, find their common point of intersection.
Solution: The given lines are
x- y- 6=0 ...(1)
4x-3y-20 =0 ...(2)
6x+5y+& =0 ...(3)
Solving (1) and (2) by cross multiplication, we get
__ x_= y 1
'20-1& -24+20 -3+4
The Straight Lines 399

x =2,y=-4
Thus, the two lines intersect at the point (2, - 4). Putting x = 2, y = - 4 in (3) , we get
6 x 2+5x x (-4)+8=0
so (2, -4) lies on (3).
Hence, the given lines are concurrent and their common point of intersection is (2, - 4).
3: Prove that the lines 2x + 3y - 13 = 0 x + 2y - 8 = 0 and 3x - y - 3
-.E~.ample = 0 are
concurrent
Solution: Solving the equations
2x+3y-13 =Oandx+2y -8=0
x y 1
We have
-24+ 26 -13 + 16 4-3 .
or x =2,y=3
The lines will be concurrent if the point (2,3) satisfies the equation of third line.
Putting the coordinates (2, 3) in 3x - y - 3 = 0, we have
3 (2)-3 -3 =0
o = 0 , which is true.
Hence, the lines are concurrent.
Example 4: Find the value of k, so that the lines
x-2y+1 =0
2x - 5y + 3 = 0 and
5x - 4y + k = 0 are concurrent.
Solution: The equation of the lines are :
x-2y+ 1 =0 ... (1)
2x-5y+3 =0 ...(2)
5x-4y+ k =0 ...(3)
Solving (1) and (2)
x
=~=--
-6+5 2-3 -5+4
x
=l:'...=..!...
-1 -1 -1
x = l,y= 1
:. The point ofintersection of(1) and (2) is (1, 1).
This pointwiII lie on (3) if5 -4 + k= 0 or k=- 1.
Thus for concurrency of(1) and (2) and (3) ,k=-l.
Example 5: Find the equation of the line which is perpendicular to the line 3x-2y+4 = 0 and
passes through the point of intersection of the lines x + 2y + 1 = 0 and y = x + 7.
[UPTU B. Pharma 2008]
Solution: The eqn. of a line perpendicular to 3x - 2y + 4 = 0 is
2x+3y+1 =0 ...(1)
400 Remedial Mathematics

Point of intersection of x + 2y + 1 = Uand y = x + 7 is


x = -5, Y = 2 is (-5, 2).
Line (1) passes through this point so
2(-5)+3(2)+1.. =0
-10+6+1.. =0
A =4
puting, we get
2x+3y+4 =0.

• ANGLE BETWEEN TWO INTERSECTING LINES


Theorem 1: Prove thatthe angle Q between the line y = mIx + C1 andy = m jX + C2 is given
m)-m2
by tan () = ---"--=-
1+m)m2
Proof: Let I) and 12 be two lines y = m) x + c) and y = m2 x + c2 respectively. Let I) intersect
12 at P making an angle e between them. Let I) and 12 meet x - axis at Rand Qrespectively. Let
I) and 12 angle a. and ~ respectively, with the positive direction of x - axis.
y

Q J3
o X
/1 '1
Fig. 9.14
The exterior angle a. = e+ ~
e =a.-~
tan e =tan(a.-~)

tan a. - tan~
tan e
1 - tan a. tan ~

or tan e= m) -m2
1+m)m2

e = tan-) m) - m2
1+m)m2
The Straight Lines 401

Remark
• The value oftan e can be bot4 positive and negative because between two lines there
we two distinct angles. If thIs value is +ve, then the angle between the lines is acute
and if it is -ve the angle is obtuse.
Theorem 2: Prove that the angle Qbetween thelinesalx +bJy + c J = Oandar + b§ + c2 = 0
alb2
is given by tan () = la2bl - 1·
ala2 + bl b2
Proof: Letm l andm2 be the slopes of the linesalx+bly+c) =Oanda r + by; + c2 =0. Then.
ml = -;1 andm 2= -b:2

Now tane =II~:)~J

tan e

tan e = la2b) - alb21


ala2 + b)b2

Hence, e = tan-II albl - alb21·


a)a2 + b)b2
Condition of perpendicularity:
Two lines are perpendicular, ifthe angle between them is a right angle i.e., a = 90°
tan a = tan 90° = 00
m)-m2
=00
1 +mlm2
=> 1 + m)m2 =-1
=> m) m2 =-1
Hence, two lines are perpendicular ifthe product of their slopes is -I.
Condition of parallelism:
Two lines are parallel, if the angle between them is either 0 or 1t, i.e., a = 0 or 1t
., tan a = tan 0 ortan 1t = 0
m)-m2
=> =0
1 + mlm2
=> m l -m2 =0
=> m l =m2
402 Remedial Mathematics

~~~~~~~I SOLVED EXAMPLES ~I~~~~~~


Example 1: Find the acute angle between the line 9x + 3y - 5 = 0 and 2x + 4y + 3 = O.
Solution: We have 9x + 3y - 5 = 0 ...(1)
2x + 4y+ 3 =0 ...(2)
9
Slope of(l) => m = -- =-3
I 3

Slope of(2) => m2 = -"42 = -"21


The acute angle between the lines is given by

tan 8 = Iml+ - m21


1 mlm2

1
~3 -
tan 8 = - + 1/21
-
1+3/2

=> tan 8 = 1-551=>tan8= 1 =>8=45°.


Example 2: IfA(- 2, 1), B(2, 3) and C( - 2, - 4) are three points, find the angle between BA
and Be.
Solution: Let m l and m2 be the slope of BA and BC respectively. Then
3 -1 2 1
m = -=-
I 2-(-2) 4 2
-4-3 7
andm = - - = -
2 -2-2 4
Let Q be the angle between BA and Be. Then
ml-m2
tan 8 = --'.-~
l+mlm2

=> 7/4-0/2)1 110/81


2
11+ 7/4 (1/2) = 15/8 = ± "3
=> 8 = tan- I (j).
Example 3: Find the angle between the following lines:
(z) x cos a l + y sin a l = PI and x cos a 2 + y sin a 2 = P2
••)
( II
X Y an d ---=
-+- x Y 1.
a b b a
Solution:
. . cos al
(i) Thesopeo
I f x cos a l +ysma l =Pllsm l = --.--=-cota l
smal
. . cosa2
The s Iope 0 f x cos Clz +Y sm Clz = P2 IS m2 = - - . - - = - cot a 2
sma2
The Strqight Lines 403

1 m
Now, tan 8 =lm - 2 1
1+mlm2
= - cot (XI + cot (X2
1 + cot (X2 cot (X2
I I
---+--
tan (XI tan (X2
1+_1_ _1_
tan (XI tan (X2
tan (XI - tan (X2 ( )
---'-----=-- = tan (X - (X
1 + tan (XI tan (X2 I 2

=> e =(XI-(X2
x y. (11 a) b
(ii) Slope of - + - = 1 IS m = - -- = - - •
a b I lib a
x y. -(lIb) a
Slopeofb"-~ = hsm 2 = (-lIa) =-b"

Hence m I m2 = ( - ;). (~) = - I

=> The lines are at right angles.


=>8=90°
Example 4: The angle between two lines is 45°. If the slope ofone ofthem is 114. Find the
slope of other.
Solution: Here, 8 = 45°, m 1 = 114 . ~et the slope of the required line be m 2•

Now

tan 450 = / ml - m2 /
l+mlm2

=>
4
1 = /11 - m / => 1 + m = ± (~_ m)
l+ml4 "4 4

. m 5 -3 -3
for + ve SIgn 1 + - = - - m => - m = - => m = - .
4 4 4 4 5
. m I 5 3 5
for - ve SIgn 1 + - = - - + m => - = - m => m = -
4 4 4 4 3

The possible slope of the lines are ~, -3.


3 5
Example 5: Find the angle between the lines:

x-yJ) -5 =OandJ)x+y-7=0 [UPTU B. Pharma 2001] .


404 Remedial Mathematics

Solution: The given two lines are:


x-yfj -5 =0. ...(1)
and fj x+ y-7 =0 ...(2)
Here m = Slope of the line (1) = __1_ = _1_
l
-fj J3
- fj r:;
m2 = Slope of the line (2) = -1- = - ,,3
clearly m l x m2 =-1.
Hence, the two lines are at right angles.
Example 6: The line joining (- 5, 7) and (0, - 2) is perpendicular to the line joining (1, 3)
and (4, x). Then find x. [UPTUB. Pharma2003]
Solution: Here m l = Slope of the line joining the points (- 5,7) and (0, -2).
-2-7 -9
0-(-5) 5
and m2 = Slope of the Iinejoining the points (1,3) and (4, x).
x-3 x-3
=--=--
4-1 3
If the given two lines are perpendicular, then
m1m2 =-1

(_~) (X;3) =-I~-9(x-3)=-15


~ x = 14/3.
Distance of a point from a line: L
Let ax +by + c = 0 be any equation ofthe line and P(x, y) be any
point in space, then the perpendicular distance d of the point p
from the line is
Fig. 9.17
d = lax +bYl +cl
l
~a2 +b2
Distance between two parallel lines:
+ + + +
Let ax by c 1 = 0 and ax by c 2 = 0 be
two equation of parallel lines, then the distance
between the two lines is given by

Fig. 9.18
The Straight Lines 405

~~~~~~I SOLVED EXAMPLES ~I~~~~~~


Example 1: Find the length ofthe perpendicular drawn from the point (- 2, 3) on the line
12x - 5y + I = o.
Solution: We know that the length of the perpendicular segment from point (x\, YI) on

ax+by+c=Ois 11 +l'IY+cl
a 2 +b 2
:.Here a =I2,b=-5,c=1,x l =2'YI=3
:. length of the required perpendicular segment

= 112X(-2)+(-5)X3+II
~(l2)2 + (_5)2

= 1-24 -15 + 11 = 1- 38 1= 38.


13 13 13
Example 2: Find the distance between the parallel lines 2x - 3y + 9 = 0 and 4x - 6y + I = O.
Solution: As the given lines are parallel, they have same distance between them throught
out. So we shall find the distance of any point on the first line from the second line (0, 3) is a
point on the line 2x - 3y + 9 = o.
Perpendicular distance ofthe point (0,3) from 4x - 6y + 1 = 0 is

4X-6X3+II
~42 + (_6)2 =
(-17)
..J52 =
17
2J13
1
Hence, the distance between the given lines is 1~ .
2"l3
Example 3: Are the points (2, - 4) and (0, 5) on the same or opposite sides of the line
2x-5y + 6 = O?
Solution: Perpendicular distance of (2, - 4) from the given line is
_ 2x2-5(-4)+6
PI - ~4+25
4+20+6
...(1)
59
Perpendicular distance of(O, 5) from the given line is
2xO-5x5+6
.J4 + 25
-25 +6
59
-19
= 59· ...(2)

Since (1) and (2) are of opposite signs, therefore, the point are on opposite sides of the
given line.
406 Remedial Mathematics

Example 4: Which ofthe lines 2x - y + 3 = 0 and x - 4y - 7 = 0 is farther from the origin?


IRGPV B. Pharma 2001]
Solution: The length of perpendicular from (xl'YI) on ax + by + c = 0 is

= laxj;b:~:cl
Length of perpendicular of2x - y + 3 = 0 from origin

= 12XO-0+31= ~
PI ~4+1 15·
and length of perpendicular of x - 4y - 7 = 0 from origin

P -10-4XO-71-
- --7
2 ~1 +16 .Iff"
as PI> P2
... 2x - y + 3 = 0 is farther from origin.
Example 5: Find the distance between the two parallel straight lines y = mx + c and
y = mx + d. [R-GPV B. Pharma 2002]

Solution: Puttingy = 0 in y = mx + c, we get x = - elm. Thus ( - : ,0 }s a point on the line

y = mx + c. Length of perpendicular from ( - : ,0) to y = mx + d is given by

p ~ mX~+d ~I~I
Example 6: Find the distance between the paral/ellines 3x + 4y = 12 and 3x + 4y = 3
[UPTU B. Pharma 2004J
Solution: The given lines are 3x+4y =12
3x+4y =3 ...(2)
Putting x = 0 we gety = 3. Thus (0, 3) is a point on the line (I). The perpendicular distance
between the lines (I) and (2) is
= the length of perpendicular from the point (0,3) to the line (2)

3xO+4x3-3 9
~9+ 16 5

1. Find the length of the perpendicular from the origin on the line 4x- 3y = 7.
2. Find the distance ofthe point (3, -2) from the line 7x-5y-29 = O. Determine whether
the point lies on the origin side of the line.
3. For what value of kwill the point (3, k) lie on the origin side of the line 2x + 3y+ 6 = o.
The Straight Lines 407

4. Find the foot of the perpendicular drawn from the point (- 2, - I) on to the line
3x+2y-5=O.
5. Show thatthe point (l, 2) is equidistant from the lines 5x - 2y- 9 = 0 and 5x - 2y + 7 = O.
6. Find the distance between the pair of parallel lines 2x- 3y+ 4 = 0 and 4x-6y-5 = O.
7. If a and b are the intercepts of a line on the x and y axis respectively and P be its

perpendicular distance from the origin then show that -;- = -\- + ~.
P a b

HINTS TO THE SELECTED PROBLEMS

4. Let P (- 2, - I) = (xl' y\) and M = (h, k) be the foot of the perpendicular On to


3x+2y-5=O.
. h-x\ k - y\ (ax! + by! +c)
Now (h, k) are gIven by - - = --=
a b a2 + b 2
h+2 = k+1 = -(-6-2-5)
=>
3 2 9+4
h+2 = k+1 =1
=>
3 2
=> h+2 =3,k+I=2
=> h=l,k=1
:. The foot of perpendicular (1, 1).

7. ~+l. = 1 =bx + ay =ab =>bx + ay-ab = 0 ...(1)


a b

I ANSWERS I

1. 7/5 2. ~, origin lie on the opposite side of the line.


,,74

3. k<4 4. (1, I) 6.
Jl3
2
408 Remedial Mathematics

OBJECTIVE EVALUATION

MULTIPLE CHOICE QUESTIONS


Choose the most appropriate one:
1. The equation of the straight line which passes through the point (3,4) and whose intercept on
y-axis is twice i.e., on x-axis is
(a) x + 2y = to (b) x-2y= 10
(c) 2x-y = to (d) 2x+y= 10.
2. The equation of the straight line which makes an angle of 15° with the positive direction of
x-axis and cuts an intercept of length 4 on the negative direction ofy-axis is:
(a) (2+ .J3)x-y-4=O (b) (2- .J3)x+y-4=O.
(c) (2- .J3)x-y-4=O (d) (2+ .J3)x+y+4=O.
3. The ratio in which the line y - x + 2 = 0 divides the line joining (3, -1) and (8, 9) is
(a) 3:2 (b) 2:3
(c) -3: 2 (d) I: 3.
4.. Find the acute angle between the pairs of lines 3x + 2y = 5 and 2x + Y + 7 = O.
5 3
(a) tan e = - (b) tan e = -
3 2
7
(c) tan e = - (,d) 90°.
4
5. Ifthe lines 13x - 6y + 7 = O. , x - y + 9 = 0 and 2x + 5y = k are concurrent, then value of k is
given by ,
(a) k= 1 (b) k=2
(c) k = 3 (d) k = 4.
6. Find the equation of the line passing through the point (-3, 7) and having equal intercept on the
coordinate axis
(a) x-y=2 (b) x+y=3
(c) x + Y = 4 (d) x-y=4.
7. Find the co-ordinates of the point of intersection of the lines x + 3 = 0 and 3y - 4 = O.

(a) (2,D (b) (3, -~)


(c) (I.~) (d) (-3,~)
8. Find the equation ofthe line passing through the point (- 3, - 7) and perpendicular to t~e y axis.
(a) y =-3 (d) y=-5 '
(c) y=-7 (d) y=-9.
9. Find the equation of the line passing through the point P(-5, 7) and parallel to the coordinate
axis.
(a)x= 5 (b) x = - 5
(c) y= 5 (d) y=-5.
10. Find the slope of a line which makes equal intercepts of opposite sign on the coordinate axis.
(a) 3 (b) 2
(c) I (d) 112
The Straight Line 409

11. The equation of the line having slope mandy-intercept c.


(a) y=mx+c (b) x=my+c
(c) y=-mx+c (d) None of there
12. The slope ofline which makes an angle of 60° with the positive x-axis.
(a) J3 (b) J2
(c) 1 (d) 2.
13. The value of k, if the slope of the line containing the points (k, 3), and (- 2,5) is 4/5.
00 ~ ~ -W
(c) -9/2 (d)1114.
14. Find the slope of the line passing through the points (- 2,3) and (5, -7).
(a)-5/6 (b) -6/7
(c) -9/10 (d) -10/7.
15. The value of k so that the lines 2x - 3y + k = 0 , 3x - 4y - 13 = 0 and 8x - Ily - 33 = 0 are
concurrent is
(a) 6 (b) -6
(c) 7 (d) -7.

ANSWERS

MULTIPLE CHOICE QUESTIONS


1. (d) 2. (c) 3. (b) 4. (c)
5. (d) 6. (c) 7. (d) 8. (c)
9. (b) 10. (c) 11. (a) 12. (a)
13. (c) 14. (d) 15. (d)

Can we do? (Frequently Asked Questions)


1. Find the equation of the straight line which makes equal intercepts on the x-axis and
passes through the point (3, - 5). [UPTU B. Pharma 2002]
2. Find the equation of the straight line the portion of which intercepted between the
axes is divided by the point (- 2,6) in the ratio 3 : 2. [UPTU B. Pharm 2007]

3. A straight line drawn through the pointA(2, 1) makes an angle 2: with positive x-axis
4
and intercepted another line x + 2y + 1 = 0 at point B. Find the length AB.
rUPTU B. Pharma 2003)
4. Find the equation of the line passing through the points (4, 3) and (7,8)
[UPTU B. Pharma 2001)
5. Find the slope and the equation of the straight line joining the points (2, -5) and (4, 1).
[UPTU B. Pharma 2002)
6. Find the equation of the straight line which divides the line joining points (2, 3) and
(- 5,8) in the ratio 3 : 4 and is also perpendicular to it. [UPTU B. Pharma 2006]
410 Remedial Mal/rcmatics

7. Find the equation of a line at a distance of 3 units from the origin such that the

perpendicular from the origin to the line makes an angle tan- 1 (~) with the positive

direction ofx-axis. [UPTU B. Pharma 20061


8. Reduce 4x + 3y - 9 = 0 to the normal form and find the distance (perpendicular
distances p) from origin. [UPTU B. Pharma 20041
9. Find the angle made by the line x cos 30° + y sin 30° + sin 120° = 0 with the positive
direction ofx-axis. [UPTU B. Pharma 2007]

10. Find the angle between the lines x - y.J3 - 5 = 0 and .J3 x + Y - 7=0
[UPTU B. Pharma 2007]
11. The line joining (- 5,7) and (0, -2) is perpendicular to the line joining (1,3) and (4, x)
then find x. [UPTU B. Pharma 2003]
12. Find the equation of the straight line which passes through (1. 2) and is perpendicular
to the line 4x - 3y = 8. [UPTU B. Pharma 2001]
13. Prove that the equation to the straight line passing through the point (a cos38, a sin38)
and perpendicular to the line x sec 8 + y cosec 8 = a is x cos 8 - y sin 8 = a cos 2 8.
IUPTU B. Pharma 2007]
14. Find the equation of perpendicular bisector of the line segment joining the points
A(2,3)andB(6,-5). IUPTU B.Pharma 2004]
15. Find the distance between the parallel lines 3x + 4y = 12 and 3x + 4y = 3.
IUPTU B. Pharma 2004]
16. Find the equation to line which passes through (2, 3) and whose slope is 3
[RG PV B. Pharma 2005]
17. Find the equation ofa line passing through the point (3, - 2) and perpendicular to the
line x - 3y + 5 = 0 IRGPV B. Pharma 2003J
18. Find the equation to the straight line passing through the points (- 3,4) and (1, - 3)
[RGPV B. Pharma 2001]
19. Which ofthe lines 2x - y + 3 = 0 and x - 4y - 7 = 0 is farther from the origin.
[RGPV B. Pharma 2001 J
20. Find the distance between the two parallel straight lines y = mx + c and y = mx + d
[RG PV B. Pharma 20021
21. A line passes through (3, 4) and sum of its intercepts on the axis is 14. Find the
equation of the straight line. [UPTU B. Pharma 2008]
22. Find the equation of the line which is perpendicular to the line 3x - 2y + 4 = 0 and
passes through the point of intersection of the lines x + 2y + 1 = o.
[UPTU B. Pharma 2008J
The Straight Lines 411

SUMMARY OF THE RESULTS

S.No Name Data about the line Equation


1. Slope intercept form slope = m, Y - intercept = c y=mx+c
2. Point slope form slope = m, one point (xl' YI) Y-YI = m(x-x l )
Y2-Yl
3. Two point form two points (xl' YI) and (x2' Y2) (Y-YI)= - - ( x - x I )
x2- xl
x Y
4. Intercept form x-intercept = a, y-intercept = b -+-=1
a b
5. Normal form length of 1. from origin on the
line =p , and inclination = a x cosa + Y sina = p
x-xl Y-Yl
6. Parametric form Horizontal component = r cos e --=--=r.
cos e
sin e .
vertical component = r sin e
fixed point = (xl'YI)

000
CHAPTER

10 FUNCTIONS AND LIMITS

II1II INTRODUCTION
Let A and B be two sets, then the rule or correspondence, which associates each element of
A to a unique element of B, is called a function from set A to set B.
If a general element of set A is denoted by x, and of set B is denoted by y, then we say
thaty is a function ofx iffor every x E A, one and only one value ofy E B can be determined.
Symbolically: Iffis a function from a setA to a set B, then we writef: A ~ B, read asfis a
function from A to B orfmaps A to B.

Domain and Range of a Function


Let an elementy E Bbe corresponded by an element x E A, theny is called the image ofx and
is denoted by j(x). Here, x is defined as the pre-image ofy.
The set A is called the domain and the set B is called the co-domain of the function f
The set of all j-images of the element ofA, is called image set or the range off and is
denoted by
j(A)or{(f(x):x EA}
Evidently, j(A)~B.

Thus amappingf: A ~B is the set of ordered pairs {(a, b): a E A, bE B} so that no two
ordered pairs have the same first element.
i.e., f= {(a, b): a E A, bE B, b = j(x) V a E A}
For Example: LetA = {-2,-1, 0, 1,2, } and B be the set of natural numbers for every x E A,
j(x) E Bandj(x)=~.
Here, A is the domain and B is the co-domain andj{a) is the value of the functionj(x) ,
when x takes the value a
i.e., when x is replaced by a.
The elements ofthe co-domain which is equal tof(x) form the range.
When x =-2, j(-2) = (-2i =4
When x=-I, j(x)= 1
When x=O, j(x)=O
When x = I, j(x) = 1
Functions and Limits 413

When x=2, j(x)=~


This can be illustrated in the following figure.

A B
Domain Co-domain Range

Fig. 10.1

Remarks
• IF f A ~ B, then a single element in A can not have more than one image in B.
However, two or more elements in A may have the same image in B.
• Every element in A must have its image in B, but every element in B may not have its
pre-image inA.
• The each element x in A, there exists a unique elementy in B such thaty = j(x).
• The unique elementy of B is called the value offatx (the image offunder x), and is
written asy= j(x).
• The range off consists of those elements in B which appear as the image of at least
one element in A. In other words, we can say range of a function is the image of its
domain.
• Rang is subset of co-domain.

_ TYPE OF FUNCTIONS
(a) One-one Function: AfunctionffromA ta-B i.e.,! A ~B is said to be one-one (or
injective) if distinct elements ofA have distinct images.
Symbolically :fis one-one iffor x l ,x2 E A,
We have
x ;ex2
=> j(x l ) ;ej(x2 )"r! x l ,x2 EA
or j(x l ) = j(x2)
XI =x2 V x\,x2 E A
Fig. 10.2 One-one Function
It is also called Univalent function.
414 Remedial Mathematics

Graphically, a function is one-one ifand only ifno f(x)


line parallel to x-axis meets the graph of the function in
more than one point.
(b) Many-one Function : A function f: A -+ B is
called many-one, if at least one element of co-
domain B has two or more than two pre-images
in domain A.
Symbolically:fis many-one, iffor xl' x 2 E A we have, -_-+3--1_'""""_-11---
0 +--+--+--+- - x
2 2 3
Xl ;o!ox2
~ j(x ) =j(x ) Fig. 10.3
l 2
This can be illustrated in the figure J 0.4

ea -\-----=01-

ec
ed

A B

Fig. 10.4 Many-one Function


Graphically, a function is many-one if and only if a line parallel to x-axis meets the graph
of the function in more than one point.
f(x)

-3 -2 -1 o 2 3

Fig.IO.S

Remark
• One-many function does not exist.
(c) Onto Function: Afunctionf A -+ B is called an
onto function, if there is no element ofB which is
not an image of some element of A i.e., every
element ofB appears as the image ofat least one
element ofA. L -_ _ _ _ _ _ _ _ _ _--'

Fig. 10.6 Onto/llnetion


This can be illustrated in Fig. 10.6.
Functions and Limits 415

Remarks
• In an onto function, Range = Co-domain
• Onto function is also called surjective.
(d) Into Function: Afunction! A ---+ B is called an
into function if there is at least one element of
the set B which has no pre-image in the set A. ' - - - - - - - - - - - - - - '
Fig. 10.7 Into/unction
This can be illustrated in figure (10.7)
A B
Remark
• In an into function, Range c Co-domain
(e) One-one into function: Afunction! A ---+B is a
one-one into function if it is both one-one and
into function, i.e., the different points in A are,
joined to different points in B and there are
some points in B which are not joined to any
point in A. Fig.IO.S One-one intofonction
Symbolically: One-one into function is defined as
(i) Rang c Co-domain A B
(ii) j(x\) ,*j(x2 ) =::>x\ ,*x2
(t) One-one Onto Function: ifa jimction fA ---+ B is
both one-one and onto i.e., the different points
in A are joined to different points in B and no
point in B is left vacant.
0+-------+0
Fig. 10.9 One-one onto/unction
Remarks
• One-one onto mapping is also known as bijec- A B
tive or one-to-one.
• For a one-one onto function, Rang = Co-domain
'*
andx\ ,*x2 =::> j(x\) j(x2 )
• j(x\) =j(x2 ) =::>x\ =x2
(g) Many-one Into Function: Afunctionf: A ---+ B
which is both many-one and into function is Fig. 10.10 Many-one into/unction
called a many one into function i.e., two or
more points in A are joined to. some points in B and there are some points in B which
are not joined to any point in A.
Therefore, for many-one into function. A B

~
(i) Rank c Co-domain
(ii)x\ ,*x2 =::> j(x\) = j(x2 ).
(h) Many-oneOnto Function: ifjUnctionf: A ---+B is
both many-one and onto function, then it is many
one onto jUnction i.e., in B, one point is joined Fig.lO.n Many-one onto/unction
to at least one point in A and two or more points
in A are joined to some points in B.
416 Remedial Mathematics

Therefore, for many-one onto function


(i) Range = Co-domain
(ii) XI -:F-X2 ~ j(x l ) = f(x2)

Working Procedure [For Checking the injectivity (one-one) ofthe Function]

STEP KNOWLEDGE
Let x andy be two arbitrary elements in the domain off
Step 1: Takef(x) = fly)
Step 2: /fwe get, X = y, after solvingf(x) = fly)
Then, f: A ~ B is one-one.
Working Procedure [For Checking the Surjectivity (onto) of a Function]

STEP KNOWLEDGE
Step 1: Take an arbitrary element y in the co-domain
Step 2: Putf(x) =y
Step 3: Solve f(x) = y for x and obtain x in terms ofy
Step 4: Get the equation ofthe form x = g(y)
Step 5: Ifx = g(y) belong to domain off,for all values ofy, thenfis onto.

~~~~~~I SOLVED EXAMPLES ~I~~~~~~


Example 1: Letf: R -+ R be a function defined by
3X -1, when x > 3
f(x) = x 2 - 2, when - 2 ~ x ~ 3
{
2x+3, when x <-2
Find (i)f(2), (ii)f( 4), (iii)f(-J), (iv) f(-3).
Solution: (i) f(2) = (2)2 - 2 = 4 - 2 = 2
(ii) f(4)=3(4)-1=12-1=11
(iii) f(-I)=(-1)2-2= 1-2=-1
(iv) f(-3) =2(-3) + 3 =-6 + 3 =-3
Example 2: For thefimctiony = +Fx, say whether it is afunction or not, /fit isfonction,
find its domain and range.
Solution: Here, we havey= +Fx ...(1)
Since y is real if x ~ 0 and is unique and fmite for each x ~ 0
Then (1) is a function with domain [0, CXl]
Again, from (1 ), y ~0 '1/ x ~ 0
Hence, Rang = [0, CXl].
Functions and Limits 417

x 3 _X2 +4x+7
Example 3: Find the domain off (x) = - - - - -
3x+JJ
Solution: Sincefis defined for all real values ofx except when 3x + 11 = O.
-11
i.e., when, x=-3-

Hence, domain of f=R_{-~I}


Example 4: Letf: N - {I} --+ N be defined by f(n) = the highest prime factor ofn. Show that
f is neither one-one nor onto. Also, find the range off
Solution: Since, we have
f(6) = the highest prime factor of 6 = 3
f(9) = the highest prime factor of9 = 3
f(12) = the highest prime factor of 12 = 3
Therefore,Jis a many-one function.
Clearly, image of any n E N - {I } is the largest prime number that divides n. So the range
offconsists of prime number only. Consequently, range off:;t: N (Co-domain)
=> fis not onto function.
Hence,Jis neither one-one nor onto. The range offis the set of all prime numbers.
2
x +3x+ 1
Example 5: State the domain ofthe function f{x) = 2 [UPTU B. Pharma 2007]
x -5x+6
Solution: Obviously f(x) is not defined when ~ - 5x + 6 =0
Now (~-5x+6) =0
=> (x-2)(x-3) =0
=> x =2, or x=3.
Sof{x) is not defined atx =2,3. Hence domainf=R- {2, 3}.
Example 6: Let A = {I, 2}. Find all one-to-onefunctionsfromA toA.
Solution: LetlA --+A be a one-one function. Then forf{I), there are two choices i.e., lor 2.
Let us first suppose f(1) = 1
As f: A --+ A is one-one
So, f(2) =2
Therefore, we have f(1) = 1,/(2)=2.
Now, let f(1) =2
Since, f: A --+ A is one-one, therefore f(2) = 1
Therefore, we have f(1) = 2 andf(2) = 1
Hence, we have two one-one functions say f and g from A to A given by
f(1) = 1,/(2) = 2
and f(2) = 1 andf{I)=2
Example6: LetA = {x fER: -I sX s I} = B. Showthatf: A ~A givenbyf(x) =xl x I is one-
one and onto.
418 Remedial Mathematics

Solution: Letx,y be any two elements in A, then


x;t:y
=> xlxl ;t:ylyl
=> J(x) ;t:J(y)
Therefore,fis one-one
Since, range of J= J(A) = B, soJ: A ~ B is onto mapping.
Hence,fis one-one and onto.

Example 7: Find the domain and range oJthefunction..f(x) = -~-5-6x-x2


Solution: Given that, fix) = -~-5 - 6x - x 2
For,fto bereal-5 -6x-x2;:: 0
=> x2+6x+5 $;0
=> x2+6x $;-5
=> x2+6x+9 $;-5+9
=> (x+3i $;4
=> Ix+3 $;4 e
=> Ix +31 $;2
=> -2 $;x+3$;2
=> -2-3 $;x$;2-3
=> -5 $;x$;-I
Therefore, domain ofJ(x) = [-5, -I]
To find the range ofJ(x), put y = J(x)

Therefore y = -~-5-6x-x2 ,y:S;O


Therefore, T=-5-6x-x2
=> x2+6x+()?+ 5) =0
For real x, discriminant ;:: 0
i.e., (6i-4 x 1 x ()?+ 5) ;::0
=> 36-4;-20 ;::0
=> -4;;::-16
=> ; $;4
=> 1;1 $;4
=> Iyl $;2
i.e., -2 $;y$;2
But Y $; 0, therefore, -2 $; y$;O
Hence, Range of J= [-2,0]
Example 8: Find the domain and range ojthe Junction
1
j(x)= - - -
2-cos3x
Functions and Limits---. 419

1
Solution: We have.f{x) = 2 3
-cos x
It is known that the value of cos 3x always lies between - 1 and 1 for all x E R.
i.e., -I ~ cos 3x::::; 1 "if x E R
Here we also get 2 - cos 3x ;t:. 0 "if x E R
~ .f{x) is defined for all x E R
Thus, domain of.f{x) is the set of real numbers.
Further, we know that the maximum value of cos 3x = 1

i.e., the maximum value of.f{x) = _1_ =! = 1


2-1 1
and minimum value of cos 3x =-1

~theminimumvalueof.f{x)= 2-~-1) =3
Hence, the range of.f{x) is [~, I]
Example 9: Find the domain and range of the following jUnctions
(i) .f{x)=sinx-cosx
(ii) .f{x) = 1 sinx 1
Solution: (i) We havef(x) = sinx-cosx

= Ji(;sinx - ~cosx)
= Jisin( x-~)
Now Domain of sin (x-~) is R (Set of Real Numbers) so domain of.f{x) is R(Set of
Real Numbers)

Range of sin ( x - ~) is [- I, I]

so range of.f{x) is [ -Ji, Ji] from (1)


(ii) Let .f{x) = 1 sin x 1
Domain of sin x is set of Real Numbers (R)
so domain ofl sinx 1 is also of Real Numbers R
Range of sin x is [- I, 1]
so Rang e ofl sinx 1is [0, 1]
Example 10: For a finite set A, iff: A -+ A is a one-one function, show that f is onto.
Solution: Let A = {aI, a2 , ... , an} be a finite set
Since! A ~ A is one-one function, therefore.f{a I ),.f{a2 ), .•. ,.f{an) are distinct elements of
the set A, but A has only n elements. Therefore,
420 Remedial Mathematics

A = (f{a\),j(~), ... ,j(an)}


=> Co-domain = Range
Hence, every element is A (co-domain) has its pre-image in the domain A.
=> f: A => A is onto.

Remark
• For a finite set A, iff A ~ A is onto function, thenfis one-one.

~~~~~~~I EXERCISE 10.1 ~I~~~~~~~


1. LetA = {-2, -1,0,2,1, 2} and! A ~Zgiven byj(x)=~-2x-3
Find (i) the range offOi) pre-image of6, - 3 and 5.
2. Find the domain and range of the functionj(x) = ~(x -1)(3 - x)

3. Find the range of the functionj(x) = (2x _ 3~(X + I)


4. Find the domain and range of the following functions.
2
I II
x _1
(I) j(x)= x-I (iI) Ixl+y=O (iiI) j(x)= :=1 (iv) y= ../x-3
(v) j(x)=~+4
5. If A = {-I, 0,2, 5, 6, Il},B= {-2, 0, 18,28, 108}andj(x)=~-x-2, findj(A).
6. Let A be the set of two positive integers. Let! A ~ zt, set of J)9sitive integers be
defmed byj(n) = p, where p is the highest prime factorofn. Ifrange off= {3}, findA.
7. Find the domain for which the functionj(x) = ~ - 1 and g(x) = I - 3x are equal.
8. Letfi: R ~ R andJi: C ~ Cbe two functions defined as.lj(x) =x3 andJi(x) = x3 . Show
that they are not equal.
9. LetA = {p, q, r, s} andB= {I, 2, 3}.Whichofthe following relations from A toB is not
a function?
(l) R\ = {(P, I), (q, 2), (r, I), (s, 2)} (il) R2 ={(P,I),(q,I),(r, I),(s, I)
(iii) R3 = {(P, I), (q, 2), (p, 2), (s, 3) (iv) R4 = {(P, 2), (q, 2), (r, 2), (s, 2)}
10. Write the following relations as sets of ordered pairs and find which of them are
functions?
(I) {(x,y):y= 3x,x e {1,2, 3},y e {3, 6, 9, 12}}
(il) {x,y):y> x+ I,x= 1,2 andy=2, 4, 6}
(iii) {(x,y):x+y=3,x,ye to, 1,2,3}}
11. Express the following relations as sets of ordered pairs, and find their range
(I) fi:A ~ R:.Ij(x)=~ + I, where A = {-I, 0, 2 ,4}
(il) .Ii: A ~ N:Ji(x) = 2xwhere A = {x: x e N, x 5, 1O}
12. LetfR ~ R be a function such that f(x) 2x. Determine
(I) range off (il) {x:y(x) = I} (iii) whetherj(x+y) =j(x).f{y) holds.
Functions and Limits 421

13. Letf: R+ ~ R. be a function such thatf(x) = log x. Detennine


(I) the image set of domain off (il) {x:f(x) =-2}
(iii) Whetherf(xy) =f(x) +fey) holds.
14. Given an example of a map, which is
(I) one-to-one but not onto (il) not one-to-one, but onto
(iii) neither one-to-one nor onto.

HINTS TO THE SELECTED PROBLEMS

2. Given that f(x) = J(x-I)(3-x)


Forfto be real (x-I)(3 -x) ~o
either (x-I) ~Oand(3-x)~0
or (x-I) :s; 0 and (3 -x):s; 0
x:s; I andx~3
Which is not possible x ~ I any x :s; 3
~ I :S;x:s;3
Hence, domain =[1,3]
To find the range of.f{x) puty = .f{x)

y = J(x-I)(3-x)
I =(x-IX3 -x)
~ I =3x-x2+x-3
~ x2-4x+ (J? +3) =0
For real x, discriminant ~o
~ (-4)2-4xlx(J?+3)~0
~ 16-4(J?+3) ~O
~ 16-41-12 ~O
~ 4-41~0
~ I-l~o
~ I~l
-1 :s;y:s; 1
Range =[-1, I]
s. A = {-1,0,2,5,6, 1I}
B = {-2,-1,0, 18,28, 108}
f(x) =x2-x-2
f(-I) =(-1 2 )-(-1)-2=0
f(O) =02 -0-2=-2
f(2) =(2i-2-2=0
f(5) =(5i-5-2= 18
f(6) =62 -6-2=28
422 Remedial Mathematics

f(l1) = 112_11-2= 108


f(A) = {0,-2, 18,28, lO8}
7. f(x) =2~-1
g(x) = 1-3x
if f(x) = g(x)
~ ~-1 = 1-3x
~ ~+3x-2 =0
~ ~-4x+x-2 =0
~ 2x(x-2) + l(x-2) =0
~ (x-2)(2x+ 1) =0
~ x =2,x=-1/2
domain = {-2,-112}
9. A = {p, q, r, s}, {1,2,3}
(I) RI={(P, I}, (q, 2), (r, 1), (s, 2)
It is a function.
(ii) R2 = {(P, l),(q, l),(r, l),(s, 1)
It is a function.
(iii) R3 = {(P, 1), (q, 2), (p, 2), (s, 3)
Here, element p has two images 1 and 2 which is not possible for the existence of
function.
So R3 is not a function.
(iv) R4 = {(P, 2), (q, 2), (r, 2), (s, 2)
It is a funation.
11. fi:A~R1i()€)=~+ 1 where A = {-1,0, 2,4}
fi(-I) =(-li+ 1 =2
NO) =02 + 1 = 1
fi(2) =22+ 1 =5
.fi(4) =42 + 1 = 17
fi(x) = {(-I, 2), (0,1), (2, 5), (4,17)
(ii) h:A ~ N:fi(x) = 2x
A = {x:x E N,x~ lO}
h(1)=2xl=2 Ji(6) =2x6=12
fi(2)=2 x 2=4 Ji(7) =2x7=14
fi(3)=2 3=6x Ji(8) =2x8=16
fi( 4) = 2 x 4 = 8 Ji(9) = 2 x 9 = 18
fi(5) =2 x 5 = 10 fi(lO) =2 x 10=20
fi(x)= {l, 2), (2,4), (3, 6), (4, 8), (5 ,10), (6,12), (7,14), (8, 16), (9,18), (10, 20)
12. f:R~R:f(x)=2x.
(i) Range off= R+, the set of positive reals.
Functions and Limits 423

(ii) {x:j(x) = I}
j(x) ='2:
I ='2:
2° ='2:
x=o
(iiI) To checkj(x + y) =j(x).j(y)
j(x + y) =2x+Y =2x . 2Y
=j(x).j(y)
Hencej(x + y) = j(x) .j(y) holds.

I ANSWERS
1. (i) j(A)= {-4,-3,0,5} (ii) <\>, to, 2},-2
2. Domain = [I,3],Range=[-I, I]
-8
3 ]- 00, 25 ]u[O, oo[
4. (i) R-{l},R-12}, (li) R ;R-R+
(iii) R- {I}, {-I, I} (iv) [3,00[, [10, 00 (v) B, (4, oo[
s. j(A)={0,-2, 18,28, 108} 6. A = {3, 6} or {3, 9} or {3, 12} etc.
7. {-2, II2} 9. (iii)
10. (i) {(1, 3), (2, 6), (3, 9), function,
(it) {(I, 4), (1, 6), (2, 4), (2, 6)}. Not function,
(iiI) {(O, 3), (1, 2), (2, I), (3, O)}, function
11. (i).fj = {(x,j(x»: x E A} = {(-I, 2), (0, I), (2, 5), (4, 17)}
(it)J2= {(x,g(x):x E A}
12. (i) Range of/= R+, the set of positive real numbers,
(it) {x:j(x) = I} = to},
(iiI)j(x + y)= j(x) .j(y) holds for allx,y E R
13. (i) R,
(it) {e-2 }
(iii) holds
14. (i) n -+ if :N-+N,
(it) n -+ Inl: Z -+ Nu to}
(iii) n -+ In12: Z -+ Nu {O}.

II1II SOME PARTICULAR FUNCTIONS


(a) Even Function: Afunctionf A -+B is said to be an evenfunction ifj(-x) = j(x)/or all
xEA.
424 Remedial Mathematics

For Example: f(x)= x?+l f(x)


2
fe-x) =x + 1
Therefore, f(x) = j(-x)
Here, f(l) = 12+ 1 =2
f(-I) = (-Ii + I =2
The graph of an even function is symmetric about the
-2 -1 2
functions axis.
Fig. 10.12
In Fig. (10.11), for the functiony= j(x), if we take any
f(x)
positive value of x, e.g.x = a,y= j(a). For the same value of
x, but in the negative axis i.e., x = - a we gety = f(-a). We
can observe that,f(a) = f(-a) i.e., for both positive and
negative values of x, we get a positive value of y. Thus
y = f(x) is an even function. ---...-----.,.---!f-----,--r-- fey)
Similar agrument holds if we get a negative value ofy -2 -1 0 4
for both negative and positive values of x i.e., y = If(x)\
Similarly, in fig.(10.12), x is a function ofy and the
same argument holds for x = f(y).
Fig. 10.13
(b) Odd Function: Afunctions! A -+B is said to bean odd
f(x)
-x if x < 0
function ifj(-x) = j(x), { x if ~ 0 X E A

ForExample:f(x)=x3 ~ j(-x) = (_x)3 =_x3 =-j(x)


The graph of an odd function is a double reflection,
-2 -1 0 2
first in the function axis and then in the other axis.
For the functiony = f(x), if we take any positive value
ofx e.g.x = a,y = f(a). For the same value of x, but in the
negative axis, i.e., x = - a we get y = fe-a}. In the graph
Fig. 10.14
(10.13), weobservethatf(-a) =-f(a).
Remark
• There are functions, which are neither even nor odd. e.gf(x) = x 3 + x? + x + 5
(c) Inverse Function: Let! A ~ B be a one-one onto function, then afunction g: B ~ A
which associates each element y E B to a unique element x E A, such that j(x) = y is called
the inverse off, i.e.,
f(x) = y<=> g(y)=x
It is denoted by;1 i.e.;1 (y) = x<=> j(x) = y A B

For Example:f(x) = 2x, where x

2
E B.

rl(x} = ~, where x E B.
G+------+l

Fig. 10.15
®
Functions and Limits 425

Remarks
• The inverse functionr 1: B ~ A is defined only whenfis one-one onto; further, if
inverse functionf- 1 exits, then it is also one-one onto.
• The inverse relation of function is symmetric.
(d) Constant Function: Afunctionf:A -+ B is said to be constantfunction ifeach element oj
domain is associated with a single element of the co-domain B. This can be illustrated in
the following figure.

A B

~
-3 -2 -1 0 2 3
-1

-2

-3

Fig. 10.16 Fig. 10.17


Graphically, a constant function will be a straight line parallel to either of two axes.
For Example:f(x) = 3 i.e.y=3
(e) Step Function: A fimctionf: A ~ B is said to be step function iff(x) = [x] where [x]
denotes the greatest integer less than or equal or x for all x E A.
For Example: If f(x) = [x],f([2.46]) =2,f([3])=3,
f([-2.29]) =-3
Graphically, it can be represented as follows;
f(x)
f(x) =0, when O::;;x < 1

,,
f(x) =1, when 1 ::;;x<2 "
I
"
I I I
f(x) =-1, when-l ::;;x<O , ,I----t

,,
,
"
"
"
..--,
Remark
,, "
" ,
,,
,~

• Step function is also known an integer "


"
function. ,
I----t ", ,

(1) Modulus Function: Afimctions f: A -+ B is -2 -1


,' "
"-"":"--'-- X
-,..---,.--+--''---'--..:..'
said to be modulus function, iff(x) = lxi, i.e., ,, 1 3 4
f(x) takes only the magnitude ofx. ,, -1
,, ,
-x if x < 0 ,,
I----t -2
Symbolically: f(x) = >0
{ Xlif x_ ,
Fig.l0.t8
426 Remedial Mathematics

The graph of the modulus function ,f(x) = Ix I is shown in Figure

-4 -3 -2 2 3 4

Fig. 10.19
Here, /(1) =/(-1)= I
/(2) =/(-2) =2
/(3) =/(-3)=3
Remark
• Modulus Function is always an even function.
(g) Linear Function: Afunctionf A ~ B o/the/orm/(x) = ax + b, where a, b E R is called
a linear function.
The graph of a linear function is always a straight line
For Example: Graph of/(x) = 4x - 2 is shown in Figure

-4 -3 -2 -1 3 4
-1

-2

Fig. 10.20
Functions and Limits 427

x o 2
f(x) 2 -2 6

(h) Quadratic Function: Afunctionf A ~ B is called a quadraticfunction ~fit is oftheform


y = d + bx + c, where a, b, c E R, a ;rOo
f(x)

-4 -3 -2 -1 234
Fig. 10.20 (a) f(x)
The graph of such a function is called parabola.
For Example:Graph of f{x) = 2x2 is shown in
(figure 10.20)

f~)
-2 -1 2

I 8 2 2 8

(i) Exponential Function: Afunctionf A -+ B ofthe


form f(x) = cr,
where a > 0 and x E R is called an
exponential function.
The exponential function d will never be negative -3 -2 -1 2 3
for all a> 0 and x E R. Therefore, domainoff(x) isR
Fig. 10.21
and range off(x) is the set of positive real numbers.
For Example: The graph of the exponential function.f{x) = 2x is shown in Fig. 10.21

-2 -1 o 2 00

1/4 112 2 4

G) Logarithmic Function: For any a > 0, a ~ 1. a function f(x) defined by f(x) = logax,
.: x> 0 is called logarithmic function.
By definition oflogarithms, we have
aY=x<=>logax=y
428 Remedial Mathematics

Here it is clear that, x >0 for all Y E R and a> 0, a::;; I. So j(x) is defined for all x> 0. Thus.
domain ofj(x) is the set of positive real numbers.
The graph of the logarithmic function is shown in fig 10.22.
f(x)

f(x) =loga x, a > 1

--~~---*--------------------
o __ x

Fig. 10.22

III ALGEBRA OF FUNCTIONS


The algebraic operations of addition, subtraction, multiplication and division, yield new
functions. Let us see the following definitions.
Definition: Let f and g be two real valued functions with domain D} and D2 respectively.
If D = DI (') D2 ::;; $, then
(I) The sum function, denoted by f + g, is defined by
if+ g) (x) = j(x) + g(x), with domain D.
(ii) The difference function, denoted by f - g is defined by
if-g) (x) = j{x)- g(x), with domainD.
(iii) The product function, denoted by f g is defined by
ifg) (x) = j(x) . g(x), with domain D.

(iv) The quotient function, denoted by f is defined by


g

f)(x) = f(X» ,with domain D', where


(g g(x
D' = {x:x E D,g(x)::;;O}::;;$
1
(v) The reciprocal function denoted by f is defined by

(~ }X)= f~X)' with domain D", where


D"= {x:x E Dl'j{x)::;;O}
(vi) If c is any real number, then scalar multiple off by c, denoted by cf, is defined as (cf)
(x) = cj{x), with domain D}
(vii) Iff is a function, then f f is denoted by f2,j2! is denoted by f3 and so on. Also
ift(x) = {f{x)}n, withdomainD}.
Functions and Limits 429

IlI!I COMPOSITION OF FUNCTIONS


Let f: A ~ Band g: B ~ C be two real valuedfunctions. Then the composition offand
g denoted by g of, such that g of: A ~ C is defined by
(go j)(x) =g(f)(x»
This is also known as function of a function or resultant of a function.
Similarly, (fo g) (x) = f(go ex»

Remarks
• Iff:X ~ Y is a one-one onto mapping, thenfo rl = Iyandf-l 0 f= Ix
• If f:X ~ Y and g:y ~ Z be two one-one onto mappings, then the mapping of
go fis also one-one and onto.
• go fmay exists whilefo gmay not exists.
• If g ofand f o g both exit, they may not be equal.
• g ofexists ifand only if the range offis a subset of domain of g. Simiiarly,Jo gexists
if range of g is a subset of domain off

General Theorems
Theorem 1: The composition offUllctiolls is associative.

Remark
• The composition offunctions is not commutative, i.e.f° g:l= g 0 f
Theorem 2: The composition ofany function with the identity function is thefunction itself.
Theorem 3: Letf: A ~B, g: B~A betwofonctionssuchthatgof=IA . Thenfisan ilyection
and g is a Surjection.
Theorem 4: Letf A ~ Band g: B ~ C be two functions, then
(i) g 0 f: A ~ C is onto => g : B ~ C is onto.
(il) g 0 f: A ~ C is one-one => f: A ~ B is one-one.
(iii) g 0 f: A ~ C is onto and g : B ~ C is one-one => f: A ~ B is onto.
(iv) g 0 f: A ~ C is one-one andf: A ~ B is onto => g : B ~ C is one-one.
Theorem 5: The inverse ofbijectivefunction is unique.
Theorem 6: Iff A ~ Band g: B ~ Care two bijectivefunctions, then g of: A ~ C is a
bijection and

ISOLVED EXAMPLES I
Example l(a): IffR~R isdefinedbyj(x) =x2 -3x + 2,findf(f(x».
Solution: Since j(x) = x2 - 3x + 2
Therefore, j(j(x» = j(x2 - 3x + 2)
=(x2-3x+2i-3(x2-3x +2)+ 2
=x4 +9x2 +4-6x3 -12x+4x2-3x2 + 9x-6 + 2
= x4 - 6x3 + 1Ox2 - 3x
430 Remedial Mathematics

Example l(b): LetfN-+R beafunctionsuchthat.f{x) =2x-3 andg: Z -+R beafunCtion


x-3
such that g(x) = --. Find g of N -+ R.
2
Solution: Consider, (go.f) (x) = gf(x) = g(2x- 3)
= 2x-3-3 = 2(x-3) =x-3
2 2
Example2: LetA = {I, 2, 3, 4, 5}, LetfA -+A andg: A -+A be defined by
f(1) = 3./(2) = 5./(3) = 3./(4) = 1./(5) = 2
g(1) = 4, g(2) = 1, g(3) = 1, g(4) = 2, g(5) = 3.
Find (10 g) and (g oj).
Solution: Here, we have,
«0 gXI) =.f{g(1»=/4) = I
«0
g)(2) =.f{g(2» =.f{I) = 3
«og)(3) =.f{g(3»=.f{I)=3
«0
g)(4) =.f{g(4» =.f(2) = 5
«0
g)(5) =.f{g(5» =.f(3) = 3
(g0.f)(1) =g(f(l)=g(3)= I
(go/)(2) = g(f(2» = g(5) = 3
(go.f)(3) = g(f(3» = g(3) = I
(g0.f)(4) = g(f(4» = g(I»=4
(go.f)(5) = g(f(5» = g(2) = 1.
Example 3: Let j. g: R -+ R be two functions defined by

f(x) = ../x-I andg(x) = ~4-x2 'If x ER.


Find (i) f+ g (ii) g +f (iii) f- g
(iv) g-f (v) fg (vi) gf
(vii) fig (viii) glf
_ Solution: Since, we have

.f{x) = ../x-I andg(x)= ~4_x2


Therefore, the domain off= [1, 00 [ = D J (say)
and domain of g = [ -2,2] = D2 (say)
Define D = D J I1D 2 = [1, 00 [11 [-2, 2] = [I, 2] *~
Then, we have

(I) «+gXx)=.f{x) + g(x)= ../x-I + ~4-x2 ,withdomainD

(il) (g +j)(x) = g(x) +.f{x) = ~4 - x 2 + ../x-I, with domain D

(iii) «- g)(x) = .f{x)- g(x) = ../x-I- ~4-x2 ,withdomainD

(iv) (g-j)(x) = g(x)-.f{x) = ~4-x2 - ../x-l


Functions and Limits 431

(v) (fg)(x)=1(x) . g(x) = ..Jx-l· ~4_X2 = ~(x-l)(4-x2), with domain D


(vi) (gf)(x) = g(x) .1(x) = ~4_x2 . ..Jx-l = ~(4-x2)(x-I) ,withdomainD

(vii) The domain of f is obtained by deleting those points x at which g(x) = 0 from D.
g

Therefore, the domain of f isD'= {x:x E D,g(x);eO} = [1,2]


g

f)(x) = f(x) = ~ =
(g g(x) ~4-x2
~( X-I)
4-x2

(viii) The domain D" of ~ is given by

D' = {x: x E D,j{x);e O} = ]1, 2]

Th""for~ (; }Xl ~ ;~:~ ~ ~~a ~-:12)


Example 4: Iff(x) =x2 and g(x) = 3x. Find the value of(g Of) for x = 1, 2, 3
Solution: Here, we have
f(1)=1 2 =1
(gof)(l) =g(f(l»=g(l)=3 xl =3
f(2) =22 =4
Therefore, (gof)(2) =g(f(2»=g(4)=3 x4= 12
Now, 1(3) =3 2 =9
(gof)(3) = g(f(3» = g(9)= 3 x 9=27.
This can be illustrated in (figure 10.23)

A B c

Fig. 10.23
3
Example 5: Show that bf: R - (OJ ~ R, given by f(x) = -
x is invertible and it is. inverse of
itself
Solution: (i)fis one-one:
Letx,y E R- to} such thatf(x) =f(y)
Then, f(x) = f(y)
3 3
x y
x=y
432 Remedial Mathematics

j(x) =j{y)
=> x=y
Since, x,y are arbitrary, therefore,j(x) = j{y)
=> x=y'ilx,y
=> f is one-one
{ii)fis onto
Let y be an arbitrary element of R - {O}, then-
j(x) =y
3
=> - =y
x
3
=> x= -
y

Therefore, for eachy E


.
R - {O}, there eXIsts -3 E R - {O}, suc h that
y

j(x) = f(~) = _3_ = Y


Y 3/y
=> fis onto.
Therefore,f is bijective and hence invertible.
To find;l, letj(x) = y, then
j(x) =y
3
=> - =y
x
3
=> x=-
y

=> ;1 (y) = ~
y
3
=> ;I(x) = - = j(x).
x
Hencefis the inverse of itself.
Example 6: Ifthefun.ctionfR ~ R is given by j(x) = x 2 + 2 and g:R ~ R be given by g(x)
x
= --1' Findfogandg of
x-
Solution: Here, we obse\'Ve that, the range off= domain of g
and range of g = domain off
Therefore, fog and g a fboth exist.

Consider, (fog)(x) =j(g(x» = 1 ~)=(_x_)2 =~+2


J~x-I x-I (x-I)
x 2 +2x 2 +2
and (g oj) (x) = g(/{x» = gC-~ + 2) = 2 =2- -
(x +2)-1 x +I
Functions and Limits 433

Example7: lffR-+Rbegivenby

2 2
f(x) = sin x + sin ( X + ~ ) + cos x cos ( .x + ~ ). V X E R
andg: R ~ R be such thatg (5/4) = 1
then show that g of:
R ~ R is constant function.
Solution: Given that f(x) = sir?x + si n2 (x + n/3) + cosxcos(x + n/3)

= ~[2sin2 x+2sin2 (x+~ )2cosxcos( x+~)]


= ~ [1- cos 2x + 1- cos ( 2x + 23 1t
) ~
+ cos ( 2x + ) + cos ] ~
= ~ [% - cos 2x - cos ( 2x + 231t
) + cos ( 2x + ~) ]

= ~ [ %- {cos 2x + cos(2x + 23 1t
) } ~
+ cos ( 2x + ) ]

= ~[%-cos( 2x+~ )cos~+ cos( 2X+~)]


= ~ [% - cos( 2x + ~ ) + cos( 2x + ~ ) ] = ~ V X E R

Now, g o.f(x) = g(j(x» = g(5/4) = 1


=> g oj(x) = 1, V X E R.
Hence, g of: R ~ R is a constant function.
Example 8: Iff R -+ R be a jUnction given by f(x) = ax + b, \;j X E R. Find the value ofa
and b such that fO f = lIt
Solution: Here, we have fof=IR
which implies, foj(x) = fix), V x E R
=> j(f(x» =x, V x E R(:.fR(x)=x)
=> j(ax + b) =x, V X E R
=> a(ax+b)+b=x, V XER
=> (if-l)x+ab+b=O,VxER
=> if - 1 = 0 and ab +b = 0
=> a =± 1 andb(a+ 1)=0
If a = 1, then b = 0
If a=-I,thenb(a+l)=O, V bER
Therefore, a = -1 and b may take any real value.
Hence, either a = 1 and b = 0, or a = - I and b can take any real value.
434 Remedial Mathematics

Example 9: Which ofthefollowingfunctions are odd or even or neither?


(I) j(x) =tanx+3 cosecx+x
(ii) j(x)=lxl+ 1
(iii) j(x) = Ix-21
Solution: (i) Here, we have;f(x) = tan x + 3 cosec x + x
~ fe-x) =tan(-x)+3 cosec(-x) + (-x)
= -tan x - 3 cosec x - x
= -(tanx + 3cosec x + x) = -f(x)
Therefore,J(x) is an odd function.
(ii) We have,J(x) = Ix I + 1
~ fe-x) =I-xl+ 1 = Ixl+ 1 =f(x)
~ j(x) is even function.
(iii) We have, j(x) = Ix-21
~ j(-x) = l-x-21 =1-(x+2) I = Ix+21
~ j(-x) #=j(x)orft.:-x)#=-j(x)
Therefore,J(x) is neither even nor odd function.

Example 10: Iff(X)=tOg (I+X) showthatf(x)+fty) = f(x+y)


I-x l+xy
[UPTU B. Pharna 200SJ
Solution: It is given that

j(x) =10g(I+X) ...(1)


I-x

~ fiy)=10 g (I+ Y ) ...(2)


l-y
Adding (1) and (2) we get

f{x)+fiy) = 10g(I+X)+ 109(l+ y


I-x l-y)
I
= log[Cl + x)(1 + y)] = [1 + x + y + XYJ ...(3)
(l-x)(l-y) l-x-y+xy

Again f(~)l+xy
=[I+2yj
l_x+y
l+xy

= IOg[l+X+ y++xy] ...(4}


l-x- y+xy
Using (3) and (4), we conclude that

j(x)+fiy) =1 x+ y)
J~I+xy
Functions and Limits 435

Example 11: Ifis an identity function what isfe j? What isff


Solution: Letfbe an identity function on a set X. Thisf: X ~Xis identity if
.f(x) = x 'V X E X ...(i)
fe.f(x) =j{f(x)} =.f(x) from (I)
=x from (i)
So fe.f(x) =x
Again .fI(x) =.f(x).f(x) = x. x
From (I) =x2

Example 12: Iff(x) = x -


3
:3 then show thatf(x) = feu = o.

. gIven
Solution: It IS . t h at j(x) = x
3 1
-'3
x

So .f(x) = f(~)

(x :3) {(~J -C:xJ}


3
_ =

3 1 1 3
x -- 3
=--x
3
x x

1
x3 = -
3
x
=> x6 = 1
=> x=1
1 =1--=0
1
Now {(f(x)} =13__
x= I 13 1

{f(~)L=1 = OJ _1 =0 3

So .f(x) =~~) =0.


Example 13: Letf: A ~ B such that.f(x) = x- 1 and g: B ~ C such that g(y) = y.. Find
fe g(y).
Solutio,n: Given that .f(x) =x-l andg(y)=y
Then feg(x) =j{g(x)} =.f(x2)=x2-1
So feg(x) =x2-1.
436 Remedial Mathematics

I x-I
Exercise 14: Given thatf(x) = --, g(x) = - - . Find the value ofg[f(x)].
I-x x
I x-I
Solution: j(x) = - , g ( x ) = -
I-x x

So g[f(x)] =g[I~J=[~IJ= I-I~~X) =x

I-x (I-x)
Hence g[f(x)] =x.

~~~~~~~I EXERCISE 10.2\~~~~~~~~


1. If A = {a, b, e, d} andfcorresponding to the Cartesian product {(a, b), (b, d), (e, a),
(d, e)}. Showthatfis one-to-one from A ontoA. Find]l.
2. If A is anon-empty setandf, g: A ~A, such thatfo g= go f= IA' Show thatfand g are
bijections and g = ]1.
3. LetX= {-2,-I, 0,1,2, 3} and Y= to, 1,2, ... , 10} andf:X ~ Ybe a function such that
j(x) =~, V x EX, Findjl (A), where A = {O, 1, 2,4}
4. Find the inverses of the following functions, if exist
I x-I
(I) j(x)=--x+4 (ii) j(x)= - ,x* 1
3 x+l

(iii)j(x) = ~I-x2 ,o~x~ 1


5. IfA = {a, b, e, d} andfcorrespondstothecartesianproduct {(a, b),(b, d),(e, a),(d, e)}.
Show thatfis one-one from A onto B. Find]l.
6. Iff R ~ R is a bijection given by j(x) = ~ + 3, Find]1 (x).
7. IffR ~ R is defined byj(x) = 3x-7. Showthatfis invertible and find]l.
8. Let fA ~ B be a function, such that
(I) A = {0,-1,-3, 2}, B= {-9,-3, 0, 6} and.f{x)= 3x. Find]l.
p
9. Letf R ~ R be given by.f{x) = (x + 1 -1, x ~ -1. Show thatfis invertible. Also find the
setS= {x:.f{x) =]1 (x)}.
10. Find]1 (3), if exist, when.f{x) = x 3 +4, wheref: R ~ R.
11. Iff= {(5, 2), (6, 3)}, g= {(2, 5), (3, 6)}. What is the range offandg? Findfo g.
12. If.f{x) = ~ - 1, g(x) = 3x + 1, then describe the following function:
(I) g 0 f (ii) f o g
(iii) go g (iv) fo!

13. If.f{x)= X-I, verify that ifo]l)(x) =x.


x+l
14. Iff R ~ Rand g: R ~ R are defined by.f{x) = x + 2 and g(x) = ~ +5. Findfo g and
go!
Functions and Limits 437

15. Iff and g are two real valued functions such that.f{x) = :x? - 5 and g(x) = 2x + 3, Find
fog.

16. Letf: R ~ R be defined by f(x) = +' x +1


Findf(f(2))

1
17. Iff(x) = - , showthatf(f{f(x)})=x.
I-x

18. Iff R~ R, where.f{x) =:x?+ 2 andg: R ~ R, whereg(x) = 1__1_, then find


I-x
(I) fog (ii) gof
°
19. LetA = {x E R: ~x ~ 1}, iffA ~A is defined by
X, if x E Q
j(x) =
{
I-x, if x~Q

then show that (f0 f)(x) = x, v X E A.


20. LetfZ ~ Z and g: Z ~ Zbe defined by.f{n) = 3n, v n E Z and g: Z ~ Zbe defined by
n / 3, if n is a multiple of \:f n E Z
g(n) = { 0, if n is not a multiple of 3 \:f n E Z
Show that gof=Izandfo g::tlz ·

HINTS TO THE SELECTED PROBLEMS

1. A = {a, b, c, d} andfA ~A

A x A = {(a, b), (b, d), (c, a), (d, c)}


It is clear that under J, each element of A is mapped to
a unique and every element of A so f is one-one and
onto
rl = {(b, a), (d, b), (c, a), (d, c)} Fig. 10.24
I
4. (i) .f{x)= -3"x+4

Let y = -'!'x+4. Solving for x


3
=> x =3(4-y)= I2-3y
So rI(x) =12-3x
x-I
(il) .f{x) = -, x ;r-I
x+I
x-I
Let y=-
x+I
(x+ l)y =(x-I)
xy-x =-(1 +y)
x(y-I) =-(1 +y)
I 438 Remedial Mathematics

_ -{I + y) 1+ Y
x- =-
y-l 1- y

so rl (x) = l+x ifx¢ 1


I-x

(iiI) f(x)=~I-x2 O:s;x:s;1

Let y = ~1-x2
Solving for 1 = l_x2
x2 = 1-1

x= ~1-i
so rl(x) = ~1-x2 ,0:S;x:S;1
8. (i) f:A-+B
A = {0,-1,-3,2} B= {-9,-3,0,6}
fix) = 3x, show that
Herej(x) is one-one onto (bijection) sorl exist
rl (x) = {CO, 6), (-3,-1), (-9, -3), (6, 2)}
(il) fA -+ B:j(x) = x2
A={1,3,5, 7,9},B={0, 1,9,25,49,81}
Herej(x) is not bijective sincej(x) is not onto (0
is the element is B which is not mapped by any
element of A underfsor l orfdoes not exist.

15• .f{x)=x2-5,g(x)=2x+3
A B
fo g(x)=j{g(x)} =j(2x+ 3)=(2x+ 3i-5 Fig.lO.2S
=4x2+9 + 12x-5 =4x2+ 12x+4
1
17. f(x) = -
I-x

fiflf(x)4} = f(fC~J=f(I_~l=f(~l
I-x I-x
I-X) 1 1 x
= f ( -=;- = 1_(I-X) = 1+ (I-x) =x+(I-x) =x
-x x
19. Wehavef:A -+AwhereA = {x E R, 0 :S;x:S; l}
X, if XEQ
f(x) =
{ I-x, if XIi!OQ
Fllnctions and Limits 439

JV(x)} = {f(X), if X e Q
f(1-x), if xr£Q
X, if xeQ
= { l-(1-x), if xr£Q

= {X, if xeQ
x, if xr£Q
so j(f(x)} = x, V X eA.

I ANSWERS
1. j l = {(b, a), (d, b), (a, c), (c, d)}
3. jl(A) = {O, -I, I, -2, 2}

4. (i) j l (x) = 12 - 3x, (ii) jl(x) = 1+ x


I-x
5. r l = {(b, a), (d, b), (a, c), (c, d)}
x+7
6. rl(x) = (x - 3)1/3, V X e R 7. rl(x) = -3-

8. (i)r l = {(-9, -3), (-3, -1), (0, 0), (6, 2)},


(ii) I-I does not exist
9. S= {O, I}
11. fog(2)=2,fog(3)=3
=>j{range) = (2,3) g(range) = (5, 6)
12. (i) gof=3:x?-2 (il) 9:x?+7x
(iii) 9x + 4 (iv) x4 - 2x2
14. gof=~+8x+ 13,fog=~+7

15. 4:x?+ 12x+4 16. ~


29

• CONCEPT OF LIMIT
Consider a function
2
j(x) = x - 9 + 6x + 18
x 2 -6x +9

The value ofj{x) at x = 2 is of the form Q, which is meaningless or indeterminate.


o
Therefore, in this case, we cannot divide:x? - 9 by x - 3, because at x = 3, x - 3 is zero. -
Now suppose x is not exactly equal to 3 but x tends to 3. Then x - 3 is not equal to zero,
thus in this case we can divide the numerator:xl- - 9 by denominator x - 3.
440 Remedial Mathematics

We therefore obtain
2
fix) = x -9 = (x-3)(x+3) =(x+ 3).
x-3 x-3
Now, if x is very close to 3, thusj(x) comes to 6

x:~: -61 = 1x2:~;6X 1= I


1 (::t I
=lx+31·
Now Ix + 3 I can be made as small as we please by letting x tends to 3.
Hence from the above observations, we observe that when x takes the fixed value 3, the
value ofj(x) comes to be a meaningless number but when x tends to 3.j(x) tends to 6. Which
in fact gives the limit ofj(x) as x tends to 3.
Mathematically, we can write
. x 2 _9
Llm-- =6.
x-t3 x-3
Limit ofJ{x) at x = a. In many cases we obtain k, = ~. k (say).
Then we call kthe limit ofj(x) atx = a.
Mathematically, we write
lim f(x) = k.
x-ta
Some Important Results on Limits
Ifj(x) and g(x) are two function, then
(i) Lim [f(x) ± g(x)] = Lim f(x) ± g(x) .
x--+a x-ta

(ii) Lim[f(x)· g(x)] = {Lim f(x)} {Lim g(x)} .


x-ta x--+a x-ta
Lim I(x)
(iii) Lim f(x) = X--+a , with Lim g(x)"* a.
x-ta g(x) Lim g(x) x-ta
x--+a
(iv) Lim [cj(x)] =c Lim j(x).
X~(I x-+a

Some Standard Limits


x" -I x"-a"
1. Lim-- =n. 2. Lim = n·a"-'.
x--+' x-I x--+a x-a

3. Lim(I+.!. J =e.
x-tro X
4. Lim(I+~J
x--+roX
=~.
L. log a (1 + x)
5. Lim (I + x)'/x =e. 6. 1m = logae.
x--+o x --+ 00 x
. aX-I L. sinS
7. Llm-- =Ioga 8. Im-- =1
x--+o x e 8--+0S .
L. tanS L. logx
9. Im--=l 10. Im-- =0.
8--+0S . x--+oo X
Functions and Limits 441

~~~~~~~I SOLVED EXAMPLES I~~~~~~~


Example 1: Evaluate thefollowing limits:
(I) lim (x 3 -xt + 1) (ii) lim (I + x+.x2 + .... + X 10)
x~I x~-I

4 3 . ax 2 +bx+c
(iii) lim ~ (iv) hm 2 ' a + b + C *- O.
x~4 x-2 x--->I cx +bx+a
Solution: (i) lim (~-.x2+ 1)= 1 _1 + 1 = 1-1 + 1 = 1.3 2
x~l

(ii) lim (I + x + .x2 + ... +x IO ) = 1 + (- 1) + (-1 i + ... + (-I) 10


x~-I
=1-1+1-1+ ... +1=1.
4 +3 lim(4x+3) 4x4+3 19
(iii) lim _x_ _ = .::!.x~-=-4_ __
x~4 x-2 lim(x-2) 4-2 2
x~4

2
r
ax +bx+c ax(I)2 +b(I)+c
---,:....:.,..._...0....:._ =
a+b+c
=I
(iv) x~ cx +bx+a
2 c(l)2 +b(I)+c a+b+c .
Example 2: Evaluate the following limits:
. 3x 2 -x-lo . (2x-3«Fx -I)
(i) hm ---,,-.-- (ii) I1m ~---,,--'-'-----'-
x~2 2 x -4 x--->I 2x2 +x-3
4
(iii) lim x _81
x--->32x 2 -5x-3
2
. 3x -x-IO (x-2)(3x+5)
Solution: (i) hm 2 = lim -'-------'--'-----'-
x--->2 x -4 x~2 (x-2)(x+2)

= lim 3x+5 =3x2+5 =!.!.


x~2 x+2 2+2 4
· (2x-3)(Fx -1) I. (2x-3)(Fx -1)
(ii) I1m = 1m ~-"":"":-----'-
x--->I 2x2 +x-3 x~1 (2x+3)(x-I)

= lim (2x-3)(Fx-I) lim _....:('--2_x_-~3:"..)_


x--->I (2x+3)(Fx +1)(Fx -I) x--->I (2x +3)(Fx + 1)
2xl-3 -1
----,=--- = --- =-
(2 xl + 3)(.Ji + 1) 5x2 10

· x4 -81 I. (x-3)(x+3)(x 2 +9)


(iii) I1m 2 = 1m ~-'-...:....--":"":""-----'-
x~32x -5x-3 x--->3 (x-3)(2x+l)
2
= lim (x+3)(x +9)
x~3 (2x+1)

(3+3)(3 2 +9) 6x18 108


= =--=-
2x3+1 7 7
442 Remedial Mathematics

' x4 -4 I' (x 2 +2)(x 2 -2)


(iv) \1m 2 = 1m ~--;=i-'----;:,=-
x~.J2 x +3J2x-S x~.J2 (x+4J2)(x-J2)
2 2
= lim (x +2),(x+J2)(x-J2) = lim (x +2),«x+J2)
x~.J2 (x+4J2),(x-J2) x~.J2 x+4J2

_ (2+2)(J2 +J2) _ sJ2 _ s


- (.fi +4.fi) - s.fi -S'
Example 3: Find the following limits

(it) lim J"m - J2a


x~a x-a
, ,/3-x-l
(iii) h m - - -
x~2 2-x
Solution: (i) Here, the function involving surds, so rationalizing the numerator

,
I1m
.J1;7 -~
2 =
I'
1m
[JI;;2 -Pi[JI;;2 +~]
r:--:; r:--:;
x~o X x~o x 2 [vfl+x 2 +vfl-x 2 ]

' 2
= \1m ---;==--===-
X~o[~ +J17]

2 =~ = 1
[~+.J1-O] 2 '
(ii) Diagonalizing the numerator, we ha"e

' ../x+a -fi; \' [../x+a -fi;][../2+a +..!2a]


\1m = 1m -=-------===,---=:----=-
x~a x-a x~a (x-a)[../x+a+fi;]
' (x+a)-2a
= \1m ~---;==,..---;===-
x~a(x-aH../x+a +fi;]
' 1
= \1m ---,,==---.==-
x~a[../x+a +fi;]
1 1
= ../a+a +fi; = 2J2a '
Functions and Limits 443

(iii) Rationalizing the numerator, we have


. .J3-x-I . [.J3-x-I][.J3-x+l]
11m = lim !:....:-c---~===--~
x-+2 2-x x-+2 (2-x)[.J3-x +1]
. (3-x)-I
= hm - - - - ; = = = - -
x-+2 (2 -x)[.J3 -x + 1]
. 2-x
= I1m - - - - ; = = = - -
x-+2 (2 - x) [..13 - x + I]
. 1
= IIm-..,,=~-
x-+2[.J3-x+I]
1 1
-=-
[..13-2 +1] 1+1 2
Example 4: Evaluate
x bX . eX_e- x
(/) lim a - (ii) h m - - -
x-+o x x-+o x
x bX . aX -I_b x +1
Solution: (i) lim a - = 11m - - - - -
x-+o x x-+o X
X
. [aX - 1 b - 1]
= 11m - - - - -
x-+o x x
. aX -1 . bX_I
= IIm---hm--
x-+o x x-+o X
= loga -10gb = log(a / b).
X -x . e2i -1 . e2x -1 2
(ii) lim e -e = hm--= h m--·-
x-+o x x-+o xe x x-+o 2x eX
2 2
= Ix- = Ix- =2.
eO 1
n
Examp Ie:
5 F In . . .Integer n so that I·1m -
· d the posItIve _3-
xn - = 108 .
x-+3 x-3
n 3n
Solution: Since lim ~ = 108
x-+3 x-3

n(3)'1-1 = 108

=> n.(3)n-1 =3(3)4--1


=> n =4.
Example 6: Evaluate
·log(I+x 3 ) . I-cosx·.Jcos2x
(I) I1m -='---- (iz) I1m 2 .
x-+o sinx x-+o x
sinx-cosx . 1+ cos2x
(iii) lim (iv) I1m 2.
x-+7t/4 x-1t/4 x-+7t/2(1t-2x)
444 Remedial MathemaIlCS

0Iog(1+x3 ) 10Iog(1+x3 )
Solution: (I) 11m 3 = 1m
0 3 0

x~o sm x x~o 3 sm x
x -3-
x

lim 2- log (l + x 3 ) ( 3 )1/xl


= x~Ox3 = lim log l+x
o (sin x)3
hm - -
x~o (1)3
x~o X

= loge =! = 1.
1 1

(
ii) r l-cosxJroS2x r l-COSXJroS2X[1+COSX~]
x~ x2 = x~ x2 1+cosx.Jcos2x

0 l-cos2 xocos2x
= \1m --:------==~
x~o x (1 + cos x.Jcos 2x )
2

0 l-cos2 x(1-2sin 2 x)
- \1m ---,----==~
- x~o x 2 (1 + cos x..Jcos 2x )

0 l-cos2 x+2cos 2 xosin 2 x


= 11m ----::-.,.-----::==___
x~o x 2 (I +cosx.Jcos2x)
2
0 sin2 x+2cos xsin2 x
= 1Im--,-----==,-
2
x---M) x (1 + cos x.Jcos 2x)

0 sm x 10 + cos 2 x )
= 11m ( -0)2
- olm ( 1 2
x~o X x~o 1 + cos x.Jcos 2x

= (1)2 x 1+ 2 (1)2 = i 0

1+1.Ji 2

o sm - + Y ) - cos (1t
o (1t - + y)
(iii) lim smx-cosx = lim 4 4
x~x/4 (x-1t/4) y~O y
01t 1to 1t o1to
sm-ocosy + cos- °sm y - cos-ocosy +sm- osmy
=lim 4 4 4 4
x~o Y
1 1 0 1 1 0
-cosy +-smy --cosy +-sm y
= lim F2 F2 F2 F2
y~O y

= lim~(Siny)=~ lim siny


y~oF2 y J2 y~O y

=~Xl=~=F2o
F2 Ji
Functions and Limits 445

1 + cos 2x l+cos2x
=
0 0

(iv) 11m 2 11m


x~1t/2 (n - 2x) x~1t/2 4 (n--x )2
2

2 cos 2(n
-- y )
= lim 2 cos x = lim L 2
x---)1t/2 (n
4 --x
)2 x---)O 2 y2
2
2
= L lim sin y = .!.( lim Siny)2 = .!.(I)2 ==.!.
2 y---)o i 2 y~O Y 2 2
Example 7: Evaluate
0 cosx - cos a
1l m - - - - - (UPTU B. Pharma 2004)
x---)a x-a
Solution: We have lim cosx - cos a o
[form -]
x---)a x-a o
0 -sinx-O 10 ( 0 ) 0
= 11m == 1m -smx =-sma
x---)a 1-0 x---)a

Example 8: Evaluate
e X+e- x _2
o

hm---::--- (UPTU B. Pharma 2007)


x---)o x2
Solution: We have
o eX +e- x -2 e 2x -2ex +1 1
[:0 e-x =
0

hm = hm ----,,---- ~]
x---)o x2 X---)o x 2eX e

lim[~(eXx-:-1)2] (lim~)o[lim(eX
x---)o eX x---)O eX
=
x---)O
_1)2)
X

= (T}W2 = I
Example 9: Evaluate the following limits:

(I) lim ~ -I (UPTU B. Pharma 2006)


x~O x

(ii) lim.JI+; -I (Meerut B. Pharma 2003)


x---)O X

esinx -I
(iii) lim - - - (Meerut oB. Pharma 2006)
.Y~O x
Solution: (t) We know that d = e 10ge a Using this, we get
0

X I ixlogea)_1
lim~
0

= hm----
x---)o X x---)o X
446 Remedial Mathematics

(x loge a) (x loge a)2 ]


·
[1+ l! + 2!
+ ... -1
= IIm=-----~------~~------~
x--+O X

_ . x[
- hm - loge a +
x(loge a)2 + ...]
x--+Ox 2!

_ . [
- hm loge a+
X(IO ge a)2]_
+ ... -logea.
x--+O 2!
(ii) Expanding ~ = (l + x)ll2 by Binomial Theorem, we get

.Jl+x 1 .O+x)1I2_1
lim - = LIm -'------'---
x--+O X x--+a X

=
lim
[1+Lx+
2
m(~-I)
2.
h .. ]-1
x

= !~;[~-ix+ . .J=~.
(iii) Expand e inx by exponential series
2
sinx sin x 1]
. esinx_I [1 +--+---+ ... -
hm---- = lim 1! 2!
x--+o x x--+O x
3
_- I'1m (Sinx) sinx sin x ... ]
- - . [1 +--+---+
x--+o x 2! 3!

= I1m (Sinx)
· - - . I'1m [I +sinx
- - +sin
- - + ...
3
x ]
x--+o X x--+o 2! 3!
=lxl=1.

x 2 +4
Example 10: Evaluate lim 2 . [UPTU B. Pharma 1995)
(x - 2)
X--+OO

Solution: Divide the Nr. and Dr. by x 2 , we get

1+0 =1.
1-0+0
Functions and Limits 447

III ONE SIDE LIMITS


(I) Right hand limit: Afunctionfis said to approach I as x approaches afrom right if
corresponding to an arbitrary positive number 8, there exists a positive number
8> 0 such that
Ij(x)-ll <E, whenevera<x<a+o.
It is written as
j(a+O) or lim j(x) =1
x~a+O

and j(a+O) = limj(a+h).


h~O

(ii) Left-hand limit: Afunctionfis said to approach to I as x approaches afrom the left,
ifcorresponding to an arbitrary positive number 8, there exists a positive number 8
> 0 such that
Ij(x)-/I <E, whenevera-o·<x<a.
It is written asj(a - 0) or lim j(x) = I
x~a-O

j(a-O) = lim j(a-h).


h~O

Ifboth, right hand limit (RHL) and left hand limit (LHL) offas x -+ a exist and are equal
in value, then their common value will be the limit off as x -+ a and function is said to be
continuous at x = a.

Remark
• If either or both of these limits do not exists, the limit of f as x ~ a does not exists.
Even if both these limits exist but are not equal in value, then also the limit offas
x ~ a does not exists.

STEP KNOWLEDGE
(1) To find the limit on rigllt, put a + h for x inf(x) and then take limit as h ~ 0
=> lim j(x) = limf(a+h)
x~a+O h~O

(il) To find tile limit on left, put a - h for x inf(x) and then take limits as h ~ 0
=> lim j(x) = limj(a-h).
x~a-O h~O

II1II LIMIT AT INFINITY AND INFINITE LIMITS


(A) Limits at infinity:
(I) Afunctionj(x) is said to tends to a limit I as x ~ -00 iffor given e> 0, however small,
there exists a positive number 0, such that
lj(x)-/I<e'V x~o
=> I-E <j(x) < 1+ E 'V X ~ 0
and we write lim j(x) = I.
X~-o:J
448 Remedial Mathematics

(il) A function f(x) is said to tends to a limit I as x ~ - 00 iffor given I: > 0, however small,
there exists a positive number 0> 0 such that
Ij(x)-/I <I: v x=:;-o
===> I-I: <j(x) <1+1: v x=:;-o
and we write lim j(x) = I.
x~-oo

(B) Infinite limits.


A function! A ~ R, where A c R is said to tend to the limit + 00 as x ~ a, if for any
given positive number 0\ > 0, there exists a positive number 2 such that °
x EA,O<lx-al<02===>j(x»0\.
Some Useful Expansions
n-l n(n-l) 2 n(n-l)(n-2) 3
(I) (1 +x-+nx+
) x+ x+ ...
2! 3!
x 2 x3
(i/) f?=l+x+-+-+ ...
2! 3!
x 2 x3
(iiI) 10ge(1 +x)= X--+-- ...
2 3

~~~~~~~I SOLVED EXAMPLES ~I~~~~~~


xifO=:;x=:;l
Example 1: Letj(x) = {
3 - x if I < x =:; 2.
Define ofj(x) atx = I exist? Give reasonsforyouranswer. [Meerut B. Sc. Biotech 2005]
Solution: RHL = lim j(x)= lim.t{l +h)
x---+\+ h---+O

= lim 3-(1 +h)= lim 2-h=2


h---+O h---+O
lliL= lim f(x) = Iimf(l-h)= limf(l-h)=l
x---+\- x---+O h---+O

Since R. H. L. is not equal to R. H. L., then lim j(x) does not exist.
x---+\

III CONTINUITY
A continuous process is one that goes on smoothly without any sudden change. Continuity
of a function can also be interpreted in a similar way. For better understanding, consider the
following figures.
The graph of the function in fig. (a) has an sudden cut at the point x = 4 whereas the
graph of the function in fig. (b) proceeds smoothly. We say that the function of fig. (b) is
continuous, while function of fig. (a) is not continuous.
Functions and Limits 449

y y

X X
0 4 0 4
(a) (b)
Fig. 10.26

Also, while defining lim /(x), the function/ mayor may not be defined at x = a. Even
x-+a
if/is defined atx = a, lim lex) mayor may not be equal to the value of the function at x = a.
x-+a
If lim lex) = lea), then we say that/is continuous at x = a.
x-+a
Continuous Functions
Continuity at a point: A function/, defined on some nbd of a point a, is said to be continuous
at a if and only if any one of the following conditions is satisfied:
(1) lim j(x)=j(a)

(ii) j(a-O) =j(a+ 0) =j(a)


(iii) For E > 0, :3 0> 0 such that
Jj(x) -j(a) J < E whenever 0 < J x - a J < 0.

~~~~~~I SOLVED EXAMPLES I~~~~~~


2
Example 1: Show thatj(x) = x -J is continuous/or all values o/x except x = 1.
x-I
Solution: If x"* 1, thenj(x)=(x+ 1)=Apolynomial
=> j(x) is continuous for all values of x"* 1.

If x = 1,j(x) is of the form Q, which is not defined and so the functionj(x) is discontinu-
o
ous atx= 1.
Example 2: Show that the/unctionf(x) is defined by
2
x ,x"* I
lex) = {
2,x= 1
is discontinuous at x = 1.
Solution: Here the value of/ex) atx = I is 2
=> /(l) =2.
Now, RHL=j(1+0)= Iim/(l+h)= lim (l+hi=1
x-+o x-+o
also lliL=j(1-0)= Iimj(l-h)= lim (I-hi =1
x-+o x-+o
450 Remedial Mathematics

Therefore, we have
.1(1 +0) =.1(1-0)*.1(1)
=> j{x) is not continuous at x = 1.
Example 3: Examine whether or not the function

.l(x) = {Si:2X, when x *0


2, whenx=O
is continuous at x = O.
Solution: Given that .l(x) =2, whenx=O
=> .1(0) =2

NowRHL=f(O+O)= lim.l(O +h)= lim [sin2(O+h)]


x--+o x--+o (O+h)

=2J:. 1im sinx =IJ


i x--+o x

and lRL =.I(~)= lim .l(O-h) = lim [sin2(0-h)] =2.


h--+O h--+O (0 - h)
Therefore, we have
.1(0+0) =.1(0-0)=.1(0)=2.
Hence,j{x) is continuous at x = 2.
Example 4: Afunction.l(x) is defined asfollows
1+XifX:=>2
j(x) =
{ 5-xifx~2
check the continuity of.l(x) at x = 2.
Solution: Here, we have
.1(2) = 1 + 2 or 5 - 2 = 3 ...(1)
Now, RHL =.1(2+0)= lim f(2+h)
x--+o
= lim [5-(2+h)]= lim [3-h)]=3 ... (ii)
h--+O h--+O
and lRL =.1(2 -0) = lim .l(2-h) = lim [1 +(2-h)] = 3 .... (iii)
h--+O h--+O
Now, from (I), (ii) and (iii), we have
.1(2+0) =.1(2)=.1(2-0)=3.
Hence, the functionj{x) is continuous atx = 2.
Example 5: Test thefollowingfunctionfor continuity at x = 0:

(I) .l(x) = xsin..!.. ,x* O,.I(x) = 0 atx= O. [Meerut B. Sc. Biotech 20061
x
1
(il) .l(x)= -lIx ,x*O,j{x)Oatx=O.
l-e
Functions and Limits 451

Solution: (i) Here, we have


lliL =j(O-O) = lim j(O-h)= lim./(-h)
h~O h~O

= lim (-h)sin(_1 )
h~O -h
· h·
= I1m 1
SIn-
h~O h
= 0 x a finite quantity lying between I and - 1 = 0
and RHL =./(O + 0) = lim itO -h) = lim j(h)
h~O h~O

· h·
= I1m 1 =0.
SIn-
h~O h
Also .1.0) = 0 given
=> ./(0 + 0) =./(O - 0) =./(0).
Hence, the function./(x) is continuous at x = O.
(ii) Here we have
lliL =./(0-0)= lim j(O-h)
h~O

= limj(-h) = lim~ =0
h~O h~O l-e
and RHL =./(0-0)= lim j(O+h)
h~O

= lim j(h) = lim l_lfh = 1.


h~O h~O l-e
Also, .10) =0
=> ./(0 + 0) "#./(O - 0) =./(0).
Hence,./(x) is discontinuous at x = 0 and this discontinuity is of first kind.
Example 6: Examine the continuity at x = 1 ojthe jitnction
j(x) = 5x-4, when 0 50x 50 1
= 4x3 - 3x, when 1 < x < 2. [Meerut M. Sc. Chemistry 2004]
Solution: At x = 1,./(1) = 5.1-4 = 1.
Now R.H.L. = Limj(x) = Lim./(l +h)
x~l+ h~O

= Lim [4(1 + h)3 -3 (l + h)], using lower rule


h~O

=[4xl-3 x l]=1.
Also L.H.L. = Limj(x) = Limj(l-h)
x~l- h~O

= Lim[5(I-h)-4] =5 x 1-4= 1
h~O

R.HL = L. H. L = 1 => lim j(x) = I


x~l

Obviously lim j(x) =./(I)


x~l

=> ./(x) is continuous at x = I.


452 Remedial Mathematics

~~~~~~~I EXERCISE 10.3 ~I~~~~~~


1. Ifj{x) =:? - 3x + 5, then findf(O).
2. Ifj{x)= 3:? +4x+3, then fmdj(O),j(1) andf(-I).

3. Ifj{x) =:? - 3x + 6, then find the value of f(3 + h) - f(3)


h

4. Ifj{x) = loge (~::) , then prove thatj{x) + j{y) = f( T)


1 __
1 x2
f(~) .
2
5. Ifj{x) = 1t , then find
I
1+-x2 4
2
1t

6. Ifj{x) = ~ , then prove that: x = f(x)


x-I f(x)-I
7. Which of the following functions are even or odd?

(i) - x3 (ii) x 50 (iii) x +! (iv)


x
= x-y
8. Ifj{x)= x-I ,then prove that: f(x)-f(y)
x +1 1+ f(x)f(y) l+xy
f(x)- f(x 2 )
I-x
9. Ifj{x) = - - , then prove that: - -I
l+x 1+ [f(x)f 2
10. Evaluate the following limits:
x 2 +2x-1
(I) lim 6:?-4x+ I (ii) lim
x~o x~1 2x+3
2 2
C~
III
r1m 2x -3x+6 (iv)
r1m--
x-I
x~2 3x 2 -6x+8 x~1 x-I
x 2 -a2 x 2 -x-6
(v) lim (VI) lim 3
x~a x-a x~3x -2x-3
. x 2 -3x 2x2 -7x+6
(vii) hm- - (viii) lim
2
x~a x -9 x~25x2 -llx+2
3x2 -4x+ I
(ix) lim 2
x~1 x -4x+3

11. Evaluate the following limits:


3x 2 + 2x+3 5-x2
(I) lim 2 (il) lim 2.
x~O 5x + 2x + 4 x~oo3x+5x

x 2 -4x+5 ~7x4 _5x 3 +4x2 +3


(iiz) lim 2 (iv) lim
x~oo x - 7 x + 12 x~oo 3x 2
Functions and Limits 453

12. Evaluate the following limits:


eX _e- x
(l) lim (I + px
x-+o
i Ix (il)
.
hm---
x-+o x
X bX
(iii) lim a - (iv) lim (I+xt-I
x-+o x x-+o x
x
(v)lim aX +b -2
x-+o x
13. The functionf(x) is defined as follows:
5x-4 O<x~1
f(x)= { 4x 3 -3x ' l<x<2

Examine its continuity at x = I.


14. A functionj(x) defmed as follows:

x2 when x< 1
j(x)= 5/2 when x = 1 [RGPVB. Pharma 20041
{ 2
x + 2 when x > 1
Does lim j(x) exists.
x-+I

ANSWERS

1. 5 2. j(O)=3,j(I)= 1O,j(-1)=2 3. 3 +h
15
5. -
17
7. (I) odd (ii) even (iii) odd (iv) odd.
2
10. (I) (il) - (iii) (iv) 2
5
5 1
(v) 2a (vi) (vii) - (viii) -
4 2 9
(Lx) - 1.

H. (I) -
3
(il) (iii) (iv)
17
5 5 3
a
12. (I) eP (il) 2 (iii) 10g- (iv) n
b
(v) logab.
13. At x = 4, function has infinite discontinuity and is continuous at all other points in
R.
14. does not exist
454 Remedial Mathematics

OBJECTIVE EVALUATION

MULTIPLE CHOICE QUESTIONS


Choose the most appropriate one:
1. IffR -4 R is defined by j{x) = x 2 - 3x + 4, for all x E R. then]1 (2) is
(a) {I, 2} (b) (1,2) (c) [1,2] (d) {- 2}
2. Let A be a set containing 10 distinct elements. Then the total number of distinct functions
from A toA is
(a) 10! (b) 1010 (c) 2 10 (d) 2 10 - 1
3. The number of bijective functions from the set A to itself, if A contains 108 elements is
(a) 108 (b) (108)! (c) (108)2 (d) 2 108
4. Iff: N x N -4 N is such thatj{m, n) = m + n for all n E N, where N is the set of all natural
numbers, then which of the following is true.
(a) fis one-one but not onto (b) fis neither one-one nor onto
(c) fis one-one and onto (d)fis onto but not one-one
5. If R denotes the set of all real numbers then the functionf: R -4 R defined by j{x) = Ix I is
(a) one-one only (b) onto only
(c) both one-one and onto (d) neither one-one nor onto
6. ThemappingfR+ -4R defined byj{x) = 10gloX, (whereR+ isthe set of all positive Number) is
(a) only one-one mapping (b) only onto mapping
(c) both one-one and onto (d) None of these
7. On the set Z of all integers define f Z - {O} -4 Z as follows:

I
n .
j{n) = 2' n IS even
0, n is odd
Then.! is
(a) onto but not one-one (b) one-one but not onto
(c) one-one and onto (d) into
8. Let the functionf: R -4 R be defined by j{x) = 2x + sinx, for x R. Then fis
E

(a) one-one and onto (b) one-one but not onto


(c) onto but not one-one (d)neither one-one nor onto
9. Iff: A -4 B is surjective then
(a) n(A) ~ nCB) (b) n(A) = nCB)
(c) n(A) ~ nCB) (d) none of these
10. If A = {2, 3, 4, 5}, then which of the following relation is a function from A to itself.
(a) fi = {(x,y):y=x+ I} (b)fi = «x,y):x+y>6}
(c) /3= «x,y):x>y} (d) f4={(x,y):x+y=7}

11.
·
The vaIue 0 f I1m
~1+J2+x -..J3 .
IS
x--+2 x-2
I 1
6~
1
(a) 5..J3 (b) (c) 7..J3 (d) 8..J3
Functions and Limits 455

x+sinx
12. lim is equal to
x4'" x-eosx
(a) 0 (b) -I (c) 1 (d) 2
e= -ell<
13. lim is equal to.
X40 X
(a) ex: -13 (b) ex: + 13
14. lim (~x2 + I -x) is equal to.
X4'"

(a) 0 (b) 1 (c) -I (d) None of these

15. r1m - - IS equaI to


sinx.
X4'" X
(a) 0 (b) I (c) -I (d) 2
r sinx+log{I-x).
16. 1m 2 IS equal to
X40 X

(a) II2 (b) II3 (c) -II3 (d) -II2


eosx-eosa
17. lim =?
x4a eotx-eota

(a) sin a (b) sin 2 a (c) sin 3 a (d) eosee 3 a.

18. lim tan3x =?


X41t12 x
(a) 0 (b) OCJ (c) 3 (d) 113.
rI m
I-eosx
--- =?
19.
X40 sin 2 x .
(a) I (b) II2 (c) -II2 (d) 0
. 3sinx-sin3x
20. I1m =?
X40 3 x .

(a) I (b) 2 (c) 3 (d) 4

FILL IN THE BLANKS


1. A funetionf A ~B is said to be if distinct element of A have distinct images inB.
2. A funetionfA ~B is said to be ifeveryelement ofB is the image of some element
of A under the mapping!
3. For into function the range of/is a subset of its co-domain.
4. Limit of a function if exist is _ __
5. The limit of the quotient is equal to the of the limit.
m m
6. lim_x_-_a_
X40 x-a
. aX-I
7. hm--= _ __
X40 X
8. A function is said to have if.
j(a + 0) = j(a - 0) "'flO)
456 Remedial Mathematics

9. A polynomial function is always _ __

~ {2 -;
x< I
10. L"fix) l~x<2
x~2

Thenf(3/2) = _ __

TRUE/FALSE
Write 'T' for true and 'F' for false statement
1. For lim fix) do exists, the functionf{x) must be defined at x = a (T/F)
x-+a
2. The limit of product is equal to the product of the limits (T/F)
3
. x -I 2
3 hm-- =- (T/F)
• -I
x-+lx2 3
4. For a functionf{x) to be continuous at x = a it is necessary that lim f{x) must exist (T/F)
x-+a
5. The function must be defined at the point of continuity (T/F)

6. The functionf{x) = jSi:X,x,*O iscontinuousatx=O (T/F)


J..,x=1
· sin3x I
7. Il m - - = (T/F)
x-+o X

8.
x-3! !
lim - - - = I (T/F)
x-+3 x

ANSWERS

MULTIPLE CHIOICE QUESTIONS


1. (a) 2. (b) 3. (b) 4. (b)
5. (d) 6. (c) 7. (a) 8. (a)
9. (c) 10. (d) 11. (a) 12. (c)
13. (a) 14. (a) 15. (b) 16. (d)
17. (c) 18. (b) 19. (b) 20. (d)

FILL IN THE BLANKS


1. one-one 7. log.a 2. onto 8. discontinuity
I
3. Into 9. Continuous 4. Unique 10.
2
5. quotient 6. mam- I

TRUE/FALSE
1. F 2. T 3. T 4. T
5. T 6. T 7. F 8. F
Functions and Limits 457

REFRESHER
Do you know? After reading this chapter you should be able to know the following
concetps:
• Let A and B be two sets, then the rule or correspondence, which associates each
element of A to a unique element of B, is called a function from set A to set B.
• A function ffrom A to B, i.e.,! A ~ B is said to be one-one (injective) if distinct
elements of A have distinct images.
• A functionf A ~ B is called many-one if at least one element of co-domain B has
two or more than two pre images in domain A.
• A functionf A ~ B is called an onto function, ifthere is no element of B which is not
an image of some element of A, i.e., every element of B appears as the image of at
least one element of A.
• A functionf:A ~ B is called an into function ifthere is at least one element of the set
B which has no pre-image in the set A.
• LetfA ~ Band g: B ~ C be two real valued function. Then the composition off and
g denoted by gofsuch that gof: A ~ C is defined by (g of) (x) = g(f(x)].
• The composition of functions is associative.
• The composition of any function with identity function is the function itself.
• Letf A ~ B, g: B ~ A be two functions such that g of= fA thenf is an injection and
g is a surjection.
• The inverse ofa bijective function is unique.
• A functionfis said to approach I as x approaches a from right if corresponding to an
arbitrary positive number E, there exist a positive number 8 > 0, such that.
Ij(x)-l) 1< E, whenever a < x < a + 8

or j(a+O)= lim f(a+h)


h~O

• A functionf is said to aproach to I as x approaches a from left if corresponding to


an arbitrary positive number E, there exists a positive number 8 > 0 such that
If(x)-/I < E, whenever a- 8 <x < a

or f(a-O) = lim f(a - h).


h~O

• A function/, defined on some nbd of a point a, is said to I be continuous at a if and


only if
j(a- 0) =j(a + 0) =j(a).

Can we do? (Frequently Asked Questions)


. x 2 +3x+ 1
1. State the domain of the functlOnj{x) = -:::-2- - - IUPTU B. Pharma 20071
x -5x+6
458 Remedial Mathematics

2. If.f{x) = IOg(I+X)oshowthat.f{X)+j(y)= f(x+y)o [UPTU B. Pharma 2005]


I-x I-xy

3.
o .JI
+x-I
Evaluate hm - - - - [UPTU B. Pharma 2005]
x~o x
cosx-cosa
0

4. E vaIuate 1un - - - - [UPTU B. Pharma 2004]


x~o x-a
o x e +e- x _2
5. Evaluate hm 2 [UPTU B. Pharma 2006, 2007]
x-.o x

6.''EvaIuate 1I maX-I
0

-- [UPTU B. Pharma 2006]


x~o x
esinx -I
7 . .f{x)= lim ,forx:;t:O
x~o x
=O,forx=O
Determine whether the function is continuous at x = 0 [UPTU B. Pharma 2006]

DOD
CHAPTER

11 DIFFERENTIATION

lID INTRODUCTION
Lety= j(x) be a function ofx where x is an independent variable andy is dependent variable.
Let 8x be any increment in the value of independent variable x and 8)' be the corresponding
increment in the value of dependent variable y, then 8y is known as the rate of change ofy
8x
with respect to x.
Definition: Derivative ofafunctionf(x) is the limiting value of 8y as 8x ~ 0 provided the
8x
limit exist finitely and it is denoted by dy .
dx

T hus, -dy = I·1m -8y = f(x+8x)- f(x) , provl·ded the I·· .


lmlt eXIst.
dx Iix-tO 8x 8x
Here, the increments 8x and 8y can be positive or negative.
Remarks
• The differential coefficient and instantaneous rate of change are also used for
derivative.

• dy, ~(Y)'Y',Y\, ~(f{x»,f'(x)orDj(x)havethesamemeaning.


dx dx dx

• The derivative ofa functionj(x) at a pointx= a is denoted byf' (a) or [~ f(X)]


dx x=a

III METHOD FOR FINDING THE DERIVATIVE USING FIRST


PRINCIPLE

STEP KNOWLEDGE
To find the derivative of a function from the first principle, we use the following steps.
Steps 1. First we take the function in the form ofy = j(x).
Steps 2. Take the small increment t5x in x and let corresponding increment in y is ~ such
thaty + ~ = j(x + t5x).
Steps 3. Find out the increment 8y = j(x + 8x) - j(x).
460 Remedial Mathematics

Steps 4. Now dividing both sides ofoy = j(x + ox) - j(x) by ox and take the limit Ox -+ O. we
get
dy = lim oy = lim f(x + ox) - f(x)
dx &X~O oX &X~O OX
Some Standard Derivatives:
1. Differential coefficient or derivative of y = x":
Lety= j(x) =x" soj(x + ox) = (x+ ox)n
dy = lim f(x+ox)- f(x) = lim (x+ox)n _xn
dx &X~O oX &X~O ()X

= lim xn [(
1+-
Ox)n -1
x
1
&X~O ox

1+n Ox + n(n-l)(0x)2 + ... -1]


[
= lim xn x 2! x
&x~o Ox
= lim xn[~+ n(n-I)ox + ...] = xn~ =nx"-I
&x~o x 2 x X

d(x") = nx"-I
So,
dx
2. Differential coefficient of a constant function:
Lety = j(x) = c, where c is a constant.
So, j(x+Ox) =c
dy = lim j(x+Ox)- j(x) = lim c-c =0
dx ax~o Ox !ix-+O Ox

So, d(c) =0.


dx
Hence, derivative of a constant function is always zero.
3. Derivative of the product of a constant with a function:
The derivative of the product of a constant with a function is equal to the product of
constant and the derivative of the function.
Let y = C j(x), where Cis a constant.

So, dy = lim (Cf(x+oX)-Cf(X»)


dx &x-+O ox
=C lim (f(X+Ox)-f(X») = C d[f(x)]
&x~O oX dx

Hence, ![cf(X)] = c ! [f(x)].


Differentiation 461

4. Derivative of sin x:
Lety= j{x) = sinx, then f(x + ox) = sin (x+ ox)

So, dy = lim (f(X+ox)- f(X)) = lim sin(x+ox)-sinx


dx fu~O ox fu--+O ox
Ox).
ox
2 cos ( x+- SIn-
lim 2 2
fu~O ox

[.: sinC-sinD = 2COS( C~ D}in( C ;D)]

lim
( Ox) .
Ox--
cos x+- SIn-
2
. ox
SIn-
2 = lim cos x+~ lim __2_
(0 )
fu~O ox fu--+O 2 fu~O ox
- -
2 2

= I
cosx . = cosx [Since lim sin x =
x--+o x
IJ
~ (sinx) = cos x
dx
5. Derivative of cos x:
Let y = j{x) = cosx, then j{x + ox) = cos(x + ox)
So dy = lim f(x+ox)-f(x) = lim cos(x+ox)-cosx
dx ox-+O ox ox-+O ox

. ( ox). ( ox)
= lim 2 Sin x+ 2 Sin -2
ox--+O ox

[Since cosC-COS D=Sin( C ;D }in( D;C)]


. ( x+
-SIn ox).
2 2
SIn (ox)

= lim ox
fu~O
2

.Ox
= lim [-Sin(x+ ox)] lim
fu--+O 2 fu--+O
SI: 2
uX
2
= - sinx. I = - sinx

Hence, ~ (cosx) = - sinx


dx
462 Remedial Mathematics

6. Derivative of tan x:
Lety=.f{x) =tanx, then.f{x + ox) =tan(x + ox)
So, dy = lim f(x+ox)-f(x)
dx 1ix-t0 ox
sin(x+ox)
--- I'1m tan(x+ox)-tanx -- I'1m cos(x + ox)
sinx
cosx
1
[
1ix-t0 ox 1ix-t0 ox
= lim [sin(x+oX)COSX-COS(x+oX)Sinx]
1ix-t0 ox cos(x + ox) cos x
= lim __ si_n..:.(x_+_ox_-_x--,-)_
1ix-t0 cos( x + ox) cos x ' ox
[',' sin A cos B - cosA sin B = sin(A - B)]

)' (sinox) 1
= o1~o ~ cos(x+o) cosx

= lim (sinox) lim 1 = 1_1_ = sec2x


1ix-t0 ox Iix-tOCOS(x+ox)cosx cos 2 x

Hence, ~ (tan x) =/sec2x.


7. Derivative of cot x:
Lety=.f{x) = cotx, then.f{x + ox) = cot(x+ ox)

So,
dy = lim f(x+ox)- f(x) = lim (cot(x+OX)-cotx).
dx 1ix-t0 ox 8x-t0 ox
Proceeding in the same way, we may get
d
-(cotx) = -cosec2x.
dx
8. Derivative of sec x:
Let y = .f{x) = sec x then.f{x + ox) = sec(x + ox)
So, dy = lim f(x+ox)- f(x)
dx 1ix-t0 ox

= lim sec(x + ox) - secx = lim (cose: + ()x) ~ )


Iix -to ox Iix -to ()X
. cosx-cos(x+ox)
= I1m - - - - ' - - - - ' -
1ix-t0 ox cos X cos(x + ox)

, 2 sm ' ( x+ ox), sm ox
= hm 2 2.
oX----)oO ox cos X cos (x + ox)
Differentiation 463

. (x+
sm ox) . Ox
sm-
= lim 2 lim __2_
Ox-+O cos x cos( x + ox) Ox-+O ox
2
sinx
= -- = secxtanx
cos 2 x
d
Hence, dx (secx) = secx tanx.

Important Formulae (to be used directly):


. C+D . D-C
cos C - cos D = 2 sm--sm--
2 2
C+D . C-D
cos C + cos D = 2cos--sm--
2 2
sm . D
. C +sm =
2· C+D
sm--cos-- C-D
2 2
C+D sin C-D
sin C - sin D = 2 cos
2 2
9. Derivative of e :
Lety= fix) = If thenj{x+ ox) = If+Ox
So, dy = lim f(x+Ox)-f(x) = lim eX+Ox_ex
dx Ox-+O ox Ox-+O ox
x Ox x X[ Ox 1]
= lim e e - e = lim e e -
Ox-+O Ox Ox-+O Ox

= lim
Ox-+O
eX [1 + 2! (ox) + (ox)2 + ...
3!
J= If
Hence, !!..- (eX) = If.
dx
10. Derivative of log,x:
Lety = fix) = log.,x, thenj{x + ox) = loge(x + ox)
So, dy = lim f(x+Ox)- f(x) = lim loge(x+ox)-logx
dx ox ox

!:
Ox-+O Ox-+O

loge ( ox)x -loge x


lim ---''-:.:...--<----
Ox-+O ox

loge (1 + oX)x -loge x


lim x
Ox-+O Ox
464 Remedial Mathematics

. loge X + IOg(1 + ?l:) -loge X


= lIm _ _ _ _-.0-----"-"-_ __
&x~o ?lx
= lim [~_~ ?lx2 + ...]
&x~o X 2x
1
x
d 1
Hence, dx (loge) = ; .

11. Derivative of logax:


Let y = j(x) = log,,x
We know log~ = loge x. logae
d d d
So, dx (log~) = dx [loge .logae ] = log~ dx [loge]

I
= logae-
X

d 1
Hence, dx (log~) = ; logae .

12. Derivative of ll":


Let y = j(x) = cf orj(x) = ~Ioga
then j(x+&) = e(x+&X)loga
dy _- (x+ox)loga xloga
lim _e____-_e_ _
So,
dx ox~O ?lx
. exloga .e&xloga _exloga . exloglt[efuloga -1]
hm-------- hm --"------"
&x~o ?lx &x~o ?lx

xloga [?lX loga (?lx loga)2 ]


e + + ...
lim I! 2!
&x~o ?lx
x log a[IOg a+ &x(log a )2 + ]
= lim e 2'
&x~o

=~Ioga . loga = cf loga


d
Hence. dx (d) = cf . loge a

lID DERIVATIVE OF THE SUM OF TWO FUNCTIONS


.. The derivative of the sum of two functions is equal to the sum of their derivatives. "
Let y = f(x)= <PI (x) + <P2 (x), then
f(x+&) =<PI (x+ ?lx) + <P2(x+ &)
Differentiation 481

(v) ~sin(atan-l x)= cos(atan-1 x)~(atan-l x)


dx dx

= ~cos(atan-l x).
a+x

1. Differentiate sinx w.r.t. x.


dy
2. Ify=3 x4Ef+ 5, then find dx'
3. Differentiate (~ + 7x + 2) (Ef - sin x)
4. Differentiate (x - 2) (x - 3) w.r.t. x. using product rule. Differentiate the same after
expanding as a polyomial. Verify that the two answers are the same.
x 2 +ex
5. Differentiate y = I 0 w.r. t. x.
ogx +2

sinx+cosx
6. Differentiate . w.r.t.x.
smx-cosx

7. Evaluate -d (. smx 2)
dx
8~· Differentiate log sinx w.r.t. x.
9. Differentiate e cosx w.r.t. x.

10. Find:, ifx= a cos O,y= b sin O.

11. Find : ' ifx = a(O + sinO),y= a(l-cosO).

12. Find : ' if x = a(t - sin t), y = a(1 - cott) [RGPV B. Pharma 2001]

13. Find : ' x = log t + sin t, y = et + cos t [RGPV B. Pharma 2002],

HINTS TO THE SELECTED PROBLEMS

2. : =. ! 4 x
(3x e + 5)

= ~(3x4eX) + ~(5)
dx dx

= 3 -d ( x 4eX) = 3 [4 d 4)]
d x ) + e x -(x
x -(e
dx dx dx
4
= 3[x Ef + Ef 4i3] = 3i3 Ef (x + 4).
482 Remedial Mathematics

5. : = ! [(.?+7x+2)(Ef-sinx)]

=(.?+7x+2). ~(Ef-sinx)+(Ef-sinx) ~ (.?+7x+2).


dx dx

=(.?+7x+2) [~(eX)
dx
-~(SinX)]
dx
+ (eX - SinX)[~(x2) +~(7x) +~(2)]
dx dx dx
= (.?+ 7x + 2) [Ef -cosx] + (Ef - sin x) (2x + 7).
= (Ef-cosx)'? +(9 Ef-7 cosx-2 sin x) x + 9 Ef -2 cosx-7 sinx.

6. dy = ~(sinx + cos
dx dx sinx - cosx
x).
(sin x - cos x) . ~(sinx + cos x) - (sin x + cos x) . ~(sinx - cosx)
dx dx
(sinx - cosx)2
= (sin x - cos x) . (cos x - sin x) - (sin x + cos x) (cosx + sin x)
(sin x - cos x)2
2
(sin2 x + cos X - 2sinx cos x) - (sin2 x + cos 2 X + 2sin x cos x)
(sin x - cosx)2
2
-------::-
(sin x - cosxi .
7. Let y = sin.? = sin t where t = .?
dy dt
Therefore dt = cos t and dx = 2x.
Hence, by chain rule, we have
dy dy dt
-=---
dx dt dx
=cos'?·2x=2xcos'?
8. Lety = log sin x = log t, where t = sin x.
dy 1 dt
Therefore, dt = t . and dx = cos x.
By chain rule. we get
dy dy dt 1
-= _.- = -.-cosx =cotx.
dx dt dx Stnx
dx dy. e
11. Wehave de =a(l +cosO), de =asm

dy

dy dx asine e
-=-= =tan-.
dx de a(l + cos e) 2
Differentiation 483

ANSWERS
1. xcosx+sinx 2. 3~~(x+4)
3. (~- cos x) x'- + (9 ~ - 7 cos x - 2 sin x) x + 9 ~ - 2 cos x - 7 sin x

(log x + 20)(2x + eX) + (x +


2
eX)(~ + 10 )
4. 2x+ 1 5.
(log x + 20)2
-2
6. 7. 2x cosx2 8. cot x 9. - sin x e Cosx
(sin x - cosx)2
cosec 2t teet - sint)
10. -cot 8 11. tan 8/2 12. 13.
(I-cost) (I + tcot!)

III DIFFERENTIATION OF IMPLICIT FUNCTIONS


A function which can be expressed in terms of independent variable x is known as explicit
function. On the other hand, A function which is not explicit, is known as implicit function, or
we can say A function which cannot be expressed directly in terms of independent variable
x is implicit function. For example, xY + Y = a is an implicit ~nction, because this function

cannot be expressed in terms of x. To find the : of implicit function, we differentiate each

term with respect to x treating y as a function of x and then separating : . A method by

which we find the : of implicit function is known as implicit differentiation.

~~~~~~I SOLVED EXAMPLES I~~~~~~~

Example 1: Find : ofthe implicitfunction aX2 + 2hxy + by + 2gx + 2fY + c = 0


Solution: Differentiating the given equation with respect to x, we get

2ax+2h(xdY + Y)+2bydY +2g +2fdy = 0


dx dx dx

dy (hx + by + f) = - (ax + hy + g)
dx
ax+hy+g
hx+by+f'

Example 2: Find: ofthefunction xJI + y + yJI + x = O.

Solution: Here, x.jl+Y + y.JI+x = 0


=> xJI+Y = - y.JI+x.
On squaring, we get
x'-(l +y) =y(l +x)or(x2 -y)+(x'-y-yx)=0
484 Remedial MatMmatics

:::) (~-y)+xy(x-y) =0
:::) (x +y)(x-y)+xy(x-y) =0
:::) x+y+xy=Oory(1 +x)+x =0
x
y=--
1+ x
Now, differentiating both sides, with respect to x we get
d d
dy = (1 + x) d; (-x) - (-x)d;(l + x) = _ (1 + x) + x
dx O+x)2 Cl+x)2

(1 + x)2 .

Example 3: Find : ' when y= sin C. [RGPV B. Pharma 2004)


Splution: We have y = sin c1

-ely = cos ex . -d (eX) = cos e...x . e-"


dx dx
dy
dx = c .cos C.
Example 4: Find : ' when y = cos ~ log sin x. [RGPV B. Pharma 2001]

Solution: We have y = cos ~ . log sin x


dy r d d c
dx = cos"X· dx (logsinx)+logsinx· dx (cos"x).

= cos~ ._._l-.~(sinx) + log sin x ·(-sin~) .~~


smx dx dx

=
cos~
-_·cosx- 1" r .1- 1-
ogsmx·sm"x
sinx 2 Fx
r log sin x x sin~
= cos"X· cot x - .
2~
Example S: Find : ' when y = tan (.; ). [RGPV B. Pharma 2001]

Solution: We have y = tan (~)


-dy = ~ tan
dx dx x
(.!.)
= sec
2
(~)- ~ ~) = sec (~
-(
2
)- ( - x12 )
Differentiation 485

Example 6: Find : ' when y = ~x2 (RGPV B. Pharma 2001 ]

. dy d 3x 2
SolutIOn: dx = dx (e )
=~x2 .6x=6xo~x2 0

Example 7: Find : ' when y = log cos x (RGPV B. Pharma 2003]


Solution: We have y = log cos x
dy did
- = -(log cos x) = -_o_ocosx
dx -dx . cosx dx

= _1_0 (-sinx) =-tanxo


cos x
dy xX· <Xl
Example 8: Find dx ofthefunctiony = x
xX'"
Solution: Here,y= x
:::> y=xY:::>logy=ylogx
Now, differentiating with respect to x, we get

! dy = y~(logx) + dy log x or! dy = y!.. + dy log x


ydx dx dx ydx x dx

:::> (! -IOgX)
y
dy -
dx
2:
x
=0
dy
:::> -
dx
=
y2
(1- y logx)x

Example 9: Find : ofthe function log xy = ~ +y 0

Solution: Here, log(xy) =~+yorlogx+ logy=~+yo


Now, differentiating both sides with respect to x, we get

!..+! dy = 2x + 2ydy or (!-2Y)dy = 2x-!..


x ydx dx y dx x

(1- 2y2) dy (2x 2 - I)


or
y dx x
dy (2x 2 -I)y
Therefore,
dx = (I _2y2)x 0

Example 10: ljx3 + y = 3axy;find dy


dx
Solution: We have,x3 + y = 3axy
Differentiating both sides with respect to x, we get
.!!-. ~ +.!!-.l = ~ (3axy)
dx dx dx
486 Remedial Mathematics

dy
=> (3y-3ax)- = 3ay-3x2
dx
dy
=> 3(V-ax) - =3(ay-~)
dx
dy = ay_x 2
Hence,
dx y2_ax

Example 11: if ~I- x 2 + JI- y2 = a(x-y), then prove that

dy P
dx = ~I- x 2

Solution: We have ~I - x 2 + ~I - i = a(x - y)


putting x = sin 9 and y = sin 4> we get

~I- sin 2 9 + ~I- sin 2 4> = a(sin e- sin 4»


=> cos 9 + cos 4> = a(sin 9 - sin 4»

=> 2cos(9;4>}os(9;4» = a{2cos(9;4>)sin(9;4>)}

9
=> cot ( ; 4» = a

=> 9 - 4> = coc l a


2
=> sin-I x - sin-I y = 2 coC! a
Differentiating w.r.t. x, we get
1 _ 1 dy =0
~I - x 2
JI - i dx

Hence, dy =
dx
JI- y2
1- x 2

III WGARITHMIC DIFFERENTIATION


To fmd the derivative of a function, which is of the form of the product of functions or
quotient of function or a function ofthe form (j(x--,<x). In this case, we take the logarithms on
both sides and then differentiate. This process is known as Logarithmic differentiation.
Differentiation 487

~~~~~~~I SOLVED EXAMPLES ~I~~~~~~

Example 1: Find : ofthefunctiony = xxx


XX
Solution: Here,y= x
Taking logarithms on both sides we get
XX
logy = log (x ) orlogy =xX logx.
Now, differentiating w.r.to x, we get
I dy d ) + -ex
d X) logx
- - = x x -(logx
ydx dx dx

..!. dy = xx.!. + XX (1 + logx)logx [.: ~exX) = XX (I + IOgx)]


ydx x dx

: =y~-l +xXlogx(l + log x)].

Hence, : = xxx [x x- 1 + XX 10gx(1 t log x)] .

Example 2: Find : ofthefollowingfunctions:


(i) y = ax-3 + bx1 + cx9/ 2 sinx (ii)y=xX
(iii) y = (1 + xY (iv) Y = (ax + bY
(v) y = (cos x/og x.
Solution: (i) We have, y = ax-3 + bx3 + cx9/2 sinx

So, dy = ~eax-3 + bx3 + cx9/ 2 sin x)


dx dx

= a..!!.-ex-3 ) + b..!!.-ex3 ) + c..!!.-ex9/ 2 sin x)


dx dx dx

=-3ax-4 + 3bx2 + c(x9/ 2 ! sin x + ! ex9/2)Sinx)

=-3ax-4+3bx2 + c(x9/2cosx+~x7/2SinX)

en) Here, y = xX.


Taking logarithms on both sides, we get
logy =xlogx.
Now, differentiating w.r.to x, we get
.Idy d d
-- = x-(logx)+-(x)logx
ydx dx dx
488 Remedial Mathematics

1 dy 1
or - - = x- + 1 logx
ydx x

or dy = y(1 + logx)
dx
dy
Hence, dx =x'{1 + log x)

(iii) Here, y = (1 + xy.


Taking logarithms on both sides, we get
logy =xlog(l + x).
Now, differentiating both sides with respect to x, we get

~ dy = x~[log(l + x)] + ~(x)log(l + x)


ydx dx dx

~ dy = x_I_+ I.log(l + x)
ydx I+x

: =y[l:X + log(l + X)].


Hence, : =(1 +XyC: x + log(l + x) J-
(iv) Here, y = (ax + by
Taking logarithms on both sides, we get
logy = x log(ax + b)
Now, differentiating both sides w.r.to x, we get

~ dy = x~log(ax + b) + ~(x) ·Iog(ax + b)


ydx dx dx

or ~ dy = x 1 d (ax + b) + I.log(ax + b)
yd.. (ax+b)dx

or ~ dy =x I a + log( ax + b)
ydx (ax+b)

or : = Y[ ax': b + log(ax + b)]


Hence, : =(ax+br[ax':b + log(ax + b)] .

(v) Here, y = (cosx)logx


Taking logarithms on both sides, we get
logy = logx log (cos x).
Differentiation 489

Now, differentiating both sides w.r.to x, we get

~ dy = logx~log(cosx) + ~logx ·log(cosx)


ydx dx dx

or ~ dy = logx_l_~(cosx) + !log(cos x)
y dx cosx dx x
1 dy sinx 1
or - - = -logx-- + -log(cosx)
y dx cosx x

or ~ dy = !log(cosx) - tanxlogx
ydx x

or : = (cosxl ogx [~IOg(cOSX) - tanxlogx]


Example 3: Differentiate (sin xf with respect to x. [RGPV B. Pharma 2002]
Solution: Lety=(sinxt
Taking log on both sides
logy = log (sin xt
log y = x log sin x.
differentiating w.r.t. x

- = xd- lOgSlllX+
1 .dy
- · d
IOgSlllX'-'X
.
Y dx dx dx .

=
1 d.
x·--·-smx+ Iogsmxx
. 1
sinx dx
cosx .
=x-.-+ 10gSlllx
smx
=x cot x + log sinx.

: = y(x cot x + log sin x)


= (sinxt [x cot x + log sin x]

Example 4: Find : when y = xX + (sin x)logx [RGPV B. Pharma 20011


Solution: We have y = xX + (sin x)logx
Let u =xX and v= (sinx)logx.
Then y=u+v
dy du dv
-=-+- ... (1)
dx ~dx dx
Consider u = xX .
Taking log on both sides,
log u = log xX
log u = X )ogx.
490 Remedial Mathematics

differentiating both sides w.r.to x


1 du 1
= x·-+logx.
u dx x
1 du
_ . - = 1 + log x
u dx
du
dx = u(1 + log x)

du
- =.r(1 + logx) ...(2)
dx ,
Consider v = (sin x)logx.
Taking log on both sides
log v = log x . log(sinx).
differentiating w.r.t. x
Idv I I 1,·1
= ogx· -.-. cosx + ogsmx· -.
v dx smx x

= logx· cot x +! logsinx.


x

dv = v(logx. cot x + !Iogsinx)


dx x

dv = (sin X)logx (log x . cot x + ! log sin x) ...(3).


dx x
Using (2) and (3) in (1), we get

: = xX(l + log x) + (sinx)logx(logx. cot x + ~logSinx).

ExampleS: Ify= (tan x)cot x + (cotx)tanx, find :

Solution: We have y = (tanx)cotx + (cot x)tan x


Let u = (tanx)Cotx and v =,(cotx)tan x
then y=u+v
dy du dy
-=-+- ...(1)
dx dx dx
Now u = (tanx)Cotx
taking log, we get
log u = log (tan x)cotx
log u = cot x log (tan x)
diff. w.r.t. x, we get
1
! dy = cotx.__.sec 2 x + log(tanx)·(-cosec 2 x)
u dx tan x
= cosec 2x - cosec2x log(tan x)
= cosec2 x(1-log(tan x)
Differentiation 491

dy
dx = (tan x)cotx . cosec 2x(1 -log(tan x» ... (2)

Now v = (cotx)tanx
taking log, we get
log v = log (cotx)tanx
= tan x log (cot x)
diffw.r.t. x, we get

.! dy = tanx._I_.(-cosec 2x) + log(cotx).sec 2 x


v dx cot x
= -sec2 x + log (cot x)· sec 2x
dv
dx = (cotx)tanx· sec2 x(log(cotx)-I) ... (3).

Using (2) and (3) in (1)~ we get

: =(tanx)cotx·cosec2x(I-log(tanx» + (cot x)tanx. sec 2x(log(cotx)- 1)

Example 6: Findthe : ofthefollowingfonctions:

1 + cosx
(i) t? log sin 2x (il) - - -
sinx
Solution: Here,y = t? log sin 2x

dy = ~(eX logsin2x) = eX ~(logsin2x) + ~(eX)logsin2x


dx dx dx dx

= eX -1-~sin2x + eX logsin2x
sin2x dx

= eX_._1_ cos 2x ~ (2x) + eX log sin 2x


sm2x dx
= 2t? cot 2x + t? log sin 2x.
1 + cosx
Oi) Here, y=
sinx
sinx~(l + cos x) - (1 + cosx)~sinx
So,
dy d
dx = dx
(1 + cosx
sinx )=
dx
(sinx)2
dx

sinx(- sin x) - (1 + cosx)(cosx)


(sin x)2
-sm· x-cosx-cos
2 2x -(l+cosx)
= (sinxp = 1- cos2 x
- (1 + cos x) -1
(1 + cosx)(l- cos x) 1- cosx
492 Remedial Mathematics

Example 7: Find the derivative of the following function


(t) ax + by + c = 0 (ii) Y = 4ax
(iii) :?+y=if (iv) :?+Y=4d.
Solution: (i) The given equation is ax + by + c = o.
Differentiating w.r.to x, takingy as a function of x, we get
d dy dy
a-(x)+b- =o=> a+b- =0
dx dx . dx

=> dy=-!!.
dx b·
(ii) The given equation is y = 4ax.
Differentiating w.r.to x, takingy as a function of x, we get
dy dy 4a dy 2a
2y- = 4a => - = - => - = - .
dx dx 2y dx Y
y
(iii) The given equation is:? + = if.
Differentiating both sides w.r.t. to x, we get
dy dy dy x
2x+2y dx =O=>2x= -2y dx => dx = -y'.
(iv) The given equation is:? + y = 4ax2 .
Differentiating both sides with respect to x, we get

2x + 2Y dy = 8ax
dx

2Y : =8ax-2x=2x(4a-l)

dy = x(4a-l)
dx y

ExampleS: Find : ofthe following functions:

.. xm ym
(t) x 3 + I = 3d (II) -
am
+- m = 1
b
(iii) x = aCt - sin t), y = a(I - cos t).
Solution: (i) Here, the given equation is x 3 + = 3d.I
Differentiating both sides, w.r. to x, we get
2 2dy 2dy .,
3x +3y - =6axor 3y - =6ax-3.r=3x(2a-x)
dx dx
dy = x(2a - x)
or
dx y2

.:\ H h· . . xm ym 1
(llJ ere, t e gIven equatIOn IS - + - m = .
am b
Differentiation 493

Differentiating, both sides w.r to x, we get

- d (m
1 - x ) +1m-d- (m)
y d
=-(1)
am dx b dx dx
mx m-J 1 m-J
dy
--+-m y - =0
am bm dx

or bmmxm- 1 + ammym-l dy =0
dx

or

or

(iii) Here, x = a (I - sin I), y = a (1 - cos I)

SO, : =a(l-cos/)and ~ =a[-(-sint)]

dy .
or dl =as1O/.

dy = dy 1dt = a sin t sint


So,
dx dx 1dt a(l- c.os I) 1- cost
2sint 12cosl 12 cost 12 t
= -.-- = cot-.
2sin2112 s1Ol12 2
dy 1
So, dx = cot2".

Example 9: Ify = Ax2? -x + 8 andfor x = 1, : = 2, then obtain the value ofA.

Solution: Here,y=Ax2?-x+ 8, and for x = I, : =2

So, dy = A!!...(x2 e X )-!!...(x)+!!...(8)


dx dx l
dx dx

or

Now, put x=1 dy =2


'dx
2 =A{e+2e} -I,
311
=> A=-=-=>A=-.
3e e e
494 Remedial Mathematics

~~~~I MISCELLENEOUNS SOLVED EXAMPLES ~I~~~


+x
Example 1: Find the differential coefficient at x = 0, ify = aX + - -
I-x ~
Solution: We have y = aX + ~I + x
1- x
Differentiating w.r.t. x, we get

-dy = a xloga+---
1(I+X)-1/2{10-X)-0+X)(-l)}
dx 2 1- x 0- x)2

IJ§-X
= a Xl oga+- - - {I-X+I+X}
2 1+ x (1- x)2

= IJ§-X{
a X loga+- - -
2 +1 x
2 }
(1- x)2
Putting x=o

(:)X=o = aOlOga+~~~:~ {(1_20)2}


1 2
= loga+-·- =loga+ 1
2 12

Example 2: Differentiate sin- J (1-


I+x
x) w.r.t. .j;.

u
Solution: Let = sin-I (1-
l+x
x) ,v= .j;
Differentiating w.r.t. x
du - (1 + x) - (1- x) dv I
dx -;=1_=(=I=_=x=)~2 (1 - x)2 ' dx = 2.j;

I+x

= ~O + x)2
l+x
- (1- x)2
(-2) dv
(1 + x)2 'dx =
1
2.j;

I dv 1
.j;(1 + x)' dx = 2.j;

du 1 / I -2
Therefore,
dv = ,- .j;(1 + x) 2.j; = 1 + x
Differentiation 495

Example 3: Ify= ~+ l. Then show that

dy = x-a
dx 2x...r;;;

SOluti~n: We have y = ~+ l
Differentiating w.r.t. x, we get

dy = _1 _ I +..ra( __
1 )
dx ..ra 2.[; 2x3/2

I x-a
= ~(x-a) = ,...-.
2x 2xvax
X
x+ex+e +
Example 4: y = e then show that
dy =----L. [UPTU B. Pharma 2003]
dx I_y·

X
x+ex+e +
Solution: We have y = e
y = £f+Y ... (i)

Differentiating w.r.t. x we get ! = eX + Y ( 1 + !) = y ( 1+ !) from (i)

!(1-y) =y

dy =-L
dx l-y

1. Find the derivatives (or differential coefficients) ofthe following functions:


. (x + 1)(x + 2)
(I) y= (x + 3)(x + 4) (ii) y = log sin ~

x4 _ 5x 2
1 + tan x
(iii) y= 5x6 + 7x (iv) y= 1 .
-tanx

!
rn
2. Find of the following functions:

0) y= JC::) (ii) y=

(iii) y= log[.J(x + 1) - .J(x -1)]


496 Remedial Mathematics

3. Find the derivatives of the following function~ :


(i) y=(x-2)(x+2)(x-3)(x+3) (ii) y=(x+ l)(aJ-21)
(iii) y = sin-I (tan x) (iv) y = (tan xy.
4. Differentiate the following functions:

(i) y=(sinx)COSX

(iii) y = sin x log x (iv) y = tf tan4 x.

5. Find : of the following functions:

(i) y = log sin x + cos-I(t!) + x4 sec x (ii) y = (logxY

(iii) log (;-) = x + y (iv) y=sec(~-2x+ I).

6. Find the derivatives of the following functions:


(i) £?x cos 3x (ii) log (sin-I x 4)
(iii) x 3 - ; - 3axy = O.

Find : for the following implicit functions.

7.xy=x+y
8. (~+IP=XY

9. sin(xy) + x =~-y
2
y
10. ytf + 2x sin x = cos Y
11. sin (x + y) = log (x + y)
X Y
12. e - = IOg(;-)

13. x· 2Y + 2x = Y
14. tan-I (~+ =9 I)
15. 3 sin (xy) + 4 cos (xy) = 5
16. ycosx=x-y
fY ~ dy x-17y
17. If V-; + fy = 6, show that dx = 17x _ y

dy cos 2 (a + y)
18. Ifsiny=xcos (a +y) prove that dx =

X'%
cosa

19. If ~I _ x' + ~I _ y' ~a(x'-l)prov.that dy ~


dx l l_x 6

I 2 2 -I Y dy x +Y
20. If logvx + y = tan - prove that - = - -
x dx x-y
Differentiation 481

(v) ~sin(a tan-1 x)= cos(a tan- 1 x) ~(a tan- 1 x)


dx dx

= ~cos(atan-l x).
a+x

1. Differentiate sin x w.r.t. x.

2. Ify= 3 x4 tr + 5, then find : .


3. Differentiate (.x2 + 7x + 2) (£f - sin x)
4. Differentiate (x - 2) (x - 3) w.r.t. x. using product rule. Differentiate the same after
expanding as a polyomial. Verify that the two answers are the same.
x 2 +ex
5. Differentiate y = I 20 w.r.t. x.
ogx +
sinx+cosx
6. Differentiate . w.r.t. x.
smx-cosx
d (.
7. Evaluate dx sm x 2)
8~ Differentiate log sin x w.r.t. x.
9. Differentiate e COS X w.r.t. x.

10. Find:, if x = a cos G,y = b sin G.

11. Find : ' if x = a(G + sin G), y = a(l - cos G).

12. Find : ' ifx= a(t- sin t),y= a(l-cot t) rRGPV B. Pharma 2001 )

13. Find : ' x = log t + sin t, y = l + cos t [RGPV B. Pharma 2002),

HINTS TO THE SELECTED PROBLEMS

2. : =" ! 4 x
(3x e + 5)

= ~(3x4eX)+~(5)
dx dx

= 3 ~(x4ex) = 3 [x4 ~(eX) + eX ~(X4)]


dx dx dx
4
= 3[x £f+£f4~] =3~ £f(x+4).
482 Remedial Mathematics

dy d .
5. dx = dx [(x2+7x+2)(eX-smx)]

=(x2+7x+2)· ! (eX-sinx) + (eX-sinx) ! (x2+7x+2).

=(x2+7x+2) [~(eX) - ~(SinX)] + (eX - SinX)[~(x2) +~(7x) + ~(2)]


dx dx dx dx dx
= (x2+ 7x + 2) (eX -cosx] + (eX - sin x) (2x + 7).
= (eX -cosx)x2 + (9 eX-7 cosx- 2 sinx)x + 9 eX -2 cosx-7 sinx.

6. dy = ~(sinx+cosx).
dx dx sin x - cosx

(sin x - cos x) . ~(sinx + cos x) - (sin x + cos x) . ~(sinx - cosx)


dx dx
(sinx - cosx)2
_ (sin x - cos x) . (cos x - sinx) - (sin x + cos x) (cos x + sin x)
(sin x - cos x)2
(sin2 x + cos 2 X - 2sinx cos x) - (sin 2 x + cos 2 x + 2sin x cos x)
(sin x - cos x)2
2
--------;:-
(sin x - cosx)2 .
7. Lety = sin x2 = sin twhere t=x2
dy dt
Therefore dt = cos t and dx = 2x.
Hence, by chain rule, we have
dy dy dt
-=_0-
dx dt dx
= cosx2· 2x= 2xcosx2.
8. Lety = log sin x = log t, where t = sin x.
dy 1 dt
Therefore, dt = t . and dx = cosx.
By chain rule. we get
dy dy dt 1
-= _.- = -.-cosx =cotx.
dx dt dx smx

11. Wehave : =a(l +cosO), de =asin9


dy

dy dx asin9 9
-=-= =tan-.
dx d9 a(l + cos 9) 2
Differentiation 483

ANSWERS
1. xcosx+sinx 2. 3x3~(x+4)
3. (~-cosx)x2 + (9 ~ -7 cosx-2 sinx)x+ 9 ~-2 cosx-7 sinx
2
(log x + 20)(2x + eX) + (x + eX)(~ + 10)
4. 2x+ 1 5.
(log x + 20)2
-2
6. 7. 2x cosx2 8. cot x 9. _ sinx eCosx
(sin x - cosx)2
cosec 2t t(i - sint)
10. -cote 11. tan e/2 12. 13.
(I-cost) (I + tcot!)

III DIFFERENTIATION OF IMPLICIT FUNCTIONS


A function which can be expressed in terms of independent variable x is known as explicit
function. On the other hand, A function which is not explicit, is known as implicit function, or
we can say A function which cannot be expressed directly in terms of independent variable
x is implicit function. For example,:xY + ;f = a is an implicit function, because this function

cannot be expressed in terms ofx. To find the : of implicit function, we differentiate each

term with respect to x treating y as a function of x and then separating : . A method by

which we find the : of implicit function is known as implicit differentiation.

~~~~~~I SOLVED EXAMPLES l~~~~~~~

Example 1: Find : ofthe implicitfunction ~ + 2hxy + by + 2gx + 2fy + c = 0


Solution: Differentiating the given equation with respect to x, we get

2ax+2h(xdY +Y)+2bydY +2g+2fdy =0


dx dx dx

dy (hx + by + f) =- (ax + hy + g)
dx
dy ax+hy+g
dx hx+by+ f·

Example 2: Find : afthefunction x~l+ y+ y~l+x =0.

Solution: Here, x..JI+Y + y~ =0


~ x..JI+Y = - y~ .
On squaring, we get
x 2(I +y) =;(1 +x)or(~-;)+(~y-;x)=O
484 Remedial Mathematics

~ (x2-y)+xy(x-y) =0
~ (x+y)(x-y)+xy(x-y) =0
~ x+y+xy=Oory(l +x)+x =0
x
~ y=---
l+x
Now, differentiating both sides, with respect to x we get
d d
dy = (l + x) d; (-x) - (-x)d;(l + x) = _ (l + x) + x
dx (l+x)2 (l+x)2

----~
(l + x)2 .

Example 3: Find : ' when y= sin l? [RGPV B. Pharma 2004)


S.olution: We have y = sin l?

-dy, = cosex . d- ( eX) =COSe·e


J J

dx dx
dy
dx = l? . cos l?

Example 4: Find : ' when y = cos £ log sin x. (RGPV B. Pharma 2001]

Solution: We have y = cos £ . log sin x

: = cos£· ~(lOgSinX)+lOgSinX. ~(COS£).


= cos£ ._._l-.~(sinx) + logsinx.(-sin£).~£
smx dx dx
cos £
= -_·cosx- 1 . . I .1_1-
ogsmx·smvx
sinx 2£
r log sin x x sin £
= cOSvX ·cotx- .

Example S: Find : ' when y = tan (.; ). [RGPV B. Pharma 2001]

Solution: We havey = tan(~)


-dy = ~ tan('!')
dx dx x

= sec2(~} ~ -(~) = sec2(~}( - :2)


1 sec2
= __
x2
(.!.)x
Differentiation 485

Example 6: Find : ' when y = t?~ [RGPV B. Pharma 20011

. dy d 3 x2
Solution: dx = dx (e )
=e3:;?- . 6x=6x· t?:;?-.

Example 7: Find : ' wheny = log cos x [RGPV B. Pharma 20031


Solution: We have y = log cos x
dy dId
dx = -dx (log cos x) =. cosx . dx . cosx

= _1_. (-sinx) =-tanx.


cos x
dy xx. '"
Example 8: Find dx ofthe function y = x
XX '"
Solution: Here,y = x
==> y =xY ==> logy= y logx
Now, differentiating with respect to x, we get

~ dy = y~(logx) + dy log x or ~ dy = y.!.. + dy log x


ydx dx dx ydx x dx

==> (~-
y
log x) dy - ~ = 0
dx x
==> dy =
dx (1- ylogx)x
i .
Example 9: Find : ofthe function log xy = x? + ;- .
Solution: Here, log(xy) =x? +;- orIogx+ logy=x? +;-.
Now, differentiating both sides with respect to x, we get

.!.. + J. dy = 2x + 2y dy or (~ - 2Y ) dy = 2x - .!..
x ydx dx y dx x

(1- 2i) dy (2x 2 -I)


or
y dx x

dy _ (2x 2 - l)y
Therefore,
dx - (1 _2y2)x .

dy
Example 10: If xl + ? = 3axy;find dx
?
Solution: We have x 3 + = 3axy
Differentiating both sides with respect to x, we get
!!...-~ +!!...-l = ~(3axy)
dx dx dx
486 Remedial Mathematics

=> (31-3ax) dy = 3ay-3x2


dx

=> 3(V-ax) dy =3(ay-x2)


dx
2
dy = ay _X
Hence,
dx i-ax
Example 11: if ~1 - x 2 + ~1 - y2 = a(x - y), then prove that

dy
dx =
n ~1- x 2

Solution: We have ~1 - x 2 + ~1- y2 = a(x - y)


putting x = sin 8 and y = sin 4> we get

~1- sin 2 8 + ~1- sin 2 4> a(sin 8 - sin 4»


=
=> cos 8 + cos 4> = a(sin 8 - sin 4»

=>

=>

=> 8 - 4> = coC! a


2
=> sin-! x - sin-! y = 2 coC! a
Differentiating w.r.t. x, we get
1 _ 1 dy =0
~l - x 2
~l - i dx

dy = / _ y2
Hence,
dx 1- x 2

III LOGARITHMIC DIFFERENTIATION


To find the derivative of a function, which is of the form of the product of functions or
quotient offunction or a function of the form (f(x--,<x). In this case, we take the logarithms on
both sides and then differentiate. This process is known as Logarithmic differentiation.
Differentiation 487

~~~~~~~I SOLVED EXAMPLES I


Example 1: Find : ofthefunctiony = xxx.
XX
Solution: Here,y= x .
Taking logarithms on both sides we get
XX
logy=log(x ) orlogy=~logx.
Now, differentiating w.r.to x, we get
1~
- - = x x -d (I ogx) + -(x
d X) logx
ydx dx dx

-.!.. ~ = xx..!.. + xX(l + 10gx)logx [.: !!.-(XX) = XX (1 + log X)]


ydx x dx

: = y[!!-I + ~ logx(l + log x)].

Hence, : = XXX [xx-I +xxlogx(ltlog x )].

Example 2: Find : ofthe followingfunctions:


(i) y = ax-3 + b~ + cJJI2 sinx (ii) Y =~
(iii) y = (1 + xy (iv) Y = (ax + by
ogx
(v) y = (cos x/ .
Solution: (i) We have, y = ax-3 + bx3 + cx9/2 sinx

S
0,
~
dx
= !!.-(ax-3 +bx3 +cx9/ 2 sinx)
dx

= a!!'-(x-3 ) + b!!'-(x3 ) + c!!.-(x9/ 2 sin x)


dx dx dx

= -3ax-4 + 3bx2 + c( x 9/ 2 ! sin x + ! (X9/2)Sinx)

=-3ax-4+ 3bx2 + c( x 9/ 2 cosx +~x7/2 sin x )

(n) Here, y =~.


Taking logarithms on both sides, we get
logy =xlogx.
Now, differentiating w.r.to x, we get

. -.!.. ~ = x!!.-(logx) + !!.-(x) log x


ydx dx dx
488 Remedial Mathematics

1 dy 1
or - - = x- + 1 logx
ydx x

or dy = yO + logx)
dx
dy
Hence, - = r(l + logx)
dx
(iii) Here, y =(1 +xt·
Taking logarithms on both sides, we get
logy =x logO + x).
Now, differentiating both sides with respect to x, we get

..!.. dy = x..:{..[log(l + x)] + ..:{..(x)log(1 + x}


ydx dx dx

..!.. dy = x _1_ + I . 10g(1 + x)


ydx l+x

: = y[l:X +IOg(l+X)].

Hence, : =(1 +xtC: x + log(l + X))-


(iv) Here, y = (ax + bt
Taking logarithms on both sides, we get
logy = x log(ax + b)
Now, differentiating both sides w.r.to x, we get
1dy d d
- - = x-10g(ax + b) + -(x) ·Iog(ax + b)
ydx dx dx
1 dy 1 d
or -- = x (ax+b)+l·log(ax+b)
yd'C (ax+b)dx
1 dy 1
"or -- = x a+ 10g(ax+b)
ydx (ax+b)

or : = y [ ax: b+ loge ax + b)]


Hence, : = (ax + bt[ ax: b + log(ax + b)] .

(v) Here, y = (cos x)logx


Taking logarithms on both sides, we get
logy = logx log (cos x).
Differentiation 489

Now, differentiating both sides w.r.to x, we get

~ dy = logx~log(cosx) + ~logx ·!og(cosx)


ydx dx dx
ldy 1 d 1
or -- = logx---(cosx) + -log(cosx)
ydx cosxdx x
1 dy sinx 1
or -- = -logx--+-log(cosx)
y dx cosx x

or ~ dy = ~ logecos x) - tan x log x


ydx x

or : = (cosx)logx [~IOg(COSX)-tanxIOgx]
Example 3: Differentiate (sin xl with respect to x. lRGPV B. Pharma 2002J
Solution: Lety=(sinxt
Taking log on both sides
logy = log (sin xt
logy = x log sin x.
differentiating w.r.t. x
1 dy d · I . d
y' dx = x dx logsmx+ ogsmx· dx ·x.

I d.
= x·--·-smx+ Iogsmxx
. 1
sinx dx
cosx .
= X-.- + logsmx
smx
= x cot x + log sin x.

: = y(x cot x + log sin x)


= (sinxt [x cotx + log sin x]

Example 4: Find: wheny =r + (sinx)logx rRGPV B. Pharma 20011


Solution: We have y =r + (sinx)logx
Let u =randv=(sinx)logx.
Then y=u+v
dy du dv
-=-+- ... (1)
dx "dx dx
Consider u =r .
Taking log on both sides,
log u =logr
log u = X logx.
490 Remedial Mathematics

differentiating both sides w.r.to x


1 du 1
= X ._+ logx.
udx x
1 du
=l+logx
u dx
du
dx = u(1 + log x)
du
dx = ~(1 + logx) ...(2)

Consider v = (sinx)logx.
Taking log on both sides
log v = logx· log(sinx).
differentiating w.r.t. x
Idv = Iogx· -.-'
1 cosx + I '
ogsmx· 1
-.
v dx smx x
= logx.cotx+..!.logsinx.
x
dv = v(logx. cot x + ..!.logsinx)
dx x

dv = (sin X)logx (log x . cot x + ..!. log sin x) ...(3).


dx x
Using (2) and (3) in (1), we get

: = XX (1 + log x) + (sin X)logx (log x . cot x + ~IOgSinX).

ExampleS: Ify= (tan x)cot x + (cotx)tanx, find :

Solution: We have y = (tanx)cotx + (cot x)tan x


Let u = (tan x)cotx and v =1 (cot x)tan x
then y=u+v
dy du dy
-=-+- ...(1)
dx dx dx
Now u = (tanx)Cotx
taking log, we get
log u = log(tanx)Cotx
log u = cot x log(tan x)
diff. w.r.t. x, we get
1 dy 1
- - = cotx·--·sec 2 x + log(tanx)·(-cosec 2x)
u dx tan x
= cosec 2x - cosec2x log(tan x)
= cosec 2 x(1-log(tan x)
Differentiation 491

: = (tanx)CotX. cosec2x(l-log(tanx)) ...(2)

Now v = (cotx)tanx
taking log, we get
log v = log (cotx)tanx
=tanxlog(cotx)
diff w.r. t. x, we get
Idy 1 2 2
- - = tanx·--·(-cosec x) + log(cotx)·sec x
v aX cot x
= -sec2 x + log (cot x)· sec2x
dv
aX = (cot x)tanx. sec 2 x(log (cotx)-I) ...(3).

Using (2) and (3) in (l), we get

: = (tanxt otx . cosec2x(l-10g (tan x)) + (cotx)tanx. sec2x(log(cotx)-l)

Example 6: Find the : ofthe followingfimctions :

1 + cosx
(i) ~ log sin 2x (ii) - - -
sinx
Solution: Here,y = ~ log sin 2x

dy = .!!...-(e X10gsin2x) = eX .!!...-(logsin2x) + .!!...-(eX) log sin 2x


aX aX aX aX

= eX -1-.!!...-sin2x + eX logsin2x
sin2x aX

= eX _._1_ cos 2x.!!...- (2x) + eX log sin 2x


sm2x aX
= 2~ cot 2x + ~ log sin 2x.
1 + cosx
(ii) Here, y=
sinx
sinx.!!...-(1 + cos x) - (1 + cos x).!!...-sin x
So,
dy d
aX = aX
(I + ) cosx
sinx =
aX
(sinx)2
aX

sin x(.... sin x) - (1 + cos x)(cos x)


(sinx)2
. 2 2
_ -sm x-cosx-cos x _ -(1 + cos x)
- (sinx)2 - 1- cos 2 X
-(1+cosx) -I
(1 + cos x)(1 - cos x) 1 - cos x
492 Remedial Mathematics

Example 7: Find the derivative of the following function


(i) ax + by+ c = 0 (it); =4ax
(iii) :? +;
= if (iv) :? +;
= 4d.
Solution: (i) The given equation is ax + by + c = O.
Differentiating w.r. to x, taking y as a function of x, we get
d dy dy
a-(x)+b- =o=> a+b- =0
dx dx . dx
dy a
=>
dx
(ii) The given equation is; = 4ax.
Differentiating w.r.to x, takingy as a function of x, we get
dy dy 4a dy 2a
2y- =4a=> - = - => - =-.
dx dx 2y dx Y
(iii) The given equation is:? + ; = if.
Differentiating both sides w.r.t. to x, we get
dy dy dy x
2x+2y dx =0=>2x= -2y dx => dx = -y'.
(iv) The given equation is:? + ; = 4ax 2 .
Differentiating both sides with respect to x, we get

2x + 2 y dy = 8ax
dx

2Y : = 8ax-2x =2x(4a-l)

dy = x(4a-l)
dx y

Example 8: Find : ofthefollowingfunctions:

xm ym
(i) x 3 +1=3d (ii) - + - = I
am bm
(iii) X = a(t-sin t),y = a(1- cos t).
Solution: (i) Here, the given equation is ~ + I = 3d.
Differentiating both sides, w.r. to x, we get
2 2dy 2dy - ,
3x +3y dx =6axor 3y dx =6ax-3x-=3x(2a-x)

dy = x(2a-x)
or
dx y2

" . . . xm ym
(11) Here, the gIven equatIOn IS - + - m = 1.
am b
Differentiation 493

Differentiating, both sides w.r to x, we get


1d 1d d
- - ( x m)+--(ym) = -(1)
am dx m
b dx dx

mx m-I 1 m-l
dy
--+-my - =0
am b m dx

or bmmxm- 1 + am mym-I dy = 0
dx

or

or

(iii) Here, x=a(t-sint),y=a(l-cost)


dx dy.
So, dt = a (1 - cos t) and dt = a[- (- SIn t)]

dy .
or di =asm t.

dy = dy I dt = a sin t sint
So, dx dx I dt a(1 - C:.os t) 1- cost
2sintl2costl2 costl2 t
= - - - = cot-
2sin2 tl2 sintl2 2.
dy t
So, dx = cot"2'

Example 9: Ify = A~t? -x + 8 andfor x = 1, : = 2. then obtain the value ofA.

Solution: Here,y=A~t?-x+ 8, and for x = 1, : =2

So, dy = A!!.-(x2 eX )-!!.-(x) + !!.-(8)


dx dx' dx dx

or

Now, put x=l dy =2


'dx
2 =A{e+2e} -1,
311
=> A=-=-=>A=-.
3e e e
494 Remedial Mathematics

I MISCELLENEOUNS SOLVED EXAMPLES I


~
+'x
Example 1: Find the differential coefficient at x = 0, ify = aX + - -
I-x

Solution:

We have y = a X + J§+x
--
I-x
, Differentiating w.r.t. x, we get

-
ely = a
xloga+---
I (I+X)-1I2 {IO-X)-(1+X)(-I)}
dx 2 1- x (1- x)2

= a Xl oga+-I~-X{I-X+I+X}
--
2 I+x (1- x)2

= I~-X
a X loga+- - - { 2 }
2 I + x (1- x)2
Putting x =0

(:1=0 = aOIOga+~J~:~ {(l_20)2}


I 2
= log a + - . - = log a + 1
2 12
Example 2: Differentiate Sin-/(I-X) w.r.t. J;.
I+x

Solution: Letu= Sin-1(I-X),v= J;


I +x
Differentiating w.r.t. x
du - (I + x) - (1- x) dv 1
dx r==(=1==)'="2 (1- x)2 ' dx = 2J;
1- 1: :

l+x (-2) dv 1
= ~(1 + x)2 - (1- x)2 (1 + x)2 ' dx = 2J;
dv 1
J;(1 + x)' dx = 2J;
du I / 1 -2
Therefore,
dv = - J;(1+x) 2J; = I+x
Differentiation 495

Example 3: Ify= ~+ to Then show that

dy x-a
dx = 2x";-;;;

Soluti~n: We have y = ~+~


Differentiating worot. x, we get

dy = _I _ I + Fa( __
1 )
dx Fa 2£ 2x3 / 2
I x-a
= --(x-a) = - -
2x3/2 2x";-;;; 0

x+ex+eX +
Example 4: y = e then show that
dy =-L (UPTU B. Pharma 2003J
dx I-yO

X
x+ex+e +
Solution: We have y = e
y = ?+y ... (i)

Differentiating wor.t. x we get : 1


= eX + y ( + : ) = y (I + : ) from (i)

dy(1-y) =y
dx
dy y
-=--
dx I-y

1. Find the derivatives (or differential coefficients) of the following functions:


(x+l)(x+2)
(ii) Y = log sin ~
o

(1) y= (x + 3)(x + 4)
x4 _ 5x 2 1+ tan x
(iii) y = 5x6 + 7x (iv) y= 0

1- tan x

2. Find : ofthe following functions:

(0 y= ~C::) (ii) y = JlGf}


(iiO y= log[~(x + 1) - ~(x -I)] (iv) y=(a2+5x+7r2 o
496 Remedial Mathematics

3. Find the derivatives of the following functions:


(i) y=(x-2)(x+2)(x-3)(x+3) (ii) y=(x+ l)(aJ-21)
(Ui) y=sin- I (tan x) (iv) y=(tanxt.
4. Differentiate the following functions:

(i) y= (sinx)COSX (ii) y = sin(e X ) +.!.. + log(x2 + x)


X
(iii) y = sin x log x (iv) y = t? tan4 x.

5. Find : of the following functions:

(i) y = log sin x + cos-I(t?) + x4 sec x (ii) y= (logxt

(Ui) IOg(~) =x+y (iv) y = sec (x 2 -2x + 1).

6. Find the derivatives of the following functions:


(i) ea
cos 3x (ii) log (sin- I x 4)
(Ui) x 3 -1-
3axy = O.

Find : for the following implicit functions.

7. xy=x+ y
8. (~+y)2=xy

9. sin(xy)+~ =~-y
y
10. yt? + 2x sin x = cos Y
11. sin (x + y) = log (x + y)

12. e
X
-
Y
= IOg(~)
13. x·2Y+2x =y
14. tan- I (X2+y) = 9
15. 3 sin (xy) + 4 cos (xy) = 5
16. y cos x = x - y

y HJ¥ dy
dx
x-17y
17. If -+ - =6, show that - = - - ' -
x 17x-y
dy cos 2 (a + y)
18. Ifsiny=xcos(a+y)provethat dx =
cosa

19. If JI-x' +Jl- y' ~a(x'-l)provethat dy ~ x'§


dx i l-x6

I 2 2 -I Y dy X +Y
20. If log" x + y = tan - prove that - = - -
x dx x-y
Differentiation 497

21. If EI + eV = EI + Y, prove that ely = _ eX (eY -I)


dx eY(eX -I)

dy logx
22. If~=EI-Y,provethat - = . IRGPV B. Pharma 2001)
dx (1 + logx)2

. dy x+tanx
23. Fmd dx' wheny = ---=2,.---- IRGPV B. Pharma 2004]
x -I
24. Differentiate w.r.t. x
2
ax +h
(ii) - - - - [RGPV B. Pharma 2001, 20021
sinx + cosx
25. Differentiate with respect to x
t! +e-x
t! _e-x
[~GPV B. Pharma 2001 r
26. Differentiate with respect to x
eX + tan x
[RGPV B. Pharma 2002]
cot x - x 2

5. Find : ' when


--

(i) y = sin2(lo&,e sec x). [RGPV B. Pharma 2002]


(ii) y=sin- I (.r+2x) [RGPV B. Pharma 2005]
(iii) y = sec(3x + 2) + (3x + I) \13 [RGPV B. Pharma 2005]

HINTS TO THE SELECTED PROBLEMS

. (x + I)(x + 2)
1. (I) y= (x + 3)(x + 4)
Taking log on both sides, we get
logy= log (x+ I) + log (x+ 2)-log(x+ 3)-log (x+4),
Idy I 1 I I
Diff. w.r.t.x, y dx = x + 1 + (x + 2) - (x + 3) - (x + 4)

~--y
dy { 2x + 3 2x + 7 }
dx (x + I)(x + 2) (x + 3)(x + 4)
2 2 2
= y{2X(X +7x+12-x2 -3x-2)+3(x +7x+12)-7 (x +3X+2»)
(x 2 + 3x + 2)(x 2 + 7x + 12)
2 2
(x + I)(x + 2) { 4x + 20x + 22 ) 4x + 20x + 22
= (x+3)(x+4) (x+3x+2)(x 2 +7x+12) = (x 2 +7x+12)2
498 Remedial Mathematics

(iJ) y = log sin :?-


dy 1 2 ,
Diff. w.r.t. x, we have dx = -.--2 . (cos x ) 2x =2xcotx-
SIOX

1 + tan x
(iii) y= I-tanx

Diff. both sides by Quotient rule


dy sec 2 x(l- tan x) + sec 2 x(1 + tan x) 2sec 2 x
-= =---7"
dx . (1- tan x)2 (1- tan x)2

2. (i)y= J(I- x) = (1- x)1/2


1+ x (1 + x)1I2
-

Diff. both sides by Quotient Rule

_!_I_~(l+x)-!~ 1
dy = 2~ 2 (~
dx (1 + x)
_ 1 {(l + x) + (1- x)} _ - 1
- - 2' (1 + x)~l- x 2 - (l + x)~l- x 2
(ii) Do as Question No.2 (i)
(iii) y = 10g[.Ji+l- ~] , diff. w.r.t. x both sides we have

dy
dx = ~x+l-~ 2~x+1
1 (1 -
I)
2~
1 -(..jx+i-~)
2(..jx+i _~)~x2 -I 2~X2 -I

(iv) Y = (2:?- + 5x + 7r2, diff. both sides w.r.t. x


dy = - 2 (4x + 5) = - 2(4x + 5)
dx (2x + 5x + 3 7t
(2x + 5x + 7)3
3. (i)y= (x-2)(x+ 2)(x-3)(x+ 3) = (x 2-4)(:?--9)
Difference w.r.t. x by product rule
dy
dx =2x(:?--9)+2x(:?--4)=4~-26x.
(ii) Do as in above Question
(iii) y = sin-1 (tan x), diffw.r.t. x

dy 1 2 cos x 2 secx ~
-= sec x = sec x = = secxvsec2x
dx ~l- tan 2 x ~cos2 x - sin 2 x .Jcos2x
Differentiation 499

(iv) y = (tan x't, taking log on both the sides


logy=x log (tan x),
1~ x 2
Diffw.r.t. x - - = log(tan x) + --sec x
y dx tan x
~
dx = y {log tan x + x sec x cosec x}
,= (tanx't {log tanx+x sec x cosec x}
4. (i) y = (sin x)COSX taking log on both the sides
logy = cos x log sin x, diffw.r.t. x
1~ . I' cosx
-- = -smx ogsmx+cosx--
y dx sinx

: = y (cos x cot x- sinx log sin x)


= (sin x)COSX (cos x cot x - sin x log sin x)

(ii) y= sin(eX ) +.!.. + log(x 2 + x),diffw.r.t.x


x
~ x x 1 1+ 2x
- = cos(e )e - - + - -
dx x2 x2 + X
(iii) Y= sinx (log x), diffw.r.t. x byproduct rule
dy sinx
- = - - + cosx(logx)
dx x
(iv) y= eX tan 4x, diffw.r.t. x by product rule
~ = eX tan4 x + 4 tan3 x sec2 x eX
dx
= eX tanX (tan3 x + 4 sec 2 x)
5. (i) y= log sin x + cos -I (eX) + x4 sec x, diff, w.r.t. x
~ cos x eX 3 4
= -.- - r.---;; + 4x sec x + x sec x tan x
dx smx '11- e2x
eX
3
= cotx- ~ +x secx(xtanx+4)
2x
1- e
(ii) y={logx't
Taking log on both sides
logy =x log (log x)
Diff. w.r.t. x both sides

.!. ~
y dx
= _x_(.!..)
logx x
+ log(log x)
ddxO' = Y {_1_logx
X)}
+ log(log

= (log xl {_1_ + IOg(lOgX)}


logx
500 Remedial Mathematics

(in) IOg(~) =x + Y
logx-Iogy =x + y,
. . 1 Idy dy
Dlffw.r.t. x both sIdes ;- - dx = 1 + dx y
dy
dx
(1 +~)
y
1+.!.-x =

dy = y(I - x)
dx x(I + y)
(iv) y = sec (.x2 - 2x + 1), diff. w.r. t. x both sides

dy =2(x-I)sec(.x2-2x+ 1) tan (.x2-2x+ 1)


.fix
= 2(x-I) sec (x- Ii tan (x- 1)2

6. (i) y = ;.x cos 3x, diff. w.r.t. x

: = 2;.x cos 3x + (- 3 sin 3x) e2x

=;.x (2 cos 3x - 3 sin 3x)


(n) Let y = log (sin -I x 4) diff. both w.r.t. x
dy 1 4x3 4x3
- = x-===
dx
I
sin- x4 ~I- x 8 sin-I x4 ~I- x 8

(in) ~ -y- 3axy = 0, diff. w.r.t. x


2
3x - 3i : - 3a( y + x : ) = 0

dye y 2 +ax+x
-dx ) 2 -ay=o

dy x 2 -ay
dx = y2 + ax .

x 1
7. xy=x+y~y= - - = 1+--
x-I x-I
. dy-I
Dlff. w.r.t.x, - = 2
dx (x-I)
8. (x 2 +li=xy

Diff.w.r.t.x, 2(x +i){2X+2Y


2
!} = x: +y
Differentiation 501

=:) {4y(x 2 + i) - x} dy
dx
= y-4x(x2 + y)

dy = y-4x(x 2 + i)
dx 4y(x 2
+ i) - x

9. sin (xy)+ ~ =x2-y ... (1)


i
.
Dlff.w.r.t.x, cos(xy) ( y +
Xdy) 1 2x dy = 2x- dx
- +2'-3"- dy
dx y y dx

=:) xcos(xy) - -2X)dy


+1 - = 2x - - 1 - ycos(xy)
( y3 dx i
=:) ( xy2 cos(xy) - 2; + y2 ) : = ~-I-l cos (xy)(Multiplying byy)
=:) {xy2 cos(xy) - 2(xy2 - i - ysin(xy) + l) dy = 2xy-1 -l cos (xy)
dx

Putting the value of'::' from (1)


y

dy 2xy2
=:)-=-----"---:=---~~--=----=-
-l cos(xy) -I
dx 2ysin(xy) + xi cos(xy) - 2xi + 3i
10. yEf + 2x sin x = cos Y

Diff. w.r.t. x, ye X + eX dy + 2 x log 2 sin x + 2 x cos x = - sin y dy


dx dx
dy = - [ye X + 2 x cosx + 2 x sinx log2]
dx (eX + sin y)
11. sin (x + y) = log (x + y)

Diff.w.r.t.x, COS(x+Y){I+ : } = X~y {I+ : }


=:) {COS(X + y) - _I_}dy
x+y dx
= _ {COS(X + y) _ _
x+y
I_}
dy
=:) dx =-1.

x
12. Ef-Y= log- = logx-Iogy
Y

Diff. w.r.t. x, we have eX - Y (I -:) = .; - ~:


50l Remedial Mathematics

:(;_eX-Y) = ~'_eX-Y
ely = y(1-xe~-Y)
dx x(l - ye X - y,)
13. x.2Y+2x=y

Diff. w.r.t x, we have 2Y + x2Y log 2 ~- + 2 log 2 = :


x

2Y + 2x log 2 = ely (1 - x . 2Y log 2)


dx

ely = [2 + 2x IOg2]
Y

dx 1- x2Y log2
14. tan -1 (.x2 +;l) = 9 =>.x2 +;l = tan 9
Diff. w.r.t. x, we have 2x + 2Y : = 0

ely x
=>dx=-y'
15. 3 sin (xy) + 4 cos (xy) = 5

DifEw.r.t.x, 3COS(XY){Y+X:}-4Sin(xy){y+x:} =0

ely {3xcos(xy) - 4xsinx(xy)} =-y{3 cos (xy)-4 sin (xy)}


dx .

: = -(~)
16. ycosx=x-y ... (1)

Diff. w.r.t.x, - ysinx + (cos x) : = 1- :

ely (1 + cos x) = 1 + Y sin x


dx
ely - 1 + ysinx - ( 1 +XSinx) 1 + xsinx + cos x
- - - - - -1- -
dx l+cosx l+cos+x (1 + cos x) (1 + cosx)2

17. Jf+H=6
=>y+x = 6Fxy
=>.x2+;l-34xy= 0

Diff. w.r.t.x, 2x + 2Y: - 34( x: + y) =0


Differentiation 503

x+ dy (y-17x)-17y =0
dx
dy = x-17y
dx 17x-y
18. siny = x cos (a + y) ... (1)

Diff. w.r.t.x, ! (cosy) = cos(a + y) -xsin(a + y{!)


dy {cosy + xsin(a + y)} = cos (a + y)
dx

dy {COSy + siny sin(a + y)} = cos (a + y)


dx cos(a + y)

From(l) dy {cos(a+ y)cosy+sinysin(a+ y)} =cos2 (a+y)


dx
dy = cos 2(a + y)
dx cosa
1
19. -log(x 2
+ y 2) = tan -1 -Y
2 x

12x + 2y dy _ 1 xdy
dx -y]
dx - 1+ ~~ [ x
2 ( ) 2
2(x + y2)

x+ydy=xdy-y
dx dx

x+ y= (x- y)dy
dx
dy = x+ y
dx x-y
20. tr +e'=tr+ Y,

D·ff..w.r.t.x, e
1 x +eY dy
dx -- e x + Y {I + dy}
dx

~_~+Y= dy(-e>'+ex+ Y )
dx
dy = eX{l-e>')
dx e>'(e X -I)
504 Remedial Mathematics

ANSWERS

4x 2 + 20x + 22
1. (i) (li) 2x cot x 2
(x 2 + 7x + 12)2

x 2(-10x 7 + 100x 5 + 21x2 - 35) 2sec 2 x


(iii) (iv)
(5x 6 + 7x)2 (1- tanx)2

2. (i) _ 1 -1
(ii)
~(1- x 2 ) 2x.Jx(x + 1)

(iii) _ 1 (iv) - 2(4x + 5)


2~x2 -1 (2x 2 + 5x + 7)3
3. (i) 4~ - 26x (ii) 8~ + 6x2 -21
(iii) sec x~r-(s-ec-2-x-) (iv) (tan xY (x sec x cosec x + log tan x)

4. (i)(sin x)COS x (cos x cot x - sin x log sin x) (ii) eX cos(eX) _ ~+ 2: + 1


x x +x
.. , sinx 1
(m) - - + cos x og x (iv) eX tan 3 x (4 sec2 x + tan x)
x

5. (i) cot x - h 1-e h


+ x 3sec x(x tan x + 4) (n) (logx)X [_1_ + log (log X)]
~gx

(1- x)y
(iii) (1 + y)x (iv) 2(x-1)sec(x2-2x+ l)tan{x2-2x +
1)

6. (i) ~ (2 cos 3x- 3 sin 3x)

2
oo') x -ay
(111 2
ax-y

1- Y -1 y-4x(x 2 +i)
7. - - or ---:- 8.
X -1 (x _1)2 4y(x 2 + y2) _ (x)

9.
2xy2 -l cosexy)-l 10.
_[ye X +2x cosx+2xsinxlog2]
2ysinxy + xy2 cosxy _ 2xy2 + 3y2 eX + siny

11.-1 12. L[xe x -


x ye
X

- Y
Y
-1]
-1

14. (-x/y)
Differentiatioll 50S

1+ysinx
15. -(y/x) 16.
1 + cosx

1
27. (i) 2 tan x sin (loge sec x) (ii) (2x + 2)
~1- (x 2 + 2x)2
(iii) 3 sec(3x +2) tan(3x +2) + 1 2/3
(3x + 1)

_ SECOND ORDER DERIVATIVES

It is known that derivative of y w.r.t. x (if exists) is denoted by : and is called the first
derivative ofy,

Further, derivative Of: w.r.t. x (ifit exists) is denoted by ~:; and is called the second
derivative ofy,

d2
Thus - y
2
(dy) = second derivative ofy w.r.t. x,
= -d -
dx dx dx
3 2
Similarly, d ;, ~(d2:;) derivative of d :; w.r.t. x,
= =
dx dx dx dx

In general dny denotes the nth derivative ofy w.r.t. x,


dx n

Other Symbols

1. : is also denoted by y 1 or y'

d 2y
2. dx is also denoted by Y2 or y"

3. : is also denoted by Dy, where D is the operator !.


d 2y d2
-2 is also denoted by d y, where D2 is the operator - 2 '
dx dx
4. Letj{x) be a differentiable function, then!' (x) denotes the first derivative of/ex) w.r.t.
d
x. Thus!' (x) = dx {{(x)}

. . d d 2 {f(x)}
Sllmlarly/" (x) = dx if' (x)} = dx 2 = second derivative of/ex) w.r.t. x.
S86 Remedial Mathematics

STEP KNOWLEDGE
To Find the Higher Ordered Derivatives
Step 1. Let the given function be y.

Step 2. (i) Differentiate the given function w.r.t. x to get :


(ii) If both base and power in the given function are variables then first take

logarithm and then differentiate to get : .


2
Step 3 . AT
)Vow diffi . dy t d y
I erentlate dx w.r.t. x to ge dx 2 ·

Step 4. If a particular expression is to be obtained, simplify the expression involved after


obtaining first derivative making use of the given relation between x andy and if
required also use the expression for first derivative obtained
After simplification find the higher derivatives.

I SOLVED EXAMPLES I
Example 1: Find the second derivatives ofthe followingfunctions
(i) y = x 3 log x (ii) e6x cos 3x
3
Solution: (i) Let y = x log x
Differentiating w.r.t. x, we get

-
dy
I 3
+ 3x2 . log x
= X •-
dx
x
Again differentiating w.r.t.x, we get
d 2y 2 I
--2 = 2x + 3x . - + 6x . log x = 5x + 6x log x
dx x
(ii) Lety = e6x cos 3x ... (1)
Differentiating w.r.t. (1) w.r.t. x, we get

.. : =e6x·6cos3x+e6x(-sin3x)·3
= 6e6x cos 3x - 3e6x sin 3x ... (2)
Again differentiating (2) w.r.t. x, we get
2
d ;, = 6[e6x. 6 cos 3x+ e6x (-3 sin 3x)] - 3 [e6x . 6 sin 3x+ e6x . 3 cos 3x]
dx
= e6x [36 cos 3x-18 sin 3x-18 sin 3x-9 cos 3x]
e6x (27 cos 3x - 54 sin 3x)
=
= 27 e6x (cos 3x- 2 sin 3x)
Example 2: Ify = e tanx , prove that
2
d dy
cos 2 x ;' - (1 + sin2x)- =0
dx dx
Differentiation S07

Solution: Given,y=etanx
.. logy=tanx ... (l)

~ dy = sec2 x or dy = y sec2 x ... (2)


ydx dx
2 dy
or cos x - =y ...(3)
dx
Differentiating again W.T.t. x, we get
2
cos 2 x d y _ 2cosxsinx dy = dy
dx 2 dx dx
2
2 d y . dy
or cos x dx 2 -(1+sm2x) dx =0.

d 2y
Example 3: Ify = ~ + sinx,jind-
2 (UPTU B. Pharma 2006)
dx
Solution: We have y = ~ + sin x

-dy = -d (X)
e +d- (.
smx)
dx dx dx
=~+cosx.
2
d y d (dy) d x
dx2 = dx dx = dx (e + cosx)

d x + -(cosx
= -(e) d )
dx dx
= ~-sinx.
d 2y dy
Example 4: Ify = a cos (log x) + b sin (log x ), show that x 2 - 2 + x - + Y = 0
dx dx
[RGPV B. Pharma 20031
Solution: Giveny = a cos (log x) + b sin (log x) ...(1)
Differentiating (1) W.T. t. x, we get

dy = -asin(logx) . .!. + bcos(logx) . .!.


.. dx x x

or x . dy = - a sin (log x) + b cos (log x)


dx
Again differentiating again w.r.t. x, we get
d 2y dy 1. 1
x - - + 1· - = - acos(logx)· - - b sm(logx)·-
~ dx x x
2
2d y dy .
or x dx 2 +x dx =-[acos(logx)+bsm(logx)]=-y [From(l)]
2
2d dy
or x dx; +x dx +y =0.
508 Remedial Mathematics

d 2y dy
Example5:lfx=(sin- 1 xi,provethat(i-x2) dx 2 = x dx +2
Solution: Given.
y = (sin -I x)2 ... ( 1)
Differentiating (1) w.r. t. x, we get
dy 2 . -I 1 r:-21 2 dy 2.-1
dx = . Sin X· ~l _ x 2 or" 1 - x- dx = Sin X

squaring, we get

o-x
2
{:f =4(sin-1 xi=4y[From(l)]
Again differentiating both sides w.r.t. x, we get
2
dy d y
(l-x 2 )2-·-+(-2x) (dy)2
- dy
= 4-
dx dx 2 dx dx

Dividing both sides by 2 : ' we have


2
2 d y dy
(l - x ) - =x·-+2
dx 2 dx'
d 2y
Example 6: Wheny = a sinx + b cosxfind dx 2 . [RGPV B. Pharma 2004]

Solution: As y = a sin x + b cos x.


on differentating w.r.t. x, we have
dy b .
dx = a cos x - Sin X.

Again differentiating w.r.t. x, we get


d 2y .
- 2 =-asInx-bcosx
dx
d 2y d 2y
- 2 =-y=> - + y =0.
dx dx 2
Example 7: lfx = a(cos t + t sin t)
d2 y
Y = a (sin t - t cos t) Find -2 .
dx
Solution: We have, x = a( cos t + t sin t)
y = a(sin t - t cos t)
dx
dt = a(
-. . )
Sin t + t cos t + Sin t = at cos t

dy
dt = a( cos t + t Sin
. t - cos t) .
= at Sin t
Differentiation 509

dy dy/dt atsint
-=--=--=tant
dx dx/ dt atcost

~(dy). dt
2
d y = !!....(dy) =
dx 2 dx dx dt dx dx
d 121
= -(tant)·-- = sec t · _ -
dt at cos t at cos t
sec 3 t
at
Example 8: Ify = x + tan x, show that
2
2 d y
cos x·---2y+2x =0
dx 2
Solution: We have y = x + tan x

dy = 1 + sec 2 x
dx
d 2y
= 2 sec x . sec x tan x
dx 2
= 2 sec2 x tan x
Now consider LHS

2 d 2y
cos x . -2- - 2y + 2x = cos 2 x(2 sec 2 x tan x) - 2(x + tan x) + 2x
dx
= 2 tanx-2x-2 tan x + 2x
=O=RHS

~~~~~~~I EXERCISE 11.4 ~I~~~~~~~


1. Find the second order derivative of the following functions:
(i) log x lRGPV B. Pharma 20051
(ii) x2 + 3x + 2
(iii) x cos x
(iv) If sin 5x
(v) sin (log x)
dy d 2y
2. Ifx=a(9-sin9),y=a(1-cos9),find dx,AIsofind dx 2 .

1- t 2 . 2t
3. Ifcosx= - - 2 andsmy= - - 2 ,0$t$1.
l+t l+t
2
Show that d ;' is independent of t.
dx
510 Remedial Mathematics

d 2y 1t
4. Ifx= 3 sin t- sin 3t,y= 3 cos t-cos 3t, find - 2 att= -.
dx 3
2
d y
5. Ify=~+tanx,showthat-2 =6x+2sec2 xtanx
dx
d 2y dy
6. Ify= A t? + Bet/x, show that dx 2 -(p + q) dx + pqy= 0 [RGPV B. Pharma 2001]

ANSWERS

(it) 2 (iii) - x cos x - 2 sin x

-sin(logx) + cos(logx)
(iv) 2~(5 cos 5x -12 sin 5x) (v) 2
X

dy 8 d 2y 1 48
2 - = cot- - - = --cosec- 4.
• dx 2' dx 2 4a 2

OBJECTIVE EVALUATION

MULTIPLE CHOICE QUESTIONS


(Choose the most appropriate one)

1. lfy = 2.x4 + 3~ + 2.x + 5 , then : =

(a) 8~ + 9x2 + 2 (b) 8x4 + 9x3 + 2 (d) None of these.

2. lfy = ( Fx + l), then : is equal to

1 1 1 1
(a) 1 + 2" (b) 1 - - (c) 1 - -
2 (d) 1+-
x 2x x 2x
d l-sin2x
3. -
dx 1+ sin2x

(a) sec2(~+x ) (b) sec2(~-x) (c) -sec2(~-x) (d) None of these.


x oo
4. Ify = xx --- then dy =
, dx

(a)
y
(d) i (c) Y (d) i
x(l+ ylogx) x(l+ ylogx) x(l-ylogx) x(l+ ylogx)

s. ~ (x2 . e" sin x) =


dx
(a) xe 2 (2sinx+x sinx+ cos x) (b) xe'" (2sinx + x sinx - cos x)
(c) xr:'(2sinx+xsinx+cosx) (d) None of these.
Differentiation 511

6. If y = x", then dy is equal to


dx
(a) 1 + logx (b) x' + logx (c) x'logx

7. Ify= ~logx+~logx+Jlogx+--oo then : =?

1
(a) 1 (b) 1
x(2y-l) x(2y+ I) (c) 2y -I (d) 2y+ 1

8. Ify=logu,x+ 10gx10+ 10g1010+ 10gxX, then : is equal to

(a) (b) --
xloge 10 x(logex)2 x loge 10 X10glO e
(c) loge 10 (d) None of these.
x loge 10 -x(log e x)2

9. If y = x,[X ,then dy is equal to


dx

(a)
2+logx
2$ (b)
2-logx
2$ (c) x
,[X [2 + 10gx]
I (d) x,[X [2 -2$10gx]
2"x
10. Ify = logco sinx. then dYequals
sx dx
(a) cot x.log cos x + tan x.log sin x (b) tan X.lOg cosx+ cotx.logsinx
(Iogsinx)2 (Iogcosx)2
(c) cot X.lOg cosx + tan x.logsin x
(d) None of these
(Iogcosx)2

11. IfxJ' = ~ -Y then dy equals:


dx
(a) x+ y x-y x-y
(b) Y (c)
x(I+logy) x(l+ logx) x(l+ logx) (d) x(l + logx)2

12. Ify =eux cos bx then dy equals:


dx
(a) eOx (a cos bx + b sin bx) (b) eox (a cos bx - b sin bx)
(c) eox (b cos bx + a sin bx) (d) eax (b cos bx - a sin bx)
x
I+e dy
13. Ify = - - x ' then - is equals.
I-e dx

14. Ify = (I - x l/4) (I - xll2) (I + xl/4), then dy equals.


dx
(a) -I (b) I (c) $ (d) -x.
I-cosx .
15. IfJ{x) = - - . - , thenf'(7tl2) IS equal to.
I-smx
(a) 0 (b) (c) 00 (d) does not exist
512 Remedial Mathematics

FILL IN THE BLANKS


1. Iff(x) = Ix - 21 and g(x) = Ix I, then g'(x) = - - ,
2x+I
2. Iff(x) = log tan -4-' thenf'(O) = _ _ '

3. Ify=f ( -2X-I)
- andf'(x)=smx ,2
,then -dy
= _ _,
x2 +1 dx
4. Iff(x) = ~ - al and g (x) = f[[(f{x»] then g '(x) = _ _ '

d(I+X2+X4)
6. If - = ax + b then a = b= ,
dx l+x+x2 ' --'--

7. Ify=log(x+ JI+x2 ),thenY2(0)= _ _ ,

TRUE/FALSE
Write 'T' for true and' F' for false statement
n dn
1. If u = ax + b, then ~ [[(ax + b)] is equal to - n [[(x)], (T/F)
dx n du

2. Ify=tan- I -4x- + tan-I (2+3X)


- - then - dy= - 5x -, (T/F)
1+ 5x2 3 - 2x dx 1+ 25x 2
d 2y dx d 2y dy d 2x
3 Ifx=f(t) andy =g(t) and i f - =0 then - , - = -,- (T/F)
. , dx 2 ' dt dt 2 dt dt 2
4. If xy = C2 , where C is a constant and if u is any function of x, then
du du
x-+y-=O (T/F)
dx dy

ANSWERS

MULTIPLE CHOICE QUESTIONS


1. (a) 2. (c) 3. (c) 4. (d)
5. (c) 6. (d) 7. (a) 8. (c)
9. (c) 10. (c) 11. (c) 12. (b)
13. (d) 14. (a) 15. (d)

FILL IN BLANKS

1. 1. 2. cosec G) , (2X-I)2{2+2X-2X2}
3. SI\1 -2-
X +I
2
(x + I)
2'

1
4. 1. 5. - 6. a = 2, b = - I 7. 0,
e

TRUE/FALSE
1. T 2. F ~ T ~ T
Differentiation 513

REFERSHER
Do you know? After reading the chapter, you should be able to know the following
concepts:

• Derivative of a functionf(x) is the limiting value of 8x as


8x
ox - 0, provided the limit
exist finitely and is denoted by : .
• The derivative of the sum of two function is equal to the sum oftheir derivatives.
d d d
dx [<p)(x). <P2(x)] = <p)(x). dx [<P2 (x)] + dx [<p)(x)]. <P2(x)

• Derivative of the quotient of two functions


d d
.:!-. [<PI(X)] = <P2(x)'d;[<PI(X)-<PI(X)'d;[<P2(X)]
dx <P2(x) [<p2(x)f
• If y is a function of u and u is a function of x, then
dy=dydy
dx du' dx
• Ifx is a function oft i.e., x = f(x) andy is also a function oft i.e.,y = g (t), then
dy dy/dt .
- = --, where t IS a parameter.
dx dx/ dt

• Its is known that derivative of y w.r.t. x (if exists) is denoted by dy is called the first
dx
derivative ofy.
d2
dy
Further, derivative of - w.r.t. x (ifit exists) is denoted by
dx dx
-f
and called the second
derivative ofy.
List of standard derivatives:

_ ~_I
1. -d ( x n) -n
d
2. -(x) = 1.
dx dx
d .
3. .:!-.(sinx) =cosx 4. -(cosx) = smx.
dx dx
d d
5. -(tanx) = sec2 x 6. - (cot x) = - cosec2 x.
dx dx
d d
7. -(sec x) = sec x tan x 8. - (cosec x) = - cosec. cot x.
dx dx
d -I I
9. -d (sm
. -I)
x = 1 10. -(cos x)=- ~
dx ~1-x2 dx 1- x 2
d I d -I __I_
ll. - (tan- I x) = - - 12. -(cot x)= 2
dx 1+x2 dx I +x
514 Remedial Mathematics

d d 1
13. -(ff)=ff 14. -(log x)= -.
dx dx e x
d 1 d _ 1
15. - (log x) = - log a 16. -(sec I x)=
dx e x e dx xN-l
d _I -1
17. -cosec x = r,
dx x'\Jx2 -1

Can we do? (Frequently Asked Questions)


• Find the derivatives the following functions with respect to x.
1. log2x [UPTU B. Pharma 2001]
2. ff + ff logx [UPTU B. Pharma 2007]
3. ff cos x [UPTUB. Pharma 20041

4. IOge(X+~) [UPTU B. Pharma 2004]

5. loge (~+ 3x) [UPTU B. Pharma 20011


6. log log log X3 [UPTU B. Pharma 2007]
7. logsin~+ 1) [UPTU B. Pharma 20041
8. sin (a tan-Ix) [UPTU B. Pharma 2005]
9. sin[cos(tanx)] [UPTU B. Pharma 2006)
10. x sinx [UPTU B. Pharma 20041
11. (sin x)logx [UPTUB. Pharma 2003]

12. Ifx=a?,y=2at, find dy [UPTU B. Pharma 2005)


dx

'" show that dy = .-L [UPTU B. Pharma 2003]


, dx 1- y

d 2y
14. Ify = ff + sin x, find dx 2 [UPTU B. Pharma 2006]

15. Ifx = a(cos t + t sin t),y = a(sin t- t cos t), find : . [UPTU B. Pharma 2008]

dy
16. Ify=(tanx)Cotx+(cotx)tanxfind dx. [UPTU B. Pharma 2008\
CHAPTER

12 INTEGRATION

lID INTRODUCTION
We have already discuss the methods of finding the derivatives of a function.f{x). We have
notice that the derivative of a function is also function (may be the function of independent
variable or constant). In this chapter we dealt with the converse of the derivative.
Consider the following question:
(I) What is the function, whose derivative is 8x7 .
(ii) What is the function, whose derivative is 8.
Obviously, x 8 and 8x be the function, whose derivative is 8x7 and 8 respectively, which
are called the integrals of the given function. So, Integral is the inverse operation of
differentiation.
For example: Ifthe derivative of tan x is sec 2 x, the integration ofsec2 x is tanx. Now if.f{x)

is the function ofx which is differentiable, such that ~ [F(x)] = .f{x). Then F(x) is known as
dx
the integral of .f{x) and denoted by F(x) = ff(x) dx and the function .f{x) is called the

integrand of ff(x) dx and the function ff(x) dx is read as "the integral of.f{x) w.r.t. x"

Remarks
• Integral is also known as primitive or antiderivative or an indefinite integral.
• The symbol f...
dx, stands for integral and taking separately the symbols is
meaningless.
• In general, integral is defined as the sum of the certain terms of a function at very small
interval.

III INDEFINITE INTEGRAL


To define the indefinite integral, let F(x) be a function and e be a constant.
d d de
So dx[F(x)+e] = dx[F(x)]+ dx =.f{x)
[.: derivative of a constant function is always zero]
516 Remedial Mathematics

f'
Bence, fl(x) dx = F(x) + C, where, the symbol' is an integral sign, and C is the
constant of integration. The constant C may have any value, but in general C is omitted and
the function F(x) is called indefmite integrandj{x).

Some Standard Results


nn+1
(I) fxndx ==--C, (ifn;e-l) (ii) fexdx = ~ + C
n+l
X
(iii) f'!'dx = loga + C (iv) f'!'dx = log x + C
x loga e X e

x
(v) faxdx = _a_ (vI) fcosx dx = sinx + C
loge a

(vii) fsinx dx = -cos x + C (viiI) fcosec 2xdx =-cotx+C

(ix) fsec 2 xdx =tanx+C (x) fsecxtanxdx = sec x + C

(xi) fcosecx.cotxdx =-cosecx+C (xiI) f dx = sin-l X +C


~(1-x2)
(xiii) f (-I)dx = cos-l x + C (xiv) f dx = sec- I x + C
~(1-x2) IxIJ(; -1)

(xv) f- dx = cosec-I x + C (xvi) f~ = tan-I x + C


I xl ~(x2 -1) l+x2

(xviI) fl+x2
-dx = cocl x + C

(xvii) fe dx = ex + C, where e is a constant.

Important Results on Integration


Result 1. Tile integral of tile product of afunction witll a constant is equal to tile product
of tile constant and integral of til at function. i.e., fCf(x)dx = C ff(x) dx
Result 2. Tile integral of tile sum or difference of two functions is equal to tile sum or
difference oftlleir integrals JUI (x) ± h(x)}dx = fii (x) dx ± fh(x) dx.

~~~~~~~I SOLVED EXAMPLES I~~~~~~~


Example 1: Evaluate the following integrals:
4
(I) f5x dx
Integratioll 517

(v) Indio
l-x 2

(vii) f ~dto
( (2_1

4 f 4 4+1 ,5
Solution: (i) We have, f5x dx = 5 x dx = 5 : + 1 = 50 x5 = x 5 + C
7+1 8
~+1 ~
7
(ii) fx dx = = +C
-n+1
x - ndx = _x__ + C; 11"* 1
(iii)
f -n+1
(iv) I(ax S +hx3 +cx+ d) dx = a fx 5 dx + h Ix 3 dx+c Ix dx + d fdx
x S+ 1 x 3+ 1 xl+1 x O+ 1
= ao--+ho--+c--+d--+C
5+1 3+1 2+1 0+1

1+1 2
= ~+logx+2x+C= ~+logx+2x+C
1+1 2

(vii) fNf2~-1
dt = sec- I
t + C.

Example 2: Evaluate the/ollowing integrals:

(I) f(cosecxcotx-3sec 2 x)dx (ii) (~ +-];;)dx


(iii) ftan 2 xdx (iv) N(I +cos2x) dx
(v) r( 4sinx + __3_)dxo
Jl 5 cos2 x 5sin 2 x
Solution: (i) We have

I(cosecxcotx-3sec 2 X)dx = I(cosecxocotxdx-3 fsec 2 xdx


= -cosec x - 3 tan x + C
518 Remedial Mathematics

x 1/2 + 1 1 X- I/ 2+1 ,
= Jb-I- + Jb 1 +c.
-+1 --+1
2 2
2Jb x 3/2
2xl/2
= +--+C.
3 Jb
2 2
(iii) We have 1= Jtan xdx = J(sec x -I)d~

= Jsec 2 xdx- Jdx =tanx-x+ C.

(iv) Wehave, JJ(1+cos2x)dx = N(1+2cos 2 X-l)dx (": cos 2x = 2 cos 2 x-I)

= J~(2Cos2 x) dx = J2 Jcosx d'C


= J2 sinx+C.
(v) We have

4sinx + - -
----,:-- 3)dx
- - - dx + -3 Jcos ec 2x dx
= -4 Jtanx
( 5cos 2 x 5sin 2 x 5 cosx 5

="54 Jtanx.secx dx+~" Jcosec 2x dx


4 3 4 3
= -secx+-(-cotx)+c = -secx--cotx+C.
5 5 5 5
Example 3: Obtain J(2-x)(x+l)dx. [RGPV B. Pharma 2001)

Solution: Consider J(2-x)(x+l)dx = JC2x+2-x2 -x)dx


2 3
2 x x
J(x+2-x )dx =-+2x--+C.
2 3
2 .

Example 4: Obtain (x + ~) dx. [RGPV B. Pharma 2001)

Solution: Consider (x + ~ r dx = (x2 + xI2 + 2 ) dx

2
= Jx dx+ J~dx+2
x
Jdx

x3 x-I x3 1
= -+-+2x+C = ---+2x+C.
3 -1 3 x
Example5:0btain J(5-4x)(1+x)dx. [RGPV B. Pharma 2003)
Integration 519

Solution: Consider f(S -4x)(l +x)dx = f{5+5X-4x-4x 2 )dx


, 3
2 x· 4x ,
f
= (5+x-4x )d'r =5x+ - - - + c .
2 3

Example 6.if~[f{x)] =3x2+2x+ l,thefindj(x).


dx

Solution: Since !f(X) =3x2 +2x+ 1

2
then f(x) = f{3x + 2x + l)dx = 3 fx 2 dx+2 fxcit' + fl.dx

x 2+ J 2.x J + J
= 3--+--+x + C =x3 +x2 +x + C.
2+1 1+1
Example 7. Evaluate thefollowing integrals:

(") (x3 + Sx - 6)
II
X
2
i
(X

(iii) f(x-a 2 )d'r

(v) r(4+3Sinx)dx.
J~ cos 2 x
Solution: (i) We have

f{ae X +x3 ) d'r = a fexdx+ fe 3 dx

(ii) We have

= fxdx+S J-!-dx-6 fx- 2 dx


X

3x J + J x-2 + J x2 6
= 2x---+5.log x - 6 - - = -+5Iog,x+-+c.
1+ 1 e -2 + 1 2 e x
(iii) We have

J= f{x-a 2 )dx = f{x 2 +a 2 -2ax)d'r

= f~ dx+if fd'r-2a fxdx


520 Remedial Mathematics

(iv) We have
2
J= ( ax3 + bx + cx + d ) u..(=
.J__ (
ax 2 + bx+c+-
d)=
.J••

x x

X 2+ 1 xl+l
= a--+b--cx+d·logex+C
2+1 1+1
ax 3 bx 2
= -+-+cx+d·logex+C.
3 2
(v) We have

n 4+3~inx)dx = n_4_+ 3sinx)dXS


J~ cos x Jl cos 2 x cos 2 x
2
= J(4sec x+3tanx.secx)dx

= 4 f(sec 2 xdx+3 ftanx.secxdx


= 4 tan x + 3 sec x + C.

• METHODS OF INTEGRATION
(I) Integration by sUbstitution: In this method, we transform the given integral into the
another standard integral of some other independent variable. The method of changing
the variable is called substitution method. To explain the method of substitution,
consider the integral

ff{~(x)}~'(x)dx.
Put ~(x) = t so ~' (x) dx = dt.
So, ff{~(x)}.~'(x)dx = ff(t)dt

Now, ff(t)dt can be easily evaluated, then substitute t = ~ (x) and get the required
result.

Some important substitution


Expression Substitution
(i) if +x2 x = a tan 8 or cot 8
(ii) if-x2 x = a sin 8 or cos 8
(iii) ~_a2 x = a sec 8 or cosec e

(iv)
JSJS
--or - -
a+x a-x
x = a cos 28

(v)

-
x-b
or ~(x-a)(x-b) x = a cos 2 e + b sin 2 e
Integration 521

~~~~~~I SOLVED EXAMPLES ~I~~~~~~


Example 1. Obtain the/ollowing integrals

(i) fx 2 sin x 3 dx (ii) f( 4x + 5)6 dx


(iii) fx(x 2 + 4)5 dx (iv) f2x 3 ~(x2 + 4)dx.

Solution: (i) The given integral is fx 2 sinx3dx .

Put x3 = t so 3x2 dx = dt.

So,
fx 2·smx3dx = -31 fsmt.dt
. = -1 (-cos t) =- --+C
3
cos
3
t

= -.!.cos~+c. (.: t=x3 )


3

(ii) The given integral is f( 4x + 5)6 dx . Put 4x + 5 = t => 4 dx =dt

7
t 6 dt =.!. ft 6dt = L~_ + C
So,
f 4 4 47
= 1 (4x+5) +C = 4x+5 +C. (on putting t = 4x + 5)
4 7 28

(iii) The given integral is Jx (x 2 + 4)5 dx .

Put xZ + 4 = t so, 2x dx = dt
5 dt _ 1 5 I t 5+1 _ 1 6
So, ft 2 - 2" ft dt 2" 5 + 1 - 12 t + C

= ~(x2+4)6+C. (on putting t = x 2 + 4)


12

(iv) The given integral is f2x 3 ~(x2 + 4)dx. s


Put (xZ +4) = t so, 2x dx = dt

So, f2x2.x~x2 +4dx = f(t-4)-Jtdt


PI2)+1 t(l/2)+1
=---4--+C=
~+I ..!.+I
2 2
= ~(x2 +4)5/2 _~(x2 +4)3/2 +C. (on putting t=xZ +4)
5 3

Example 2. Evaluate f dx [UPTU B. Pharma 2001]


5 +4cosx
522 Remedial Mathematics

Solution: Let / = J5 +4cosx


dx J dx

Divide numerator and denominator by cos 2 ~ , we get


2

sec 2 ~dx 2
/= J 2
rec x/2 dx
9+ tan 2 x/2
sec 2 ~+8
2

put tan ~ = t, and differentiate in w.r.t. x, we get


2
1 2 x
-sec -dx =dt
2 2
2dt 2 -I t
Then /= J32 +t2 = 3tan 3+ C

tan-
= 32 tan -\ (-3-2Xl + C.
Some Special Cases of Method of Substitution
Case I: Linear function ofx i.e., (ax + b). If the integrand is a function of the form (ax +b),
the integral ofthe same form ofthe function is obtained by dividing the integral of function
by the coefficient ofx in the ax + b i.e., if
Jf(x)dx =$ (x),

the we have Jf(ax+b)dx= $(ax+b).


a

~~~~~~I SOLVED EXAMPLES ~I~~~~~~

Example 1. Obtain the integral JJ1-(cx+d)2


dx .

Solution: Here, the given integrals = JJ1-(ex+d)2


dx .

dx
Put ex + d = t, so e dx = dt or dx = -
e

So, J dx = ! J~ = ! sin-I t
Jl-(ex+d)2 e ~ e

= !sin- I (ex+d)+C (on putting t = ex + d)


e
Integration 523

Example 2: Obtain JI 06x dx.

Solution: Put 6x = t so 6 dx = dt

dt
or rfx = - .
6

So, fI0
6x
dx = flOt ~ ="61 f10t dt

(on putting t = 6x)

Some Important Results


II I (ax+b)II+1
(I) r(ax+b) dx= 1 ,n:;e-I
J a n+

(ii) e dx = -1 eax ,a :;e 0


fax
a

(/'/'/') fax+b
- - = -1
dx
a
Ioge (ax + b) ,a:;e 0

ax+h
(iv) Jeax+bdx = 7,a:;e0

(v) f(ax+1b)" dx_- a(n-l)(ax+b)


1
-
11 I' a -:;: 0, n:;e 1

(vi) fcos(ax+b)dx= sin(ax+b) ,a:;eO


a

(vii) fSin(ax+b)dx = - ~cos(ax + b), a -:;: 0


a

(viii) fsec(ax + b)tan (ax + b)dx = ~ sec (ax + b), a:;e 0


a

(ix) fcosec 2 (ax + b)dx = ~ cot (ax + b), a -:;: 0


a
d
cx+d 1 a cx+
(x)
fa dx = ---,
c loge a
c:;eO

(xi) f-cosec(ax + b)cot(ax + b)dx = ~ cosec (ax +b).


a
Case II: Integral oftire type f[~(x)]"~'(x)dx
In this case, we put ~ (x) = t. So,~' (x) dx = dt, then we get,
t ll + 1
j(x)= ft"dt=-+c,(n-:;:I)
n+l
524 Remedial Mathematics

Now, put t = c\l (x) in above result, we get


1
+c.
J
c\l(xtc\l'(x)dx = [c\l(xt+
n+ 1
2
Example 3: Evaluate the integral Stan 3 x.sec xdx. s

So1ution: Here, the given integral is Jtan 3 x.sec 2 xdx.


Put tan x = t, so sec2 x dx = dt
t 3+1 t4
so, we get Jtan 3 x.sec 2 xdx = Jt 3 dt= --+C = -+C
3+ 1 4
4
= tan x +C (putting t = tan x)
4
Case III: Function o/tlle type (a 2 ±x2)
In this case, put x = a· t so, dx = a.d!, and then substitute these value in given integral and

then integrate. After integration, put! = ~, we get the required result.


a
For example: Evaluate J 2 1 2 dx.
a +x

Solution: The given integral is fa 2 1 2 dx .


+x
Put x = at
dx = adt

So,

x
Now, put t = - , we get
a

1 2 dx = .!tan-1 (~).
fa 2
+x a a
Similarly, we can obtain the following results:

(I) fJa 1_x


2 2
dx = sin- 1 ~
a
(iii) Jx~x2 - a 2 dx
1 = .!. sec- i (~) (iv) JxJx-dx_a = -1 cosec-l(X)
-
a a 2 2 a a
Integratioll 525

Case IV: Function ofthe type Jxn-1f(xn)dx.

In this case, put xn = t ~ nxn - I dx = dt, substitute these values in given integral and ther
integrate and after integration, put t =~, to get the required result.

~~~~~~~I SOLVED EXAMPLES ~I~~~~~~


4 5
Example 1: Evaluate the integral Jx tan x dx.

J
Solution: The give integral is x tan x dx
4 5

Put x 5 = t so, 5x4 dx = dt

~ x4 = dx = dt.
5
4 5
So, we get Jx tanx dx = ~ Jtantdt s = ~ sec 2 t + C

= ! sec 2 t + C (on putting t = x 5 )


5
Example 2: Evaluate the integral Jx 3 sinx4 dx.

Solution: The given integral is Jx 3 sinx4 dx .


Puts x4 = t
~ 4x3 dx = dt

or x.i dx = dt
4
3 4
So,s we get Jx sinx dx = ~ Jsintdt= ~(-cost)+C
=- 1
-cosx.. + C (on putting t = x 4 )
4

Case V: Integral ofthe type Jf'(x)


f(x)
dx.

In this case, put f(x) = t. So,f'(x) dx =dt.

So, we get Jf'(x)


f(x)
dx = Jdx = lot ts
t
= logj{x) [on putting t = j{x)]
n-I
For example, Evaluate the integral fnx n dx.
x
Solution: Put xn = t so ~ - 1 dx = dt
n-I d
So. f~dx = f~ =logt+C
xn t
=Iog~+ C (on putting t = xn)
=n 10gx+C.
526 Remedial Mathematics

More Important Results


(i) fcotx dx = log sinx

(ii) fsecx dx = log (secx+ tan x) =-log(secx-tanx) = log tan (~+~)


(iii) ftan x dx = -log cos x = log sec x [UPTU B. Pharma 2002]

(iv) fcosecx dx = -log (cosec x + cot x) = log tan ( ~).


= log( cosec x - cot x)

~~~~~~I SOLVED EXAMPLES I~~~~~~


eX +e- x
Example 1: Evaluate the integral f X -x dx.
e -e
Solution: Put eX - e-x so, (eX + e-X ) dx = dt
ex +e-x
Therefore, f
eX _e- x
dx = f-dtt = log t + C
= log (eX _e-X ) + C. (on putting t = eX - e-X )
3x2 +2x
Example 2: Evaluate f 3 2 dx .
x +x +1
· 3x2 +2x
Solution: Let 1= f 3 2 dx
x +x +1
put x 3 + ~ + 1 = t, differentiating w.r. t. x
we get (3~ + 2x) dx = dt

Then I = fdt = log t + C = log (x 3 + x 2 + 1) + C.


t
Example 3: Evaluate the following integrals:

2xsinx2 dx
f cosx 2 (il)
e210gX -1) -dx
1
(1) ( e210gx +1 x
. 2
2 xsmx dx
Solution: (i) Here, the given integral is f
cosx 2 .
Put~=t so 2xdx =dt

2xsinx2dx fSintd
There tiore f 2 = - - t.
cosx cost
Again put cos t = v, so - sin t r;lt = dv
Integration 527

= f-dv
sint
Therefore, f--dx
cost
-
v
=-Iogv

= - log cos t + C
= - log cos :x? + c (on putting v = cos t)
= - log cos:x? + c (on putting t = x 2 )
= log sec:x? + C

(ii) The given integral is (


e210gX -1) -dx
I
e210gx +1 x
I
Put log x = t so, - dx = dt
x

Therefore,
e210gX
21
J
-1 1
-dx = f-2-I-
e
21
-1
dt =
i
fe + e_rdt
I
-e-
I

(
e ogx +1 x e +I
(Dividing numerator and denominator by el )
=Iog(el+e-~+C
= log (e logx + e -IOgx) + C (on putting t = log x)

Example 4: Evaluate fIe


x+vx
dx . IRGPV B. Pharma 20031

Solution: Let x = ? so that dx = 21 ell

f _ l-dx = f~dt
2
= 2 I-I dt
x+~ t +t 1+1
= 2 log (t + 1) + C

= 2 log (Fx + 1) + C.
Example 5: Evaluate J~dx. [RGPV B. Pharma 20011
1-3x
Solution: Put (I - 3:x?) = t
-6xdx = at
dt
xdx= -
-6

J_ x - dx = _'!'fdt = -.!.loo-(1-3x 2 )+C.


1-3x 2 6 t 6!:>

SinFx
Fx dx
Example 6: Evaluate f IRGPV B. Pharma 20041

Solution: Put Fx =t

=> .!.x-1/2 dx = dt
2
dx
=> - =2dt
Fx
528 Remedial Mathematics

-- fsinx2dt=2 fsin+dt =-2cost+C


fsinFxdx
Fx
= - 2 cos Fx + c.
cotx
Example 7: Evaluate f I (. ) dx . IRGPV B. Pharma 20021
og smx

'. - f cotx dx
SolutIOn. Let / - I (. )
og smx
put log(sin x) = t
or cot xdx = dt
dt
/=
ft =Iogt+ C

= log [log sin x] + C.


sinxcosxdx
Example 8: Evaluate facos 2
x+bsm x
. 2 [RGPV B. Pharma 2002)

sinxcosxdx
Solution: Let / = facos 2
x+bsm x
. 2 dx

put a cos 2 x + b sin 2 x = t


=> (-2a cos x sin x + 2b sin x cos x) dx = dt
=> (2b - 2a) sinx cos x dx = dt
dt
sin x cos xdx =
2b-2a
dt 1
/= I
(2b-2a)
-- =
t f
2(b - a)
logt

I log [a cos 2 x + b sin 2 x] + C.


2(b-a)
sinx
Example 9: Evaluate fsm. (x-a) dx )UPTU B. Pharma 2007)

Solution: Put (x-a)=t


So that dx = dt
f sinx dx = Fin(t+a) dt
sin(x-a) sin!
= fsintcosa~ cost sin a dt
smt

= cos a fdt + sin a feot! dt


= cos a. t + sin a log I sin tl + C
= (x-a) cos a + sin a log Isin (x-a)1 + c.
Integration 529

Example 10: Evaluate


x .
e -SIUX dx
(i) feX +cosx IUPTU B. Pharma 20011

rUPTU B. Pharma 20011

IUPTU B. Pharma 20021

(iv) Jlogsecx
tan x dx fUPTU B. Pharma 20031

Solution: (i) Put t? + cos x = t


So that (~- sin x) dx = dt

ex - sin x Jdt
fe X
+SIUX
• dx = - = log t + C = log (~ + cos x) + C.
t
x2
(ii) We have /= J1+ x 6 dx

putting x 3 = t => 3x2 dx = dt

/= .!.J~dx
6
= '!'f-t- dt
2
3 1+ x 3 1+ t

f
= -1- -2t- dt = -logO
1 + t2) + ~
ro

3 x 2 1+ t2 6

(iii) / = JO + log X )2 dx
x
1
Let 1 + log x = t => - dx = dt
x
3 (1 + log x)3
/ = fl2 dt = -t +C = + C.
3 3
(iv) Put log sec x = t
1
So that - - sec x tan x dx = dt
secx
tan x dx = at

tanx dx fl-t dt
flogsecx = = log t + C

= log (log sec x) + C.


530 Remedial Mathematics

Example 11: Find


2
l+x dx
f1+x4

Solution: Consider /=

(1+~) dx
= f( x--,;1)2 +2

Let (x - ~) = t. Then (1 + x~ ) dx = dt

/-- ft 2 +(.J2)2
dl = _1_tan- 1 _1_ + C
.J2 12
/ = ~ tan-1 ( ~l) + C.
Example 12: Evaluate the following integral:

(I) f OgX
x
dx (il) rinxcosx dx'
1+sin 2 x
logx
Solution: (i) Here, the given integral is f dx.
x
1
Put log x = t, so - dx = dt
x
Iogxdx - f d - t 2 c- (Iogx) 2 C
So,
f -
x
- - t 1- -+ -
2
+
2
(on putting t = log x)

(ii) Here, put 1 + sin 2 x = 1

Put 2 sin x. cos x = dt


1 f
Therefore, fsinxcosx
.
l+sm x
2
2
1
dx = - tdt = -log t
2
=-
1
2
log (1 + sin 2 x) + C.

III INTEGRATION BY PARTS


The method of 'integration by parts' is very powerful method of finding the integral. If the
integrand is the product of two different functions, then the method of "integration by parts"
can be used.
If fix) and g(x) are two functions ofx , then
ff(x)g(x)dx =j(x) fg(x)dx- f[!f(x){fg(X)dx}]dx.
llltegration 531

The method of 'integration by parts, can be written into words as follows:


The integral of the product of two function = First function x integral of the second
function - integral of (the derivative offirstfunction x integral of the secondfunction)

STEP KNOWLEDGE

• There is no general rule to choose the first and second function, but remember the
following points:
(I) If the second function is not given the unity may be taken as the second function.
(ii) The integral of second function must be known.
(iii) If necessary, then the above formula can be applied more than once.
(iv) If the integral is of the form fxnf(x) dx where n is positive, then ~ must be
taken as the first function.
(v) Trick using here is ILATE which stands for
I ~ Inverse Trigonometric, L ~ Logarithmic function
A ~ Algebraic function, T ~ Trigonometric function
E ~ Exponential function.

~~~~~~~I SOLVED EXAMPLES ~I~~~~~~


Example 1: Evaluate the following integrals:

(i) fx 2 sin x dx (ii) f x 2e3x dx.

Solution: CO Here, the given integral is fx 2 sin x dx .


Now, integrating by parts, taking x 2 as first function, we get
2
fx sinxdx =x2 fsinxdx- f{~x2 fSinxdx}dx
= _x2 cos X - f{2x.(-cosx)}dx
= -x2 cosx + 2 fx.cosxdx

=-x2 cosx + 2 [x. fcosxdx- f{! (x). fcosx dx}dx ]

=-x2cosx+2 [xsinx- fsinx~]


= - x2 cos x + 2x sin x + 2 cos x + C.
where C is the constant of integration.
(ii) Here, the given integral is fx 2e3X dx.
Now, integrating by parts, taking x 2 as the first function, we get
2 3X 3X 2 3X
fx e dx =x2 fe dx- f{! x fe dx}dx

x 2e3x e3x x 2 e3x 2


= -3-- f2X dx = -3--3 fx.e 3X dx.
J
532 Remedial Mathematics

Example 2: Evaluate
x x dx
(I)
Je----,,-
(1 +X)2
(UPTU B. Pharma 2007)

(il) JeX(-sinx+2cosx)dx. [UPTU B. Pharma 2004)

fex (x+l)-l dx
Solution: (i) We have Jex-x-dx
(1+x)2
=
J (x+l)2

= Jex[-I _ _I
x+l (x+l)2
Jdx.
= Jex[f(x) + f'(x)]dx wherej(x) = _1_
x+l
x x
= I(x) Je dx+ fe f'(x)dx

x x
= ~ j(x) - Je f'(x) dx + Je f'(x)dx
applying integration by parts to the first integral taking ~ as the second function.
1
~ j(x)+C =~- +c.
x+l
(iz) We have jeX(-sinx+2cosx)dx s

= j(_e 2X sinx + 2e 2x cosx)dx

_e 2x 2e 2x
-2--1 (2 sinx-cosx) +-2--1 (2 cosx+ sinx) + C

I
2 +1 2 +1

[ e
a +b
eOX
~
.: jeoxsinbxdx= 2 2 (asinbx-bcosbx+C

and fe~ cosbxdx = 2 2 (acosbx+bcosbx)+C.

e 2x
a +b

= - [-2 sinx+ cosx+4 cosx+ 2 sin x] + C


5
= e2x cos x + C.
Integration 533

Example 3: Evaluate f~ log x dx (UPTU B. Pharma 20081


x

Solution: We have f~logX


x
dx
II I
=> Integrating by parts, we get

= 10gx{ -;) - f;{;) dx

= -!...Iogx +
x
f-I
x
dx
2

1 1
= --Iogx--+C.
x x
Example 4: Evaluate the/ollowing integrals.

(I) f(a2_~2)3/2dx (ii) f(loge x )2 dx (iv)


n
fx logxdx .

Solution: 0) Here the given integral is f(a 2 1 2 3/2 dx .


-x )
Put x = a sin e
=> dx = a cos e d8

So,

~ fsec 2 8d8
a

12 I {
=-tan8+C=- x } +c.
a 2 a ~a2 _x 2
(ii) Here, given integral is f(lOge x)2 dx.
Here, the second function is not given so, we take second function as unity.

Therefore, f(loge x)2.1dx = (loge xi fl dx - 2


f{! (loge x ) fl.dx }dX

=x(logex)2- f 210ge x !....xdx


X

=x(logexi-2 flogex.1 dx

=x(loge x i-2 [f lOge x fl dx- f{! loge x fl dx}dx ]


= X (loge x)2 - 2x logex + 2 J-!-x dx
X
=x (logex)2 -2x loge x + 2x + C.
534 Remedial Mathematics

2
(iiI) Here, the given integral is Jx e dx.
X

Integrating by parts, taking x2 as the first function, we get


22 2 2 2 2
Jx e dx = x Je dx- J{!x Je dx}dx

=x2~-2 Jx.exdx

=x2~-2[X JeXctx- J{!xJex ctx}ctx]


= x2 ~ - 2x~ - 2~ + C
= ~ (x2 - 2x + 2) + C

(iv) Here, given integral is Jx n logxdx. Now, integrating by parts, taking logx as the
first function, we get
n n n
Jx logxdx = log x Jx ctx - J{! logx Jx dx}dx s

x n+1 x n+1 JI
;n+1
=logx·--- ---dx =logx·--- J--dx
xn
n+1 x n+1 n+l n+1
xn+1 I x n+1
= log x - - - - - - - - + C
n+l n+1 (n+l)
x n +1
= --(logx-n+I)+C
n+1
Example 5: Integrate log x. [RGPV B. Pharma 2004].
Solution: Jlogx dx = J(logx).l dx

= (log x.) . x- f!'.x


x
dx

Integrating by parts taking I as second function


= x log x - J1dx = x log x - x = x (log x - I)

Example6:Evaluate Jxtan- I xdx. [RGPVB.Pharma2001]

Solution: Jxtan- I x = tan-I x JXdx- {! tan-I x Jxdx Jdx.


xx 2
I
I x2
=tan- x -2'J + 2 dx
2 I x

=tan-I x xx - - -
2
1JI + x 2 -I dx
2 2 l+x2

= tan-I x. XX x
2

2
-![ 2
Jldx- J-I-dx]
l+x2
2
-I x .1 I -I
=tan xx-- -x+ - tan x+ C.
2 2 2
Integration 535

2
Example 7: Evaluate flog (1 + x ) dx. [RGPV B. Pharma 2001]

2
Solution: We have flog (1 + x )dx = log (1
I
+~) fl.dx - {! log(1 + x 2 ) fl.dx Jdx.
= log (1 +~)x- f~.2x.xdx
I+x

=xlog(l+x2)-2 J--dx
X2

1+ x
2

I+x 2 -1
=x log (1 + x 2)-2 J I+x 2 dx
=xlog(l +X
2
)-2[ •fldx- J~dx]
I+x
=x log (I + x 2) -2 [x-tan- 1 x] + C
=x log (1 + ~)-2x+ 2 tan- 1 x + C.

ExampleS: f+
x -I
dx . [RGPV B. Pharm 2004]

Solution: Putx2 - I = t
=> 2xdx = dt
dt
xdx = - .
2

J-x -x-Id x
2
= -I JI-dt =-logt
2 t
I
2
I
= -log(~-I)+C.
2
Example 9: Evaluate fxe ox dt. (RGPV B. Pharama 2003]

Solution: fxe
ox
dt =x (~eax ) - (~.eax) dx
Integrating by parts taking x as find function.

= !... eax _ ~ reoxdx = !... eax __l_eax = ~(x-~) e ax + C.


a aJ' a a2 a a
Example 10: Evaluate Jxlogxdx. (RGPV B. Pharma 2002]
2
x2
Solution: Jxlogxdx = (log x). -
2
- J--
1 x
x 2
dx

Integrating by parts taking x as second function.

= ~~ logx- ~ fxdx
2 2
1 2 1 2
= -x logx- -x +C.
2 4 {
536 Remedial Mathematics

Example 11: Integrate tan -Ix IRGPVB. Pharma 20011

Solution: ftan- I xdx= ftan- I x.ldx


Integrating by part x taking x as first function.
f
=xtan- I x- -1 - 2x
-dx
2 1+x2
1
=xtan- I x- -Iog(l +~)+C.
2
Two Important Formulae
eOX
ax cos bx dx = -2--2 (a cos bx + b sin bx) + C
(i) fe
a +b
ax
(ii) Je sin bx dx = ~ (a sin bx - b cos bx) + C
ax
a +b
Example 12: Evaluate the following integrals:
3
(i) fx 2 cosx dx (ii) fx cos x dx

(iii) fx 2 sinx dx (iv) fex.cosx dx


2
Solution: (i) Here, the given integral is fx cos x dx . Integrating by parts, taking x 2 as first
function, we get
2 2
fx cosxdx =x fcosxdx- {~(x2)·fcoSXdxJdx
=x2 sinx-2 fxsinxdx

=~ sinx-2[ fx(-cosx)- f{~ (x) fSinxdx}dx]


= ~ sin x - 2 [- x cos x - f cos x dx ]
= ~ sin x + 2x cos x - 2 sin x + C.
3
(ii) Here, the given integral is fx cosx dx .
Integrating by parts, taking x 3 as the first function, we get
3 3
fx cosxdx = x fcosxdx- {! (x)3 fcosxdx Jdx

=x3 sinx-3 fx 2 sinxdx

= x 3 sinx-3 [x 2 fsinxdx- f{2x·fsinxdx}dx]

= x 3 sinx+3x2 cosx-6[x.sinx- fsinxdx]


=x3 sinx + 3x2 cosx- 6x sinx- 6 cosx + C.
where C is the constant of integration.
= (x 3 - 6x) sin x + (3x 2 - 6) cos x + C
Integration 537

(iii) Here, the given integral is fx 2 sinx dx .


Integrating by parts, taking ~ as the first function, we get
2 2
fx sinx dx = x fsinxdx- f(2x fsinxdx)dx

=-~ cosx + 2 fxcosxdx s

=-~cosx+2 [xsinx- fsinxdx]


= - x 2 cos x + 2x sin x + 2 cos x + C
(iv)Let f = fex.cosxdx.
Integrating by parts, taking cos x as the first function, we get
fexcosxdx =cosx.~+ fsinx.exdx.
Again integrating by parts, we get
fex cosx dx = ex cos x + sinx~- fcosx.exdx
+
I = ~ cos x ~ sins x - I
2I = ~ cos x + ~ sin x

1= !(~ cosx+~sinx)= eX (cosx+sinx)+C


2 2

III INTEGRATION BY PARTIAL FRACTIONS

If the given function is of the type I(x) ,wherej{x) andg(x) both are polynomials, then we
g(x)
may assume that the numeratorj{x) and denominator g(x) have no common polynomial factor
and that the degree ofj{x) is less than the degree of g(x). If the degree ofnumeratorj{x) is
greater than or equal to the degree of denominator g(x), then we have to dividej{x), by g(x)

so that I(x) = hex) + rex) , where hex) is the quotient (a polynomial) and r (x) is the
g(x) g(x) ;
remainder whose degree is less than the degree of g(x}. The method in which we change the

I(x) into partial fractions, depends on factors of the denominator. We know that every
g(x)
polynomial can be expressed as a product oflinear and irreducible quadratic factors with real
coefficient. So, we have the following cases:
(I) If all the factor of denominator are linear and non repeated, then for each linear non-

repeated factor (ax + b), there corresponds a fraction of the form _A_, where A is
ax+b
a constant, is to be obtained.
(ii) If all factors in the denominator are linear but some of them are repeated, then for each
t
linear factor (ax + b repeating n times, there correspond the sum of r partial fractions.
538 Remedial Mathematics

(iii) Ifthe factor in the denominator are linear and irreducible but quadratic factor and are
non repeated, then for each irreducible quadratic factor a:? + bx +c, which occurs
only one in the denominator, there correspond a partial fractions of the form
A
2
ax +bx+c

~~~~~~~I SOLVED EXAMPLES ~I~~~~~~


x+4
Example 1: Evaluate the integral J3+2x-x 2 dx .

x+4
Solution: Here, the given integral is J3+2x-x 2 dx

x+4 x+4 A B
Consider --+--.
---- =
(3-x)(1+x) 3-x I+x
So, x+4=(l+x)A+(3-x)B

Putting x = 3, we get A = 7.. and putting x = -I, we get B = ~


4 4
x+4 7 3
So, we have ----::- +
3+2x-x 2 4(3 -x) 4(1 +x)
x+4 dx = 7.. J-I-dx~ J-I_
Hence, J3+2x-x 2 4 3-x 4 I+x
dx; Integrating

7 3
=- -log(x-3)+ -Iog(l +x)+C
4 4
3x+2
Example 2: Evaluate the integral J(x-2)(x+ 1) 2 dx .

3x+2
Solution: Here, the given integral is J(x-2)(x+ I) 2 dx .

3x+2 ABC
Consider = --+--+---
(x-2)(x+1)2 (x-2) (x+l) (x+I)2
or (3x+2) = A (x + 1)2 + B(x- 2)(x + 1) + C (x- 2)
. 8 d . I
On puttmg x = 2, we get A = '9' an putting x = - I, we get C = "3 .
Now, on comparing the coefficient of x2 , we get A + B = 0
8
i.e., B=-A=--
9
3x+2 8 8
So, we have 2 ---- +---
(x-2)(x+ I) 9(x-2) 9(x+l) 3(x+I)2
Integration 539

Hence I 3x+2 dx =! f_l_ dx _! f-I-dx+~ f 1 dx


, (x-2)(x+I)2 9 x-2 9 x+1 3 (x+I)2
8 8 1 1
= - log (x - 2) - - log (x + 1) - - - - + C
9 9 3 (x + 1)

= !IOg(X-2)_ 1 +C
9 x+1 3(x+l) ,
Example 3: Evaluate

(1) fX:X2 [UPTUB. Pharma 20051

(ii) fX-X
dx 3 [UPTU B. Pharma 2004]
3
(iii) f(x -l)(x - 2)(x - 3) dx
X
[UPTU B. Pharma 2006)

·
SoIUhOR: (') vve have
1 nr - -12 = 1 = -I - -1- , reso I" , I tifaCtIons,
vmg mto partla ' so
x+x x(1 + x) x I + x

fx~x2 = R~-l~Jdx
I~dx- I-I dx
=
x l+x
= log lxi-log II + xl + c.

=IOgl~I+c.
I+x
1 1 I ABC
(ii) Here, - - 3
= 2 = = - + - - + - - (say),
x-x x(1-x ) x(1 +x)(1-x) X I-x (1 +x)
To find A suppress x in the given fraction and put x = 0 in the remaining fraction.

So A= I = I similarly B = _1_ = ~ and C=-1/2


(1 + 0)(1 - 0) 1(1 + 1) 2 .
I I 1
Then -- =-+---
x-x 3 X 2(1-x) 2(1+x)

f-x-x
I - dx
3
= f'!"dx+'!"f-I_dx_'!"f-I_ dx
x 2 I-x 2 (l+x)
I I
= log x - -log (1 - x) - - log (1 + x) + Cs
2 2
= log x - ~ 10g(1 - x 2 ) +C
2

= ~IOg[~)+c.
2 (1-x 2)
540 Remedial Mathematics

(iiI) Here degree ofthe numerator is not less or degree lower than the denominator. We
divide the numerator by the denominator till the remainder is of lessor degree than
the denominator

~ ABC
------- = 1+--+--+--
~-Q~-~~-~ ~-Q ~-~ ~-~
3 3
1 1 2
We have A = (1-2)(1-3) = "2,B= (2-1)(2-3) =-8
3
3 27
and C = (3-1)(3-2) = 2"

x3 I 8 27
.. = 1+ ------+---
(x-Q(x-2)(x-3 2(x-l) (x-2) 2(x-3)
3
Hence I(x-l)(x-2)(x-3)
x dx - II dx + I dx
z(x-l)
I(x-2)
8 dx I 27 dx
+ 2(x-3)

=x + "21 log 1x-11-8Iog Ix-21 + 2"


27
log 1x-31 + c.

7X 2 +3x+l
Example 3: Evaluate the integral f x(x + 1)

7x 2 +3x+l
Solution: Here, the given integral is f x(x + 1)
To change the integrand into partial fraction, first we divide (7~ + 3x + 1) by (~+ x),
because the degree of numerator must be less than the degree of denominator. So we get
7x 2 +3x+l 1-4x
- - - - =7+--
x(x+l) x2+x

So, f-
2
7x- -+3X+l f f
l 4x
- = 7 d x +-- -
x(x+l) x 2 +1
l - 4X
x +xf
=7x+ -2--dx ... (1)

l - 4x
f
Now, to finds -2--dx.
x +x
1-4x A B
Let -2 - =-+--
x +x x x+l
=> 1-4x =A (x-l)+ Bx ... (2)
putting x = - 1 => B = - 5 and putting x = 0 => A = 1
1-4x 5
Thus,
x2+x =~- x+l
Integration 541

So, I-4X = II--5 I- 1d x =Iogx-5 log (x + 1)


I-2--dx
x +x x x+I
Hence, by equation (1), we get
7x2 +3x+I
J x(x+I) dx=7x+logx-5log(x+I)+C.

Example 4: Ix2 +xx _ 6 dx [RGPV B. Pharma 2002J

Solution: /= I 2 Xdx = J( 2;( 3) dx.


x +x-6 x- x+
x A B
Let =--+--
(x-2)(x+3) (x-2) (x+3)·
x =A(x+3)+B(x-2) ...(1)

Putting x =-3 in eq. (1), =>-3 =-B =>B= ~.


5

Putting x = 2 in eq. (1), => 2 = 5A => A = ~.


5
~f_I-dx+~f-I-dx
Ix 2 +x-6
x =
5 x-2 5 x+3
2 3
= -log(x-2)+ -log(x+3)+C.
5 5

Example 5: Evaluate f(2x + I)(x


x 2
+ I)
dx [RGPV B. Pharma 2004J

Solution: Let! = J(2x+I)(x


x 2
+1)
dx

. x A Bx+C
puttmg (2x + 1)(x 2 + 1) = (2x + 1) + (x 2 + I)

x A(x 2 +1)+(Bx+C)(2x+l)
(2x+1)(x 2 +1) (2x+I)(x2+1)
x =A (x2+ 1)+(Bx + C)(2x+ 1)
putting x = -1/2 and 0
1
when x=--
2
-.!. = ~A=-~
2 4 5
when x =0
=> O=A+C

C=~
5
542 Remedial Mathematics

equating the coefficient of x 2 is both side of equation


o =A +2B
B = A =~.
2 5
dx=-'3.f dr: +~JX+2dx
f(2x + l)(x
x
2 + I) 5 (2x + I) 5 x 2 + 1 .

2 J dx 1 Jxdx 2 J 1
="5 (2x + 1) + 5" x 2 + 1 +"5 dx x 2 + 1 dx .

= -~log(2x+I)+~log(x2 +1) + '3. tan-I x+C


5 10 5

Example 6: Evaluate f(x 2 dx


+ l)(x-l)
[RGPV B. Pharma 2003]

dx
Solution: Let ]-
- f(x 2 +l)(x-l)
A Bx+C
putting ----;;;----- = - - + - -
(x 2 +I)(x-I) (x-I) x 2 +1
::::> I = A (x 2 + 1) + (Bx + C) (x - 1).
putting x=landO.
When x = I
::::> 2A = I
::::> A =112
when x = 0 ::::> A - C = 1
::::> C =112-1 =-112
equating the coeff ~ = 0 in both sides of the equation.
A +B=O

B=-~.
2

Now f 2 1 dx =.!. f_l_dx_'!'fx+I dx


(x +I)(x-I) 2 x-I 2 x2 +1

= ~ f-I- dx - .!. f xdx -.!. f-I- dx


2 x -I 2 x2 +1 2 x2 +1 .

= .!.log(x-I)- ~log(~+ 1)- ~tan-IC+ C.


2 4 2
2x+ 1 dx
Example 7. Evaluate f (2x + 3) (3x _ 4) . [RGPV B. Pharma 2001)

'.
SolutIOn. LetI-
- f(2x+3)(3x-4)
(2x + I) dx
Integration 543

2x+l A B
Let ---+---
(2x + 3)(3x - 4) (2x + 3) (3x - 4)
=> (2x+ 1) = A (3x-4) + B(2x + 3) ...(1)
putting x = 4/3 in eq (1)

=> ~+I=B(~+3)
17 11 11
=> -B=-=>B=-
3 3 17
3
When x= --
2

=> -3 + 1 A ( - ~ - 4)
=

-2= -17 A
2

=> A=~.
17
f(2x+3)(3x-4)
2x + 1 dx = ~
17 f(2X+3)
1 dx + .!.!. dx
17 f(3X-4)·
2 11 .
= -log(2x+3)+ -log(3x-4)+C.
17 51
2x dx
f
Example 8: Evaluate (x -I)(x + 3) . (RGPV B. Pharm 20021

2x A B
Solution: Let ( 1)( 3) = - - + - - .
x- x+ (x-I) (x+3)
2x A(x+3)+B(x-I)
=>
(x-I)(x+3) (x-I)(x+3)
=> 2x = A (x + 3) + B (x - 1) (1)
putting x = - 3 in eq (1) => - 6 = - 4 B

=> B=~=~.
4 2
put in x = 1 in eq (1) => 2 = 4A => A = 112

f(x-I)(x+3)
2x dx
=
1
2"
f(x-I)
dx 3 f dx
+2" x+3
1 3
= - log(x-I)+ -log(x+3)+C.
2 2
x2
Example 9: Evaluate f;x -x 2
- 2x
dx (RGPV B. Pharma 2001)
544 Remedial Mathematics

x2 2
Solution:l= fx 3 2
-x -2x
dx = f x(x 2X
-x-2)
dx

=I x dx=I x dx
(x 2 -x-2) (x-2)(x+ 1) .
x A B
Let =--+--.
(x-2)(x+I) (x-2) (x+l)
x A(x+I)+B(x-2)
(x-2)(x+I) (x-I)(x+l)
x =A (x + 1)+ B(x-2) (1)
B = 1/3.
putting x =-1 in(J)

Putting x = 2 in eq (l) , => 2 = 3A => A = ~.


3

~ I-1-dx+! I dx
fx 3
x2
_x 2 -2x
dx =
3 x-2 3 x+I
2 1
= -log (x - 2) + - log (x + 1) + C.
3 3

Example 10: Evaluate the integral I( x- I)~2x+ 1) dx.•


Solution: Here, the given integral is I(x -1)~2X + 1) dx .
. x A B
ConsIder = - -+---
(x-I)(2x+I) (x-I) (2x+l)
=> x=(2x+I)A+(x-I)B.
. cr x = 1 we get A
N ow, puttm = -3'land · cr x = - -I we get B = -I
puttll1
b ' b 2' 3
x
Thus, ---+---
(x-I)(2x+I) 3(x-l) 3(2x+l)

Hence, f x dx = !f_l_dx+!f-I- dx
(x-I)(2x+l) 3 x-I 3 2x+I
1 1
= "3 log (x - 1) + "6 log (2x + 1) + C
2
Example 11: Evaluate the integral f(x-I)x (x-2) dx.
3

2
Solution: Here, the given integral is f(x-I).~ (x-2) dx.
integration 545

x2 ABC D
Let = --+--+--+-- ... (1)
(x-I)\x-2) x-I (x_I)2 (x-I)3 x-2
=>x2=(x-li(x-2)A +(x-l)(x-2)B+(x-2)C+ (x-I)3 D ... (2)
put x = I => C = - I
put x = 2 => D = 4
Now, we want to find the coefficients A and B.
On comparing the coefficient ofx3 in (1). we get
A +D=O.
Now comparing the constants terms in (1), we get
-2A + 2B-2C-D =0
Putting the values of C and D, we get
A =-4,B =-3
2
x 4 3 I 4
Thus, ---::---- = - - - - - - - - + - -
(x-I)\x-2) x-I (x_I)2 (x-I)3 x-2'

Hence f(x-I)3(x-2)
x2 dx=-4 f-dx+-
I 3 f- -I d x - f--dx+4
1 f--dx
I
, x-I 2 (x_I)2 (x_I)3 x-2
3 I
=log(x-I)+ --+ 2 +4Iog(x-2)+C
x-I 2(x-l)

(X-2)
x-I
3
=4log - - + - + I
x-I 2(x-I)2
+C.

Evaluate the following Integrals

1. JSinFx
Fx dx
3. J(4x+2)~x2 +x+l dx 4. fcosecx.log(cosecx-cotx)dx

5. J
sec 2 (2 tan- 1 x) dx
1+x2
6. J1- sin 2x dx
x+cos 2 x
7. J cosec: dx 8. JI-cotx dx
log tan- 1+ cot x
2

9. J4x3~5-x2 dx

11. J. x
dx
x
sm-+tan-
2 2
546 Remedial Mathematics

HINTS TO THE SELECTED PROBLEMS

1. /= fit dx

Put Fx =t
_l_dx =dt
2.Jx
~ =2dt
Fx
J =-2 cos .Jx +C

I dx
Put - =t=> - 2 =-dt
x x
2 rrl+cos 2t) 2 l+cos 2X)
f
J=- cos tdt =- J~ 2 dt ( ':cos x = -2 - -

= - i I(1 + cos2t)dt, Integrating

= -i (1+ Si~2t)+C
1 -!sin~+C,
= __
2x 4 x
3. J = f(4x+2)~x2+X+ldx
P~ ~+x+l=t
=> (2x+I)dx=dt
J =2 It ll2 dt,

t 3/2 4
= 2-+C = _t 3/2 +C
3/2 3
=.i (~+x+ 1)3/2+C
3
4. /= fcosecx,log(cosecx-cotx)dx
Put log (cosec x - cot x) = t
I
- - - - - ( - cosec x cot x + cosec2 x) dx = dt
cosecx-cotx

t2 1
J= ftdt = -+C= -{loglcosecx-cotxl}2+C
2 2
lntegration 547 \

Put tan-I x = 1

_l_dx =dt
l+x2
2
1= fsec (2t) dt, Integrating
tan2t .
= ~+C puttIng the value of t, we have

= -1 tan (2 tan_I x) + C
2
6. 1= f I - sin 2x dx
2
x+cos x
Put x + cos 2 x = t
(l - 2 sin x cos x) dx = dt
(l - sin 2x) dx = dt

1= Jdt, Integrating
t
I = log t + C = log Ix + cos 2 xl + C
cosecx x
7. 1= flogtan~ dx; put log tan -2 = t, differentiating
2

We get -I- ( sec 2 - -dx


x
I
2 2
=dt x)
tan-
2

=> dx =dt
2 sin x / 2 cos x / 2
=> cosec x dx = dt
dt .
t =
f-,Integratmg
t
= log t + C
putting the value of t

= I
log log tan ~I + C
8. 1= JI-cotx dx = Js~nx -cosx dx
l+cotx smx+cosx
Put cos x + sin x = t
=> - (sin x - cos x) dx = dt
548 Remedial Mathematics

dl
T= -
ft = -log 1 + C,

Putting the value of 1


/ = -log I cos x + sin xl + C

9. /= f4x3~5-x2 dx
= f4x3~5-x2dx

= f4x2.~5-x2 ·xdx
2 dt
Put5-x-=I=>-xdx= -
2

/=4 f(5-t)v't( - ~)
=-2 f(5v't -p/2)dt, Integrating

t 3/2 2 512 }
=-2 { 5 3/2 -Sf +C

= - 20 13/2 +~t512 +C , putting the value of t


3 5
=_ 20 (5_x2)3/2+ ~(5-x2)512+C
3 5
X
10. /- f(x+l)e dx
- cos 2 (xe x )
Put x eX = t => (eX + x eX) dx = dt
=> eX (x + 1) dx = dt

/ = Jcosd~ t = fsec 2 t dt ; Integrating

= tan t + C, putting value of t


= tan (xeX) + C

ANSWERS I

1 1. 2 C
1. -2 cosJ; 2. ----sm-+ s
2x 4 x
3. ~ (x2 + x + 1)3/2 + C
1
4. - {log Icosec x - cot xl}2 + C
3 2
5. '!'tan (2 tan- 1 x) + C 6. log ~+ cos2 xl + C
.; .~ 2
7. log Ilog tan ~I + C 8. -log Icos x + sin xl + C
Integration 549

10. tan (xeX) + C

11. J21 log Icosec (1t/4 -x/2) - cot (1t/4 + x/2) + C

~~~~~I MORE SOLVED EXAMPLES I~~~~~~

Exampel:Evaluate f~dx.
l+e
1 1 eX
Solution: I = f--dx
x = f--dx = f - dx x
l+e- l+~ I+e
eX
Putting 1 + eX = t~ eX dx=dt

1= fdt = logltl + C = logll + eXl + C


t
sin2x
Example 2: Evaluate fa sm x+b cos x dx
2. 2 2 2

sin2x
Solution: We have 1= f 2. 2 2 2
a sm x+b cos x
Let a2 sin 2 x + b 2 cos 2 x = t
~ (2a2 sin x cos x - 2b 2 sin 2x cos x) dx = dt
~ (a2 sin 2x- b2 sin 2x) dx = dt
. dt
sm 2x dx = -2--2
a -b
Putting in (1), we get
dt dt
/= ft(a 2
-b)
2 = -2--2 log It1/ + C
a-b

= ~ log 1a2 sin2 x + b2 cos2 xl + c.


a -b
dt
Example 3: Find fx{l +x ) n

dt f nxn-1dxn
Solution: Let! = fx(1+xn) = X (1+x )
(multiplyby~-l into Nr. and Dr.)
put (I+~=t~xn=t-l

~ - 1 dx = dt ~ ~ - 1 dx = dt
n
550 Remedial Mathematics

Therefore, 1 dt
1= -;; (t-1)t
f
= ~
n
n_
J\J-1
1
__ !)dt
t
(by partial fraction)

Integrating I = ~ {log It - 11- log ItI + c = ~ log It -11 +C


n n t

= ~IOg/~I+c
n xn +1
( ... putting the value of t)

(2x+5)
Example 4: Evaluate fvxI 2
+3x+ 1
. dx

Solution: Let/= f (2x +5) = f (2x2+3) +2 dx


Jx +3x+l Jx +3x+1
2

_J (2x+3) dx+ J 2 dx
- ~x2 +3x+ 1 ~x2 +3x+ 1 '" (1)

Suppose I = J (2x+3) dx
1 ~x2 +3x+1
put ~+3x+2 = t
(2x+ 3) dx = dt

Now 11 = f~ =

2dx dx •
12 = f-Jx2+3x+l = 2 (23991) 1/2
x + x+---+
4 4

=210 g !( x+%)+~x2 +3X+l!+C2


Now 1=11 + 12

Example 5: Evaluate J cotx dx.


log sin x
Solution: Let/= J cotx dx
logsinx
Putting log sin x = t:::::;> _._1_ cos x dx = dt
smx
cot x dt = dt

Then I = fdt = log ItI + c


t
= log I log sinxl + C
Integration 551

Example 6: Evaluate fFxx(1x +1 dx


Solution: Let/= fFxx( 1 x +1
dx

Putting Fx + I = t, differentiating w.r.t. x


_1_dx =dt
2Fx

~ =2dt
Fx
2dt r
Then j = -
tf= 2 log It + C = 2 log I
'
"X + II + C.

Example 7: Evaluate Jsec3 xtanxdx .

Solution: Let j= Jsec 3 xtanxdx

j= Jsec 2 x(secxtanx)dx
Putting, sec x = t, differentiating w.r.t. x, we get

secxtanxdx=dt
3
t
Then /= rt 2 dt = -+C
J' 3'
Now, putting the value of t, we have
3
_ sec x C
j - --+.
3
2
logx
Example 8: Evaluate f--dx.
x
2
Solution: Let j = flogxx dx

= P log x dx
x
Putting log x = t, and differentiating
1
We get - dx = dt
x
2t2
Then /=2 ftdt = T+C=F+C
After putting the value of t, we have I = (log x)2 + C.
2 tan-I x 3
Example 9: Evaluate fx l+x
6 dx
552 Remedial Mathematics

Solution: Let

Let us put tan-I x 3 = t, differentiating


1 2_
---:-3-::-2 . 3x dx - dt
l+(x )
2
x dt
--dx =-
1+x6 3

1 t2
Then 1= 3' Jt dt = 6" + C
Putting the value of t, we gets
(tan-I x 3 )2
j= +C.
6

Example 10: Evaluate In n1sin- 1 x

l_x2
dx .

• IenlSin-l x
Solution: Let! = ~ dx
,,1- x-
1
putting sin- x = t, differentiating w.r.t. x,s we get
1
,-:--;; dx = dt
"1-x2
enlSin7"1 x
---+C. (After putting the value of t)
m

1. Evaluate the following integrals:

(I) (2+3:0 x )dxg


(ii) 2
f3sec (3x+9)dx

(iiI) I(8-6xi
1 dx (iv)
f 2;':3
2. Evaluate the following integrals:
f sin2x dx 7
(I) (ii) Jsec xtanxdx
cos 2 2x

(iil) fxlogx
1 dx (iv) fxsinx 2 dx
Integration SS3

3. Evaluate the following integrals:


eX
(l) f a dx (ii) f-d x
b+ce x I +e 2x

(iii)
2
pec (1ogx) dx (iv) f 2
sec x dx
x 1+ tanx .
4. Evaluate the following integrals:

(l) fcosecxdx (ii) f cotx dx


logsinx
3
(iii) f~dx
1+x8 .
S. Evaluate the following integrals:

(/) f2. IOg logxdx


x
(ii) fxJ']+;
1 dx

(iii) NCa 2 -x 2 )dx (iv) fxsin xcosxdx.


6. Evaluate the following integrals:
(I) fxcos 2xdx (ii) fxcos 2xsinxdx

(iii) f 1 dx
x2~(1 +x2)
7. Evaluate the following integrals:

Ci)
flog sin-I x dx
(ii) fJ(l J2dx dx
~(l-x2) + cos2x) .
8. Evaluate the following integrals:

(i) fXdx
.JI
+x
(ii) fJl +sinxdx
flog(l;X2) dx.
(iii) fcos 4 xdx (iv)
x
9. Evaluate the following integrals:

(/) 3X (ii) f xll2 log x dx


fxe dx

(iii) f l+x dx (iv) seX


-[x(1ogx)2 +2Iogx]dx.
(2+x)2 x
10. Evaluate the following integrals:

(l) f tanx dx (ii)


eX
f-(xlogx+l)dx
secx+cosx x
f sin2x+l dx
(iii) (iv) fcosx.c~s 2x.cos 3xdx.
~(x+sin2 x)
554 Remedial Mathematics

11. Evaluate the following integrals:

(/) f.}1 + cosx dx (ii) r~S2x dx.


smx
12. Evaluate the following integrals:

(i) Jcotxdx dx (ii) J x+3 dx


logsinx ~(1-x2)

(iii)
f cosx-sinx dx
(iv) J(sinx -cosx)2 dx.
(cosx+sinx)2
13. Evaluate the following integrals:

(l) fx{I+(logx)2}
I dx (ii)
J 6x-8
3x2 -8x+5 dx

(iii) r o t dx .

14. Evaluate the following integrals:


xex dx 4x 3
(l) (ii) J -4 d x
f(X+I)2 5x +7

(iiI) J cos2x dx (iv) Fin~x-a) dx.


(cos x + sinx) smx
15. Evaluate the following integrals:

(l) x
3
J(x-a)(x-b)(x-c) dx (ii) f x-l
(x + 1)(x 2 + 1) dx

(iii) f 3xdx (iv) J


x2
dx.
(x_l)2(x+3) (x-a)(x-b)
16. Evaluate the following integrals:
3x 2
(l)
f(X+l)(X+2)(X+3) dx
(il) feX +e1 2x dx
(iii) f 2x+ 1 dx
(tv)
x 2 +8
f- dx
(x+2)(x_l)2 x 3 +4x .
17. Evaluate the following integrals:

(i)
f x2+4x+l dx
x 3 +2x2 -xx2
(ii) f(x -1)(2x
x
+ 1)
dx

(iiI) f(x 2 + l)(x


2x
2 + 3) .
dx
Integration 555

18. Evaluate the following integrals:


(I) fx.sin-I xdx (ii) fxsin-3 xdx

(iii) fsinxlog(cosx)dx (iv) fsinxlog(secx+tanx)dx.


19. Evaluate the following integrals:

(i)
X 7
r
+ dx
2x+3
20. Evaluate the following integrals:

(1) fsec 4 xtanxdx.

HINTS TO THE SELECTED PROBLEMS

7 6
2. (ii) fsec xtanxdx = fsec x secxtanxdx s, Put sec x = t and then integrate

(iii) fxlogx
1 dx
1
Put log x = t => - dx = dt
x

(iv) t = 'f . 2dx


)xsmx Put ~ = t => x dx = dt
2
/ = f..!..sintdt =- ..!..cos t + C=-..!.. cosx2 + C
2 2 2
adx
3. (l) /= - -
b+cex f
= fb~-x
ae
-x

+c
dx [divide Nr and Dr by eX]

Put b e-x + c = t => b e-x dx = dt

/ = ba fdt
t = - ba log t + C
=-::b log (c + be-X) + C
eX
(ii) /=
f-l+e- d x 2x
dt •
Put eX = t => eX dx = dt, / = f--2' i~tegrating
I+t
= tan-I t + C = tan-I eX + C
2
sec (log x) dx
(iii) / = f
x
I 1
Putlogx=t=> -dx=dt/= fsec 2 tdt, integrating
x
= tan t + C = tan (log x) + C
556 Remedial Mathematics

2
. ) / = Jsec x dx dx puttmg
( IV . 1 + tan x = t
l+tanx
4. (I) /= fcosecxdx = J-.-1-dx = J ~ x
smx 2sin-cos-
2 2
2
= ~ fsec x / 2 dx [dividing Nr and Dr by cos2 x!2]
2 tanx!2

Put tan ~ = t
2
1 2 x
- sec -dx =dt
2 2
dt. .
/ = Jt" mtegratmg

= log t + C = log ( tan ~) + C

5. (iii) f ~ a - x dx = f ~ .1 dx, integrating by parts


2 2

= ~a2_x2 .x- fi(a2-x2rI/2(-2x)Xdx

a 2 _x 2 _a 2
=X~a2_x2 - f ~ dx
a -x
= x 12
va -x 2 - f~a2-x2 +a 2 f h
2 2
a -x
=x~a 2 -x 2 -/+~ sin- 1 ~+C
a

2I = x I + ~ sin- 1 ~ + C
va2 -x 2 a
2
/ = ~ I 2 2+ ~ sin-I ~ + C
2 va -x 2 a
(iv) /= fxsinxcosxdx

= i Ix(2sinxcosx)dx

= i fxsin2xdx (integrating by parts)

= i fx( -C~2X) - R-C~2X)dx


xcos2x sin2x C
= +--+
4 8
= ~ (sin 2x-2x cos 2x) + C
8
Integration 557

6. (il) /= Ix cos2x sin x dx = ~{x(2SinX cos2x) dx

= ~ fx(sin 3x - sin x) dx

= ~ fxsin3x dx - fxsinx dx, integrating by part

= ~[x( -C~S3X) + Si~3X _ x(-cosx) + sin x] + C


= ~(sin3x - 3xcos3x) + .!.(xcosx - sinx) + C
18 2

(iii) fx 2Q
I +x2
'put x = tan 9 and integrate

log (sin- i x) .
7. (1) /= f ~ = dx, put sm- i x
l-x 2
= t and integrate by part.
Taking I as II function

(il) /= J .,fidx f .,fidx = Isecxdx = log (sec x +tanx)+C


..II + cos2x ~2 cos 2 x

8. (t) /= f xdx = f{(x+I)-I} dx= f{.jl+x--I-}dx


.jl+x .jl+x .jl+x

= [~(l+X)3/2 _2CI+x)I12 ]
X
(it) NI +sin x dx Use: ..JI + sin x = cos 2 -+ 2·2X
2
sm -+ 2sm-cos-
'x x
222

x . x)2 ( x . x)
( cos'2+ sm '2 = cos'2+ sm '2

t
(iii) lcos 4
x)dx Use: cos4 x = (cos2 x)2 = ( I + c0s 2x )2
2
2
. Slog(1+x ) dx
(IV) /= 2
X

= flog(l+x)2.~dx, integrating by parts


x

=- .!. log (I + xl) -


x
f x(l+x)
1 2\ dx

=- .!. log (1 + xl) + I 2 dx 2


X l+x

= - .!. log (1 + xl) + 2tan- i x + C


x
558 Remedial Mathematics

9. (I) 1= fxe 3x dx, integrating by parts taking x as first function.

(il) I = Jx ll 2 log x dx, integrating by parts taking x as first function.

(iii) 1= S(21+x
+x
)2 dx =
f2 + x - I
(2+x)
{ 1
2 dx = J - - -
1}
2+x (2+x)
2 dx. integrating

1
=log(2+x)+ - - + C
(2+x)

(iv) 1= feX[x(logx)2 + logx]dx = Je [(lOgX)2


x
+~logX Jdx
2ex
= fe x (log x)2dx + J-logx dx
x
integrating by parts, I integral
2 2ex
= Je X(log x)2 - f-(logx) eX dx+ f-(logx)dx
x x
/= S tan x dx = f sinx dx = J sinx dx
secx+cosx cosx(secx+cosx) l+cos 2 x
put cos x = t =:> -sin x dx = dt
-dt
= --2 integrating = coC1 +C=coC1(cosx)+C
f1+t
sin2x+1
(iii) t = f,\}x+sm
I .
x
2dx

Put x + sin2 x + t =:> (1 + 2 sin x cos x) dx = dt


=:> (1 + sin x) dx = dt

1= f~ ;integrating = 2~x+sin2+x +C
(iv) /= fcosxcos2xcos3x dx = i f(2cosxcoS2x)cos3x dx

= i J(cos3x + cosx)cos3x dx = ~ f(2cos 2 3x+ 2cosxcos3x)dx

= ± JW+cos6x)+cos4x+cos2x} dx

. . 1( sin6x sin4x sin2X)


mtegratmg = - X + - - + - - + - - +C
4 6 4 2

= ~(l2x+2 sin 6x+ 3 sin4x+ 6 sin2x) + C


48
11. (i) 1= fJ1 + cosx dx
Use I+cosx =2cos 2 ~
2
Integration 559

(ii) ] = r~S 2x dx
smx
Use: cos 2x = 1 - 2 sin2 x
cotx
12. ] = fIogsmx
. dx ; put log sin x = t => cot x dx = dt

dt. .
t = Jt' mtegratmg
= log t + C = log x sin x + C

..
(ll)]= f rzdx
x+3 =
f rzdx+
x f rzdx
3
=
f rz+3sm
xdx . -I x
vI - x~ vI - x~ vi - x~ vi - x~
Put 1 - x 2 = t in I integrating x dx = _ dt
2

=-~f~ i
+3sin- x+C

lJi
=----+3 sin-I x + C
2 t(112)

=- ~1-x2 +3 sin- i x+ C
cosx-sinx
(iii) / = f(cosx+smx). dx put cos x + sin x = t and then integrating
2

dx 1·
13. (i) ]= fx{1 + (logx)2} putlogx=t => -dx=dt
x
dl. .
= f-1+1- 2 ' mtegratmg
= tan- i 1 + C = tan- i (log x) + C

(ii) ]= f 6x-8 dx
3x2 -8x+5
Put 3x2 - 8x + 5 = 1 and integrate

(iii) fCosFx dx
Fx
Put Fx = 1 and integral

14. (l) /-
-
f(xxe+ x1)2 dx -- f(x+l-1)e
(x + 1)2
X
f x( 1 1)
= e x + 1- (x + 1)2 dx
Now do same as in Question 9 (iv).

(iii) ] = fcosx+smx
cos 2~ dx

Use: cos 2x = cos 2 X - sin 2 x


560 Remedial Mathematics

(iv) [= rlll~X-a) dx = pinxcosx.-sinacosx = dx


smx smx
= f(cosa-sinacotx)dx
= x cos a - sin a log sin x + C
3
15. (I) J(x-a)(x-b)(x-c)
x
dx

~ ~ ~
= J(a-b)(a-c)(x-a) + (b-a)(b-c)(x-b) + (c-a)(c-b)(x-c)
After breaking into partial fraction

=x+
~
log(x-a)+
~
log(x-b) +
2 log(x-c)+C
~-~~-~ 0-~0-~ ~-~~-~

(il) [= J(x+l)(x
x-1 dx = n __I_+_2_1_)dx By partial fraction
2 +1) Jl x+l x +1
Now integrating we have
I
/ = -log (x + I) + -log (x 2 + I) + C
2
2 2 2
x dx {a b
(iii) j=
J(x-a)(x-b)
=
f(a-b)(x-a)
-
(a-b)(x-b)
} dx

a2 b2
=x+ --log (x -a)- --log(x-b)+C
a-b a-b

3
16 j= 3x dx = { 3 -12- + 27}dx, mtegratmg
. .
. J(X+l)(x+2)(X+3) f 2(x+l) (x+2) 2(x+3)

3 27
= -log(x+l)-12Iog(x+2)+ -log(x+3)+C
2 2

JeX (ldx+ eX) = ( -eX1 - -1+l)dx = (x


X

(il) f dx = - e --e-- dx, mtegratmg ). •


eX + e2x eX eX + 1

~ ~
X

=_ log (e- X
+ 1) + C=- + IOg(e + 1)+ C
eX eX eX
2 2
(iii) /= Jxx +4x
3
+ 8 dx = J x
x(x
2
+8
+4x)
dx=
( -
2 - -x-) dx
x x 2 +4
(By breaking into partial fraction)
1
=2Iogx- -log(x2+4)+C
2
Integration 561

2x
17. (iii) /= f(x 2 + 1)(x2 + 3) dx
Putx2=t:::::>2xdx=dt

/= f(t+I~:t+3) = (2(/+1) 2(t~3»)dt


1 1
= -log(t+ 1)- -log(t+3)+C
2 2

1 ,2 1
= -log(.x-+ 1)- - log(x-+3)+C= -log -
,,
- +C.
1 (X2 + 1)
2 2 2 x 2 +3

18. (1) / = fx sin-I x dx, integrating by parts, we get


= -sm x- f r:--? x+- f -x
2 2 1 1 2 2
X • -I x dx ,= -sm
x . -I dx
r:--? -I
2 2\fI-x2 2 2 \f1-x2
2 2
= _x sin- 1 x+.!.{f l-x dx- J--=dx=}
2 2 ~I-x2 ~I-x2

=
2
x . -I x+-
-sm
2
1
2
{J 2 dx -
r:--?I
\fl-x- J~1-x2 dx }

x+- [{ -~ +-sm x}
x . -I I x 2 I . -I . -I
= -sm
2 -sm ] x +C
2 .... 2 2 2
2
x . -I x+-x\fl-x-
= -sm 1 r:--?I
2 I. -I x+ C
--sm
2 4 4
(ii) fxsin 3 xdx
Use: sin 3x = 3 sin x - 4 sin3 x and then integrating by parts.
(iv) /= fsinxlog(secx+tanx)dx, integrating by parts.
II I

= -cos x log (sec x + tan x) - f-cosxsecxdx


= - cos x log (sec x + tan x) + x + C.
3
20. (I) Jsec 4 xtanxdx = fsec x (sec x tan x) dx
Put secx= t:::::> sec x tan x dx = dt

3 t
4
sec 4 x
/ = Jt dt = 4+ C = - 4 - +c.
562 Remedial Mathematics

ANSWERS

1. (i) (2 + 3logx)2 1
(ii) tan (3x + 9) (iii)
6 6(8-6x) ,
. 1
(IV) -log(2x+3).
2
1
2. (i) -sec x (ii) .!. sec 7 x (iii) log log x
2 7
I
(iv) - -cos x 2 .
2

3. (i) - ~ log (c + be--,) (il) tan-I €f (iii) tan log x


b
(iv) log (1+ tan x).
. x
4. (I) log tan "2 (ii) log (log sin x)

5. (i) log x (log log x-I) (ii) -2 log (cosec 8 + cot 8) where x = tan2 8,
2
x
(iii) "2"a--x-+T sm
122 a . -I(X)
-;;' I
(iv) g(sin2x-2xcos2x).

6. (i) .!. (2x sin 2x + cos 2x) (ii) ~ (sin 3x - 3x cos 3x) + .!. (x cos x - sin x)
4 18 2

(i ii) ..j1;;2
x
7. (i) sin-I x [log sin- 1 x-I] (ii) log (sec x + tan x),

8. (i) [~(l+X)3/2_2(\+X)I/2J (ii) 2 ( s in ~- cos ~) + C


(. .) 41(34 + sm. -4x8 + sm. 2)x + C
/II - - X

(iv) -.!.log (1+~) + 2 tan- 1 x.


x

9. (i) e: (x-~)x
(ii) ~x3/2Iogx_ix3/2
3 9
1
(iii) - - + log (2 + x) (iv) (log xi €f.
2+x
10. (i) coe 1 cos x (ii) €f log x,

(iii) 2~(x+sin2 x), Civ) J...(2sin6x+3sin4x+6sin2x+ 12x).


48
11. (i) 2.fisin!", (ii) log tan!" + 2 cos x.
2 2
Integration 563

12. (i) log log sin x (iv) 3 sin-I x- ~(1-x2),

(iii) - . - - - (iv) x+ !cos2x


SInX+cosx 2
13. (1) tan-I log x (ii) log (3x2 - 8x + 5) (iii) 2 sin Fx .
14. (i) _e_
l+x
x
(ii) ! log (5x4 + 7)
5
(iii) sin x + cos x

(iv) x cos a - sin alog sin x.


a3 b3
15. (i) x+ log(x-a)+ log(x-b)
~-~~-~ ~-~~-~

c3 1
+ log (x-c) (ii) -log(x2+ I)-log(x+ I),
(c-a)(c-b) 2

(iii) ~IOg(X-l)_i_l_
10 x+3 4x-1
2
a b2
(iv) --log(x-a)---log(x-b)+C
a-b a-b

16. (i) i log (x + 1)-12Iog (x + 2) + 27 log (x + 3),


2 2

(ii) log (l+e:X)-e lx , (iii) ..!..IOg(X-I) __I_,


3 x+2 x-I

(iv) 2Iogx-! log (x2 + 4).


2

17. (i) log (x2_1) (ii) ..!.. log (x - I) + ! log (2x + I),
x+2 3 2
1 x2 + I
(iii) -log-2- +c.
2 x +3
2
.) x . -I x ~I 2 I._I
18 • (1 -Sin x+-\ji-x- --Sin
244
I
(ij) 36 [3x (cos 3x- 9 cos x) - sin 3x + 27 sinx],
(iii) cos x (I -log cos x), (iv) x - cos x log (sec x + tan x).

19. (i) 2x + ..!..log (2x + 3) (ii) ..!.. log (1 + x 3 ).


2 3

20. (i) ! sec 4 x


4
~~

564 Remedial Mathematics

III DEFINITE INTEGRAL

Ifj(x) is a continuous and non-negative function over a closed interval [a, b] then J: f(x)dx

is called the definite integral ofj(x) between the limits a and b (b > a).

If Jf(x)dx = F(x) + c, then J: f(x)dx = [F(x) + c]~ = F(b)- F(a) is a defmite value.

Here, a is called the lower limit and b is called the upper limit and the interval ra, b] is
called the range of integration.
Remarks

· J: f(x) dx represents the area bounded by the lines x = a and a = b.

• If F(b) - F(a) in not a definite value, then the integral J: f(x)dx is indefinite.

III PROPERTIES OF DEFINITE INTEGRALS

1. J: f(x)dx =0.
2. TIre value of definite integral is independent of tIre variable of integration.

Le., J: f(x)dx = J: f(u)du.

then J:f(x)dx = [F(x)]: =F(b)-F(a)= J:f(u)du

3. J: f(x)dx =- J: f(x)dx.

4. J;f(x)dx+ J:f(x)dx= J:f(x),wherea<c<b.

a
6. lfj(x) is an even/unction o/x, then sa f(x)dx
-a
= 2 r
Jo
f(x)dxand ifj(x) is an odd

function then raf(x)dx=O.

7.
a
J: f(x)dx =2 J: f(x)dx iff(2a-x)=j(x)
2a
and Jo f(x) =0 ifj(2a-x)=-j(x).

8. J;a f(x)dx = n J: f(x)dx ifj(x + ma) = j(x)for all integral values o/m and n is
positive integer.
Integration 565

~~~~~~I SOLVED EXAMPLES I~~~~~~=


rl (tan -I x)2
Example 1: Evaluate Jo 2 dx [RGPV B. Pharma 2002]
l+x
I
Solution: Put tan-Ix = t, so that - I2 dx = dt.
+x
Also, when x = 0, t = tan- I = 0

and when x = 0., t = tan- I I = ~


4.

~l(tan-1 x)2 dx = f1t/2t2dt= ['!'+3J1t/4 =.!.x 1t


3
=~.
Jo l+x2 0 3 0 3 64 192

[RGPV B. Pharma 2001]

Solution: Using integrating by parts taking x1 as 1st function we have

f ~ x 2e2dx = [x1 eX]J - f~2xexdx


= [x1eX] 1_2 [xex- f I eXdx]1
° 0
o
°
=[x2eX]ol_2 [xeX-eX]b
= (l.el-a x eO) -2 [(l.el-el)-(O-eO)
=e-2(O+ 1)=e-2.

Example 3: EvaIuate f °1t/2 cosx.sm. 2


x dx. [RGPV B. Pharma 2001]

Solution: Putting sin x = t so that cos x dx = dt.

also when x = 0 t = sin 0 = 0 and when x = ~ t = sin ~ = 1.


" 2' 2

r1t/2cosx.sin 2 xdx = rlt2dt=[t3] =.!.[r3]I=.!.(I3_0)=.!.s


Jo Jo 3 3 ° 3 3
2 dx
f
Example4:Evaluate I (x+3)(x+4) rRGPV B. Pharam 2005]

2 dx I A B
Solution: Letl= f (x+3)(x+4) and (x+3)(x+4) = x+3 + (x+4)
I s

A(x+4)+ B(x+3)
(x+3)(x+4) x+3)
I =A(x+4)+B(x+3).
Putting x =-4, -3.
When x=-4
566 Remedial Mathematics

1 =-B
or B=-l
When x =-3
l=A
2 1 f2 dx J2 dx
]= J1 (x+5)(x+4) dx= 1 x+3 - 1 x+4 .
= [log (x + 3)? - [log (x + 4)]
= [log 5 -log 4] - [log 6 -log 5]
= log 5 -log 4 -log 6 + log 5
= 210g 5 -log4 -log 6
= 210g 5 -log 24.
= log 25/24.
Example 5: Evaluate thefollowing integrals
7t/2 7t/2
(I) f log tan x dx
0 (ii) f 0 10g(1 + tan 9) d9
7t xsinx dx
fol+sinx (iv) f07t/21ogsmx
. dx .
(iI)
Solution: (I) Consider,] = f;/2 10g tan x dx.

Now, ]= f;/210gtan(%-x) dx [: f: f(x)dx= f: f(a-x)dx ]


7t/2
1= f0 logcotxdx

.
On addmg, we get 21 =
f7t/2
0
r7t/2
log tan x dx J 0 log cot x dx
7t/2 f7t/2
= f 0 log(tanxcotx)dx = 0 10g1 =0
Hence, 2/=0=>1=0.

(il) Consider = 1 f; 14 10g (1 + tan 9) d9


1 = J;/4 10g [1 +tan(~-9 )]d9 [: J: f(x)dx = J: f(a-x)dx ]
= r7t/410g[1+ I-tan9]d9 = r7t/410g( 2 )d9
Jo 1+tan9 Jo 1+tan9
r 7t/4
= Jo [log2-log(1+tan9)]d9

7t/4 f7t/4 1t
= f0 log 2 de - log (l + tan 9)de = - log 2 - 1
0 4
1t
So, 2/= '41og2
1t
Hence 1= 810g2
Integration 567

.,. _ r1t xsinx dx


(w) Here / - J,o 1 .
+smx
_ r1t (1t-x)sin(1t-x) dx ( )
Now /- Jo l+sin(1t-x) ... f:f(x)~= f:f(a-x)dx
= r1t (1t-x)sinx dx = I1t1tsinx dx-J1t xsinx dx
Jo l+sinx ol+sinx ol+sinx
= J1t 1t sinx dx-/
o 1+ sinx

2J =
1t 1tsinx =1t
I1t sinx
So,. Io 1+ sin x 0 1+ sin x
1t
=1t.J1t(1 1 )dx=1t r (I_I-Sinx)dx
o l+sinx Jo cos 2 x

=1t f;[I-SeC 2 x+secxtanx]dx

=1t[x-tanx+secx]~ =1t(1t-2)
2J =1t(1t-2)

Hence, /=1t(~-I)
(iv) Here /= r/2logSin xdx ..

Also, / = Jo
r1t/2 logsm
. (1t2'-x ) dx C f: f(x)dx = f; f(a-x)dx )
(,,,/2
/ = Jo logcosxdx.

On adding, we get
nl2 . f1t/2
2J = f 0 (logsmx)dx+ 0 logcosxdx

rn/2 .
Jo log(smxcosx) dx = Jornl2 log (sin2x) (n/2
-2- dx = Jo logsin2xdx-
(n12
Jo log2dx
Let 2x = t for first integral, then on differentiating, we get
2dx = dt
1 (n . [ ]1t/2 1 (nI2. 1t
Now 2J = 2' Jo logsmtdt- (x log 2) 0 = 2'.2 Jo logsmt dt -2'log2

=- f
1 nl2 logsmtdt--Iog2
. 1t = 1n/2. 1t
logsmtdt --log2
2 0 202
1t
2J =/- 2'log2
1t
Hence, /=- 2'log2.
568 Remedial Mathematics

Example 6: Evaluate the following integrals:

(ii)

Solution: Given integral is f:13x -11 dx .

I
3x-l;whenx ~-
Now, 3
13x-ll =
{ -(3x -1), when x < ~

3 f1l3 f3
So, f o13x-lldx= 0 -(3x-l)dx+ 113 (3x-l)dx

2
= [_ 3x +x]1/3 +[3x2 _x]3 65
2 0 3 1/3 6

(ii) Here, the given integral is f; Icos x Idx.


n
cosx,O:S;x:s;-
Now I cosx 1= 2
{ -cosx ,~:s; x:S; n.

1t f1t/2 fO
So, f o Icos x I dx = 0 cos x dx + 1t/2 (-cos x) dx

= f 1t/2 cosxdx- fO cosxdx


o 1t/2
. ]1t/2 -smx1t/2=
= [smxo [. ]1t 2

Example 7: Evaluate the integral f; log(l +cosx)dx.

Solution: We have, 1= f; log (l + cos x) dx


Now, 1= f; log {I +cos(n -x)}dx = S;log(l-cosx)dx

On addings 2I = f1t log (l + cos x) dx + f1t/2 log(l- cos x) dx


0 0

= f;log(l+cosx) (l-cosx)dx

= f;logsin 2 xdx =2 fo1tlogsinxdx

~ 4 S;l2logsinxdx (using property 7)

21 =4( -~log2)
Integration 569

So, ] =-1t log 2.

Hence, ] =1t log(~)


n/2 ~ 1t
Example 8: Show that
Io ~
"smx +"cosx
~ dx = -
4
(UPTU (B. Tech) 2005]

n/2 -Jsin x
Solution: Let ]=
Io -Jsinx + -Jcosx
dx

Then

= r o
/2
~. dx
-Jcosx +-Jsmx
[.: r
a
Jo
f(x)dx = r
a
Jo
f(a-X)dx]

Adding (1) and (2), we get

21= rn/2[ ~ + ~ Jdx


Jo -Jsinx +-Jcosx -Jsinx +-Jcosx

= sn/2 ~ + ~ dx = sn/21. dx = [x]~/2 = 2:


o -Jsin x + -Jcosx 0 2

so ]= ~
4
I
rl sin- x
Example 9: Evaluate the integral Jo --x- dx .

I
rl sin- x
Solution: Wehave,]= Jo --x- dx .

Putting x = sin 8
~ dx = cos 8 de.
n/2
So, ]= r 0 8 cot 8d8.
Now, integrating by parts w.r.t. 8, we get
. ]n/2
]= [ 8.logsm8 0 - rn/2 10gsm8d8
0 .

=0- [-~IOg2 ] = ~ log2.


r"" + x2) dx
Example 10: Evaluate the integral Jo log - - - - 2 .
(1 x l+x
570 Remedial Mathematics

r'Xl ( 1+ X2) dx
Solution: We have, 1= Jo log - - --2 .
x l+x
Now, putting x = tan 9 => 9 = tan-I x, de = dx 2 .
l+x
2
So, 1= flt/210g(sec e).d9
Jo tan 9

= flt/210g( 1 )d9 = flt/2 10g ( 2 )d9


Jo sin9.cos9 0 2sin9cos9
lt/2 Jlt/2.
=
J0 10g2d9- 0 10gsm29d9

=
n
-.1og2- Ilt/2 log 29 d9.
2 0
Let 29=1
=> 2de =dt,
then 1= -n Il
og2--
2 2
Ilt 10gsmt
0
. dt
= .z!..log2-2.! (lt/210gsinxdx (using property 7)
2 2 Jo

= %.10g 2-(-%10g2) =n10g2. C J;/210gSinxdx=%10g2)


11 xtanx
Example 11: (a) Evaluate the integral 0 1 secx+tanx dx.
Solution: Here, I = JlOI xtanx dx ...(1)
secx+tanx
1_(11 (n-x)tan(n-x) dx ((lIf()dx (lI ( )dx)
Now,
- Jo sec(n-x)+tan(n-x) Jo x = Jo f a-x
= (11 (n-x)tanx.
...(2)
Jo secx+tanx
On adding, (1) and (2) we get
2I = JlI xtanx dx + (lI(n-x)tanx dx
o sec x + tan x Jo secx tan x
- fll ntanx dx
o secx+tanx
= (11 ntanx(secx-tanx) dx
Jo sec 2 x - tan 2 x
=n J;[secxtanx-sec 2 x+l]dX =n[secx-tanx+x]~
=n [{sec n-tan n + n} - (s~c 0 -tan.O + O}]
=n(-l +n-l)=n(n-2)

So; I =n(%-l):" ",


Integration 571

lCI2 dx
Example 12: (b) Evaluate 0 i 4"
5+ smx
lCI2dx flCI2 dx
Solution: Let1= 0f 5 + 4'
smx = 0 5 +smx . /2 cosx /2
Divide Nr and Dr by cos2 x/2 we have

1= r /2 2
sec x/2dx = r /2 2
sec x/2dx
o 5sec2~+8tan~ 0 5(I+tan2~)+8tanx/2
Put tan ~ = t ~ .!. sec 2 ~ dx = dt ~ sec2 ~ dx = 2 dt
222 2
When x =0
~ t=O
x
x= -
2
~ t=1

Then
1=
,I
J0
2dt 2
2+8t+5='5 JO 28
dt,I
5t t +-1+1
5

2 dt 25[ (t+±lll
= 5 fo (t+~)'.(~( 53 3/~
I

tan-I 0

='32[ tan -1 3-tan -14]


'3
Example 13: Evaluate f~sin-I xdx.
Solution: Let 1= f~sin-I xdx
Putting x = sin e
dx = cos 9 de
When x=O
=> 9 =0
When x=1

=> 9=~
2
Then 1= f; 9 cos 9 d9; integrating by parts, we get

1= (9sin9)S/2 - f;/2sin9d9

1t 1C12 1t 1t-2
= 2'+[cos9]0 = 2'-1 = -2-
572 Remedial Mathematics

sin4 xdx
1t/2
1. Evaluate the integral 1o
sin 4 x+cos 4 x .
1t/2 sinx-cosx
2. Evaluate the integral 0 - - . - - -i
l+smxcosx
4 {2X+3,O~X~3
3. Evaluate the integral
fo f(x)dx, wherej(x) =
3x,3~x ~4

1t/2 xsinxcosx dx
4. Evaluate Jo cos 4 x+sin4 x
r1t xsinx
5. Evaluate the integral Jo 2 dx.
l+cos x
1t xdx 1t
6. Showthat io a 2
cos 2 x+b 2 sin 2 x 2ab
.
7. Evaluate the integral Jo
r7tl2 log (tan x + cot x)dx.
t -I
8. Evaluate f e sm
0 x dx

HINTS TO THE SELECTED PROBLEMS

21= f1t/2 dx = 2:
o 2
1t
Hence /=-
4
2. /= f1t/2 sinx-cosx dx
o 1+ cos x sin x

r/2 s;n(%-x )-005(%-X) dx

o l+COS(~-x )sin(~-x)
Integration 573

= (1t/2 cosx-sinx dx
Jo l+sinxcosx
Adding (1) and (2)2/=0 =>/=0

3. /= f:f(x)dx= g(2x+3)dx+ f:3xdx

2
= [x +3x]5 +[3;2 I
= 18 + 24 _ 27 = 42 _ 27 = 57 .
222
1t/2 xsinxcosxdx
4. /= 1 4
o cos x+sin4x

f,'I2(~-X )sin(~ -x}OS(~-x) d<


(%-X)sin4 (%-X)
4
o COS

(1t/2 xsinxcosxdx dx = (1t/2 xsinxcosxdx dx


Jo cos 4 X + sin4 x Jo sin 4 x + cos 4 X
7t (1t/2 sinxcosxdx 4
=> 2I = - JIO • 4 4 dx Divide Nr and Dr by cos x
2 sm x+cos x
2
2I = '3:. f1t/2 tan xsec x dx
2 0 4
l+tan x
Put tan2 x = 1
=> 2 tan x sec2 x dx = dl

= -
1
7t 00 1 dl
- - - d x Whenx=O=>I=O
2 0 21+/ 2 I

Whenx =7t/2
=> 1= 00

2
7t -1 00 7t(7t ) 7t
= "4[tan 1]0 ="4 "2 - 0 = 8
2
7t
So, /=-
16
/_ (1t xsinx dx
... (1)
5. - Jo l+cos 2 x
= r (7t -x)sin(7t - x) dx
o l+cos 2 (7t-x)
... (2)

(1t(x+7t-x)sinx (1t sinx


Adding(1)and(2)2/= Jo 2 dx=7t Jo 2 dx
1+ cos x 1+ cos x
Let cos x = 1
574 Remedial Matliemalics

sin x cIx =-dt


x =0
t =1
X =7t
t=-1

Now r-I dt
21 =-7t Jo l+t 2 =-7t [tan
-I
th-I =-7t [7t
-4"-4"7tJ
7t 2
21=-
2
7t 2
J=-
4

6. J= J; a2 cos2(7t _ ;:b 2 sin 2(7t -x)

r (7t-x)
o a 2 cos 2(7t -x) + b2 sin 2(7t - x)
= r1t
(7t-x) cIx
Jo a 2 cos 2 x+b 2 sin 2 x
Adding (1) and (2) we get

[s:a f(x)cIx = 2 s: f(x)dx;f(2a-x) = f(x) ]

Diving Nr and Dr by cos2 x


1t/2 sec2 xcix
1= 7t Jo a -ril 2 tan 2 x
2

Put tanx=t
When x=o
=> t=O
7t
When x=-
2
t = 00
oo dt 7t roo dt
J = 7t J a2 + b2 t 2 -
0 b2 J0 -(-a-)-=2--
_ +t 2
b
oo
7t 1 _I t
= b 2 a / b [tan a/ bJ0
Integration 575

/= :b[tan-
1
~I = ~(%-O) = ;:b
2
rtl2 f1t'2 (Sin2 X+COS x)
7. / = f log(tanx+cotx)dx = 0 log. dx
o smxcosx

= f1t1210g( 1 )dx
o sinxcosx
= f;/2 -(logcosx+logsinx)dx

=- [f;1210gSinXdx+ f;/210gCOSXdx ]

= -2 f;12 log sin xdx

[-: f;/210gSinx dx = f;1210gCosxdx = -%log2 ]

=-2( -%log2 )=~IOg2


8. /= f~
1
sin
e - x dx
Put sin- 1 x = t
~ sin t = x
~ cos t dt =dx
When x = 0
~ t=O
When x=I~t=1[/2

/= ro 12 [ 1
i costdt= 2 2 f[i cost+i sint]
1 +1
]1t12
0

= .!. [e1t12 (0 + 1)-1]


2
[Since (1t12 eox cos bx dx = 2 1 2 [aeOX cosbx+beox sinbx]
Jo a +b
= .!. (ertl2 - 1)
2

ANSWERS
1[
1. 2.0 3. 57
4 2
1[2 1[2
4. 5. 7. 1[ log 2
16 4
e1t12 -1
8.
2
576 Remedial Mathematics

SOME MORE SOLVED PROBLEMS


RELATED TO DEFINITE INTEGRALS

Example 1: Evaluate f; xsin 6 xcos 4 xdx.

Solution: Here 1= f; xsin 6 xcos 4 xdx

6 4
= 1t f; sin xcos xdx-1

Hence, 21 = 1t f; sin 6 xcos 4 xdx = 21t f;/2 sin 6 xcos 4 xdx

1= 1t f;12 sin 6
xcos 4 xdx

5.3 .1.3 .1 1t 31t 2


= 1t. .- = - (By Walli's fonnula)
10.8.6.4.2 2 512
f7t/2
Example 2: Evaluate Jo log sin 2x dx.

7t/2 .
Solution: Let f 0 logsm2xdx. ... (1)

Put2x=t
2dx = dt, we get

1=.!. f7tlog sin tdt = .!..2 f7t/2logsintdt


2 Jo 2 Jo

= fo7t/210gSin(~-t)dt
7t/2
= f0 log cos t dt. ... (2)

Adding (1) and (2), we get


f7t/2 . J7t/2 f7t/2 (sin2t)
2/= Jo (logsmt)dt+ 0 logcostdt= 0 log -2- dt

7t/2 f7t/2 1t
= f log sin 2t dt - log2dt = 1- -log 2
o 0 2
1t
Hence, 1=- -log2
2
Integration 577

f1t/2 sin 2 x 1
Example 3: Show that J,O. dx = r.:;-Iog( F2 + 1) .
(SID X + cos X) v2

f1t/2 sin 2 x
Solution: Let /= J, dx ... (1)
o (sinx+cosx)

= r/2 . 2(1t x )
SID 2- dx

o Sin(~-X )+cOS(~-X)
Adding (1) and (2), we get
1t/2 sin2 x i1t/2 COS 2 X
li= io
dx+ dx
sinx+cosx 0 cosx+sinx

=
r
0
/2 dx
sinx+cosx = Jo
r1t/2
(I.
(1 I F2) dx
1 )dx
Ji SIDX + F2cos x
1 (1t/2 dx
= J2 Jo cos(x-1t/4)

= ~ f;/2 sec( X-~)dx

= ~IOg[sec( x-~)+tan( x_~)I/2

= _I 10 [(Ji +1)(J2 -I)] = _I 10 (F2 +1)2


F2 g (Ji-1)(J2+I) J2 g
= ~.210g(F2 +1)2

Hence, /= i IOg (F2+1).

~~~~~~~I EXERCISE 12.41~~~~~~~~


Show that
1t/2 1t
f1t x log sin x dx = .!. 1t2 10g .!.
1.
Jo 2 2
2.
io xcotxdx = -log2.
2

3. f1t/2[~J2
o sme
de =1tlog2. .4.
I
I
0
sin -I
--dx = -log2.
x 2
1t
578 Remedial Mathematics

1t/4 1t 1t
5. Jo log(1+tan9)d9 = -log2,
8 4

r1t/2 ~ dx = ~ r1t/2 f:os2 x dx 1


8. Jo (' ) = r,;-log(Ji+l),
7. Jo .Jtanx+.Jcotx 4' smx+oosx ",2

1t
2
x sin 2x sin ( ~ cos x ) 8
10. Jo 2x-1t = ;'

HINTS TO THE SELECTED PROBLEMS

1. /= J;xlogsinxdx = J;(1t-x)logsin(1t-x)dx = J;1tlogsinxdx

=> 21 = 1t J;logsinxdx
7t/2 ,
= 21t J0 logsmxdx

CJ: a
f(x)dx =2 J: f(x)dxif f(2a-x) = f(Xf)
a

= 21t( -~log2)
=1t2 log!
2
9
3. /= r 1t/2(-,- 1t/2)2
d9 = r 92 cosec2 9d9 = (_92 cot 9)Q'2- r -29cot9d9
'lC/2
Jo sm9 Jo Jo
= (-0 + 0)-2 J;/2 -9cot9d9 = +2 [(9lQg Sin 9)~/2 - J;/2logSin 9d9 ]

=+2 [(0-0)- J;l2l0gSin9d9]

=+2 (~IOg2) [Since J;12 log sin 9 = -21tlOg2]


=1tlog2
7t/4
5. /= J 0 log (1 + tan 9)d9

= J;/4 10g {1 +tan(~+9 )}d9


= r1t/4l0g{1+ tan1t/4-tan9 '}dx
Jo l+tan1t/4tan9

= r1t/410g{1+ I-tan9}d9 = r1t/410g{ 2 }dx


Jo, l+tan9 Jo 1+tan9
r
= Jo
7t/4
log2dx- J1t/4
0 log (1 +tan 9)d9
Integration 579

2I=log2[x] 1tI4= ~log2


o 4
7t
/= glog2

7. / = J1t12 .Jtii.TU dx ... (1)


o ..Jtanx +..Jcotx

... (2)

_ f1t'2 _ 7t
Adding (l) and (2) 21- dx - -
o 2
7t
/=-
4
r1t12 cos 2 xdx
8 • /= JI0 ... (1)
sinx+cosx

= r/2 .
cos
2(7t
'2- x
)
dx

o sin(~-x )+cos(~-x)
r 2 /2
sin x dx
= 0 cosx+sinx
... (2)

2 2
21= f1t'2 cos x+sin x dx
Adding (1) and (2)
o cosx+sinx

=
r/2 cosx+sinx
0
dx

1t12 dx
= J 0 .2X. x x
I-2sm -+2sm-cos-
2 2 2
Dividing Nr and Dr by cos2 ~
2
r1t12 sec 2 x/2dx
21- JI
o 1+2tan~-tan2~
2 2
Lettan ~ = t => sec2 ~ dx = 2 dt
2 2
580 Remedial Mathematics

Also when x = 0 ~ t = 0 x = ~ => t = 1


2
2/- rl 2dt -2 rl dt
- Jo 21+I-t 2 - Jo (J2)2 _(t_I)2

~2' 2~[IOglt:::n
= _1 [lOg J2 _IOg(J2-1)]
2J2 J2 J2+1

= _1_10g{J2-1}=_I_IOg J2+1 s
J2 J2+1 J2 J2-1

=_110 {(J2+1)(J2+1)}
J2 g (J2 -1)(J2 +1)

= ~10g(J2+1)2= ~10g(J2+1)

So /= ~10g(J2+1)

9. /= f:sinm ncos 2m +1 xdx =f(x),say


Here,f(x) = sinmx cos 2m + 1 x
f(1t-x) = sinm(1t -x) CQs(2m+ I) (1t-x)
= - sinmx cos2m + 1 X =.f(-x)
So / =0 Since f: f(x)=Oiff(2a-x)=-f(x).

OBJECTIVE EVALUATION

MULTIPLE CHOICE QUESTIONS


Choose the most appropriate one:
3\ogx 4 \
1. fe
(a)
+ It dx is equal to
(x
log (x4 + 1) + C (b) _log(x4+ I)+C.
(c) .!..log(x4 +I)+C (d) -.!..Iog (x4 + I) + C.
4 4

2. J.1 tan4 ..rx sec2 ..rx dx is equal to.

(a) ~ tan 5 ..rx + C (b) '!"tanS..rx+ c.


s S
(c) 2 tan 5 ..rx + C (d) None of these.
Integration 581

rot x. tan x dx
3. 2 is equal to
sec x-I
(a) cotx+x+ C (b) cotx-x+ C
(e) -cotx +x+ C (d)-cotx-x + c.
2 3
4. ( I+x+-+-+-
x x ) dx is equal to:
2! 3!
(a) e'+C (b)-e'+C
(e) e"x+ C (d) None of these.
5. fFx +-Ix-2
dx =?
.

(a) .!. [x3/2 - (x - 2)3/2] + C (b) .!. [x3/2 + (x + 2)3/2] + C


'3 3
(e) ![x 312 + (x - 2)3/2] + C (d) ! [x 3/2 _ (x - 2)3/2] + C .
2 2
6. f(x 2 + 5) dx =

x3 x3
(a) - +5x (b) - -5x
3 3
x3 x3
(e) - +5x+2 (d) - + 5x + I.
3 3
7. f(eolOgx + exlogU)dx is equal to

x o +1 x o+ 1
(a) --+if +C (b) --+logx+C.
a+1 a+1
x u+ 1 aX
(e) --+--+C (d) None of these.
a+I loga
8. fsecxlog(secx+ tan x) dx is equal to:
(a) sec 2 + se~ tan x + C (b) [log (sec x + tan x)] + C.
1
(c) - [log (sec x + tan x)] + C (d) None of these.
2
cose# .
9. fe# Fx dx equal to IS

(a) sin e# + C (b) 2 sin e# + C

(b) cos e
Fx +C (d) 2 cos e# + C.
10. f-ll-sin2xdx =
(a) sin x + cos x + C (b) sinx-cosx + C
(e) secx+tanx + C (d) secx-tanx + C.
II. Ifj{x) = f(a + b -x), then f: x f(x)dx is equal to

(a) (a+b) lXf(x)dx (b) i(a+b) l Xf (x)dx

(e) (b-a) lXf(x)dx (d) i(b-a) lXf(X)dx.


582 Remedial Mathematics

12. r
• 1
x+2
Jx 2 +2x-3
dx

2J3 1 2J3 1
(a) ----log3 (b) -+-log3
3 2 3 2
2J3 I J33 +2) 2J3 1 J33+2)
(b) ---Iog( (d) ---Iog(
3 2 3 2
. r/2 xsinx .
13. The value of the mtegral - - d x IS
-1t/3 cosx
(a) re/3 -log tan 3re/2 (b) 2 (2re/3 -log tan 5rt/2)
(e) 3 (re/2-log sin re/12) (d) None of these.
14. The value of r e,/xdx is
(a) e2 (b) 2e2
(e) 4e2 (d) 3e2.
15. The value of the integral

(a) rt/4
r1
/2
logtanxdx is
(b) rt/4
(e) 0 (d) re
2
16. I1t12 . sin x dx is equal to
1 smx+cosx
(a) rt!2 (b) J2log( J2 + 1)

(e) ~ log ( J2 + 1) (d) None of these.

17. The value of the integral r 12 3/2 dx


O(l+x)

(a) 112 (b) !J2


2
(b) 1 (d) J2
2 1t
18. IfI= I: sin xdx, then.

2
(a) /=2 S:1t sin xdx (d) /= 4 I1t/2
0 sin 2 xdx

(e) /= S:1tcos 2 xdx (d) /= 11t/4


0
. 2
sm xdx.

19. The value ofthe integral III (x+ ~(x2 + l)dx is


(a) 0 (b) log2
(e) log 112 (d) None of these.
20. The value of r /2
-1t/2
sin[log(x+N +1)]dx is
(a) 1 (b) -1
(e) 0 (d) None of these.
Integration 583

FILL IN THE BLANKS


X +6e-X
l. If f 4ege x
-4e
-x dx =Ax+ Blog (ge 2x
-4) + CthenA = ,B _ _ _ and C-= _ __
2. f x(x3I + I) dx --
3. fe 2x 1 -2x dx =A tan- I 2x+ C then A =
+e
eX
4. f .J1+e- 2x
dx-
-

5. f sinx-cosx
1 dx =

6. f sm x+ 1dx =
. 3
2
cos x
7. f l .IS=
2sinx+sin2x

8.
fJl:X3 dx =
2x
The value of f
e -I
9. dx is = -2--
eX +1
1t/2 1
10. The value of 0 f 4 cos 2 x+ 9'sm 2 x dxis= _ __
11. The value of f sec xcos 2xdxis = _ __
2 2

f1t/2 3
12. The value of Jo tan xdx is= _ __

13. The value of f4 ~ dx is = _ __


o l+v2x+1
"" x 212
14. The value of
f xe-
-0
dxis= _ __

f1t/2 2
15. The value of Jo x cosxdx is= _ __

TRUE/FALSE

1. fa f(x)dx = I: [f(x) + f(-x)]dx (T/F)

2. IIo .J(1 + x)(1 + x3) dx in less then or equal to ~8 (T/F)

3. I: I f(x) Idx = 0 => I: (f(x))2 dx =0 (TIF)

4. I: f(sinx) dx = 2 1: 12
f(sinx)dx (TIF)

5. Ifj{x) is an odd function then Io f(cosx)dx = 2 Jo


1t
(1t/2
f(cosx)dx (TIF)
584 Remedial Mathematics

ANSWERS

MULTIPLE CHOICE QUESTIONS


1. (c) 2. (a) 3. (d) 4. (a)
5. (c) 6. (d) 7. (c) 8. (d)
9. (b) 10. (a) 11. (b) 12. (b)
13. (b) 14. (b) 15. (c) 16. (c)
17. (b) 18. (a), (b) 19. (a) 20. (c)

FILL IN BLANKS
3 35
1. A= -- B= - CER.
2' 36' 2. .!.IOgl4-1
3 x +1
+ C 3.
2
1
4. sin- 1 (If) + C 5. J2 log tan (x12 -n/8) + C
1 1
6. - log leos xl + 2"1 eos2 x + tan x + c.. 7. - log (1- eos x) + - log (1 + eos x) + CA
6 6

8. ~ log (x 3/2 + ~1+x2 ) + C 9. log (If + e-~ + C.


3
10. tanx+ sin 2x-2x + C. 11. n/12
2
1 n
12. -log(e/2) 13. 2-log2 14. 0 15. --2.
2 4

TRUE/FALSE
1. True 2. False 3. True 4. True
5. False

Do you know? After reading this chapter you should be able to know the following
concepts:
• Letf(x) be a function and c be a constant.
d d dc
dx [f(x)+C] = dx [f(x)] + dx =F(x).

Hence ff(x)dx =F(x)+C, where the symbol f is an integral sign and Cis constant
of integration. The function F(x) is called the indefinite integral of integrand fix)
• The integral of the product ofa function with a constant is equal to the product of the
constant and integral of that function.
• The integral of the sum or difference of two function is equal to the sum or difference
of their integrals.

fLfi(x)±h(x)dx = f.li(x)dx± fh(x)dx


Integration 585

• Ifj{x) andg(x) are two functions of x , then

ff(x)g(x)dx = f(x) fg(x)dx- f! f(x){ fg(x)dx}dx .

• Ifj{x) is a continuous and non-negative function over a closed internal [a, b] then

! f(x)dx is called the defmite integral ofj(x) between the limits a and b. (b > a), then

t f(x)dx = [f(x) + CJ: =f(b)-f(a) is a definite value. Here, a is called the lower and
b is called the upper limit and the interval [a, b] is called constant of integration.
Can we do? (Frequently Asked Questions)
Evaluate the following integrals:

1. f cosx dx [UPTU B. Pharma 2001]


2
sin x
x .
2. f e -smx dx [UPTU B. Pharma 2001]
eX +cosx
x 2dx
3.
fl +x6 [UPTU B. Pharma 2001]

4. f tanx dx [UPTU B. Pharma 2001]


logsecx
fO + logx)2 dx (UPTU B. Pharma 2002]
5.
x
6.
f5+~OSX [UPTU B. Pharma 2001]

7.
f sinx dx (UPTU B. Pharma 2007]
sin(x-a)
8. fx 3sinx 2dx (UPTU B. Pharma 2004]

x 2 tan- 1 x
9. (UPTU B. Pharma 2006]
f 1+x2 dx
. -I
10. fxsm x dx (UPTU B. Pharma 2006J
~1_x2

11. feX(l+x)2
x dx [UPTU B. Pharma 2007J

12. fe 2x (-sin x + 2cosx)dx [UPTU B. Pharma 2004]

13. fa 2 ~x2 IUPTU B. Pharma 2003]

14. [UPTU B. Pharma 2005]


fx:x2
586 Remedial Mathematics

15.
JX~X3 [UPTUB. Pharma 2004)

3
16. J(x-1)(x-2)(x-3)
x ch [UPTU B. Pharma 2006)

x2 1
17. J + ch [UPTUB. Pharma 20(5)
x4 +1

18.
J1t/4
1t/2 cosa cosec ada 2
[UPTU B. Pharma 2002)

19. Sh owthat r
o
/2 .Jsinx
.Jsinx+.Jcosx
ch = -7t
4
[UPTU B. Pharma 2005]

20. Evaluate J-;-log


x
x ch [UPTU B. Pharma 2008)

DOD
Appendix-I

"MENSURATION AND ITS PHARMACEUTICAL APPLICALITONS"


Some important results:
1. Volume of cuboid = length x breadth x height.
2. Curved surface area of cuboid = 2 x height (length + breadth).
3. Total surface area = 2 [length x breadth + breadth x height + length x height].
4. Volume of a cube = (edge)3.
5. Curved surface area of cube = 4 (edge i.
6. Total surface area ofa cube = 6 (edge)2.
7. Cylinder: (Right circular cylinder):
Let r be the radius of the base and h be the height of the cylinder. Then,
(I) Volume = 7t,2h.
(ii) Curved surface area = 27trh.
(iii) Total surface area = 27t (h +r).
8. Cone: (Right circular cone)
Let r be the radius, h the height and 1the slant height ofthe cone. The

(I) Volume= .!.7tr 2h.


3
(il) Curved surface area = 7tr (/+ r).

(iii) Total surface area 1= ~h2 + r2


9. Sphere: Let r be radius of the hemisphere. Then,

(I) Volume= ~7tr3 .


3
(iz) Surface area = 47t,2.
10. Hemi sphere: Let r be the radius of the hemi-sphere.

(I) Volume= 3. 7tr3 .


3
(iz) Curved surface area = 27t,2.
(iii) Total surface area = 37t,-2.
588 Remedial Mathematics

~~~~~~~ SOLVED EXAMPLES

Example 1: The metal cylinder ofradues 18 cms and height 80 cms is melted to prepare
spheres ofdiameter 12 cms. Find the number ofspheres prepared
[RGPV B. Pharma 20021
Solution: Volume of cylinder = 1t?h
=1t x (l8i x 80
= 25920 1t cubic cm.
4 3
Volume of sphere = -1tr
3
4
=-1t x (6)3
3
= 2881t cubic cm.
259201t
Number of spheres prepared = 2 = 90.
881t
Example 2: Find the volume ofa capsule having a cylinder ofheight 5 mm and radius 4 mm
on both sides of cylinder, there are hemispheres of radius 4 mm.
[RGPV B. Pharma 2005]
Solution: Height ofthe cylinder, h = 5 mm.

(-.J....-1 ---..-1)
Shape

r = radius of the cylinder = radius of hemisphere = 4 mm.


Volume of capsule = volume of cylinder + 2 x volume of hemisphere

=1t?h+2(~1tr3 )
22 2 4 22 3
= -x(4) x5+-x-x(4)
7 3 7

= 1760 + 5632 = 10912 =519.619cubicmm.


7 21 21
Example 3: Write the volume ofcapsule in the shape ofcylinder bounded by hemisphere (as
shown in fig given below on the ends.
length = 8 mm. diameter = 4 mm . [RGPV B. Pharma 2003]

.,--;
Appendix 589

Solution: Volume of capsule = volume of cylinder + 2 x volume of hemisphere

=1tYlh + 2 x ~ 1t?
3

=1t(2i x 8+ ±
3
x 1t x (2)3

321t 1281t .
= 321t + - = -- cubic mm.
3 3
= 134.01 cubie mm.
Example 4: Find the volume ofa right circular cone ofradius 3 cm and height 7 cm and also
find the surface area of the whole one. [RGPV B. Pharma 2001]

Solution: Volume of cone = .!.1tr2h


3
1 22
= -x-x3x3x7
3 7
= 66 cubic cm.
Surface area = 1tr (I + r)

= 22 x 3 (.J58 + 3)
7
22
= -x3(7.483+3)
7
=66 x 10.483 = 98.8489 cm.
7
Example 5. Write the volume oftablet in the shape ofcylinder top bounded by
cone (as shown is fig) on one end
height of cone = 5 mm.
height of cylinder =5 mm.
radius of cylinder = 4 mm. [RGPV B. Pharma 2007]
Solution: Let hI = height of cone = 5 mm.
h2 = height of cylinder = 5 mm.
r = radius of cone = radius of cylinder = 4 mm.
Volume of tablet = Volume of cone + volume of cylinder
1
= -1tYlh l + 1tYlh2·
3

=.!.1t(4i x 5+1t(4i x 5
3

= (j 1)+ 80 1t = ~ x 80 1t

= 320 x 22 = 335.24 cubic mm.


3 7
590 Remedial Mathematics

EXERCISE
1. The base radii of two right circular cones ofthe same height are in the ratio 3 : 5. Find
the ratio of their volumes.
2. The circumference of the base of a 9 m high wooden solid cone in 44 m. Find the
volume ofthe cone .(use 11 = 2217.)
3. Circumference of the edge of hemispherical bowl in 132 em. Find the capacity of the
bowl. (use 11 = 2217).
4. How many spherical lead shots each 4.21 em in diameter can be obtained from a
rectangular solid lead with dimensions 66 em, 42 em and 21 em?
5. A solid cylinder has a total surface area 462 sy. em. Its {;urved surface area is one third
of the total surface area. Find the volume of the cylinder.
6. A rectangular sheet of paper 44 em x 18 em is rolled along its length and a cylinder is
formed. Find the volume ofthe cylinder [use 11 = 2717].

I ANSWERS I
1. 9: 25. 2. 462m3 . 4. 1500.
3
5. 53gem . 6. 2772em3 .
tiC 1t9U•IfIJ TABLES RELATED TO MATHEMATICS I
COMMON LOGARITHMS

! 6.." 1 2 3 4 5 6 7 8 9
·x 0 1 2 3 I 4 5 6 7 8 9
+ ADD
10 I .0000 0043 0086 012810170 0212 42 4 8 13 17 21 25 29 34 38
I I 0212· 0253 0294 0334 0374 40 4 8 12 16 20 24 28 32 36
II 1 0414 1,0453 0492 053 1 10569 0607 39 4 8 12 16 19 23 27 31 35
, I i 0607 0645 0682 0719 0755 37 4 7 11 15 19 22 26 30 33
121.079210828 0864 089910934 0969 35 4 7 11 14 18 21 25 28 32
0969 1004 1038 1072 1106 34 3 7 10 14 17 20 24 27 31
13 ."391"73 1206 1239 1271 1303 33 3 7 10 13 16 20 23 26 30
1303 1335 1367 1399 1430 32 3 6 10 13 16 19 22 26 29
14 .1461 1492 1523 1553 1584 1614 1644 1673 1703 1732 30 3 6 9 12 15 18 21 24 27
15 .1761 1790 1818 1847 1875 1903 1931 1959 1987 2014 28 3 6 8 11 14 17 20 22 25
16 .2041 2068 2095 2122 2148 2175 2201 2227 2253 2279 26 3 5 8 10 13 16 18 21 23
17 .2304 2330 2355 2380 2405 2430 2455 2480 2504 2529 25 2 5 7 10 12 15 17 20 22
18 .2553 2577 2601 2625 2648 2672 2695 2718 2742 2765 24 2 5 7 10 12 14 17 19 22
19 .2788 2810 2833 2856 2878 2900 2923 2945 2967 2989 22 2 4 71 9 11 13 15 18 20
20 .3010 3032 3054 3075 3096 3118 3139 3160 3181 3201 21 2 4 6 18 11 13! 15 ]7 19
21 3222 3243 3263 3284 3304 3324 334513365 3385 3404 20 124 6 8 JO 12114 16 18
22 .3424 3444 3464 3483 3502 3522 3541 3560 3579 3598 19 2 4 6 8 10 11 13 15 17
23 .3617 3636 3655 3674 3692 3711 372913747 3766 3784 1812 4 5 7 9 111 13 14 16
24 .3802 3820 3838 3856 3874 3892 3909 3927 3945 3962 18 2 4 5 7 9 11 113 14 16
25 .3979 3997 4014 4031 4048 4065 408214099 4116 4133 17 2 3 5 7 9 10 112 14 15
I
26 .4150 4166 4183 4200 4216 4232 4249 4265 4281 4298 16 2 3 5 6 8 10 I3 14
27 .4314 4330 4346 4362 4378 4393
28 .4472 4487 4502 4518 4533 4548
4409 4425 4440 4456
4564 4579 4594 4609
16 2 3
15 2 3
5 6
5 6
8
8
10 II
91 11
1"
13
12
14
14
29 .4624 4639 4654 4669 4683 4698 4713 4728 4742 4757 15 I 3 4 6 7 9 10 12 13
30 .4771 4786 4800 4814 4829 4843 485714871 4886 4900 14 1 3 4 6 7 8 10 II 13
31 .4914 4928 4942 4955 4969 4983 4997 5011 5024 5038 14 1 3 4 6 7 8 10 II 13
32 .5051 5065 5079 5092 5105 5119 5132 5145 5159 5172 13 I 3 4 5 7 8 9 10 12
33 .5185 5198 5211 5224 5237 5250 5263 5276 5289 5302 13 I 3 4 5 6 8 9 10 12
34 .5315 5328 5340 5353 5366 5378 5391 5403 5416 5428 13 I 3 4 5 6 8 9 10 12
35 .5441 5453 5465 5478 5490 5502 5514 5527 5539 5551 12 1 2 415 6 7 8 10 11
36 .5563 5573 5587 5599 5611 5623 5635 5647 5668 5670 12 I 2 415 6 7 8 10 11
37 .5682 5694 5705 5717 5729 5740 5752 5763 5775 5786 12 I 2 4 5 6 7 8 10 11
38 5798 5809 5821 5832 5843 5855 5866 5877 5888 5899 11 1 2 314 6 7 8 9 10
39 .5911 5922 5933 5944 5955 5966 5977 5988 5999 6010 11 1 2 3 4 6 7 8 9 10
40 .6021 6031 6042 6053 6064 6075 608:1 6096 6107 6117 11 1 2 3 4 8 7 8 9 10
41 .6128 6138 6149 6160 6170 6180 6191 6201 6212 6222 10 1 2 3 4 8 6 7 8 9
42 .6232 6243 6253 6263 6274 6284 6294 6304 6314 6325 10 1 2 3 4 8 617 8 9
43 .6335 6345 6355 6365 6375 6385 6395 6405 6415 6425 10 1 2 .3 4 8 7 8 9
44 .6435 6444 6454 6464 6474 6484 6493 6503 6513 6522 10 1 2 3 4 8 661 7 8 9
45 .6532 6542 6551 6561 6571 6580 6590 6599 6609 6618 10 1 2 3 4 8 7 8 9
61
46 6628 6637 6646 6656 6665 6675
47 6721 6730 6739 6749 6758 6767
48\.6812 6821 6830 6839 6848 6857
49 .6902 6911 6920 6928 6937 6946
6684 6693 6702 6712
6776 6785 6794 6803
6866 6875 6884 6893
6955 6964 6972 6981
9 12
9 12
9 I 2
9 I 2
3 4
3 4
3 4
3 4
8
8
4
4
;I: 7 8
7 8
5 6 7 8
5 6 7 8
No. log No. log
n=3.14159 0.49715 In x = loge x = (lIM) log 10 X (lIM) = 2.30259 o ~6222
e = 271828 0.43429 log" = log 10 X = M loge X M= 0.43429 1.63778
P 1 2 3 4 5 6 7 8 9 10
log r!' 04343 0.8686 1.3029 1.7372 2.1715 26058 J 0401 34754 39087 4.3429
log e-P 1.5657 1 1314 2.6971 22628 38285 33942 49599 4.5256 4.0913 56571
COMMON LOGARITHMS

1 6m 123)4561789
.,01123456789
I + ADD
50 .6990 6998 7007 7016 7024 7033 7042 7050 7059 7067 9 1 2 3 !4 4 5 6 7 8
51 .7076\7084 7093 7101 7110 7118 7126 7135 7143 7152 8 I 2 2 3 4 5 6 6 7
521.7160 7168 7177 7185 7193 7202 7210 7218 7226 7235 8 1 2 2 345 667
531.724317251 7259 7267 7275 7284 7292 7300 7308 7316 8 I 2 2 3 4 5 6 6 7
541.7324 7332 7340 7348 7356 7364 7372 7380 7388 7396 8 1 2 2 I :; 4 5 6 6 7
55 .7404 7412 7419 7427 7433 7443 7451 7459 7466 7474 8 I 2 2 3 4 516 6 7
56 .7482 7490 7497 7505 7513 7520 7528 7536 7543 7551 8 I 2
2 3 4 516 6 7
57 .7559 7566 7574 7582 7589 7597 7604 7612 7619 7627 8 1 2
2 3 4 5 6 6 7
58 .7634 7642 7649 7657 7664 7672 7679 7686 7694 7701 8 I 2
2 3 4 5 6 6 7
59 .7709 1 7716 7723 7731 7738 7745 7752 7760 7767 7774 7 1 1
2, 3 4 4 5 6 6
60 .778217789 7796 7803 7810 7818 7825 7832 7839 7846 7 I I 2 3 4 4 5 6 6
61 .7853 7860 7868 7875 7882 7889 7896 7903 7910 7917 7 1 1 2 3 4 415 6 6
62 .7924, 7931 7938 7945 7952 7959 7966 7973 7980 7987 1 7 I I 2 3 3 4 [5 6 6
63 .7993 f 8000 8007 8014 8021 8028 8035 8041 8048 8055 1 711 1 2 3 3 4 5 6 6
64 .8062 8069 8075 8082 8089 8096 8101 8109 81\6 8182 I 7 I 1 2 3 3 4 5 6 6
65 .8129 8136 8142 814918156 8J62 8169 8176 8182 8189 '\7 I I 2 3 3 4 5 r) 6
66 8195 8202 8209 8215 8222 8228 8235 8241 8248 8254 7 11 I 2 3 3 4 5 6 6
67 .8261 8267 8274 8280 8287 8293 8299 8306 8312 8319 6 I 1 2 2 3 4 ' 4 5 5
68 .8325 8331 8338 8344 8351 8357 8363 8370 8376 8382 6 I I 2 2 3 4 14 5 5
698338 8395 8401 8407 8414 8420 8426 8432 8439 8445,1 6 1 1 2 2 3 4 14 5 5
70 .8451 8457 8463 8470 8476 8482 8488 8494 8500 8506 I 6 I I 2 2 3 4 4 5 5
71 8513 8519 8525 8531 8537 8543 8549 8555 8561 8567 6 11 1 2 234 4 5 5
72 8573 8579 8585 8691 8597 8603 8609 8615 8621 8627 6 I I 2 2 3 4 4 5 5
73 .8633 8639 8645 8651 8657 8663 8669 8675 8681 8686 6 I 1 2 2 3 4 4 5 5
74 8692 R698 870.+ 8710 8716 8722 8727 8733 8739 8745 6 I I 2 2 3 4 4 5 5
75 .8751 8756 8762 9768 8774 8779 8785 8791 8797 8802 6 I I 2 2 3 4 4 5 5
76 88m! 8814 8820 8825 8831 8837 8842 8843 8854 8859 6 112234455
77 .8865 8871 8876 8882 8887 8893 8899 8904 8910 8915 6 112234455
78 .8921 8927 8932 8938 8943 8949 8954 8960 8965 8971 6 112234455
79 .8976 8982 8987 8993 8998 9004 9009 9015 9020 9025 6 112234455
80 .9031 9036 9042 9047 9053 9058 9063 9069 9074 9079 5 112233445
81 .9085 9090 9096 9101 9106 9112 9117 ,9122 9128 9133 5 112233445
82 .9138 9143 9149 9154 9159 9165 9170 19175 9180 9186 5 1112233445

I::;;~~::;
83 .9191 9196 9201 9206 9212 9217 9222 9227 9232 9238 5
84 .9243 9248 9253 9258 9263 9269 9274 9279 9284 9289 5
I 85 .9294 9299 9304 9309 9315 9320 9325 '19330 9335 9340 5 1112233445
86 .9345 9350 9355 ,;>360 9365 9370 9375 9380 9385 9390 5 112233445
87 .9395 9400 9305 9410 9415 9420 9425 9430 9435 9440 5 10 11223344
88 .9445 9450 9455 9460 9465 9469 9474 , 9479 9484 9489 5 011223344
89 .9494 9499 950.+ 9509 9513 9518 9523 \9528 9533 9538 5 011223344
90 .9542 9547 9552 9557 9~62 9566 9571 9576 9581 9586 5 011223344
91 9590 9595 9600 9605 9609 9614 9619 9624 9628 9633 5 Oi1223344
92 9638 9643 9647 9652 9657 9661 9666 9671 9675 9680 5 I 011223344
93 968;5 9689 9694 9699 9703 9708 971 J 9717 9722 9727 5 10112233.14
94 .9731 9736 9741 9745 9750 9754 9759 9763 9768 9773 5 011223344
95 9777 9782 '1786 9791 9795 9800 9805 9809 9814 9818 5 0112233·t4
96 9823 9827 9832 9836 9841 9845 '1850 9854 9859 Y863 4011222334
97 .0368 9Sn 9377 9881 9886 9890 9894 9899 9903 9908 4011222334
98, 9912 99i7 '1921 992619930 9934 9939 9943 9948 9952 4011222)34
99: .9956 ! 9<;61 9965 9969 9974 9978 9983 9987 9991 9996 401 122233·1
ANTILOGARITHMS Hf
! An 1 2 3 4 5 6 7 8 9
A 0 1 2 3 4 5 6 I7 8 9

lr
- + ADD
.00 1000 1002 1005 1007 1009 1012 1014 1016 1019 1021 2 0 0 I I I 2 2
.01 1023 1026 1028 1030 1033 1035 1038 1040 1042 1045 2 0 0 I 1 I I I 2 2
.021 1047 01050 1052 1054 1057 1059 1062 1064 1067 10691 2 10 0 I I I III 2 2
0311072 1074 1076 1079 1081 1084 1086 1089 1091 1094 2 0 0 I I 1 III 2 2
04 ' 1096 1099 1102 1104 1107 1109 1112 1114 1117 1119 3 0 I I 1 I 212 ..."- 3
.0511122 1125 1127 1130 1132 1135 1138 1140 1143 1146 3 0 I I 1 I ,.,12 2 3
.06 1 114& 1151 1153 1156 1159 1161 1164 1167 1169 II 72 3 0 I I I I ;12 2 3
.07 1175 1178- II 80 1183 1186 1189 1191 1194 1197 1199 3 0 I I I I 212 2 3
.08 1202 1205 1208 1211 1213 1216 1219 1222 1225 1227 3 0 I I I I 2 2 2 3
.09 1230 1233 1236 1239- 1242 1245 1247 1250 1253 1256 3 0 I I I I 2 2 2 3
.10 1259 1262 1265 1268 1271 1274 1276 1279 1282 1285 3 0 I I I I 2 2 2 3
.11 1288 1291 1294 1297 1300 1303 1306 1309 1312 1315 3 0 I I I 2 2 2 2 3
.12 l318 1321 1324 1327 1330 1334 1337 1340 1343 1346 3 0 I 2 2 2 2 3
1352 1355 1358 1361 1365 1368 1371 1374 III
.1311349 3 0 I I I 2 2 2 2 3
1384 1387 1390 1393 1396 1400 1403 1377,
.14,1380 1406 1409 3 0 I 2 2 2 2 3
.15 1413 1416 1419 1422 1426 1429 1432 1435 1439 1442 3 0 I III
I I 2 212 2 3
I
.16 1445 1449 1452 145511459 1462 1466 1469 1472 1476/ 3 0 I I, I 2 212 2 3
.17 1479 1483 1486 1489 1493 1496 1507 1510 4 0
150011503 I
112 2 2 3 3 4
.18 1514 1517 1521 1524 1528 1531 1535 1538 1542 1545 4 0 I I 2 2 213 3 4
.19 1549 1552 1556 1560 1563 1567 1570 1574 1578 1581 1 4 0 I 1 i 2 2 213 3 4
.;W 1585 1589 1592 1596 1600 1603 160711611 1614 1618 4 0 I I 2 2 3 3 4
21
21 1622 1626 1629 1633 1637 1641 1644 1648 1652 1656 4 0 1 1 2 2 2 3 3 4
.22 1660 1663 1667 1671 1675 1679 1683 1687 1690 1694 4 0 1 I 2 2 2 3 3 4
.23 1698 1702 1706 1710 1714 1718 1722 1726 1730 1734 4 0 I 1 2 2 2 3 3 4
.24 1738 1742 1746 1750 1754 1758 1762 1766 1770 1774 4 0 1 I 2 2 2 3 3 4
.25 1778 1782 1786 1791 1795 1799 1803 1807 1811 1816 4 0 I 1 2 2 2 3 3 4
.26 1820 1824 1828 1832 1837 1841 \845 1849 1854 1858 4 0 1 I 2 2 2 3 3 4
.27 1862 1866 1871 1875 1879 1884 1888 1892 1897 1901 4 0 I 1 2 2 2 3 3 4
.28 1905 1910 1914 1919 1923 1928 1932 1936 1941 1945 4 0 I 1 2 2 2 3 3 4
.29 1950 1954 1959 1963 1968 1972 1977 1982 1986 1991 4 0 I I 2 2 2 3 3 4
.30 1995 2000 2004 2009 2014 2018 2023 2028 2032 2037 5 0 I I 2 2 3 3 4 4
.31 2042 2046 2051 2056 2061 2065 2070 2075 2080 2084 5 0 I I 2 2 3 3 4 4
.32 2089 2094 2099 2104 2109 21I3 2118 2123 2128 2133 5 0 I I 2 2 3 3 4 4
.33 2138 2143 2148 2153 2158 2163 2168 2173 2178 2183 5 1 I 2 2 3 3 4 4 5
.34 2188 2193 2198 2203 2208 2213 2218 2223 2228 2234 5 I I 2 2 3 3 4 4 5
.35 2239 2244 2249 2254 2259 2265 2270 2275 2280 2286 5 I I 2 2 3 3 4 4 5
.36 2291 22% 2301 2307 2312 2317 2323 2328 2333 2339 5 I I 2 2 3 3 4 4 5
.37 2344 2350 2355 2360 2366 2371 2377 2382 2388 2393 6 I I 2 2 3 4 4 5 5
.38 2399 2404 2410 2415 2421 2427 2432 2338 2443 2449 6 1 I 2 2 3 4 4 5 5
.39 2455 2460 2466 2472 2477 2483 2489 2495 2500 2506 6 I 1 2 2 3 4 4 5 5
.40 2512 2518 2523 2529 2535 2541 2547 2553 2559 2564 6 I I 2 2 3 4 4 5 5
.41 2570 2576 2582 2588 2594 2600 2606 2612 2618 2624 6 1 I 2 2 3 4 4 5 5
.42 2630 2636 2642 2649 2655 2661 2667 2673 2679 2685 6 I I 2 2 3 4 4 5 5
.43 2692 2698 2704 2710 2716 2723 2729 2735 2742 2748 6 1 I 2 2 3 4 4 5 5
.44 2754 2761 2767 2773 2780 2786 2793 2799 2805 2812 6 I I 2 2 3 4 4 5 5
.45 2818 2825 2831 2838 2844 2851 2858 2864 2871 2877 7 1 I 2 3 3 4 5 6 6
I
.46 2884 2891 2897 2904 2911 2917 2924 2931 2938 2944 7 I I 2 3 3 4 5 6 6
.47 2951 2958 2965 2972 2979 2985 2992 2999 3006 3013 7 I I 2 3 3 4 5 6 6
.48 3020 3027 3034 3041 13048 3055 3062 3069 3076 3083 7 I I 2 3 4 4 5 6 6
.49 3090 3097 3105 31I2 3119 3126 3133 3141 3148 3155 7 I I 2 3 445 6 6
ANTILOGARITHMS 1()¥

llm 1 2 3 4 5 6 7 8 9
x 0 1 2 3 4 5 6 7 8 9
+ ADD
.50 3162 3170 3177 3184 3192 3199 3206 3214 3221 3228 7 I 1 2 3 4 415 6 6
.51 3236 3243 3251 3258 3266 3273 3281 3289 3296 3304 8 1 2 2 3 4 516 6 7
.52 3311 3319 3327 3334 3342 3550 3357 3365 3373 3381 8 1 2 2 3 4 5 6 6 7
.53 3388 3396 3404 3412 3420 3428 3436 3443 3451 3459 8 1 2 2 3 456 6 7
.54 3467 3475 3483 3491 3499 3508 3516 3524 3532 3540 8 1 2 2 3 4 5 6 6 7
.55 3548 3556 3565 3573 3581 3589 3597 3606 3614 3622 8 I 2 2 3 4 5 6 6 7
56 3631 3639 3648 3656 3664 3673 3681 3690 3698 3707 8 I 2 2 3 4 5 6 6 7
.57 3715 3724 3733 3741 3750 3758 3767 3776 3784 3793 9 I 2 3 4 4 5 6 7 8
.58 3802 3811 3819 3828 3837 3846 3855 3864 3873 3882 9 I 2 3 4 4 5 6 7'8
.59 3890 3899 3908 3917 3926 3926 3945 3954 3863 3972 9 I 2 3 4 5 5 6 7 8
-.60 3981 3990 3999 4009 4018 4027 4036 4046 4055 4064 9 I 2 3 4 5 5 6 7 8
.61 4074 4083 4093 4102 4111 4121 4130 4140 4150 4159 10 I 2 3 4 5 6 7 8 9
.62 4169 4178 4188 4198 4201 4217 4227 4236 4246 4256 10 I 2 3 4 5 6 7 8 9
.63 4266 4276 4285 4295 4305 4315 4325 4335 4345 4355 10 I 2 3 4 5 6 7 8 9
.64 4365 4375 4385 4395 4406 4416 4426 4436 4446 4457 10 1 2 3 4 5 6 7 8 9
.65 4467 4477 4487 4498 4508 4519 4529 4539 4550 4560 10 II 2 3 4 5 6 7 8 9
.66 4571 4581 4592 4603 4613 4624 4634 4645 4656 4667 11 II
2 3 4 5 7 8 9 10
.67 4677 4688 4699 4710 4721 4732 4742 4753 4764 4775 II 1 2 3 4 5 7 8 9 10
.68 4786 4797 4808 4819 4831 4842 4853 4864 4875 4883 11 I 2 3 4 6 7 8 9 10
.69 4898 4909 4920 4932 4943 4955 4966 4977 4989 5000 11 1 2 3 4 6 7 8 9 10
.70 5012 5023 5035 5047 5058 5070' 5082 5093 5105 5117 12 I 2 4 5 6 7 8 lOll
.71 5129 5140 5152 5164 5176 5188 5200 5212 5224 5236 12 1 2 4 5 6 7 8 10 II
.72 5248 560 5212 5284 5297 5309 5321 5335 5346 5358 12 1 2 4 5 6 7 8 10 11
.73 5370 5383 5395 5408 5420 5433 5445 5458 5470 5483 12 I 2 4 5 6 7 8 10 II
.74 5495 5508 5521 5534 5546 5559 5512 5585 5598 5610 13 I 3 4 5 6 8 9 10 12
.rr- J023- 5636 5649 5662 5675 5689 5702 5715 5728 5741 13 I 3 4 5 7 8 9 10 12
.76 5754 5768 5781 5794 5808 5821 5834 5848 5861 5875 13 1 3 4 5 7 8 9 10 12
.77 5888 5902 5916 5929 5943 5957 5970 5984 5998 6012 14 I 3 4 6 7 8101113
.78 6026 6039 6053 6067 6081 6095 6109 6124 6138 6152 14 I 3 4 6 7 8101113
.79 6166 6180 6194 6209 6223 6237 6252 6266 6281 6295 14 I 3 4 6 7 8 10 II 13
.80 6310 6324 6339 6353 6368 6383 6397 6412 6427 6442 IS I 3 4 6 7 9 10 1213
.81 6457 6471 6486 6501 6516 6531 6546 6561 6577 6592 15 2 3 5 6 8 9 II 12 14
.82 6607 6622 6637 6653 6668 6683 6699 6714 6730 6745 15 2 3 5 6 8 9 11 12 14
.83 6761 6776 6792 6808 6823 6839 6855 6871 6887 6902 16 2 3 5 6 8 10 II 13 14
.84 6918 6934 6950 6996 6982 6998 7015 7731 7047 7063 16 2 3 5 6 8 10 II 13 14
.85 7079 7096 7112 7129 7145 7161 7178 7194 7211 1228 16 2 3 5 6 8 10 II 13 14
.86 1244 1261 7278 7295 7311 7328 7345 7362 7379 7396 17 2 3 5 7 8 10 121415
.87 7413 7430 7447 7564 7482 7499 7516 7534 7551 7568 17 2 3 5 7 910 121415
.88 7586 7603 7621 7638 7656 7674 7691 7709 7127 7745 18 2 4 5 7 9 11 13 14 16
.89 7762 7780 7798 7816 7834 7852 7870 7889 7907 7925 18 2 4 5 7 9 11 13 14 16
.90 7943 7962 7980 7998 8017 8035 8054 8072 8091 8110 18 2 4 5 7 9 11 131416
.91 8128 8147 8166 818S 8204 8222 8241 8260 8279 8299 19 2 4 6 8 1011 1315 17
.92 8318 8337 8356 8375 8395 8414 8433 8453 8412 8492 19 • 2 4 6 81011 1315 17
.93 8511 8531 8551 8570 8590 8610 8630 8650 8670 8690 20 2 4 6 8 1012 141618
.94 8710 8730 8750 8770 8790 8810 8831 8851 8812 8892 20 2 4 6 8 1012 141618
95 8913 8933 8954 8974 8995 9016 9036 9057 9078 9099 21 2 4 6 8 1013 151719
.96 9120 9141 9162 9183 9204 9226 9247 9268 9290 9311 21 2 4 6 81113 15 17 19
.97 9333 9354 9376 9397 9419 9441 9462 9484 9506 9528 22 2 4 7 9 11 13 151820
.98 9550 9512 9594 9616 9638 9661 9683 9705 9127 9750 22 2 4 7 9 11 13 15 1820
.99 9772 9795 9817 9840 9863 9886 9908 9931 9954 9977 23 2 5 7 9 11 14 161821

You might also like